Текст
                    1ГН.Литвиненко А.Г.Мордкович
ПРАКТИКУМ
ПО ЭЛЕМЕНТАРНОЙ
МАТЕМАТИКЕ
АЛГЕБРА .
ТРМГПНПкЛРТРШ


В. Н. ЛИТВИНЕНКО А. Г. МОРДКОВИЧ ПРАКТИКУМ ПО ЭЛЕМЕНТАРНОЙ МАТЕМАТИКЕ АЛГЕБРА. ТРИГОНОМЕТРИЯ Учебное пособие для студентов физико-математических специальностей педагогических институтов и учителей Рекомендовано Министерством образования Российской Федерации 3-е издание, переработанное и дополненное МОСКВА «ABF» 1995
ББК22.1 Л64 Литвиненко В. Н., Мордкович А. Г. Л64 Практикум по элементарной математике: Алгебра. Тригонометрия: Учеб. пособие для студентов физ.-мат. спец. пед. ин- тов. — 3-е изд., перераб. и доп. — М.: «ABF», 1995 — 352 с: ил. - ISBN 5-87484-023-0. Настоящее пособие предназначено студентам физико-математических факультетов педагогических институтов и университетов и имеет своей целью дать студентам и преподавателям педвузов материалы для практических занятий по многосеместровому курсу "Элементарная математика и практикум по решению математических задач", который занимает важное место в профессиональной подготовке будущего учителя. Эта книга не только и не столько задачник, сколько практикум. Это нашло свое отражение в структуре книги: каждый параграф, кроме упражнений для самостоятельного решения, содержит необходимый теоретический материал и довольно большое число различных по трудности примеров с подробными решениями. Книга будет полезна значительно более широкому контингенту читателей — это поступающие в вузы, учащиеся старших классов общеобразовательных школ, преподаватели математики. _ 2404010000-025 л _ ___ _ t ЛОШ4(03)-95 6e3°fo"№ ББК22Л Учебное издание Литвиненко Виктор Николаевич Мордкович Александр Григорьевич ПРАКТИКУМ ПО ЭЛЕМЕНТАРНОЙ МАТЕМАТИКЕ АЛГЕБРА. ТРИГОНОМЕТРИЯ Сдано в набор 06.06.95. Подписано к печати 20.06.95. Формат 108X84 1/32 Бум. кн. журнальная Гарнит. Литерат. Печать офсетная. Усл.печл. 22. Усл.кр.-отт. 22,68. Уч.-издл. 20,84. Тираж 10 000 экз. Заказ 442. Издательство «ABF», тел. 138-18-62; 291-16-50 АО «Астра-семь». 121019, Москва, Филипповский пер., 13. ISBN 5-87484-023-0 © Литвиненко В. Н., Мордкович А. Г., 1995
Часть I. АЛ ГЕБРА Глава I. ТОЖДЕСТВЕННЫЕ ПРЕОБРАЗОВАНИЯ f 1. РАЗЛОЖЕНИЕ МНОГОЧЛЕНОВ НА МНОЖИТЕЛИ При решении многих алгебраических задач бывает необходимо данный многочлен представить в виде произведения двух или более многочленов или в виде произведения многочлена на одночлен, содержащий не менее одной переменной. Однако не каждый многочлен допускает разложение на множители над полем действительных чисел. Например, многочлены х+3, д^+бя+Ю разложить на множители нельзя. Такие многочлены называются неприводимыми. Разложение многочлена на множители считается законченным, если все полученные множители неприводимы. При разложении многочленов на множители применяются различные приемы: вынесение общего множителя за скобки, группировка, использование формул сокращенного умножения и др. Рассмотрим несколько примеров применения этих приемов. Пример 1. Разложим на множители многочлены: 1) f(6)223626362 рр 1) f(a;6)=a2-2a36-2a63+62; 2) /(a) = a4-5a3-8 0 Р 1) С ) /() 8a + 40. Решение. 1) Сгруппируем, например, первый член со вторым, а третий — с четвертым и вынесем за скобки общие множители. Получим / (а; Ь)=а2 (1 -2а6)+ Ъ2 (1 -2aft)=(l -2а6) (а2 + б2). 2) Получаем f (а)=аг (a-5)-8(a-5)=(a-5)(a3-8) = ==(a-5)(a-2)(a2 + 2a + 4). Пример 2. Разложим на множители /(a;6)=4a2-12a6 + 562. Решение. 1-й способ. Дополним выражение 4a2— \2ab до полного квадрата. Получим 4a2— I2ab+9b2. Тогда /(a;ft)= 2 X (2a - 36 + 26)=(2a - 56) (2a - 6). 2-й способ. Представим второй член заданного трехчлена следующим образом: — 12а6 = — 2а6 — Юаб. Тогда /(а; 6)=(4а2 — 2а6)+ +(-10а6 + 562)=2а(2а-6)-56(2а-6)=(2а-56)(2а-6). Пример 3. Разложим на множители /(a)=a3-7a2 + 7a+15. Решение. 1-й способ. Представим второй и третий члены за-
данного многочлена следующим образом: — 7а2 = —За2—4а2; 7а=12а — 5а. Тогда /(а) = а3 —За2 —4а2+12а —5а+15. Сгруппируем слагаемые попарно и в каждой группе вынесем за скобки общие множители: " (а)==(аз__3а2)-(4а2- 12а)-(5а-15)= =а2(а-3)-4а(а-3)-5(а-3)=(а-3)(а2-4а-5). Осталось разложить на множители многочлен а2 —4а—5. Это можно сделать, дополнив выражение а2 —4а до полного квадрата. Получим а2 — 4а — 5 = (а2 — 4а + 4)-9±=(а — 2)2-9 = (а — 2 + 3) (а — 2 — 3) = =(а+1)(а — 5). Можно поступить и по-другому, а именно представить второй член следующим образом: — 4а=а — 5а. Тогда ()(+)(+)() И наконец, можно найти ai = — 1 и аг = 5— корни уравнения a2 —4a —5 = 0. Тогда, воспользовавшись формулой разложения квадратного трехчлена на множители (ax2 + fcjt+c = a(x—X\)X Х{х—х2)\ получим а2 — 4а—5 = (а+1)(а—5). Итак, f(a)*(a-3)(a+l)(a-5). 2-й способ. Представим третий член так: 7a=15a — 7a—a. Тогда /(a)=(a3-a)-(7a2 + 7a)+(15a+15)=a(a-l)(a4-l)-7a(a+l)+ +15 (a+ !)=(<*+1) (a2-a-7a+ 15)=(a+1) (a2-8a+15)= =(a+l)(a-3)(a-5). Пример 4. Разложим на множители f(a;b;-c)=ab(a+b)—bc(b + c)+ac(a—c). Решение. Воспользуемся тем, что выражение в первых скобках есть сумма выражений, содержащихся во вторых и третьих скобках: a-\-b = (b + c)+(a — с). Тогда f()(( )+())( + ) () =a& (6 + c)+aft (a—c)— be (b-\-c)+ac (a — c). Выполним далее группировку членов и вынесем общий множитель за скобки. Получим: Пример 5. Разложим на множители /(a) = a4—10a2+169. Решение. Заметив, что a4 + 169=(a2)2+132, и дополнив эту сумму до полного квадрата, получим: ()( ) =(a2+13)2-(6a)2=(a2-6a+13)(a2+6a+13). На этом разложение на множители закончено, так как дискриминанты квадратных трехчленов а2 — 6а + 13 и а2 + 6а+13 отрицательны и, значит, эти трехчлены нельзя разложить на линейные множители. Пример 6. Разложим на множители /(a; b)=as 2 * 4
р е ш е н и е. Так как а*-Ьв=(а2?-(Ь2У=(а2-Ь2)(аА+а2Ь2+ЬА\ то /(а; Ь)=(а2-Ь2)(аА + а2 2 Но Таким образом, f (а; Ь)=(а2 + аЬ + Ь2)(а2-а Пример 7. Разложим на множители /() + + + Решение. Нетрудно увидеть, что выражению / (а) до полного куба суммы не хватает слагаемого 8. Поэтому можно записать + (а + 3).2+4)=(а+1)(а2 Пример 8. Докажем, что если а 6 N и/ (а) = а4 + 6а3 + 11 а2 + 6а, то / (а) • 24*. Решение. Представим 6а3 и 11а2 в виде суммы подобных членов: 6а3 = а3 + 5а3 и 11а2 = 5а2 + 6а2. Тогда + 6a)=a3(a+l)+5a2(a + l)+6a(a+l)= Но из четырех последовательных натуральных чисел хотя бы одно делится на 3, а два числа являются четными, причем одно из них делится и на 4. Значит, произведение этих четырех чисел делится на 3*2*4. Таким образом, / (а): 24. П р и м е р 9. Докажем, что если / (а)=а2 (а2 +14)+49, где а — нечетное число, то / (а) I 64. Решение. Заметим, что f(a)=a4+14a2 + 49=(a2 + 7)2. Так как а нечетно, то а = 2л —1, где n£N. Тогда /(а) = /(2л— 1)= =((2л-1)2 + 7)2=(4л2-4л+8)2==16(л2-/1 + 2)2. Полученное выражение делится на 16. Поэтому, чтобы доказать, что f (а) :64, достаточно показать, что (л2 — п + 2)2:4. Рассмотрим два возможных случая: 1) п — четное число и 2) п — нечетное число. 1) Если п четно, то п2 тоже четно, и, следовательно, п2 — п + 2 четно, т. е. (л2 — л+ 2): 2, поэтому (л2 — я + 2)2:4, а значит, / (в): 64. 2) Если п нечетно, то п2 тоже нечетно, но тогда л2 —л четно и л2 —я+ 2 четно. Таким образом, и в этом случае ((а): 64. Упражнения Разложите на множители. 1. а<-1. 2. а4- 18а2 + 81. 3. а* + *-а*-\. 4. а5 + 3а4-4а3-12а2. Напомним, что знак «;> •значает «делится на» (без остатка).
5. а4 + 2а3—2а — 1. 6. V2 7. 8. 9. а8-f а4 + 1. 10. а4 + 324. 11. а4 + 6\ 12. а6-1. 13. аб+1. 14. а'2-2а6+1. 15. а4 + 4а2-|5. 16. 4а4 + 5а2+1. 17. 18. а4 + За3+4а2-6а-12. 19. а3 + а-2. 20. 2а3-а2+3. 21. а3 + 5а2 + 3а —9. 22. 2а26+4а62 -а2с +ас2 -462с +2Ьс2 -4аЬс. 23. (ab + ac+bc)(a+b + c)—abc. 24. ()2 ()( + ) 25. а3 (а2 -7)2- 36а. 26. (а + 6)5—(аъ + Ь5). 27. а262(6-а) + 62с2(с-6)+а2с2(а-с). 28. 29. (+ ^ 30. (а2 + а+3)(а2 + а+4)-12. 31. в(а+1)(а + 2)(а+3)+1. 32. (а+1)(а + 3)(а+5)(а+7)+15. 33. 2(а2+2а-1)2 + 5(а2+2а~1)(а2+1)+2(а2+1)2. 34. (а- 35. (а- 36. 37. 38. а2Ь+аЬ2+а2с+ас2+Ь2с+Ь<?+Шс. 39. а4 + 64 + с4—2а262—2а2с2—262с2. 40. а5+а4+а3+а2+а+1. 41. о4 + 2а3+За2 + 2а+1. 42. а4 — 2а36— 8а262—баб3 — б4. 43. 44. 45. Докажите, что если a£Nt то (а5—5а3+4а) • 120. 46. Докажите, что если а — число, взаимно-простое с 6, то (а2 —1) • 24. 47. Докажите, что если а£ЛГ, то (2а3+3а2+а) \ 6. 48. При каких значениях a£N выражение а4+4 является простым числом? а а2 а3 49. Докажите, что если а — число четное, то у^ + -g- + ^т — число целое. а5 а4 7а3 5а2 а 50. Докажите, что если a£Nt то -^ +f2 + 24 +l2 +"5""~число целое-
§ 2. ТОЖДЕСТВЕННЫЕ ПРЕОБРАЗОВАНИЯ РАЦИОНАЛЬНЫХ ВЫРАЖЕНИЙ Замена аналитического выражения другим, тождественно равным ему на некотором множестве, называется тождественным преобразованием данного выражения на этом множестве. При выполнении тождественных преобразований выражения возможно изменение его области определения. Так, приводя подобные члены при упрощении выражения *2 + 3x-5+V*-V*> (1) мы расширяем его область определения: данное выражение определено лишь при х^О, тогда как получающийся после упрощения многочлен *2 + 3-5 (2) определен при любых значениях х. Выражения (1) и (2) тождественно равны лишь на множестве [0; оо). Область определения выражения может измениться и после сокращения дроби. Так, алгебраическая дробь определена при хФ 19хФ—2. После сокращения на дс— 1 получается дробь х+2 ' W определенная при х^ —2. Выражения (3) и (4) тождественно равны на множестве (— оо; —-2)U( — 2; 1)11(1; <»)• Изменение области определения выражения возможно и в результате некоторых других преобразований, поэтому, выполнив преобразования данного выражения, нужно всегда знать, на каком множестве оно тождественно полученному. Алгебраическое выражение / (а; Ь\ с\...) называют рациональным, если относительно входящих в него букв а, &, с, ... не выполняется никаких других операций, кроме операций сложения, умножения, вычитания, деления и возведения в целую степень. Пример 1. Упростим выражение /(a; fr)= Решение. Разложим на множители каким-нибудь способом числитель заданной дроби. Например, представим ab в виде суммы подобных членов: ab = 2ab — ab. Тогда 2a2 + ab — Ь2 = 22 + 2Ь 6&2 2() Итак, f(a; Ь)=(а+ьЦ2°-ь)=2а-Ь. Так как сокращение на а + Ь может быть выполнено лишь при условии, что а + ЬФО, то / (а; Ь)= =2а —6, если аФ—Ь.
Пример 2. Упростим выражение /(g)=a<710a'+169. Решение. Разложив на множители числитель, получим (см. пример 5, с. 4) а4 — 10а2 + 169=(а2 + 6а+13)(а2- Значит, f(a)^^ Так как <r + 6a-f- 13 не обращается в нуль ни при каком действительном значении а (а2 + 6а+13=(а + 3)2 + 4>0), то f(a)=a2 — 6a+ 13 при всех значениях а. Пример 3. Упростим выражение V (* 2fl Решение. Так как a2 + 3a-f2 = (a + I)(a-f2), a2 + = (а+1)(я + 3), a2-f5a + 6 = (a + 2)(a + 3), TQ наименьший общий знаменатель трех первых дробей равен (a+l)(a + 2)(a + 3). Тогда, выполняя указанные действия, получим: V (a+l)(a + 2)(a V __/ 2a2 + 6a +4 \2 V (a+l)(a + 2)(a + 3)/ 2 2 # (a + 3)2 „ 2 Итак, /(а) = 2, если аФ — 1, аФ—2, аФ—3. Пример 4. Упростим выражение «2 с2 f/д. fr. ~\_ Q I ' V • ' / In h\fn r\ ' (h Решение. Приведя все дроби к наименьшему общему знаменателю, получим /(а; Ь\ с)=а^Ь~^2^-^-^Ь)' Заметив* что Ь — с=(а-— с) — (а— 6), преобразуем числитель следующим образом: Таким образом, /(a; 6;c)=l, если афЬ, афс, Ьфс. Пример 5. Докажем, что если a-f- & + с = 0, то Решение. Так как а+Ь + с = 0, то a= —ft —с. Тогда a3 3 3 3 3 3 3 33 2 Ho = 3aftc. 8 -c)3 + ft3 + ^3=-(ft + ^)3 + fc3 + ^3=-(&3 + 3ft2c + )+ ft3 + c3= -(3ft2c + 3ftc2)= —3ftс (b + c). c=— a. Таким образом, a3 + ft3 + ^ = — 3ftc( — a)=
Пример 6. Докажем, что если а + Ь + с=0, где афЪ, , то Решение. Рассмотрим произведение первого множителя на первую дробь второго множителя: (Ъ — с с —а \ с (а—Ь , 6 — с , с—а \ с __ j , Ь2-Ьс+ас — а2 с _ * , а& а—Ь с (a~-fe)--(a2 — Но по условию а + fr = — с. Поэтому для рассматриваемого произ- 2с2 ведения получаем 1 Н—— . Аналогично произведение первого множителя на вторую дробь второго множителя равно 1 -\—— , а на третью дробь равно 1 -\ . Сложим полученные результаты 1+4+1+^ + 1+— 1 аЬ ' ' be ' ' са =3 + 2(4 + ^ + —)- ' \ab ' be ' ас / абс Так как a? + b3 + c3 = 3abc (см. пример 5, с. 8), то 3 . 2(a3 + 63 + c3) ^g . 2>3а6с =д ' аЬс "■ абс ' что и требовалось доказать. В следующих примерах тождественные преобразования рациональных выражений выступают не как цель, а как средство решения задач с использованием метода математической индукции. Метод математической индукции формулируется следующим образом: Утверждение, зависящее от натурального числа п, справедливо для любого п, если выполнены два условия: а) утверждение справедливо для п=1; б) из справедливости утверждения для n = k (при любом натуральном значении к) вытекает его справедливость и для п = k +1. Доказательство по методу математической индукции проводится так. Сначала доказываемое утверждение проверяется для п = 1. Эту часть доказательства называют базисом индукции. Следующую часть доказательства называют индукционным шагом. В этой части доказывают справедливость утверждения для n = fe+l в предпо- лбжении справедливости утверждения для n = k (предположение индукции).
Пример7. Докажем, что + 2 + 3 + ... + п^ Решение. При я = 1 утверждение справедливо, так как __»(1 + 0(2+1) предположим, что оно верно при n — k, т. е. 6. докажем< что тогда оно верно „ при /г = & + 1, т. е. В самом деле, 12 + 22 + 32 + ...+ 6 Тем самым доказана справедливость утверждения для любого натурального числа п. Пример 8. Докажем, что 13 + 23 + 33 + ... + /г3 = (^^П)*. Решение. При п = 1 утверждение справедливо, так как 13 = = Г i^tii j # Предположим, что оно верно при n = fe, т. е. l3 + 23 + 33 + ... + fe3= ( v" ) • Докажем, что тогда оно верно и при п = к-\-1, т. е. В самом деле, 13 + 23 + 33 + + Л3+(*+1)3= | (fc | l)3_J Тем самым доказана справедливость утверждения для любого натурального числа п. Пример 9. Докажем, что (32л+1+40л-67):64 (5) при любом натуральном п. Решение. Если п= 1, то 33-f 40-1 —67 = 0, но 0 : 64. Значит, при л=1 утверждение (5) верно. Предположим, что оно верно при л=&, т. е. (32Л+! + 40Л—-67): 64. Докажем, что тогда оно верно и ю
при n = k+1. В самом деле, имеем 32А+3 +40 (* +1)-67=9.32*+1 + 40*279(32Л+1+40*67)320/ + 5769(32Л+1+40Л67)+ 4 + 64(9-5*). Каждое слагаемое делится на 64, следовательно, и вся сумма делится на 64. Итак, утверждение (5) верно при всех л£ЛЛ Пример 10. Докажем, что И 3224 (6) при любом натуральном п. Решение. При л= 1 утверждение справедливо, так как 1+6 + + 11+6=24, а 24: 24. Допустим, что утверждение (6) верно при n=k9 т. е. (&4+6Л3 + + 11Л2 + 6&) -24. Докажем, что тогда оно верно и при л = Л + 1. Действительно, имеем (Л + 1)4+6(А+1)3+11 (Л +1)4-6 (£+1)= 46fe3llfc26/024(fe2+l)4(*3+llfc) ((() Если мы теперь докажем, что (Л3+11/0:6 (7) при всех k, то этим будет доказано, что ( ) ( ) +11 (k+1)2 + 6 (6+1): 24, а тогда по методу математической индукции получается, что заданное выражение делится на 24. Перед нами новая задача, для решения которой снова используем метод математической индукции. Проверим прежде всего, справедливо ли утверждение (7) при k = l. Это очевидно: (1 + 11): 6. Пусть утверждение (7) верно при £ = т, т. е. (т3+11т):6. Докажем, что тогда оно верно и при ft = m+l. В самом деле, Из двух последовательных натуральных чисел т, (т +1) одно обязательно четно, значит, (m(m + l)):2t a (3m(m+l)):6. Но тогда ((m3+llm)+12+3m(m + l)):6. Отсюда заключаем, что (Л3+Ш) :6 при любом натуральном k. Утверждение (7) доказано. Таким образом, утверждение (6) верно для всех n£N. Заметим, что рассмотренный пример может быть решен и без применения метода математической индукции (см. пример 8, с. 5). Можно и утверждение &3+11Л:6 доказать, не применяя метод математической индукции, например, так: Здесь каждое слагаемое делится на 6 (первое — как произведение трех подряд идущих натуральных чисел), значит и вся сумма делится на 6. Пример 11. Докажем, что сумма кубов трех последовательных натуральных чисел делится на 9. 11
Решение. 1-й споеоб. Надо доказать, что ("3 + (" + 1)3+(л + 2)3):9 (8) при любом натуральном п. Проверим прежде всего, справедливо ли утверждение (8) при я = 1. Имеем 13+23+33 = 36, но 36 :9, следовательно, при л=1 утверждение верно. Предположим, что утверждение (8) верно при я = &, т. е. Докажем, что тогда оно верно и при м = &+1. В самом деле, (Л + 1)3+(* + 2)3 + (Л + 3)3(й+1)3+(/ + 2)3 + Л3 + 9Л2 + 27* + 27 (()( ))( + ) Поскольку каждое слагаемое полученной суммы делится на 9 (первое слагаемое — по предположению индукции, второе — как содержащее множитель 9), то и сумма делится на 9. По принципу математической индукции заключаем, что утверждение верно при всех n£N. 2-й способ. Имеем п3+(п +1 )3 + (п + 2)3 = я3+(м3 + Зл2 +3л +1)+ + (/г34-6/г2+12А1 + 8) = 3/г3+9/г2+15п + 9 = 3(п3 + 5п) + 9(л2+1) = = 3 {пг+11л)— 18п +9 (п2+ 1). Из соотношения (7) следует, что (я3+11л): 6, тогда тем более (п3-|-1 \п): 3, значит 3 (л3+1 In): 9. Так как, далее, 18л : 9 и 9 (п2+1): 9, то и вся сумма 3(л2+11м)— 18л+9(л2+ 1) делится на 9. Упраж нения Сократите дроби. оЧ-оЧ-аЧ-1 53. Js^e . 54. ^ 55. Упростите выражения. _J 1_ __ 2a _ 4a3 _ 8a7 58* i—a i4_a T+tf jqp? "Pf?* 59. -!— + - l * 2 * 4 * 8 ' eo i , L # a(a+l)^ (a+l)(a+2j ' (a-f 2)(a-f 3) ^ (а+3)(а+4) ^ (a+4)(a+5) * a , a2 + a—1 , a2 —a—1 2a3 61. -r^r + ^_ \ , _. + з i h . i—r- 12
J_ 1 a b + c ~a~T+c 04. / «\ / q-r-6 , b + c с +a Д3 —g3 Л, f^ 1+<Л , 2b — bc2 ' V ~t~a —с с /* a 1 a " a2 + 2ab-\-2b2 a2-2ab + 2b2 4b2 (a2 + 2b2) ^ 4b2 (a2-2b2)' a — b b — c c—a (a—b)(b — c)(c—a) a + b + b + c "^"с + а"4" {а+Ь)(Ь + с){с+а) ' 71. Докажите тождество: ^^.c c—a a—6 (a-b){a-c)^ (b-c){b-a) + (с-а)(с~6) a-6 + &- 72. Докажите тождество: rfc)(rfq) (rffl)(rf6) _ 2 6) 73. Докажите, что из равенства а24-62 + с2==а6 + ас+6с следует, что а=Ь = 74. Докажите, что если (a~6)2+(6-c)2+(c-a)2=(a + ^-2c)2+(6 + c2 + (c+a—26)2, то а = Ь = с. 75. Докажите, что (а — 1) (а—3) (а—4) (а—6) +10— положительное число при любом а 6R. 76. Найдите наименьшее значение выражения (а—1) (а—3) (а—4) (a—6)+10. 77. Докажите, что если a + & + c=0, то •»o tt I . m . n t a , 6 , с л /2 , m2 , n2 4 78. Докажите, что если p—r- H =1 и -Н 1 =0, то -т+гг Н—г = 1- а о с I m n а о с 79. Докажите, что если -_£_ + -А_ + —— =0, где а^=6, а^с, Ьфс, то о — с с—а а — b Л^=0. ft П 1,1.1 1 1,1,1 0. Докажите, что если Ь-т-Н = 4-А4- • то "^" + "м* "^ "?": где п — нечетное натуральное число. Докажите тождества методом математической индукции*. 81. * В упражнениях 81—119 предполагается, что n£N. 13
83. 85. я» 0,1,2, 86-F+F+*+ • — 87 1-3 2(2я + 1)# я(я + 1.2-3 S»3»4 п(л + 1)(я + 2) 2 \2 (n 90. 91. 2.1Ч3. 2t 1.2.3.4 + 2.3.4.5 + "> + /г(м + 1)(« _J/J 1 ~" 3 V6 93. 94. 7 + 77 + 777 + ...+777...7=- n цифр 95. (« + 1)(л + 2)...(я + л)=2".1 •, где хф\. (Юя+1~9л-10) Выведите формулы для сумм. 1 . 1 . . 1 99. Sn = 100. Sn 101. Sn + 6-11 т'"^(5я- +32-4t + ...+(- Докажите тождества. 102. 14 , где
Докажите справедливость утверждений. 107. (62я-1) • 35. 108. (4П+15п~1): 9. 109. (25я+3+5я-Зя+2) I 17. ПО. (62я+Зя+2+Зя) : 11. 111. (32я+2-8я-9): 64. 112. (33я+2 + 5.23я+1) : 19. 113. 2я+5.34я+53п+|): 37. 114. (7я+2+82я+1): 57. 115. (llrt+2 + 122n+I): 133. 116. (2я+2.Зя+5/г-4) : 25. 117. (52я+1+2п+4+2я+1) : 23. 118. (32п+2.52я-33я+2.22я) : 1053. § 3. ТОЖДЕСТВЕННЫЕ ПРЕОБРАЗОВАНИЯ ИРРАЦИОНАЛЬНЫХ ВЫРАЖЕНИЙ Алгебраическое выражение, содержащее операции извлечения корня из переменной или возведения переменной в рациональную степень, не являющуюся целым числом, называется иррациональным относительно этой переменной. Напомним определение арифметического корня. Если а>0 и n£N9 пф\, то существует только одно неотрицательное число х такое, что выполняется равенство x"=a. Это число х называется арифметическим корнем п-й степени из неотрицательного числа а и обозначается °sja. Из сказанного выше следует, что равенство -^49 = 7 является верным, тогда как равенства ^/59=— j или ^/49=±7 являются неверными. Если п — нечетное число, пф\ и а<0, то под V^ понимают такое отрицательное число jc, что x" = a. Из определения следует, что =( \о\% если л —четное число, \ а, если п — нечетное число, пФ 1. Например, \^||; V Если а>0, &>0, то: 1.° V^=VS-V6. Это свойство распространяется на произведение любого числа сомножителей, например: V8-27.125=V8-V27-Vl25=2-3-5=30. 2.° -\/4.=3t£, если ЬФО. 15
Замечание. Если а<0 и b<Ot то свойства 1° и 2° принимают вид 3.° ^)k = \f^. Например, (W 4.° Vw^V^- Например, 5 о тпг^ш ( Vla*l, если т четное, \ V^» если т нечетное Например, W= Замечание. Если показатели корней — нечетные числа, то свойства 1° — 5° выполняются и для а<0, 6<0, и для а&<0. Напомним определение степени с рациональным показателем. 1) Если афО, то а°=1. 2) Если а>0, то ая =V^" (Л> т — натуральные числа, п>2). 3) Если а>0, то a~~r=-V (г — положительное рациональное число). 4) Если a<0, m£Zy то а~т=-^г Напомним еще основные свойства степеней с рациональными показателями. Если а>0, &>0, г и 5 — произвольные рациональные числа, то: 1.° a'-as=a'+s. 4.° (a&)r = ar..6r. З.0 (aj=ars. Пример 1. Упростим выражение Решение. Сначала упростим каждый из имеющихся радикалов: После этого заданное выражение примет вид: Л =(4 л/2 + 3 V^-7 л/2) (6 л/2-Ю V5-2 V2)= =(3 л/5- 3 V2) (4 л/2 - Ю л/5). Далее получаем: Пример 2. Упростим выражение A=$J(2—y/7)A. ° J(/) Решение. По свойству 5° корней получаем A=-yJ\2—л/7|. Но 16
2-л/7<0. а поэтому Л=У—(2—л/7)=\ . Пример 3. Упростим выражение А=-\/27—Юл/2. Решение. Ясно, что выражение упростится, если окажется, что под знаком корня содержится полный квадрат разности каких-то двух чисел. Представим Юл/2 как удвоенное произведение двух чисел, сумма квадратов которых будет равна 27, т. е. 10 л/2 = 2 -у2*5. Таким образом, Л=У2 — 2л/2-5-Ь25=У(л/2 — 5)2= 1л/2 — 5|, и так как л/2 —5<0, то Л = 5—л/2. Пример 4. Проверим равенство -\/10 — 4л/б+У15 —6л/б=1. Решение. Так как 10 — 4 л/б = 4 — 2.2-л/6+6=(2 —>/б)2, а 15-бУб==9-2>ЗУб+6=(3-Уб)2, то л/Ю-4л/6+л/15-6л/6= 12—х/б| + |3—s/6|. Далее, так как 2<л/б, a 3>л/б, то 12—л/61 = —(2—л/б), а |3-Уб|=3-Уб. Итак, У10- Пример 5. Упростим выражение Л = ^9 V3— 11 ^ Решение. Рассуждая, как в примере 3, запишем подкоренное выражение в виде куба разности двух чисел. Имеем: 9->/3 = 3-V3 + 6V3=(V3)3 + 3V3-(V2)2 и И V V V V Xл/2+(л/2)3. Таким образом, Пример 6. Освободимся от иррациональности в знаменателе дроби А=—J—. V2-1 Решение. Умножив числитель и знаменатель дроби на неполный квадрат суммы чисел V2 и 1, получим: (V2)3-!3 Пример 7. Освободимся от иррациональности в знаменателе дроби А=5 1+м^л/5 Решение. Освободимся сначала от л/3 в знаменателе, для чего умножим числитель и знаменатель дроби на выражение, сопряженное знаменателю: А ^ 3(1+У2+л/3) (l+V2)2~3 лД) 2->/2 17
Теперь освободимся от ^2 в знаменателе: Пример 8. Вычислим сумму Решение. Пусть У20+УЗ^+У20—У392==Л. Возведем в куб обе части этого равенства. Получим: (20+л/392)+3 (V20+V392)2 У20- X (V20—л/^2)2 + (20 - л/392) или где Таким образом, 40 + зУ202-(У392)2.Л==Л3, 40 + 6Л=Л3, Л3- -6Л-40 = 0. Но Л3-6Л-40=(Л3-4Л2)+(4Л2-16Л)+(10Л-40)= —Л2 (Л -4)+4Л (Л -4)+ Ю (Л -4)=(Л -4) (Л2 + 4Л +10), Так как Л2 + 4Л + 10=(Л2 + 4Л + 4)+6=(Л + 2)2 + 6#0, то равенство (Л — 4)(Л2 + 4Л+ 10)=0 выполняется только при Л =4. Итак, yy Пример 9. Преобразуем выражение к виду, не содержащему знаков корня и модуля. Решение. Так как Уа2 —4а+4=У(а—2)2= |а—2| и Уа2 + 6а + 9=У(а + 3)2=|а + 3|, то /(а)=|а—2| + |а + 3|. Точки ai = — 3 и а2 = 2 разбивают числовую прямую на промежутки (— оо; —3), [—3; 2) и [2; оо). Рассмотрим заданное выражение на каждом из этих промежутков. При а<—3 имеем: |а — 2| = —a-f-2, |а + 3|=—а—3, т. е. /(а)=—а + 2 —а—3~—2а—1. При —3<а<2 имеем: |а — 2| = — а + 2, |а + 3|=а+3, и тогда /(а)= — а + 2 + а+3 = 5. Наконец, при а>2 имеем: |а —2|=а—2, |а+3|=а+3, —2а—1, если а<— 3, Итак, f{a)=l 5, если —3<а<2, 2a+l, если (a)=J Пример 10. Упростим выражение /(а; Ь)= где а>0, &>0. 18
Решение. / (a, ft)= -h/а-Ы Так как а>0, &>0, то VS + 6>0 и, следовательно, \-\[а+Ь\ = fe- Значит, Теперь нужно рассмотреть два случая: 1) ■yja—b'^0,2) -yJa--b<cO. В первом случае имеем: \^Ja — b\=^Ja—b, и, следовательно, =2^Ь. s/b Во втором случае |-\/а —&| =-—(Уа—-6), и, следовательно, > м— Уд + Ь+л/д — b _2->Jab •\[b b (2^Jb9 если Итак, f(a, 6)={2л^5 _ I —*—, если Пример 11. Упростим выражение Решение. Выполним преобразования по действиям: 3) V5(3V5-W)+VW 4) V?:V?=1. Итак, /(a, 6)=1, если Пример 12. Упростим выражение Решение. Освободимся от иррациональности в знаменателе сначала первой, а затем второй дроби. Имеем: 19
2) л/а —л/а—1 (л/я —^a- Итак, f (a)=^Ja— 1, если a>l. Упражнения Найдите значения выражений. 119. 2a2 — аЬ — Ъ2 при а=У5+1 и b = -yfi—\. 120. 2а2 —5а6 + 2&2 при a = ^+V§ и V 121. З 122. -362 при при ^^- и Ь^=^, V5-V2 V5+V2 ,23. *±MZ± при х—у+\ 124. при дг= 2ab 1 JL ± ± 125. 2а(1+*2)2 (x+il+x2)2)-1 при jc=y((а^1)2-(6а-1)2). Упростите выражения. 135. 137. 139. 141. 20 Л/2-У2-- Освободитесь от иррациональности в знаменателях дробей. 1 ,зв __!_ У5-УГ ■ VlS-W б+л/З 1 136. 138. 140. 142. +л/ 2+л/б V4+V6+V9'
Проверьте равенства 143. а) л/9^ Vv8+V-v^-i-VV8-V^7 ^ 14/• V27-3Vt8+3Vl2-V5 6+4л/2 , 6-4V2 У 9 151. Докажите тождества, указав область определения. 1 2 152. 153. 2а 1 5 2 Т аТ-аТ =0. 154. У>/3-У5> л/8+2 УГб—^ VV20+Vl2. Ve-2 VT5- Докажите тождества. Упростите выражения.
—х/55 , l—Vaifr \ 1—Vab — ,62 а+6 a— 163. а V<» V3aft-2eл/в*-лА»Э* (7+4 л/3). 164' ,в5. ,66. (a>0) ,e8 169. \ 1-- ■+- Ш\.'. -V^2 + 8a+16. т 170. /Г^а , УГ + аУП-Д)"3 \ > -jA—g >vrb l+a „3. („«=£=+ 174. a3+g6+l 22 /( V -)"'+2V5.
178. (-Jab — ab(a+-yjab)-1):— z—(a>0, 6>0). 179. fl~~ 180. 1 t § 4. ТОЖДЕСТВЕННЫЕ ПРЕОБРАЗОВАНИЯ ПОКАЗАТЕЛЬНЫХ И ЛОГАРИФМИЧЕСКИХ ВЫРАЖЕНИЙ Напомним основные сведения о логарифмах. Пусть а — положительное, отличное от 1 число. Число х называется логарифмом числа N по основанию а, если ax=N. Например, log2 16=4, так как 24=16, log^^-= — 8, так как 8=^-. Вообще, logear = r. Из определения логарифма следует, что, во-первых, записи x = \ogaN и ax=N выражают одну и ту же зависимость между числами a, x, W; во-вторых, число N должно быть положительным; в-третьих, если а>0, аф\, N>09 то aXo**N = N. (1) Тождество (1) является по существу математической записью определения логарифма; его называют также основным логарифмическим тождеством. Для любого положительного числа N и любого положительного, отличного от 1 числа а существует только одно действительное число х такое, что х=\oga N. Отсюда, в частности, следует, что если N\ = N2, то loga N] = loga N2, где N\ > О, N2 > 0. Напомним основные свойства логарифмов: Если N\*N2>0, то 1°. l0ge(tf|.tf,)-l0g. |ЛМ +l0ga \N2\. 2°. l0gaJ±=l Если, в частности, N\>0, N2>0, то \N\\=N\> |Л^2|=Л^2 и мы получаем: \oga(N\'N2)=\oga N\ + loga N2, Ioga4i=loge Л^| — loga N2. 3°. Если N>0, и-бЛ, то logaAP = iilogflW; если ц = 2/п(т=±1; ±2; ...), то loga ЛР = |1 logfl \N\. 4°. Если Л^>0, 6>0, Ьф\у то \OgaN=^dLt Это тождество принято называть формулой перехода к новому основанию. Из него, в частности, при N = b следует, что loga &=j-i—. 23
5°. Если N>0, \i£R9 то logfl W=logflli №. Рассмотрим примеры. 1—i- log725 Пример 1. Вычислим 49 Решение. Воспользовавшись тем, что 49 = 72 и что при возведении степени в степень показатели степеней перемножаются, получаем: 2-±log725 Здесь показатель степени можно преобразовать следующим образом: 2 —4-log7 25 = 2 — Iog75 = log749 — log7 5 = log7^-. 2--i-log725 log7^? Итак, 7 =7 . Но из тождества (1) следует, что 49 1 log7 -г- AQ 1 —г I°g7 25 7 5 =^-. Таким образом, 49 4 =9,8. Пример 2. Вычислим lg 25, если lg 2 = а. Решение. Имеем lg 25 = 2 lg 5. Выразим теперь число 5 через числа 10 и 2 (т. е. через данное основание и число, логарифм которого известен), пользуясь операциями умножения, деления и возведения в степень. Так как 5=-у-, то 2 lg 5 = 2 lg — =2 (lg 10 — -Ig2)=2(l-a). П )() ример 3. Вычислим log3 18, если log3 12=a. Решение. 1-й способ. Как и в предыдущем примере, упростим log3 18: log3 18 = log3(32.2)= Значит, нам нужно вычислить log3 2, зная, что log3 12=a. Выразим число 2 через числа 3 и 12 (данное основание и число, логарифм которого известен), пользуясь операциями умножения, деления и возведения в степень. =-д/—, Имеем: 2=-д/—, но тогда Таким образом, log3 18=^р 2-й способ. Имеем: log3 18 = 2 + log3 2. Введем обозначение Iog32=x, тогда log3 18 = 2+*. Далее, log3 12 = log3 (3-22)=l +2 log3 2=1 +2х. Но по условию log3 12 = а, следовательно, l+2x=a, откуда 24
Таким образом, log3 18 = 2 + *=^ Пример 4. Вычислим Iog49 16, если logu 28 = а. Решение. Использовав формулы 5° и 3°, получим: Введем обозначение log72=A:. Тогда Iog49 16 = 2*. Имеем далее: oQ=log7 28^log7(22«7)===2 log7 14 Iog7(2.7) log72 + log77 Так как по условию logl428==a, то задача сводится к решению 2*+1 а—\ уравнения 7\ = а, откуда находим х= . Таким образом, log4g 16 = 2x=-^z:-^. Пример 5. Вычислим logi2 60, если Iog630 = a, Iogi524 = fe. Решение. loa, fin - log2 m =log2 (4'3'5) = 812 log212 Iog2(4.3) log212 Iog2(4.3) 2+log23 Введем обозначения: Iog23=jc, Iog25=y. Тогда Iogi260 = 2+x " Далее имеем: Iog215 108,(3-5) x+ym Таким образом, задача сводится к решению следующей системы уравнений: Из этой системы находим: >_6 + 3 аб _2а —6 —2 + аб б1 • У~~ б1 Тогда logl260=2-S±|^. Упражнения Вычислите. !82. а) log8 log4 log2 16; б) lg lg д^ /1fi\l0g>253 /ft\log815 183а)Ш) ^«й) а) Зб^^+Ю1"182—3logt36; б) 81log53+27Iog936+3log79. 25
185. a) lg(2~log1/3V3-logV3y); б) lg(7 — log2 log3 186. a) Iog3 7.|og7 5«log54+1; 6) log3 2»log4 3«log5 4^1og65'log7 6»log8 7. 187. a) 2log35-5leg32; б) г^^-.2^^. Вычислите. 188. lg 1250, если lg 2 = 0,3010. 189. logioo 40, если Iog25 = a. 190. Iog35, если Iog62 = a, Iog65=6. 191. logs 16, если log|227 = a. 192. Iog3528, если logl47=a, Iogu5 = &. 193. log^V**» если logfl27 = 6, a>0, аф\. 194. logs 3,38, если Ig2 = a, lg 13 = 6. 195. Iog2360, если Iog320 = a, log3 15=6. 196. Iog27s60, если log|25 = a, log12ll=6. 197. logca6, если logafc=p, logb k = q, logcfc = r, где a, bt c, k — положительные числа, отличные от 1. 198. logo* -rr, если logo* a = k, где a, b — положительные числа, причем аЬФ 1. у 199. logafrt^, если logfl k = 2, log^ = 3, logc/j=6, где a, 6, с — положительные числа, отличные от 1. Докажите тождества. 200. 6log'Wog'*. б) (logo, < 204. logaAj 205. lgg*t =-o-(lga+lg^)» если a2 + b2 = 206. lg^±^=-L(lga + lg6), если a2+4* 207. logc+6 a+logc-b a=2 logf+6 a«logc-6 a, если a2 + 62 = c2, a>0, 6>0, c>0/ Упростите выражения. 208. (loge b + Iog6 a + 2) (logfl 6 — logfl6 b) \ogb a — 1. 209. l^M* -. (lOga Ь + lOg» a + 1) l0ga y / bg i op a logioofr \ о t~« / i t\ 210. \o iga >d Igft I . 211. 212. 26 lg a г \ j_2 g *
213. 214. VVl°g* a + logj 6 + 2 +~2 — log* a — loge b. 215. 2l0ga2 6((logeV^ + logftV^)2-6og«4/—+ l0g64/-|.)2), вСЛИ \ \ V а у о / / § 5. ДОКАЗАТЕЛЬСТВО НЕРАВЕНСТВ В настоящем параграфе речь идет о неравенствах, справедливость которых требуется доказать на заданном множестве значений переменных. Если такое множество не указано, то подразумевается, что эти переменные могут принимать любые действительные значения. 1. Доказательство неравенств с помощью определения. По определению считается, что a>ft, если разность а — Ь — положительное число. Поэтому для доказательства неравенства f (a, 6,..., k)>g (a, ft, ..., k) на заданном множестве значений a, ft, ..., k необходимо составить разность /(a, ft, ..., k)—g(a, ft, ..., k) и убедиться в том, что она положительна при заданных значениях a, ft, ..., k (аналогично применяется этот способ для доказательства неравенств вида f<g> f>8> f<S). Пример 1. Докажем, что если a^O, ft^O, то (неравенство Коши). (1) Доказательство. Составим разность ^ -\Jab и выясним ее знак. Имеем g±*--~y5ft = fl-2 V"*+6 =Ы~а~^ь? . Ясно, что выражение \ча~~Уь> неотрицательно при любых неотрицательных значениях а и ft. Значит, и разность ^^—-у/аЬ неотрицательна, а это означает, что a^b ^^lab. Отметим, что знак равенства имеет место лишь при a = ft. Пример 2. Докажем, что если ab>Of то Доказательство. Имеем: _(а-Ь)2 6 • а ) at ~ ah ' Так как aft>0, то ^""/^ >0, причем знак равенства имеет 27
место лишь при а = Ь. Итак, разность (-тН )~~2 неотрицательна, т. е. неравенство (2) доказано. Пример 3. Докажем, что (3) Доказательство. Рассмотрим разность Перегруппировав члены этой разности, получим: Последнее выражение положительно при любых значениях а, Ьу с. Неравенство (3) доказано. Пример 4. Докажем, что если a + 6 + c^O, то a3 + b3 + c3>3abc. (4) Доказательство. Рассмотрим разность а3 + Ь3 + с3 — — ЪаЬс, в которой сумму а3 + Ь3 дополним до куба суммы. Получим: а3 + Ь3 + с3 - Шс =а3 + За2& + 3aft2 + Ь3 + с3 - 3a2b - 2 f 3 Разложив сумму кубов (a + ft)3 + c3 на множители, получим Так как по условию a+fr+c^O, то полученное выражение неотрицательно. Отсюда следует истинность неравенства (4). Заметим, что знак равенства в неравенстве (4) имеет место в случае, когда а + &Н-с=0, а также когда а=6=с. 2. Синтетический метод доказательства неравенств. Суть этого метода заключается в том, что с помощью ряда преобразований доказываемое неравенство выводят из некоторых известных (опорных) неравенств. В качестве опорных могут использоваться, например, такие неравенства: а) а2>0; б) —--^-^аЬ, где а>0, &>0; в) f4~~>0, где а6>0; г) ax2 + bx + c>0, где а>0 и Ь2 — 4ас<0. Пример 5. Докажем, что если а^О, Ь^О, с^О, то 28
Доказательство. Возьмем в качестве опорного неравенство Коши: а±±.£+£ 2 "*" 2 2>V-2 Так как, в свою очередь, ^—^^/аЬ и £~— ^-yfcd, то Значит, 2 2 2 ^ У abed. 2 + 2 Таким образом, Проанализировав доказательство, приходим к выводу, что знак равенства в неравенстве (5) имеет место тогда и только тогда, когда a = &, c = rf и ~—=-:^-, т. е. когда a=6 = c=rf. Пример 6. Докажем, что (^-j^V >л!, где n£N, n>\. Доказательство. Возьмем в качестве опорных следующие неравенства Коши: Перемножив эти п неравенств, получим: Итак, (^)" (6) () Так как по условию я=тМ, то первое из опорных неравенств Коши может быть только строгим. Но тогда и после перемножения опорных неравенств полученное неравенство (6) должно быть строгим. Таким образом, (^р-) >п\9 что и требовалось доказать. 29
Пример 7. Докажем, что если а>0, 6>0, с>0, то Доказательство, У-й способ. Возьмем в качестве опорных следующие неравенства: Т+Т>* Т+Т>2' Т+Т>2 (эти неравенства становятся равенствами в случаях, когда соответственно а = &, а = с и 6 = с). Сложив их, получим -^Ч Ь—+ Далее выполним ряд несложных преобразований: a + b + c , q + fr + c , 6 ' 'a ' с Вынося теперь a+b + c за скобки, получим: Знак равенства имеет место лишь в случае, когда а = 6 = с. 2-й способ. Неравенство (7) можно доказать по определению. Имеем = i +iL+.£_+J_+1 +i_|_^+£_+1 _9 = oca cab e f^-+j5—2 W-+--2 ) + fJL+-£—2 ab ' ac ' fo Значит, неравенство (7) справедливо. Пример 8. Докажем, что если n£N, n> 1, то —4-—4-—4- +—<1 Доказательство. Имеем: Т=2Т<Т2'; Т==1Гз"<'2Т; Тб"==ТТ<зТ; '"' п2 П'П (п — \)п 30
Сложив эти (л—1) неравенств, получим: j , 1 , 1 , , JL^-J | ! | ! l i 4 "*" 9 "^ le"^^"^^2 "^ 12 ^ 2-3 ^3.4 "Г#""Г(я-1)л "~ Ь2 ^ 2-3 "*" 3-4 "r-"r (n-l)n V1 2 /^\2 3 >M 3. Доказательство неравенств методом от противного. Пример 9. Докажем, что если а>0, 6>0, с>0, rf>0, то (9) Доказательство. Нам надо доказать, что для любых неотрицательных значений а, &, с, d выполняется неравенство (9). Предположим противное, что существует набор неотрицательных значений а, 6, с, d, для которого неравенство (9) неверно, т. е. выполняется неравенство V(a + c)(& + rf)<V^+V^ • Так как обе части этого неравенства неотрицательны, то при возведении их в квадрат получим: откуда bc + ad<2-\/abcd , и далее bc+ad Но это противоречит неравенству Кош и. Значит, наше предположение неверно, а потому справедливо неравенство (9). Пример 10. Докажем, что если а^О, 6^0, с^О, то Доказательство. Предположим, что существует набор неотрицательных значений а, Ь, с, для которого неравенство (10) неверно, т. е. выполняется неравенство При возведении обеих его частей в квадрат получим: Г 3 ) >2 Г^"' и далее (а+6 + ^)2>3(а2 + 62+с2), 31
+ 2c2-2ab-2ac-2bc<0, (a-bf+(b-c)2+(a-c)2<0. Последнее неравенство не является верным, так как сумма квадратов не может быть отрицательным числом. Значит, неверно и наше предположение, а потому справедливо неравенство (10). Замечание. Пусть даны п неотрицательных чисел аи a2, ..., ая. Введем в рассмотрение следующие величины: Нп=-. j г- — среднее гармоническое, J I \ р _| L Gn=Vfli*«2»...«an — среднее геометрическое, ЛЛ= 2-г*...-г п —среднее арифметическое, п __ среднее квадратическое чисел аи п Между этими величинами существует такая зависимость: Некоторые частные случаи этой зависимости нами уже доказаны. Так, в примерах 1 и 5 доказаны неравенства G2<^2 и G4<^4; из неравенства, доказанного в примере 7, следует зависимость Я3<Л3; наконец, в примере 10 доказано неравенство /43<Q3. 4. Доказательство неравенств методом математической индукции. Пример 11. Докажем, что если n£Ny n>3, то 2Л>2п + 1. (11) До к а з а те л ьст в о. При п = 3 неравенство (И) верно: 23>2-3+1. Предположим, что неравенство (11) выполняется при п = Л(Л>3), т. е. предположим, что 2*>2*+1, и докажем, что тогда неравенство (11) выполняется и при л = Л+1, т. е. докажем, что 2*+1> 2* + 3. В самом деле, имеем: 2*+l=2-2*>2 (2k+ 1) = 4Л + 2 = =(2Л+3)+(2Л—1). Итак, 2Л+| >(2Л + 3)+(2Л-1). Но 2Л—1>0 при любом натуральном значении к. Следовательно, тем более 24+1>2А + 3. Согласно принципу математической индукции можно сделать вывод о том, что неравенство (11) справедливо при всех п^З Пример 12. Докажем, что если n£N, то 1+т+т+т+-+2^т>т- Доказательство. Выражение, содержащееся в левой 32
части неравенства (12), представляет собой сумму дробей, зна* менатели которых — натуральные числа от 1 до 2Л—1. При п=1 оно обращается в верное числовое неравенство \>—. Предположим, что неравенство (12) выполняется при n = fef т. е. С 1 I I I 1 I I I -^ k Докажем, что тогда неравенство (12) справедливо и при i==k+l, т. е. о ,,1,1, , 1 ^fe+l В самом деле, S*+l=(l +-L+.L+...+-,!_ где Pk=^ Выражение Р* представляет собой сумму 2* дробей, каждая из которых больше чем г*+т- Значит, Р*=—+-J—+ ■ Итак, S»>-*■-, Рк>\-. Но тогда i-=*±i. т. е. На основании принципа математической индукции заключаем, что неравенство (12) справедливо для любого n£N. Пример 13. Докажем, что при л>2 неравенство (13) выполняется для любых аи яг, ...f Q>n. Доказательство. Докажем справедливость неравенства (13) при п=2, т. е. Iai + a2|<|a,| + |a2|. (14) Предположим, что существует пара {аи Дг), Для которой неравенство (14) неверно, т. е. \а{+а2\ > |ai| + |a2|. Возведем обе части последнего неравенства в квадрат, получим \а{+а2\2> ^(Iai| + |a2|)2, т. е. ai+2ala2 + a2>a? + 2|ai| |a2| -f-ai (мы трижды воспользовались тем, что |jc|2=x2). Отсюда aia2>|aia2|, Что неверно, поскольку для любого х верно неравенство \х\^х. зз
Наше предположение неверно, неравенство (14) доказано. Предположим теперь, что неравенство (13) выполняется при n = k, т. е. |ai+a2 + -.. + a*Klail + |a2|+...+ |a*|. Докажем, что тогда неравенство (13) справедливо и при я = &+1, т. е. |ai+a2 + ... + a* + a*+iK |ai| + |a2| +...+ I a» I + la*+il- В самом деле, \а\+а2 + .'. + ак + ак+\\ = \{а1+а2 + ... + ак)+ + a*+, | < \ax +a2 + ... + a*| + \ak+{ \<\ax\ + \a2\ + ...+ \ak\ + + \ak+x\. На основании принципа математической индукции заключаем, что неравенство (13) справедливо при п2 Упражнения Докажите неравенства. 217. Если а>0, 6>0, то 218. Если а>0, 6>0, то 216. 219. Если a-f 6^0, а=^0, 6^0, то 220. Если a-f&X), то 221. 22 222. T+f Если а> — 1, то а3 2 22 224. 225. 226. Если а + &>0, то 227. 229. 230. 231. 232. 233. 234. 235. 236. 237. { Если . 228. а4 +/>\ где , то а2 Если а^О, Ь^О, с^О, то Если т, я, Л — натуральные числа, то mn-\-mk-\-nk^Zmnk. Если а^О, 6^0, с^О, то (a+6)(6 + c)(a-f-c)^8a6c. Если а>0, £>0, с>0, a + 6-fc = l, то (1 — а)(1 —6)(1 — c)^8abc. Если а^О, 6^0, с^О, то (a-|-l)(6-j-l)(c-T"Ci)(6-|-c)^ 1ба6с. Если а>0, 6>0, с>0, rf>0, To a4-f Ь4 + с4 + d4^4a6crf. >, то ^ 240. Iog23+log32>2. 241. а24-2 7 242. а24-а4-2 34
243. Если а>0, 6>0, с>0, то ~ + у+— 244. Если а>0, Ь>0, с>0, то аб (a + b)+bc(b + c)+ac (a + c)^6abc. 245. Если «>0. 6>0. то g^^^ 246. Если аи а2, ..., ал — неотрицательные числа, причем ai»a2«...«a« то(1+а,)(1+а2)...(1+ап)>2\ 247. Если п = 2, 3, 4, ..., то УТ+"\/2+-+л/л>л- 248. Если л = 2, 3, 4, ..., то л!>л 2 249. Если „=2. 3. 4 то ^^^4 250. Если а>0, 6>0, то -г + 251. Если а>0, 252. Если а>0, Ь>0, то 253. Если а>0, 6>0, то 254. 255. Если а>0, 6>0, то 256. Если а + 6>1, то а4 257. Если а>0, ^>0, с>0, то 258. Если аЬсфО, ab + ac + ЬсфО, то т т 4 oso a~\~b-\"C-\~d la2-\-b2 + 259. <^ _ 260. Если а>0, 6>0, c>0, d>0, то J_ 1 261. la + 6l>|a|-16|. 262*. Если jc> —1, л>2, то 263. Если л>5, то 2я > л2. 264. Если л > 10, то 2я > л3. 265. Если л>2, то ^<1+~+~ + ...+-^. 266. Если л>2, то 267. Если л>2, 268. Если л>2, * в упражнениях 262—268 предполагается, что n£N. 35 268. Если «>2. то l+i.+i.+ ...
} в. СРАВНЕНИЕ ЗНАЧЕНИЙ ЧИСЛОВЫХ ВЫРАЖЕНИЙ Если даны два действительных числа, то в большинстве случаев сразу ясно, какое из них больше, например 8>3, У&>-\/5. Нетрудно установить, что V5<Vl000. В самом деле, V^<2, a VlOO6>2, значит, V5<Vl6o6- Пусть теперь а=УЗ, ft =^/2. Оба числа принадлежат интервалу (1; 2), но какое из них больше, пока неясно. Для установления знака неравенства между этими числами проведем следующее рассуждение. Предположим, что a>ft, т. е. что У§>^2. Возведя обе части последнего неравенства в шестую степень, получим (V3)6>(V2)6, т. е. 9>8. Итак, a>&^KV3)6>(V2)6^9>8. Так как 9>8 — верное неравенство, то и равносильное ему неравенство а>Ь верно. Если бы мы предположили, что a<ft, то получили бы а<Ьо ^KV3)6<(V2)^9<8. Так как 9 < 8 — неверное неравенство, то и a<ft неверно, а поскольку афЬ, то остается только одна возможность: a>ft. Можно было сравнить числа а и ft, преобразовав их с помощью приведения радикалов к одинаковым показателям: Ясно, что a>ft. Пример 1. Сравним числа а и ft, если: 1) а= ; 2) a = log,3, ft = log3l,l; 3) a = log23, ft = log32; "7 4) a Решение. 1) Предположим, что а>Ь. Тогда, используя свойства числовых неравенств, последовательно получаем: V)(V l+Vl56>V22T, Итак, a>bo-J\56>32. Но неравенство -\fl56>32 ложно. Значит, и предположение а>Ь ложно. Ясно, что ложно и предположение а=6. Таким образом, остается одна возможность: a<ft. 2) Сравним числа а и ft с нулем. Имеем a = log i 3<log i 1 = =0, 6 = log3 l,l>log3 1=0. Итак, a<0, но 6>0. Значит, a<ft. 3) Сравнение чисел a и ft с нулем не позволяет сопоставить эти числа между собой, так как оба они положительны. Попробуем сравнить числа а и ft с 1. Имеем a = log2 3>log2 2=1, ft = = log32<log33=l. Итак, а>1, но ft<l. Значит, a>ft. 4) Оценим числа а и ft и снизу, и сверху. Выполним эту оценку с точностью до 1, а если такой точности окажется недостаточно, то увеличим ее при последующей оценке. 36
Имеем 2<л/5<3, 5<У30<6, 7<л/50<8, т. е. 14<л/5 + +V30+V50<17. Итак, 14<а<17. Далее 3<лА<><4, 4< <V20<5,- 7<V60<8, т. е. 14<Vl0+V20+V60<17. Итак, 14<ft<17. Полученные оценки чисел а и ft не позволяют сравнить их между собой. Оценим тогда числа а и ft с точностью до 0,1. Имеем: 2,2<л/5<2,3 3,1 <VlO<3,2 5,4<л/30<5,5 и 4,4<V20<4,5 7 <л/50<7,1 7.7<-v/60<7.8 14,6<а<14,9 15,2<&<15,5 Итак, а£ (14,6; 14,9), а 66(15,2; 15,5). Значит, a<ft. Пример 2. Расположим в порядке возрастания числа a=log2 3, & = log6 9, с = logs 17. Решение. Сравним сначала числа а и ft. Это можно сделать двумя способами. 1-й способ. Оценим числа а и ft с точностью до 1. Имеем: Iog22<log23<log24, т. е 1<а<2; Iog66<log6 9<log636, т. е. 1<6<2. Числа а и ft принадлежат интервалу (1; 2). Сравним каждое из них с серединой этого интервала, т. е. с числом —. Предположим, что Iog23>—, тогда последовательно будем иметь: з о о А так как а>у<*^9>8, то о>>-~ верное равенство. о Предположим, что ft> —, тогда J7 Последнее неравенство неверно, а так как &>—<^81>216, то и b >— неверно. Значит, Ь<—. о О Итак, Д>—, *<"2"» значит» а>Ь. 2-й способ. Рассмотрим разность а—6. Имеем: log26 37
Значит, а>Ь. Сравним теперь числа а и с. Выше мы установили, что о —<а<2. Число с заключено также в этих пределах. Действи- тельно, logs 17<logs 25=2 и logs 17>log5Vf25 = log5 5T =-|-. Сравним тогда числа а и с с серединой интервала (—; 2 Y т. е. с числом 4-- 4 Предположим, что а>—. Тогда, используя свойства неравенств, получим последовательно: 7 Последнее неравенство ложно. Ложно и неравенство а^ —. Таким образом, а<—. Предположим, что с> — . Тогда logs 17>~-<=^ 17>54 ^ 174>57. Последнее неравенство истинно. Значит, наше предположение, что с> — , верно. Итак, а< —, с>-г> и» значит,. а<с. Таким образом, Ь<а<с. Упражнения Сравните числа а и Ь. 269. а = \/5, 6 = V6. 270. a = 271. а=1 + -~, 6=2 (V2-I). 272. e-6. 6 =
275. a) a VV [Щ б) a = V5 —V2, 6=V6—V3- 276. a) a 6) a 277. a) a = log42, 6 = log0>0625 0,25; 6) a = log45, 6 = log , —. fe ^D 278. a) a = log426, 6 = log6 17; 6) a = log i -\/3, 6 = log l л/2- T 3 279. a) a = log23, & 6) a = log3 16, 6 = logI6729. 280. a) a=log6 14, 6 = log7 18; 6) a=log20 80, b = log80 640. 281. a) a = log4 12, 6 = log6 13; 6) a=logi5 5,5, ft = log2 10. 6) a=3(lg7-lg5), 6 = 283. a=r-!—+1—!—, 6=2. l0g2 П l0g4.5 Я 284. Расположите в порядке возрастания числа а, Ь, с, rf, если a = logs7, 6 = log83, c=^t rf = log, 5. 1 Глава II. РЕШЕНИЕ УРАВНЕНИЙ, СИСТЕМ УРАВНЕНИЙ И НЕРАВЕНСТВ § 7. РАВНОСИЛЬНОСТЬ УРАВНЕНИЙ Два уравнения называются равносильными, если множества их корней совпадают, в частности если оба уравнения не имеют корней. Например, равносильны уравнения lg x=0 и -у/х=\ (каждое из них имеет единственный корень х=1); равносильны уравнения 2* ('-О;—1 и -фс=х (каждое из них имеет два корня: 0 и 1). Если каждый корень уравнения f(x)=g(x) является в то же время корнем уравнения f\ (x)=g\ (x\ полученного с помощью некоторых преобразований из уравнения f(x)=g(x)y то уравнение f{ (x)=gi (x) называют следствием уравнения f(x)=g(x). Так, уравнение (х— 1)(х—2)=0 является следствием уравнения х—1=0 (а уравнение *—1=0 не является следствием уравнения (х— 1)(лг—2)=0). Если каждое из двух уравнений является следствием другого из них, то такие уравнения являются равносильными. Несколько уравнений с одной переменной образуют совокуп- 39
ность уравнений, если ставится задача об отыскании всех таких значений переменной, каждое из которых удовлетворяет по крайней мере одному из заданных уравнений. Уравнения, образующие совокупность, записываются в столбик с помощью квадратной скобки, например: Г 2х+1 L 4jc —3 = jc2 или с помощью знака «;», например: Решением совокупности уравнений является объединение множеств корней уравнений, составляющих данную совокупность. Если при выполнении преобразований уравнение f(x) = g(x) свелось к уравнению f{(x)=g\(x) (или к совокупности уравнений), некоторые корни которого (которой) не являются корнями уравнения f(x) = g(x\ то эти корни уравнения f\(x)=g\(x) называют посторонними корнями уравнения f{x) = g(x). Например, возведя в квадрат обе части уравнения л[х=—ху получим уравнение х = х2, имеющее два корня: 0 и 1. Значение х = 0 удовлетворяет уравнению -фс=—лг, тогда как значение jc=1 не удовлетворяет уравнению -фс=—х, т. е. является для него посторонним корнем. Если при выполнении преобразований уравнение f (x) — g(x) свелось к уравнению f\ (x)=g\ (x) (или к совокупности уравнений), причем некоторые корни уравнения f(x) = g(x) не являются корнями уравнения f\(x) = g\(x)9 то в таких случаях говорят о потере корней. Например, уравнение (х— 1)2=х— 1 имеет два корня: jci = 1 и х2 = 2. Если обе части этого уравнения разделить на <р(х)=х— 1, то получим уравнение х—1 = 1, которое имеет только один корень: дг=2. Таким образом, при делении обеих частей уравнения на ф(х)=х—1 произошла потеря одного корня. При решении уравнений обычно выполняются различные преобразования, в результате которых заданное уравнение сводится к более простому уравнению (или совокупности уравнений). При этом важно знать, какие из преобразований сводят данное уравнение к равносильному, какие приводят к уравнению-следствию, а какие — к потере корней. Теорема 1. Если к обеим частям уравнения f (х) = g (x) прибавить одно и то же выражение ф (jc), которое имеет смысл при всех х из области определения уравнения f{x) = g(x), то получится уравнение f (x) + q> (*)=£(*) +Ф (х\ равносильное данному. Например, если к обеим частям уравнения Зх2 + 2х —5 = = 7jc—1 прибавить выражение q>(*)= —-7jc+ 1, получится уравнение 3*2 + 2х— 5 + ( — 7х+ 1) = 7л: — 1 +( — 7х+ 1), равносильное исходному уравнению, так как выражение ф(х)=— 7х+1 имеет смысл при всех значениях х из области определения исходного уравнения. 40
Если же к обеим частям уравнения дс2=1 прибавить выражение q>(x)=V*> то получится уравнение х2+-\[х=\-\-^/х, которое неравносильно исходному уравнению, так как выражение ф(х)=^/* имеет смысл не при всех х из области определения уравнения, а только при значениях х>0. Прибавив к обеим частям уравнения х2=1 выражение q>(x)=-yjx, мы сузили область определения уравнения, что могло привести к потере решений. В данном случае х= — 1 является корнем уравнения х2=1, но не является корнем уравнения x2+V*=l+л/*- Следует понимать, что в теореме 1 речь идет только об одном преобразовании — прибавлении к обеим частям уравнения одного и того же выражения: Последующее же приведение подобных членов (если оно возможно) — это новое преобразование уравнения. Приведение подобных членов может привести к уравнению, которое неравносильно исходному. Так, если к обеим частям уравнения jt2+2x+lg*=lg*— 1 прибавить ф(*)= = — lgjc, то получится уравнение x2+2jt+lg х—lg x=lg x—l — — lg jc, равносильное исходному, ибо выражение <р (х) = — lg x имеет смысл при всех х из области определения исходного уравнения. Однако если в последнем уравнении выполнить приведение подобных членов, то получится уравнение jc2 + 2jc= —1, неравносильное исходному. Уничтожение в обеих частях заданного уравнения выражения lg x привело к расширению области определения уравнения, в результате чего могли появиться посторонние корни. В нашем случае это и произошло: значение х= — 1 является корнем уравнения *2 + 2л:= — 1, но не являетея корнем уравнения jr + 2jt+lg x=lg x—l. Следствие. Уравнения f(x)-\-q>(x) = g(x) и f(x)=g(x)— — ф (х) равносильны. Теорема 2. Если обе части уравнения f(x)=g(x) умножить или разделить на одно и то же выражение <р(х), которое имеет смысл при всех значениях х из области определения данного уравнения и нигде в этой области определения не обращается в нуль, то получится уравнение f (*)-ф(*)-« w-ф w («ли Jg- равносильное данному. Так, если обе части уравнения х =2 умножить на выражение ф(х)=х, то получится уравнение х2——=2лг, равносильное исходному, так как выражение <р(х)=х имеет смысл при всех значениях х из области определения исходного уравнения (хФО) и нигде в этой области не обращается в нуль. Если обе части уравнения х —2 = 0 умножить на ф(х)=х + 3, то получим уравнение (jc—2)(х+3)=0, неравносильное данному, так как при х=— 3, принадлежащем области определения исход- 41
ного уравнения, выражение ф(х)=л: + 3 обращается в нуль, хотя оно имеет смысл при всех х из области определения уравнения х — 2=0. Как нетрудно видеть, в данном случае умножение обеих частей уравнения на выражение ф(х) = *+3 привело к появлению постороннего корня х=— 3. Еще пример. Если обе части уравнения х — 4=x(-\fx — 2) разделить на выражение (p(x)=-yfx — 2, то получится уравнение *~~4 =х™х— ) ^ неравносильное исходному уравнению, так как, ■ух —2 -\pc-'2 хотя выражение <p(x)=yx — 2 имеет смысл при всех значениях х из области определения исходного уравнения, оно обращается в нуль при значении х=4, которое входит в область определения исходного уравнения. Если же взять уравнение х—4=х(-фс-\-2) и разделить обе его части на ф(#)=л/* + 2, то получим уравнение -фс — 2=х, равносильное данному, ибо выражение ф(х)=-7* + 2 имеет смысл всюду в области определения данного уравнения (х^О) и нигде в этой области определения не обращается в нуль. Обращаем внимание читателя на то, что в теореме 2 речь идет только об одном преобразовании — умножении (или делении) обеих частей уравнения на одно и то же выражение. Последующее же сокращение дроби (если оно возможно) — это новое преобразование уравнения. Так, умножив обе части уравнения на выражение ф(л:)=2*, мы выполним первое преобразование, которое приводит к уравнению 2х^1) + 2х2=6х. Последующее же сокращение дроби 2х(*+1) на 2х есть новое преобразование: оно приводит к уравнению х+1+2*2 = 6х. Это сокращение может привести и к уравнению, которое неравносильно заданному. Аналогично если обе части уравнения *2-"5*+6=0 умножить на ф(х)=х—2, то получится уравнение (*2—5*+6Н*"~2) = 0, равносильное данному, так как выражение ф(х)=лс —2 имеет смысл при всех значениях х из области определения данного уравнения (хФ2) и нигде в этой области определения не обращается в нуль. Однако если в левой части полученного уравнения выполнить сокращение на х — 2, то получится уравнение х2 — 5х+6 = 0, неравносильное данному: значение х=2 является корнем последнего уравнения, но не удовлетворяет заданному уравнению, т. е. является для заданного уравнения посторонним корнем. Дело в том, что при сокращении дроби произошло расширение области определения, а мы уже отмечали, что это может привести к появлению посторонних корней. Следствие. Если обе части уравнения умножить (или разделить) на одно и то же отличное от нуля число, то получится уравнение, равносильное данному. 42
Например, умножив обе части уравнения —5г~=^Ф- на 6, получим уравнение 3jc+3 = 2jc+6, равносильное данному. Тео ре м а 3. Ерли обе части уравнения f(x)=g(x), где f (x)*g (х)^0 при всех значениях х из области определения уравнения, возвести в одну и ту же натуральную степень п, то получится уравнение (/ (x))n = (g(x))n, равносильное данному. Если, например, обе части уравнения 2л:— 1 =у*— 1 возвести в квадрат, то получим уравнение (2х—1)2=(Улс—1) , равносильное данному, так как при всех х из области определения данного уравнения (*>1) обе части уравнения неотрицательны. Если же возвести в квадрат обе части уравнения дс—6=V*> то получим уравнение (л:—б)2=(V^)2» ° котором нельзя утверждать, что оно равносильно заданному, так как при некоторых значениях х из области определения заданного уравнения (х^О) левая часть уравнения принимает отрицательные значения (например, при х = 2 имеем х — 6=—4<0), а правая часть всегда неотрицательна. Действительно, уравнение (jc — б)2 == (V^)2 преобразуется к виду г2 — 13х+36 = 0, откуда *i=9, х2 = 4. Но х=4— посторонний корень для исходного уравнения. Отметим, что в теореме 3 говорится только об одном преобразовании — возведении обеих частей уравнения в одну и ту же натуральную степень. Последующее же освобождение от знака корня (если оно возможно) — это новое преобразование уравнения. Освобождение от знака корня может привести к расширению области определения уравнения, а потому и к уравнению, неравносильному заданному. Замечания. 1. Теорема 3 выполняется только для уравнений над полем действительных чисел. 2. Если п — нечетное число, то в формулировке теоремы 3 можно опустить условие: f (x)-g(x)^0 при всех х из области определения уравнения. При решении уравнений приходится также применять преобразования, не оговоренные теоремами 1, 2 и 3, т. е. такие преобразования, которые могут привести к появлению посторонних корней или даже к потере корней. Причиной появления посторонних корней или потери корней могут быть преобразования, выполняемые с помощью формул, изменяющих области определения уравнения. Таковы, например, формулы: loge (xy)=log. x+logo y, xl0*' »—y, «tff tgxctgx=l, sinjc= ^-, tg(jc+j _ 43
Во всех случаях, когда преобразование, выполненное в процессе решения уравнения, приводит к уравнению, являющемуся следствием заданного уравнения, но не установлена равносильность полученного уравнения и заданного, необходима проверка найденных корней. Она является неотъемлемой частью решения уравнения. Решение в этом случае не может считаться законченным, если не сделана проверка. Как же проверяются найденные корни? В качестве основных можно указать следующие два способа проверки: 1) путем подстановки каждого из найденных корней в заданное уравнение; 2) путем доказательства равносильности выполняемых преобразований уравнения на всех этапах решения. В некоторых случаях оказывается целесообразней делать проверку по-другому (например, с помощью области определения заданного уравнения). Примеры выполнения такой проверки читатель найдет ниже. Пример 1. Решим уравнение У2х + 5=8 —«Ух — 1. Решение. Возведем обе части уравнения в квадрат. Получим 2а: + 5=(8—У*— 1 )2, и далее 16У*— 1 =58 — х. Снова выполним возведение в квадрат: 256 (х — 1)=(58—х)2, и далее х2 — 372х + + 3620 = 0, откуда xi = 10, х2 = 362. Анализируя выполненные преобразования, можно утверждать только то, что каждое новое уравнение является следствием предыдущего. Но это значит, что в процессе решения могли появиться посторонние корни, а потому найденные корни необходимо проверить. Проверка. В данном случае найденные корни нетрудно прове- рить подстановкой их в заданное уравнение. Проверим Х| == 10. Имеем У2х! + 5=У2.10 + 5 = 5 и 8—УхУ— 1 =8—УЮ-1 =5. Таким образом, при х=10 обе части заданного уравнения принимают одинаковые числовые значения. Значит, х= 10 — корень данного уравнения. Проверим х2 = 362. Имеем: У2х2 + у^ИТ—У362— =У2-362 + 5 = 27, а — И. При х = 362 левая и правая части исходного уравнения принимают различные числовые значения. Значит, х=362— посторонний корень. Итак, наше уравнение имеет единственный корень: х=10. Пример 2. Решим уравнение У3х-+-1 = 3+Ух— 1. Решение. Возведем обе части уравнения в квадрат: Зх+1 =(3+Ух=1)2, и далее 6 Ух— 1 = 2х—7. Еще раз выполним возведение в квадрат: 36 (х — 1)=(2х—7)2, и далее 4х2 —64х+85 = 0, откуда находим: _16—3VJ9 2 2~' 44
Проверка. Ясно, что проверка найденных корней их подстановкой в исходное уравнение сопряжена со значительными вычислительными трудностями. Поэтому выберем другой способ проверки. Область определения заданного уравнения такова: х^1. В этой области первое возведение в квадрат является равносильным преобразованием уравнения. Второе возведение в квадрат мы применили к уравнению 6У* — 1 = 2х — 7. Этому уравнению могут удовлетворять только такие значения х, которые удовлетворяют неравенству 2х—7^0, т. е. х^3,5. Нетрудно установить, что неравенство "16~^2 ^^,5 истинно, а неравенство 1 ~~п ^3,5 ложно. Значит, х2 =—=~-^ посторонний корень, а х\= ^ 9 — единственный корень заданного уравнения. Пример 3. Решим уравнение Ig(^2 —7л: + 3)—lg(2jc+l)=lg(A:2 + 7jc —3)—lg(2jc —1). Решение. Преобразуем уравнение к виду j 8 2х+\ 2 откуда находим: *i=0, *2=-=-. о Проверка. Поскольку каждое уравнение, полученное на том или ином этапе решения, является только следствием предыдущего, то потери корней произойти не могло, посторонние же корни могли появиться, причем только за счет расширения области определения заданного уравнения. Поэтому проверку можно осуществить в данном случае с помощью системы неравенств, обусловливающих область определения заданного уравнения, т. е. с помощью следующей системы неравенств: ( 2х+1>0, х2 + 7дг —3>0, 2х-1>0. Ни jci = 0, ни *2=— не удовлетворяют последнему неравенству 5 системы, а, значит, являются посторонними корнями. Итак, уравнение не имеет корней. Упражнения Докажите, что следующие уравнения не имеют корней. 285. ^ГГ^^ 286. VF^V+V 45
287. log2 (x2 — 288. 2log"(jf + 2 289. Ух — 1+л/2 — x = x —5. (x3- l)+log4 (1 — x4) = Vx. + 3) 291. )g(10 —х2) = 292. 2log'(Jf-3) = 2 293. ==\. 294. 2. j •3 Равносильны ли следующие пары уравнений. 295. х2 + 1=л/х и 296. х2-1 297. х3 + х = 0 и —— = 0. 298. 299. РавнФСильны ли следующие уравнения и совокупности уравнений (ответ поясните) . 305. (х-4)(х+3)=0 и х-4=0; х+3=0. =0 и ж—4=0; хА—г-т =0. 306. (х-4 307. (х-4 308. Ух^ 309. У2^ 310. (х- 311. (2- 312. ^-= х— 0 и х- О и V^^^O; VF+3==0. 0 и V^^^O; л/*ТЗ=0. -Jt)=0 и х-3=0; lg(2-x)=0. -3)=0 и 2—jc = 0; lg(x-3)=0. и д»-а*- ~э-1=0. Решите уравнения и сделайте проверку. Если имеются посторонние корни, то выясните причину их появления. 314. 2-х ^ 2 2х—д =0. 31в.^+ 25 1 13 2лг—1 п 4х2-1 27 1-2х* 46
=I4- 320. -jn^x — УП)^ 321. -v^T3+V§^=:2=7. 322. ^fix~=2 = 2 ->/*+2"-2. 323. -V^TT+V*II3=2 V*- 324- !g (54—Jt3)=3 lg x. 325. lg(*-2)+lg(*-3)=l-lg5. 326. V^V^TF Q07 lg(to5) _ 1 lg (2s-5) 327' * 328 § 8. РАЦИОНАЛЬНЫЕ УРАВНЕНИЯ В настоящем параграфе рассматриваются уравнения вида Р (х)=0, Р (х\ -г-^- =0, где Р(х) и Q(x) — многочлены, а также уравнения вида f(x) = g{x), где f (х) и g(x) — рациональные выражения*. Напомним некоторые сведения из алгебры. 1. Если х=а — корень многочлена Р (х), то Р(х) делится без остатка на двучлен х—а. 2. Пусть все коэффициенты многочлена Р (х) — целые числа, причем старший коэффициент равен 1. Если такой многочлен имеет своим корнем рациональное число, то это число целое. 3. Пусть все коэффициенты многочлена Р (х)=аохп + а\Хп~1 + + ... + а„ — целые числа. Если корнем многочлена является целое число Ь, то b — делитель свободного члена ап (необходимое условие существования целочисленного корня). Отметим, что при решении целых рациональных уравнений преобразования, выполняемые в процессе решения, приводят только к уравнениям, равносильным заданному. Поэтому, естественно, найденные корни не проверяют и упоминать об этом в каждом конкретном случае не следует. При решении же дробно-рациональных уравнений выполняется умножение обеих частей уравнения на одно и то же выражение Q (х), что может привести к появлению посторонних корней. Поэтому при решении дробно-рациональных уравнений проверка необходима. При решении рациональных (и других) уравнений основными являются следующие методы: 1) разложение на множители; 2) введение новых (вспомогательных) переменных. Метод разложения на множители заключается в следующем: если * Здесь и всюду в дальнейшем, когда речь идет о решении уравнении, систем уравнении, неравенств, мы будем находить только действительные решения, не оговаривая этого особо каждый раз. 47
то всякое решение уравнения /W=o (1) является решением совокупности уравнений Ы*)=0;Ых) = 0; -.; f«W=O. Обратное утверждение, вообще говоря, неверно: не всякое решение совокупности уравнений (2) является решением уравнения (1). Так, например, решение уравнения *lzi3f±l/iL±2+2)=0 (3) сводится к решению совокупности уравнений 0. (4) X Решениями совокупности (4) являются значения: jci = 1, *2 = 2, ж3 = 0, х<=—р Но при х=\ не определено выражение 2_ , а при лг=О не определено выражение -L—. Таким образом, значения х= 1, л: = 0 не являются корнями уравнения (3). Вообще при решении уравнения (1) методом разложения на множители из найденных корней уравнений совокупности (2) корнями уравнения (1) являются те и только те значения х, которые принадлежат области определения уравнения (1). Пример 1. Решим уравнение х3 + 2*2 + З Решение. Разложим левую часть уравнения на множители. Имеем х2(х + 2) + 3(л: + 2) = 0, и далее (х + 2) {х2 + 3) = 0. Последнее уравнение равносильно совокупности уравнений Из первого уравнения получаем х\= — 2. Второе уравнение не имеет действительных корней. Ответ: *= —2. Пример 2. Решим уравнение xA + xz + 3x2 + 2x + 2 = 0. Решение. Попытки выполнить в левой части уравнения группировку аналогично тому, как это было сделано в примере 1, оказываются неудачными. Поэтому попытаемся какой-нибудь член уравнения представить в виде суммы нескольких слагаемых таким образом, чтобы группировка, позволяющая получить «удачное» разложение на множители, была осуществима. Положим Зх2 = х2 + 2х2. Тогда получим (*4 + x3 + x2) + (2x2 + 2jt+2)==0, и далее х2 (х2 + х+ 1) + 2 {х2 + х+ 1)=0, 48
Остается решить совокупность уравнений Ни одно из них действительных корней не имеет. Значит, заданное уравнение не имеет действительных корней. Пример 3. Решим уравнение хъ + 4х2 — 24=0. Решение. Можно пытаться решить это уравнение, как в предыдущих примерах 1 и 2, разложением на множители (представив 4jt в виде суммы — 2х2 + 6х2, получить уравнение х — 2*2-+- + 6х2 —24 = 0, а затем х2 (х — 2) + 6(х —2)(х + 2)=0 и т. д.). Мы покажем на этом примере так называемый метод подбора, с помощью которого отыскивается целый корень уравнения. Используя необходимое условие существования целочисленного корня, выпишем делители свободного члена: <х=±1; ±2; ±3; ±4; ±6; ±8; ±12; ±24. Теперь начинаем пробы. Подставим вместо х в данное уравнение а=1. Получаем l3-f-4-12 — 24=^0. Таким образом, х= 1 не является корнем уравнения. Продолжаем пробы: а= — 1, (—l)3-h4(—l)2 —24=^=0; а = 2, 23 + 4-2J — 24=0. Итак, jci=2 — корень уравнения. Воспользуемся тем, что многочлен х3 + 4дс2 —24 делится без остатка на х—2. Выполним это деление: — 24 х-2 "б*2-12* 12*—24 ~12лг —24 0 Таким образом, *3+4х2 — 24=(х — 2)(л:2 + 6лс+12), а значит, исходное уравнение принимает вид: Это уравнение равносильно совокупности уравнений (решение первого из которых уже найдено) х — 2=0; х2 + 6х+ 12 = 0. Второе уравнение совокупности не имеет корней. Заданное уравнение имеет один действительный корень *i=2. Замечание. Покажем схематически другой способ разложения многочлена на множители с учетом найденного целого корня без деления углом: Р (2)=23+4.22—24 р (х)-Р(2) =(jc3-23)+4 (*2-22) Я (ж) — Р(2)=(*—2)(a:2 + 2a: + 4)+4(jc-2)(jc + 2)=(^ Но Р(2)=0, значит, Р (х)=(х-2)(х2+Ьх+\2). 49
Пример 4. Решим уравнение 21х3+х2 — 5х—-1=0. Решение. Уравнения, левая часть которых представляет собой, многочлен с целыми коэффициентами и свободным членом, равным 1 или —1, легко преобразуются в приведенные уравнения с помощью почленного деления на х в старшей степени (нетрудно видеть, что такое деление не приводит к потере корней, так как *=0 не является корнем уравнения, свободный член которого отличен от 0) и последующей заменой — на у. В нашем примере получаем: Полагая —=у, приходим к уравнению 21 +у — 5у2 — у3=0, и далее у3-\-Бу2—у — 21=0. Найдя методом проб, как в примере 3, целый корень уравнения i/i = — 3 и разделив многочлен у* + 5у2 — £/ — 21 на у + 3, получим квадратный трехчлен у2 + 2у — 7 с корнями У2,з = = — 1 ±2 V2. Так как х=—, то хх = —~, х2,з = 1±^ . У 6 7 Пример 5. Решим уравнение 4х3 — 1 Ojc2 + 1 4jc — 5 = 0. Решение. Здесь мы применим еще один способ преобразования неприведенного уравнения в приведенное (цель такого преобразования ясна: приведенное уравнение имеет своими рациональными корнями только целые числа, а способ отыскания целочисленных корней у нас имеется). Умножим обе части заданного уравнения на такое число, чтобы коэффициент при х3 стал кубом некоторого целого числа. В нашем случае таким множителем может служить число 2. Умножим обе части уравнения на 2: Положим теперь у = 2х9 тогда уравнение примет вид: У3 — 5i/2+14i/-10 = 0. Как и в предыдущих примерах, находим корни приведенного уравнения. Здесь только один корень у\ = 1 (проверьте!). Так как х=-|-, то Xi=— единственный корень заданного уравнения. Пример 6. Решим уравнение х6 — Решение. Применим метод введения новой переменной. Положим у=х3. Тогда заданное уравнение примет вид: у2 — 9у+8 = 0, откуда находим: j/i = l, У2 = 8. Теперь задача сводится к решению совокупности уравнений: *3=1; г* = 8, откуда jci = 1 и х2 = 2. Пример 7. Решим уравнение Решение. Положим у = х2 + х + 4. Тогда заданное уравнение примет вид: 50
Решим это уравнение как квадратное относительно у: Итак, t/! = —3jc, t/2= —5x. Таким образом, задача сводится к решению следующей совокупности уравнений: jc2+x + 4=— Зх; jt2+x+4=— 5л:. Из этой совокупности находим: х1(2= — 2, хз,4= — 3±л/5- Пример 8. Решим уравнение 2 = 0. (5) Решение. Заданное уравнение имеет интересную особенность: отношение его первого коэффициента к свободному члену и квадрат отношения второго коэффициента к предпоследнему равны между собой. Уравнения с такой особенностью называются возвратными. На этом примере мы покажем способ решения возвратного уравнения четвертой степени. Разделим обе части уравнения на х2 (это не приведет к потере корня, так как значение х=0 не является корнем заданного уравнения). Получим: и далее 3(x2+i-)-2(x+f)+4 = 0. (6) Положим дс+— =у, тогда (х+—) =у2, а потому х2+4-=У2—4. X \ X / X Заменив в уравнении (6) х-\ на у, a jc2^—г на у*— 4, получим: 3(у2 — 4) —2у+4=0, откуда находим t/i=2, у2=—4"- Теперь задача свелась к решению совокупности уравнений: Эти уравнения не имеют действительных корней, значит, и заданное уравнение не имеет корней. Пример 9. Решим уравнение лс2+/ * .2 =27. Решение. Левая часть уравнения представляет собой сумму квадратов. Это наталкивает на мысль добавить к обеим частям уравнения такое выражение, чтобы левая часть обратилась в полный квадрат суммы. Итак, прибавив к обеим частям уравнения выражение -2*-^, получим: 51
.-*.) =27-6-4 Преобразовав выражение в скобках, получим X Положим теперь у=—тт • Тогда уравнение примет вид у + 6у — x-j-o — 27 = 0, откуда yi = — 9, уг = 3. Задача свелась к решению совокупности уравнений X2 X2 т=~~9; 7+з =3* Первое уравнение не имеет корней, из второго находим *i,2 = =-s"±^r-. Оба найденные значения удовлетворяют условию х+3=^=0, а потому являются корнями исходного уравнения. Упражнения Решите уравнения. 331. дг4 — 1 = 0. 332. х6—64=0. 333. jc3+jc—2=0. 334. ^~4л:2 +jc+6=0. 335. 336. 337. 2*4 — 21*3 + 74*2—105*+50=0. 338. *4 + 5*3 + 4*2 — 24jc — 24 = 0. QQO у 11М £ у' I А у** ^^^ у* —Х«, id. у ^^ ^. !■ С\ 340. jc5 + 4jc4 —6х3 —24jc2 — 27jc —108=0. 341. -' 0 +t) - "T5 • 345. 2jc4-jc3H-5^—jc+3=0. 346. 2jc4 — 4jc3H-13jc2—6x+15 = 347. ^ + 4х2-2л:-8=0. 348. x3-ЗлгЧ4л:-12=0. 349. jc3 + 3jc + 4=0. 350. x8—15*4 —16=0. 351. (^2- 352- Ч1 + ftt =2Д 353- 52
1 ,2 355. p_3JC+3 "*" *2-3x+4 356. х3 —^"J^l.Jt =2- 357. х(*-1)(*-2)(ж-3)-15. 358. (*-l)*(*+l)(*+2)-24. 359. (x+1)(*+2)(jc+3)(*+4)=3. 360. (8* + 7)2(4*+3)(jc+1)=4,5. 361. (*-4,5)4+(*-5,5)4 = l. 362. (*+3)4 + (*+5)4=16. 363. 10jc3-3x2~2jc+1=0. 364. 4лг3 —3jc—1 =0. 365. 38^ + 7^ — 8^—1=0. 366. 4*3+ 6X2+ 4*+1=0. 367. 16*3 —28x2+4*+3=0. 368. 100*3 —120*2+47*—6=0. 369. 6jc3-13jc2 + 9jc-2=0. 370. 4*3 + б*2+ 5*+69=0. 371. 3x3 — 2х*+х —10=0. 372. 32*3-24x2- 12jc-77=0. 373. 4^ + 2^—8^+3=0. 374. 2(jc2+jt) -7(jc+i.) +9=0. 375. 376. x2+jc+jc-4^2=4. 377. ^ 378. jc4~2jc3-jc2-2jc+1=0. 379. 380. 381. 16^ + 8^ — 7^ + 2^+1=0. 382. *4-8* + 63 = 0. 383. — 6x—9 § 9. УРАВНЕНИЯ, СОДЕРЖАЩИЕ ПЕРЕМЕННУЮ ПОД ЗНАКОМ МОДУЛЯ При решении уравнений, содержащих переменную под знаком модуля, применяются чаще всего следующие методы: 1) раскрытие модуля по определению; 2) возведение обеих частей уравнения в квадрат; 3) метод разбиения на промежутки. Пример 1. Решим уравнение |2х-3|=5. (1) Решение. 1-й способ. Так как по определению / f(*)f если /(*)><>, I/WI = (-/(*), если /(х)<0, то уравнение (1) равносильно следующей совокупности двух смешанных систем: 53
2л: — 3>0, r 2jc —3<C0, 2jc-3 = 5; [ -(2x-3) = 5. Из первой системы этой совокупности находим х\ = 4, а из второй JC2 = — 1. 2-й способ. Так как обе части уравнения (1) — выражения одинаковых знаков, то в соответствии с теоремой 3 (см. с. 43) уравнение (1) равносильно следующему уравнению: (|2х-3|)2 = 52. Принимая во внимание, что (1/(*)1)2==(/(*))2» получим уравнение (2л; — 3)2 = 25, равносильное уравнению (1). Решая последнее уравнение, находим корни уравнения (1): лг,=4, х2= — 1. Пример 2. Решим уравнение |2*-3|=х+1. (2) Решение. Это уравнение, как и предыдущее, может быть решено двумя способами. При решении первым способом будет получена равносильная уравнению (2) следующая совокупность смешанных систем: 2л; — 3>0, (2л; — 3<0, 2jc-3 = jc+1; \—(2* —3) = х+1, о откуда находим Xi =4, л;2=—. о Чтобы для решения уравнения (2) можно было воспользоваться вторым способом, необходимо предварительно убедиться, что обе части уравнения (2) — выражения одинаковых знаков (см. теорему 3, с. 43). Так как |2л; — 3| ^0, то ясно, что, когда л:+1 <0, уравнение (2) решений не имеет. Если же Х+ 1 ^0, то в этом случае обе части уравнения (2) — выражения неотрицательные, т. е. уравнение (2) равносильно следующей смешанной системе: откуда *,=4, *2=-|-. Пример 3. Решим уравнение |2х-3|=х-2. (3) Решение. Применим для решения второй способ. Ясно, что ес- ли х — 2<0, то уравнение (3) не имеет корней, так как |2л; — 3| ^0. В случае же, когда х-— 2^0, обе части уравнения (3) неотрицательны и поэтому, возводя обе эти части в квадрат и освобождаясь таким образом от знака модуля, получим равносильную уравнению (3) следующую смешанную систему: 54
{(2x-3)2=(*-2)2, или Последняя система решений не имеет, а следовательно, и уравнение (3) корней не имеет. Пример 4. Решим уравнение (4) Решение. Нетрудно убедиться, что способ решения возведением в квадрат (второй способ) здесь наиболее целесообразен. Действительно, при решении этим способом мы получим одно уравнение, равносильное уравнению (4): (2х-3)2=(х + 7)2, откуда л = 10, *2=—f-. Пример 5. Решим уравнение =&. (5) Решение. В данном случае более предпочтительным является метод разбиения на промежутки (третий способ). Нанесем на числовую прямую значение х, при котором 3—* = 0, и значение х, при котором дг + 2 = 0. Числовая прямая при этом разобьется на промежутки (— оо; —2), [—2; 3], (3; оо). Решим уравнение (5) на каждом из этих промежутков, т. е. решим равносильную уравнению (5) совокупность смешанных систем: г -оо<л;< — 2, f — 2<х<3, f 3<*<oo, \3 — x+x + 2 = 5; \3 — x—x —2 = 5; \ —3+x—x— 2 = 5 или rx<— 2, r — 2<x<3, r*>3, (5 = 5; \*=-2; ( -5 = 5. Решением первой системы этой совокупности является луч (— оо; —2), из второй системы находим, что х= —2, а третья система решений не имеет. Объединяя решения этих трех систем, получаем решение уравнения (5): (— оо; —2]. Пример 6. Решим уравнение |х-2|-М*-1|=х-3. (6) Решение. Уравнение (6) очень похоже на уравнение, решенное в предыдущем примере, т. е. на первый взгляд может показаться, что его целесообразней всего решать методом разбиения на промежутки. Однако из уравнения (6) ясно, что * — 3>0, т. е. дс>3, а тогда и х—2>0, и х—1>0. Таким образом, уравнение (6) равно- 55
сильно смешанной системе г л: — 2+jc— 1 =jc—3, которая равно- \*>3, сильна системе гх = 0, не имеющей решений. Итак, уравнение (6) 3 \ корней не имеет. Пример 7. Решим уравнение |х-|2х+3||=Зх-1. (7) Решение. Раскроем сначала, пользуясь определением, т. е. применяя первый способ, «внутренний» модуль. Получим равносильную уравнению (7) совокупность смешанных систем: |U-(2jc+3)|=3x-1; или 3 2 ' (8) _лг_3|=3х—1; ^ |3х+3|=3х—1. Решим сначала первую систему совокупности (8). Раскрывая модуль по определению, получим следующую совокупность смешанных систем, равносильную первой системе совокупности (8): 2 ' 15= _(_л;_з)=За:—1. Первая из этих систем решений не имеет, а из второй системы находим х=2. Теперь решим вторую систему совокупности (8). Ясно, что при х< —т- выражение 3#— 1 отрицательно. Это значит, что уравнение этой системы корней не имеет. Итак, корнем уравнения (7) является jc=2. Упражнения Решите уравнения. 384. a) |jt|+*3=0; б) UI-2*3=0. 385. а) (*+1) (|*|-1)=-0,5; б) (2*-1)(|*| + 1)=3. 38в. а) ^=^+*; б) l*+±-xJ*j*L. 387. а) ^=5-,. б) ^=2*. 388. а) 7—4л:= |4лг—7|; б) Зх—5= IЗдсг—5|. 389. a) U-7| = U+9|;6) U+3| = |2x-l|. 390. а) 2 |х+Ц = |лг—3|; б) U—2| —3 U+3|. 56
391. а) |3х+2| = |2х-3|; б) |6х+5| = |1-х|. 392. a) |х2+х-1|=2х-1; б) 1х2 —х —31«= —х—I. 393. а) 2 1х2 + 2х-5|=х-1; б) 2 |х2-х| =,х2 + 1 394. а) х2+3|х|+2=0; б) 2x2-1x1 -15=0. 395. а) (х+1)2-2|х+1| + 1=0; б) х2+2х-3 |х+1| +3=0. 396. а) |х| + |х+1| = 1; б) |х+1| + |х+2|=2. 397. a) |jc—11 — |jc—21 — 1; б) U_2|+ |4-x| =3. 398. а) |х-1| + |х-2| = 1; б) 2 jx + 3| - U —4|=4. 399. а) |х-2| + |х-3| + |2х-8|=9; б) |х+1|-|х-2| + |Зх+6|=5. 400. а) |2х+1|-|3-х| = |х-4|; б) |х-1| + 11-2x1=2 |х|. 401. а) |х|-2 |х + 1 1+3 |х+2|=0; б) |х+1|-|х|+3|х-1|=2. 402. а) |х|-2|х+1|+3|2х-4| = 1; б) |х|+2|х+1|-3|х-3|=0. 403. а) |х2-9|+ |х-2| =5; б) |х2-1| + |х+1|=0. 404. a) ix2 —4| —|9—х2! =5; б) |х2-9| + 1x2-41 =5. 405. а) |х—х2—1| = |2х—3—х2!; б) 1x2+2x1-|2-х| = |х2-х|. 406. а) ||3—2х| — 1|=2 |х|; б) ||х+4|-2х|=Зх-1. 407. а) |2 |х— 11 +3х—41 =х—2; б) |Зх-|2х-5||=х+5. 408. а) |-2х-|Зх+4|+5|=1-5х; б) |-5х-3|2х-3|+2|=И+х. 4*.>£=£!+3 ,:в) >**-*!+'— 4,0. a) } г 2-I2X+X2! § 10. СИСТЕМЫ РАЦИОНАЛЬНЫХ УРАВНЕНИЙ К Основные понятия. Несколько уравнений с двумя переменными xf у образуют систему, если ставится задача об отыскании всех таких пар (дс; у\ которые удовлетворяют каждому из заданных Уравнений. Каждая такая пара называется решением системы. Решить систему уравнений — значит найти все ее решения. Множество решений системы может быть, ц частности, пустым — в этом случае говорят, что система не имеет решений или что эта система несовместна. Несколько систем уравнений с двумя переменными х, у образуют совокупность систем, если ставится задача об отыскании всех таких ПаР (*; у), каждая из которых удовлетворяет по крайней мере одной из заданных систем. Каждая такая пара называется решением совокупности систем. 57
Процесс решения систему уравнений состоит, как правило, в последовательном переходе с помощью некоторых преобразований от данной системы к другой, более простой, затем к еще более простой и т. д. Если в результате некоторых преобразований системы f\ (*; y)=gi (*; У\ hi)g%{x\y)\ (1) мы перешли к системе и если при этом каждое решение системы (1) является в то же время решением системы (2), то система (2) называется следствием системы (1). Следствием системы уравнений может быть и одно уравнение. Например, уравнение Зх—2у=3 является следствием системы \х — ?>у=—2. (как сумма уравнений системы). Вообще следствием системы уравнений может быть система как с меньшим, так и с большим числом уравнений. Так, система есть следствие системы Две системы уравнений называются равносильными, если множества их решений совпадают. Ясно, что две системы равносильны тогда и только тогда, когда вторая является следствием первой и первая является следствием второй. Отсюда, в частности, следует, что если к системе уравнений добавить еще одно уравнение, являющееся следствием данной системы, то новая система будет равносильна исходной. Если же опустить какое-либо уравнение системы, то полученная система уравнений (или одно оставшееся уравнение) будет следствием исходной системы. Приведем две теоремы, применяющиеся при решении систем уравнений. 58
Теорема 1. Если уравнение р\(х; y)=gUx; у) равносильно уравнению fi(x; y)=g\ (х; у) (или является его следствием), а уравнение f2 (x\ y)=g2 (*; у) равносильно уравнению f2(x; y)=g2(x\ у) (или является его следствием), то система 1/Н*; y) = gf2{x\ у) равносильна системе г/,(*; y) = gx{x\ у), 1Ы*; y)=g2{x\ у) (или является ее следствием). Теорема 2. Если уравнение f(x; y)~g(x; у) является следствием уравнений f\(x; y) = gx(x\ у) и f2(x; y) = g2(x; у), то каждая из систем *; У)=82{х\ у), (f\ (*; y)=g\ (х; у), a(h(x\ y)=g2(x\ у \f(x;y) = B(x;y) U[f(x; y)=g(x; у) стемы (h(x* y)=gi(x; у), X /2 (x; y)=g2 (x; у), является следствием системы а система { f\(x; y) = g\(x; у), h{\ y)=g2(x; у), f(x; y)=g{x; у) равносильна системе (3). В частности, следствиями системы (3) будут такие системы: (fi(x; y)=g\(x; у), \ f. (x; y)±h (x; y)=gi (x; y)±g2 (x; у); (4) Г fi (x; y)=gi (x; у), ,,, \ /. (х; y)-h (x; (/)=g! (x; y)-g2 (x; у); W Г /■ (х; y)=gi (x; у), . , \(f2(x;y))2=(g2(x;y)f. W Если не существует таких пар (х; у\ при которых оба выражения '2 \х\ у) и g2 \х\ у) одновременно обращаются в нуль, то уравнение М^=^П/} Равносильно Уравнению f2 {x\ y)=g2 {x\ у). Тогда системе (3) равносильна следующая система: 59
h (xi У) g2 (x; y) * Ее следствием, в свою очередь, является система: Таким образом, приходим к следующему выводу: если не существует таких пар (х; у), при которых оба выражения /г (х; у) и g (х; у) одновременно обращаются в нуль, то система !/i (*; J/)=gi (x; У) Ы*; y)_g\{x\ у) (7) h (*; у) g2 (*; у) является следствием системы (3). Если, решая систему, мы преобразовали ее в систему, являющуюся следствием исходной, то найденные решения новой системы безусловно подлежат проверке (например, подстановкой найденных значений переменных в исходную систему). В дальнейшем будут полезными следующие утверждения: 1. Система (4) равносильна системе (3). 2. Если не существует таких пар (х; у), при которых обе части уравнения f\ (x; y)=g\ {x; у) одновременно обращаются в нуль, то система (5) равносильна системе (3). 3. Система (6) равносильна системе (3), если для любых х, у из области определения системы (3) выполняется неравенство h(x\ y)-g2{x; y)>0. 4. Если не существует таких пар (х; у), при которых одновременно обращаются в нуль обе части второго уравнения системы (3), то система (7) равносильна системе (3). Отметим еще один результат, вытекающий из теорем 1 и 2. Теорема 3. Если совокупность уравнений hi (x; y)=g2\(x\ y)y '*; y)=g22{x\ у); 0s=g2k{X't у) равносильна уравнению /2 (x\ y)=g2 {x\ у) (или является его следствием), то совокупность систем t\{x\ y)=g\{x\ у\ Ы (*; y)=g2\ (x\ у)\ fi(x\ y)=g\{x\ y\ /22 (x; у) = £22(х; у); ft (*; y)=gt (*; у), f»(*'' y)=^*(jf; у) 60
равносильна системе (3) (или является ее следствием). В частности, следствием системы / h (*; y)=gi (x; у), U2i (х; у)-Ы(х; y)-...-hk{x; t/)=0 является совокупность систем: /i (x; y) = g\ (х; у), ( ft (x; «/)=£, (х; у), ( /, (х; y)=g, (x; у), Ы (*; У)=0; 1 Ь (х; у) = 0; ...; [ f2k (X; у)=0. Пример 1. Решим систему *j (8) Решение. Перемножив уравнения системы (8), получим систему являющуюся следствием исходной. Второе уравнение системы (9) путем несложных преобразований сводится к уравнению, ху — 8у которое является следствием второго уравнения системы (9). Тогда в силу теоремы 1 система [ху=8 является следствием системы (9). Вычтем теперь первое уравнение системы (10) из второго. Получим систему (ху = 8, и далее {ху=8, X Система (11) в силу теоремы 2 является следствием системы (10). Перемножив уравнения системы (11), получим систему {^=16, (12) 61
которая является следствием системы (11). Из второго уравнения системы (12) находим */i=2, у2=—2, а из первого уравнения соответственно*i =4, *2= —4. Итак, система (12) имеет следующие решения: (4; 2) и (—4; —2). Проверка. Поскольку система (12) является в конечном счете следствием системы (8), то найденные решения необходимо проверить. Это можно сделать, например, с помощью подстановки решений системы (12) в уравнения системы (8). Проверка показывает, что оба решения системы (12) являются решениями и системы (8). Таким образом, решения системы (8): (4; 2), ( — 4; —2). Пример 2. Решим систему t— 4 { { = — 9. Решение. Сложив эти уравнения, получим новое уравнение: xy+xz+yz= — 7. Присоединив это уравнение к уравнениям заданной системы, получим систему, равносильную по теореме 2 заданной системе: — 7, I xy + xz=— 4, I yz+yx= — 1, \zx+zy=— 9. Заменим второе уравнение этой системы разностью первых двух уравнений, третье уравнение — разностью первого и третьего, а четвертое — разностью первого и четвертого, кроме того, опустим первое уравнение. Получим систему yz, xz=— 6, \ху = равносильную данной в силу теоремы 2 и утверждения 1. Перемножив эти три уравнения, получим уравнение (jq/z)2 = 36, приписав которое к уравнениям предыдущей системы придем к равносильной системе У, *z=— б, лгу = 2 (здесь снова используется теорема 2), которой, в свою очередь, по теореме 3 равносильна следующая совокупность систем: 62
Решим первую систему этой совокупности. Разделив последовательно первое уравнение системы на второе, третье и четвертое уравнения, получим х=— 2, (/= — 1, 2 = 3. Аналогично из второй системы находим # = 2, у=1, z= — 3. Итак, совокупность систем, а тем самым и равносильная ей исходная система имеют следующие решения: ( — 2; —1; 3), (2; 1; —3). 2. Основные методы решения систем уравнений. Остановимся на трех основных методах: 1) метод линейного преобразования системы (или метод алгебраического сложения); 2) метод подстановки; 3) метод замены переменных. Метод линейного преобразования системы основан на следующей теореме: Теорема 4. Если Л= | ?! ?21 #0, то система <a{fx(x; \ b{f, (х; y)+a2f2(x; y)=0, равносильна системе В частности, если ai = l, a2 = 0, 6i = l, &2=±1, то получаем систему равносильную исходной по утверждению 1. Эта теорема распространяется на случай, когда число уравнений больше двух. Например, для трех уравнений с тремя переменными имеет место следующая теорема: Теорема 4.' Если Л= Ь\ Ь2 Ь% С\ С2 Сз то система равносильна системе >, у; z)=0, /2 (х; у; z)=0, /з (*; у; z)=0. Метод подстановки основан на следующей теореме: Теорема 5. Система уравнений \f(F(y);y)=g{F(y);y) 63
равносильна системе {x = F(y), \f(x; y)=g(x; у). Так, равносильными будут следующие системы: =*2у — 5, (х=2у—5, у-5)2 + «/2=2(2«/-5)+</ и [х2+у2 2 Следствие. Если уравнение x=F(y) (или у—Ф(х)) равносильно уравнению <р(х; у)=0, то система \f(F(y);y)=g(F(y);y) или (у=Ф{х), \f(x; Ф(x))=g(x■, Ф(х)) равносильна системе (<р(х; у)=0, \f(x; y) = g(x; у). Например, система уравнений (у2+х=2(х-5), равносильна следующей системе: (х=у2 + Ю, Для системы трех уравнений с тремя переменными соответствующая теорема формулируется следующим образом: Теорема 5'. Система уравнений М; V, F{x; y))=gi{x; у; F {х; у)), h{x\ у, F(x; y))=gi{x; у; F (х; у)), z=F(x;y) {! равносильна следующей системе: 7i (x; У\ 2)=gi (x; у; г), h{x\ У, z)=g2{x; у; г), z==F(jc; у). Метод замены переменных состоит в следующем. Если F\ (x; y)=f\ (ф1 (х; у); ф2 (х\ у)), F2(x; y)=hW\{x\ у); <рг(*; у)), то систему 64
(Ft(x; t/) = 0, \F2(x- y) = 0 с помощью новых переменных ц>\ {х\ у) = и, ср2 (х\ y)=v можно записать в виде (f[{u\ v) = 0, I/2 (и; v) = 0. Пусть {и\\ v\)t (u2\ V2), ..., (un\ vn) — решения последней системы. Тогда задача сводится к решению следующей совокупности систем: ( ф1 (*; у) = ии Г Ф1 (a:; (/) = w2, f q>\ (x\ y) = Un, \ Ф2 (a:;- y) = v\\ \ч>2(х\ y) = v2: ...; \ ф2 (х\ y) = vn. Решения этой совокупности будут одновременно и решениями системы Рассмотрим примеры применения этих методов при решении систем уравнений. Пример 3. Решим систему f x2= 13х-\-4у, \у2=:4х+\3у. Решение. Вычтем второе уравнение из первого. Тогда по теореме 4 система равносильна исходной. Рассмотрим первое уравнение полученной системы. Имеем: (х — у)(х + у) = 9(х — у), и далее {у)( у) В итоге мы приходим к следующей системе, равносильной исходной по теореме 1: По теореме 3 эта система равносильна следующей совокупности систем: Каждую из этих систем решим методом подстановки. Первая система преобразуется к виду (х = у, | у2 = \у -f-1 Зу, откуда находим: f ATi=O, /jc2=17, lyi=0; (t/2=17. Вторая система совокупности преобразуется к виду 65
Из уравнения у2 = 4 (9 — у)+ \3у находим: */3=12, у4=—3, и далее из соотношения х = 9 — у получаем лг3=—3, *4 = 12. В итоге мы нашли следующие четыре решения: (0; 0), (17; 17), (-3; 12), (12; -3). Проверка. Поскольку в процессе решения заданной системы выполнялись только равносильные преобразования, то найденные решения являются и решениями исходной системы. Пример 4. Решим систему уравнений Решение. Применим метод подстановки. Имеем: = 2— у — z, ) и далее I О /О «. , J*/2-2=-3,' Последние два уравнения полученной системы, в свою очередь, образуют систему двух уравнений с двумя переменными. Решим эту систему методом подстановки. Имеем: ( = 2-3, т. е. Jt, = z- Из последнего уравнения находим: 2| = 1, 22 = 3. Из уравнения у = 2 — 3 получаем соответственно: у{ = —2, у2 = 0, а из уравнения х = 2 — у — 2 находим х\ = 3, Хч = — 1. Итак, получили следующие решения: (3; —2; 1), (—1; 0; 3). Пример 5. Решим систему уравнений (xy-\-z'2 = 2, Решение. Заменим первое уравнение системы разностью первого и второго уравнений, второе — разностью второго и третьего, а третье оставим без изменения. Тогда получим систему: ! т. е. систему 66
которая по теореме 4 равносильна заданной. Имеем далее: По теореме 3 этой системе равносильна следующая совокупность систем: — (/ = 0, (z + Jt—# = 0, J Решим системы этой совокупности методом подстановки. Из первой системы находим: (1; 1; 1), (—1; —1; —1); из второй: (л/2; 0; л/2), (—л/2; 0; —л/2); из третьей: (л/2; л/2; 0), (-л/2; -л/2; 0);_ из четвертой: (0; л/2; л/2), (0; — л/2; —л/2). Проверка. В процессе решения все преобразования были равносильными, поэтому найденные восемь решений являются и решениями заданной системы уравнений. 3. Однородные системы. Система двух уравнений с двумя переменными вида называется однородной (левые части обоих уравнений — однородные многочлены степени п от двух переменных). Однородные системы решаются комбинацией двух методов: линейного преобразования и введения новых переменных. Пример 6. Найдем решения системы: {; Решение. Первое уравнение системы — однородное (напомним, что так называются уравнения вида f (x, t/) = 6, где f (xt у) — однородный многочлен). Заметим, что если положить #=0, то из уравнения 3jc2+лгг/ — 2г/2 = 0 находим х = 0. Но пара (0; 0) не удовлетворяет второму уравнению системы, поэтому уфО, и, следовательно, обе части однородного уравнения Зл:2 -+- дг£/ — 2£/2 = 0 можно разделить на у2 (это не приведет к потере корней). Получим: Ц-+Ц—Ц=\, и далее з(— )*+-—2 = 0, У У У У \ У / У откуда находим, что —= — 1 или —=-|"» т- е- * = —У или Х=4-У- у у з з Теперь задача свелась к решению совокупности систем уравнений: (2 2х*-3х 67
Первая из этих систем несовместна, а вторая имеет два решения: (2; 3), ( — 2; —3). Это и будут решения заданной системы. Пример 7. Решим систему г Зх2 — 8ху-\-4у2 = 0, \5х2-7ху-6у2 = 0. Решение. Заметим прежде всего, что в данном случае пара (0; 0) удовлетворяет системе. Пусть теперь уФО. Разделив на у2 обе части каждого из однородных уравнений второй степени, образующих заданную систему, получим откуда находим I—=2; —=-^- У~ ' У ~ 3 ' JL=2- JL=_J_. Значит, —=2. у Положим y = t, тогда х = 2/. Заметим, что при / = 0 и х = 0, и у = 0. Таким образом, решения заданной системы — это пары вида (2/; 0. где t£R. Пример 8. Решим систему уравнений З*2 — 2xi/= 160, 2-3;n/--2i/2 = 8. (13) Решение. Умножим обе части второго уравнения на 20 и вычтем полученное уравнение из первого уравнения системы: Зх2-2ху =160 20а:2-60а:// — 40*/2=160 — 17х2 + 58ху + 40i/2 = 0. Мы получили следующую систему, равносильную системе (13): г Зх2 — 2xf/ = 160, \ 17л:2 — 58x1/ — 40t/2 = 0. (14) Рассмотрим однородное уравнение 17х2 — 58ху — 40i/2=0. (15) Если */ = 0, то из этого уравнения получаем х=0. Но пара (0; 0) не удовлетворяет исходной системе. Значит, уФО и поэтому, разделив обе части уравнения (15) на у2, получим уравнение, равносильное уравнению (15): 68
Положив и——, придем к квадратному уравнению корни которого Ы|=4, и2= —Tf- Значит, уравнение (15) равносильно совокупности уравнений: —=4; —=——, и соответственно система (14) равносильна совокупности систем: х л ( х ю ! Зх2 — 2ху= 160; ( За:2 — 2ху= 160. Применив к каждой из этих систем метод подстановки, находим следующие решения: (8; 2), ( — 8;- —2), \Ъ\ —-у-) » ( — 5; -^-J . Поскольку в процессе решения заданной системы выполнялись только равносильные преобразования, то найденные решения являются и решениями исходной системы. Пример 9. Решим систему уравнений Решение. Выполним алгебраическое сложение уравнений системы (16): • 2 .(-1) 2х3 — х2у — 2ху2+у3=0. Получим систему: (2x3-x2y I *» + *»= 1. равносильную заданной. Рассмотрим уравнение 2х3 — х2у — / + У Как и в предыдущем примере, здесь можно было бы разделить обе части уравнения на уъ. Однако в данном случае проще разложить левую часть на множители: л'2(2х-*/)-</2(2*-</) = 0, и далее (2х-у)(х-у)(х + у)=0. Значит, система (17) равносильна следующей совокупности: 69
Применив метод подстановки к каждой из этих систем, находим следующие решения системы (16): (-ф-; ^р-), (-*£-', -тг-)- \3 3 / \ 2 2/ Пример 10. Решим систему уравнений Решение. Воспользуемся очевидным равенством: х44-у4 СисГема (18) примет тогда вид: (*2 + 42)2-*V = 91, ((Xt-b!fy)( х Л-У — ху = 79 или | х +у — xy = т. е. Применяя для решения системы (19) метод линейного преобразования (сложив и вычтя уравнения), получим систему из которой уже нетрудно найти следующие решения: (3; 1), (1; 3), (-3; -1), (-1; -3). 4. Симметрические системы. Напомним основные сведения о симметрических выражениях. Выражение F(x; у) называется симметрическим, если оно при замене переменных х на у, у на х не изменится. Так, симметрическими будут следующие выражения: F(x; y) = x2 + 3xy + y\ F{x\ у)=л^+У+2ху+^+^ Основными симметрическими многочленами с двумя переменными считаются х-\-у и ху. Все остальные симметрические многочлены с двумя переменными могут быть выражены через основные. Положив для краткости и = х+у, v=xy, получаем, например: 2-v и т. д. Система, все уравнения которой симметрические, называется симметрической. Ее можно решить методом замены переменных, выбрав в качестве новых переменных основные симметрические многочлены. Пример 11. Решим систему уравнений 70
Решение. Положим Так как х3 + у3 = и3 — 2>uv, то заданная система сводится к следующей: Из этой системы находим: Теперь остается решить следующую совокупность систем: Решения этой совокупности, а с нею и исходной системы таковы: (1;2), (2; 1). Замечание. Вернемся снова к системе, рассмотренной нами в примере 10: Jx4 + *y+j/4=:91, \х2-ху+у2=7. Эта система симметрическая, а потому, как и предыдущая, может быть преобразована к более простому виду с помощью введения новых переменных: и далее Получим: |((и2 —2и)2-—2 (и2 — 2v)2 — u2=91, К-2 Из второго уравнения этой системы находим: w2=3y-f-7. С помощью этой подстановки первое уравнение системы преобразуется к виду: (Зи + 7 — 2v)2 — и2 =91, откуда находим: и = 3. Из уравнения ы2 = Зи + 7 находим «i.2= d=4. Итак, система имеет два решения: Значит, исходная система равносильна совокупности систем: х+у=49 Эта совокупность приводит к тем же решениям, что были получены выше, в примере 10. 71
Упражнения Решите системы уравнений, 411. 413. ( ж—У—1. {j4V-41. 415. |2jc2—3«/=23, 412. Г2х-|/=1, 414. f 3x+5i/=2, 416. 417. Г 2*-3t/-;«/ = 4, | 3*+i/+3;a/=3. 419. Г х2 (*+(/) = 80, \ 2^+3^=29. 418. f 5x2+14x/=19, 421. Г 2х—«/=-1, 423. 1 3x + 4i/ + 6z=-l. 425. fV 420. f *-j/ = 2, 422. 424. 426. Г Jt|/=2, I 2 I jc+2t/ —2z = l 427. [ 3jq/ + 3x2 —3i/2 —2x —*/= — 7, 13. 428. Г 429. fjc + t/z = 2, 430. \х^у=-^ 432. 443. Г jc2—Злг£/-Ьг/2= — 1, {x—y=-jxy, 431. Г-тгН—^=2 2 , 2 5 4 3,4 X2 + y2=-p-Xy. I ; =7 2 V*+i/ л: — г/ 433. 435. 438. х + У , ^—1 _ x-t/"fx + t/ 2 ' Злг — ху-2у2=0. 440. Г 4л:2 —3xt/ —1/2=0, 442. 444. f 5x2- 72
446. ( x3-i/=\9(x-y\ 447. ( *4-1/4 = 15, \ x3 + y2 = 7 (x+y). \ x3y — xyz = 6. 448. [ x2 + xy+y2=\9(x-y)2, 449. f x2 + Axy-2y2 = 5 (x + y\ 450. 452. 454. 456. 458. 460. 462. Г ^3-y3 = 451. Г 453. J i x^ у 3 * 457. 459. 461. * + t/=3. У + </5_31 •2-3. 463. (x2-i 464. \z2-xy=-\. 466. (x2y = x + y — z, (x — y + z = Q, 470. x 3 ' 2 ~ 5 ' 473. _3^_=2 474. x + y ' | t/ ' z ' a: -^1=3 x+z \ x ' у ' z 5yz ±+±+1.3. xy'yzzx L
476. 478. (x+y=3z, 479. (*- ^ J § 11. ЗАДАЧИ НА СОСТАВЛЕНИЕ УРАВНЕНИЙ И СИСТЕМ УРАВНЕНИЙ Решение задач на составление уравнений (или систем уравнений) обычно осуществляется в три этапа: 1) выбор неизвестного, обозначаемого, как правило, через х (или нескольких неизвестных, обозначаемых х, у, z, ...), и составление уравнения (или системы уравнений), связывающего некоторой зависимостью выбранное неизвестное с величинами, заданными условием задачи; 2) решение полученного уравнения (или системы уравнений); 3) отбор решений по смыслу задачи. 1. Задачи на числовые зависимости. При решении задач на числовые зависимости могут оказаться полезными следующие сведения: 1) Если натуральное число А имеет п знаков, то A=an-iX X lO""1 +--- + 01* 10 + а0, где ао, #i, аг, ..., ап-\ соответственно количество единиц, десятков, сотен, ... в числе Л. 2) Если при делении натурального числа А на натуральное число В в частном получается q, а в остатке г (г<В), то A=Bq-\-r. Пример 1. Найдем двузначное число, если известно, что единиц в нем на 2 больше, чем десятков, и что произведение искомого числа на сумму его цифр равно 144. Решение. Пусть в искомом числе х единиц. Тогда в нем (х — 2) десятков, и, следовательно, оно равно (а: — 2)10 + л:=11х — 20. Сумма цифр искомого числа равна (х — 2)+х = 2х — 2. Таким образом, получаем следующее уравнение: (11х — 20) (2х — 2)= 144. 13 Это уравнение имеет два корня: х\=4 и X2=jr- Так как х и (* — 2) — это цифры числа, то x£N и —2<9. Из найденных значений х этим условиям удовлетворяет только значение *i=4. Тогда искомым числом является число 24. Пример 2. Найдем два двузначных числа, о которых известно следующее: если к первому числу приписать справа второе число, а затем еще цифру 0, то получится пятизначное число, которое при делении на квадрат второго числа дает в частном 39, а в остатке 575; если же к первому числу приписать справа второе и затем из 74
составленного таким образом числа вычесть другое число, полученное приписыванием справа первого числа ко второму, то разность будет раБна 1287. Решение. Пусть х — первое число и у — второе. После приписывания справа числа у к числу х получится четырехзначное число *• 100 + е/, а после приписывания к этому числу справа цифры О получится (х-100-f-£/) 10. Так как при делении этого числа на число у2 в частном получится 39 и в остатке 575, то (лМ00 + (/)10 = (/2.39 + 575. Это — первое уравнение составляемой системы уравнений. После приписывания справа двузначного числа х к двузначному числу у получится четырехзначное число #•100 + *. Таким образом, получаем второе уравнение: (*.100+у) —(у100 + *)= 1287. Итак, мы приходим к следующей системе уравнений: ( \ 99а: —99г/=1287. Эта система имеет два решения: (48; 35) и (Щ-\ ~~lSr По смыслу задачи хну — натуральные числа, причем ^^ и 10^ (/^99. Из найденных решений этим условиям удовлетворяет только первое решение, т. е. искомыми являются числа 48 и 35. 2. Задачи на прогрессии. Числовая последовательность (ап) называется арифметической прогрессией, если существует число d, такое, что для любого n£N выполняется равенство пп+\~ап + (1\ число d называется разностью прогрессии. Последовательность (Ьп\ у которой Ь\Ф0, называется геометрической прогрессией, если существует число q=£0, такое, что для любого n£N выполняется равенство Ьп+\^=Ьп*д\ число q называется знаменателем прогрессии. Основные свойства арифметической прогрессии: 1) а„ = а| + £*(л-1). 2) S«=2!±S5:rtt где Sn=ax+a2 + ... + an. 3) Последовательность (ап) является арифметической прогрессией тогда и только тогда, когда для любого n£N выполняется равенство aR+1==2&±£&±2 (характеристическое свойство арифметической прогрессии). Основные свойства геометрической прогрессии: 1) ЬЯ = Ь1ЧГ-1. 2) sn=b{V~*n) , если Чф\, и Sn = nbu если q=\ (S,, = 6,+ + ) 3) Последовательность (Ьп) является геометрической прогрессией тогда и только тогда, когда для любого n£N выполняется равенство 75
=-yJbn*bn+2 (характеристическое свойство геометрической прогрессии). На практике вместо равенства |6rt+1| =yfert.6rt+2 удобнее использовать равносильное ему равенство bl+\=bn*bn+2- 4) Если геометрическая прогрессия является бесконечно убы- оо вающей, т. е. \q\<\, то 5 = т~—, где S=2fert. Пример 3. Найдем трехзначное число, цифры которого образуют арифметическую прогрессию и которое делится на 45. Решение. Пусть х — цифра сотен, у — цифра десятков и z — цифра единиц искомого числа. Так как числа х, у, z образуют арифметическую прогрессию, то у=-^—. Так как искомое число делится на 45, то оно делится на 5 и на 9. Значит, оно оканчивается либо цифрой 0, либо цифрой 5 и сумма его цифр делится на 9. Итак, возникают две возможности, и в соответствии с этим мы приходим к совокупности двух систем уравнений (отличающихся только первым уравнением): 2 = 0, /2 = 5, = 9fe, где ( Из первой системы находим гл: = Перебрав целые значения у от 1 до 9, убеждаемся, что этой системе уравнений удовлетворяют лишь пары (6; 3), (12; 6), (18; 9). Однако по смыслу задачи х и у — целые числа, удовлетворяющие неравенствам 1<х^9 и 0<у<9. Этим условиям удовлетворяет только пара (6; 3). Из второй системы находим Аналогично, перебрав целые значения у от 1 до 9, убеждаемся, что второй системе уравнений удовлетворяют пары (1; 3) и (7; 6). Итак, условиям задачи удовлетворяют три числа: 630, 135, 765. Пример 4. Найдем пятый член бесконечно убывающей геометрической прогрессии, если известно, что ее сумма равна 9, а сумма квадратов ее членов равна 40,5. Решение. Пусть последовательность 6j, &2, &з, .... Ьп> ...— бесконечно убывающая геометрическая прогрессия. Тогда Рассмотрим последовательность 6?, Ь\, 6§, ..., Ь2п, ... . Члены этой последовательности также образуют бесконечно убывающую геомет- 76
рическую прогрессию. Первый член этой прогрессии равен 6?, а ее знаменатель равен q2. Тогда ' , =40,5. Таким образом, мы получаем следующую систему уравнений: Решив ее, получим 61=6, q=—. Тогда b$ = b\q*=—. 3 27 Пример 5. Три числа составляют геометрическую прогрессию. Если от третьего числа отнять 4, то числа составят арифметическую прогрессию. Если же от второго и третьего членов полученной арифметической прогрессии отнять по 1, то снова получим геометрическую прогрессию. Найдем эти числа. Решение. Пусть х, у, z — искомые числа. Так как они являются последовательными членами геометрической прогрессии, то, воспользовавшись характеристическим свойством геометрической прогрессии, получим y2 = xz. Так как, далее, числа х% у, (z—-4) являются последовательными членами арифметической прогрессии, то, воспользовавшись характеристическим свойством арифметической прогрессии, получим i/=^ii|z^2# Так как, наконец, числа х> (#—!)» (2 — 5) являются последовательными членами геометрической прогрессии, то (у— l)2 = x(z—-5). Таким образом, мы получаем следующую систему уравнений: Она имеет два решения: (1; 3; 9) и ( —; —; -5-). Эти значения х> У> z удовлетворяют условию задачи. Таким образом, искомые числа: 1, 3 и 9 или -~» 4" и ^- У У У 3. Задачи на совместную работу. Содержание задач этого типа сводится обычно к следующему. Некоторую работу, объем которой не указывается и не является искомым (например, перепечатка рукописи, рытье котлована, заполнение резервуара и т. д.), выполняют несколько человек или механизмов, работающих равномерно (т. е. с постоянной для каждого из них производительностью). В таких задачах объем всей работы, которая должна быть выполнена, принимается за единицу. Время /, требующееся для выполнения всей работы, и V — производительность труда, т. е. величина работы, выполняемой за единицу времени, связаны соотношением 77
Пример 6. Первому трактору на вспашку всего поля требуется на 2 ч меньше, чем третьему, и на 1 ч больше, чем второму. При совместной работе первого и второго тракторов поле может быть вспахано за 1 ч 12 мин. Какое время на вспашку поля будет затрачено при совместной работе всех трех тракторов? Решение. Примем величину работы (в данном случае это вспашка всего поля) за единицу. Пусть х ч — время, необходимое для вспашки поля первому трактору, у ч — второму и z ч — третьему трактору. Тогда производительность первого трактора, 1 1 х второго и третьего. По условию задачи z — х = 2 и х — у=\. Далее, так как при совместной работе первого и второго тракторов выполняется (—| j часть работы в час, а вся работа выполняется ими за 1 ч 12 мин, т. е. за — ч, то ~г(-—h—) = Ь В итоге приходим к следующей системе уравнений: Решив эту систему, получим (3; 2; 5), ( — 0,4; —0,6; 2,4). По смыслу задачи х>1, у>0 и z>2. Из найденных решений этим условиям удовлетворяет только первое решение. Теперь ответим на вопрос задачи. При совместной работе трех тракторов производительность труда составит у+^+у» т* е* ^-. Значит, время на вспашку поля тремя тракторами состав- ляет 2f- ч. Пример 7. Имеющиеся в совхозе комбайны, работая вместе, могут убрать урожай за одни сутки. Однако по плану комбайны вступали в работу последовательно: в первый час работал лишь один комбайн, во второй — два, в третий — три и т. д. до тех пор, пока не начали работать все комбайны, действовавшие вместе до полной уборки урожая. Время работы, предусмотренное планом, уменьшилось бы на 6 ч, если бы с самого начала уборки постоянно работали все комбайны, за исключением пяти. Сколько комбайнов было в совхозе? Решение. Примем величину всей работы равной 1 и введем три переменные: п — число комбайнов в совхозе, х — производительность труда каждого комбайна за 1 ч, t ч — время совместной работы всех комбайнов по плану. По условию п комбайнов, производительность каждого из которых х, могут выполнить работу за 24 ч, т. е. 24пх= 1. 78
По плану в первый час действовал один комбайн, объем работы, выполненной им за этот час, равен х. Во второй час действовали два комбайна, объем работы, выполненной ими за этот час, равен 2х. В третий час три комбайна выполнили объем работы, равный Зх, и т. д., в (п—1)-й час (/г — 1) комбайн выполнил объем работы, равный (/г-— 1) х. После этого в течение t ч действовали все п комбайнов, объем выполненной ими работы равен ntx. Плановая работа комбайнов в итоге описывается следующим уравнением: l. (3) Заметим, что х-\-2х-\-...-\-(п — I)х есть сумма (/2 — 1) членов арифметической прогрессии (ап), у которой ai=x, d = x. Значит, x + 2x+...+(n-l)*=*£±GL^ и уравнение (3) принимает вид: Наконец, из условия следует, что если бы с самого начала работали (п — 5) комбайнов, то работа длилась бы не (п—1+0 ч, что было предусмотрено планом, а на 6 ч меньше, т. е. ((п — 1 + /)—6) ч, тогда (n + t — 7) (л — 5)х=1. В итоге получаем следующую систему уравнений относительно переменных n, x, t: — 7)(nx — 5л')=1. Из первого уравнения находим пл: = ~-. Подставив это выражение во второе и третье уравнения системы, получим: Далее система без труда решается методом подстановки. Из первого уравнения *=2Г» из второго /= и / в третье уравнение, получим: =2Г~» из второго /=49~Аг. Подставив эти значения х (я + 35)(я-5)| 48л "- 79
откуда находим, что Я|=25, п2= — 7. По смыслу задачи п>1. Этому условию удовлетворяют только /i t = 25. Значит, в совхозе было 25 комбайнов. 4. Задачи на сплавь) и смеси. Решение этих задач связано с понятиями «концентрация», «процентное содержание», «проба», «влажность» и т. д. и основано на следующих допущениях: 1. Все рассматриваемые смеси (сплавы, растворы) однородны. 2. Не делается различия между литром как единицей емкости и литром как единицей массы. Если смесь (сплав, раствор) массы т состоит из веществ Л, Ву С (которые имеют массы соответственно т\у тг, /яз, то величина — (соответственно —, —) называется концентрацией вещества А т\ mm/ r (соответственно В, С) в смеси. Величина —-100% (соответственно \ т \ — •100%, — -100% ) называется процентным содержанием вещества А (соответственно В, С) в смеси. Ясно, что — +— +— = 1, т т т т. е. от концентрации двух веществ зависит концентрация третьего. При составлении уравнения обычно прослеживают содержание какого-нибудь одного вещества из тех, которые сплавляются (сме- шиваютсй и т. д.). Пример 8. Имеется кусок сплава меди с оловом массой 12 кг, содержащий 45% меди. Сколько чистого олова надо прибавить к этому сплаву, чтобы получившийся новый сплав содержал 40% меди? Решение. Спла*в состоит из меди и олова. Проследим за содержанием одного из этих веществ, например, олова в первоначальном сплаве и в полученном. В 12 кг сплава было 45% меди, а олова в нем было 55%, т. е. 12•— кг олова. Пусть к первоначальному сплаву добавили 1UU х кг олова. Тогда получилось (12+х) кг нового сплава, в котором олова стало 60%, т. е. 60^+^ кг. Таким образом, получается следующее уравнение: 55-12 ■ 60(12 + *) 100 ' 100 Решив это уравнение, найдем, что *=1,5. По смыслу задачи х>0. Найденное значение х этому условию удовлетворяет. Итак, к первоначальному сплаву следует добавить 1,5 кг олова. Замечание. Наметим коротко составление уравнения, основанное на прослеживании за содержанием в первоначальном и полученном сплавах меди, а не олова. В первоначальном сплаве меди было 12-у^ кг. Добавили х кг олова (меди не добавляли). Тогда получилось (12 + *) кг нового 80
лло/ 40(12+*) _ 12-45 сплава, в котором меди 40%, т. е. —-^—- кг. Получаем уравнение = (12+х)40 ~" 100 Пример 9. Имеется сталь двух сортов с содержанием никеля 5% и 40%. Сколько стали одного и другого сорта следует взять, чтобы после переплавки получить 140 т стали с содержанием никеля 30%? Решение. Проследим за содержанием никеля в сплавах. Взяв для переплавки х т стали, содержащей 5% никеля, непосредственно никеля взяли при этом х-—т, а взяв для переплавки у т стали, содержащей 40% никеля, никеля взяли при этом г/*-^— т. Так как в полученных 140 т нового сплава никеля стало содер- ОЛ жаться 30%, т. е. 140•— т, то получаем следующее уравнение: 1UU 5 . 40 ,.30-140 100 "^ 100 У 100 ' Кроме того, х+у=Ш. Таким образом, приходим к следующей системе уравнений: ( 5х + 40у= 140-30, Из этой системы находим х = 40, у=100. По смыслу задачи 0<Cx<Cl40, 0<t/<140. Найденные значения х и у этим условиям удовлетворяют. Итак, стали с 5%-ным содержанием никеля следует взять 40 т, а стали с 40%-ным содержанием никеля следует взять 100 т. Пример 10. Из сосуда, содержащего 54 л чистой кислоты, вылили несколько литров и после этого долили сосуд водой до прежнего объема. Затем из сосуда вылили смеси столько же литров, как и в первый раз. В результате в смеси, оставшейся в сосуде, осталось чистой кислоты 24 л. Сколько кислоты вылили в первый раз? Решение. Пусть в первый раз было вылито х л кислоты. Тогда в сосуде осталось (54—х) л кислоты. Долив сосуд водой, получили 54 л смеси, содержащей (54 — х) л кислоты. Значит, в 1 л смеси содержится 4Т*л кислоты. Во второй раз из сосуда вылили х л о4 смеси, т. е. кислоты вылили —=т^* л. 54 Итак, в первый раз было вылито х л кислоты, во второй -^—а: л кислоты, а всего за два раза вылили 54 — 24 = 30 (л) кислоты. Таким образом, получаем следующее уравнение: 81
Решив это уравнение, находим два корня: jci = 90, лгг = 18. Flo смыслу задачи 0<х<54. Из найденных значений х этому условию удовлетворяет только х=18. Следовательно, в первый раз вылили 18 л кислоты. Пример 11. Сосуд емкостью 8 л заполнен смесью кислорода и азота, причем на долю кислорода приходится 16% емкости сосуда. Из этого сосуда выпускают некоторое количество смеси, дополняют сосуд до прежнего объема азотом и вновь выпускают такое же количество смеси, после чего опять дополняют сосуд азотом до 8 л. В результате в сосуде стало 9% кислорода. Сколько литров смеси выпускали из сосуда каждый раз? Решение. Пусть из сосуда выпускали каждый раз х л смеси и впускали в него х л азота. Тогда после первого выпускания в сосуде осталось (8 — х) л смеси, а кислорода в этой смеси осталось (8 —х)-0,16 л. Подсчитаем теперь, сколько кислорода осталось после второго выпускания смеси. Так как после впускания в сосуд х л азота оставшийся в нем кислород стал содержаться в 8 л смеси, то в 1 л смеси оказалось (8-*Q)Q'16 л кислорода. о После того как произвели второе выпускание смеси, ее. в сосуде осталось снова (8 — х) л, но в ней всего °'16'_ ~~*'«(8 —*) л о кислорода. Итак, мы подсчитали, что в смеси осталось (8~*) 0»! л кисло- о рода. Это по условию задачи составляет 9% от 8 л, т. е. 8-0,09 л. Таким образом, мы приходим к следующему уравнению: (З-*)2 0.16=3 0>00 8 Из этого уравнения находим jci = 14 и *2 = 2. По смыслу задачи 0<лс<8. Из найденных решений этому условию удовлетворяет только х = 2. Значит, выпускали каждый раз по 2 л смеси. Пример 12. Имеется два сплава с различным процентным содержанием меди. Масса первого сплава а кг, а второго Ь кг. От каждого из сплавов отделили по куску равной массы и каждую из отделенных частей сплавили с остатком другого куска. В новых сплавах процентное содержание меди стало одинаковым. Какова масса каждого из отрезанных кусков? Решение. Пусть х кг — масса каждого из отрезанных кусков, у% — процентное содержание меди в первом сплаве, z% — процентное содержание меди во втором сплаве. После перестановки частей массы х кг в полученном первом сплаве (рис. 1) меди будет ^щ~У+щ z> a процентное содержа- 82
Рис. 1 Рис. 2 а — х х ние меди будет равно Ж!_Ш1. юо%, т. е. равно (fl^ В полученном втором сплаве (рис. 2) меди будет -у5?г Ь—х х +-^Уу а процентное содержание меди будет равно - X ХЮ0%, т. е. равно —Х)*+ХУ t По условию в полученных сплавах процентное содержание меди одинаково. Значит, мы приходим к уравнению (д — x)y + xz (6 — x)z + xy а ~ b " (1) Последовательно имеем: aby — bxy -f bzx = abz — axz + axyy (aby — abz) — (bxy — bxz) — (axy — axz) = 0, ab (y — z) — bx (y — z) — ax (y — z) = 0, (y — z) (ab — ax — bx) = 0. По условию y=£z, значит, ab — ax — bx=Oy откуда находим *=-^-т. Переменные у и z исключились в процессе решения полученного уравнения. По смыслу задачи 0<*<а и 0<л:<&. Убедимся, что найденное значение х этим условиям удовлетворяет. Действительно, так как ab>0 и а + 6>0, то ясно, что х=-2—>0. Далее, так как a-f-o b<ca-\-b и а > 0, то ab < а (а + Ь)у откуда ввиду того, что а + b > 0, получим -тт<я, или х<а. Аналогично убеждаемся, что x<Cb. Итак, масса каждого из отрезанных кусков равна -^гт кг. 5. Задачи на движение. При решении этих задач принимают следующие допущения: 1. Если нет специальных оговорок, то движение считают равномерным. 2. Скорость считается величиной положительной. 83
3. Всякие переходы на новый режим движения, на новое направление движения считают происходящими мгновенно. 4. Если тело с собственной скоростью х движется по реке, скорость течения которой равна у, то скорость движения тела по течению считается равной (х-\-у\ а против течения — равной (х—у). Пример 13. Из пункта А в пункт В отправляются три велосипедиста. Первый из них едет со скоростью 10 км/ч. Второй отправляется через полчаса после первого и едет со скоростью 8 км/ч. Какова скорость третьего велосипедиста, если известно, что он выезжает через полчаса после второго и что он догоняет первого через 4 ч после того, как он догонит второго? Решение. Выразим время, которое потребуется третьему велосипедисту, чтобы догнать первого и второго велосипедистов. Пусть скорость третьего велосипедиста равна х км/ч. Тогда, сокращая расстояние до первого велосипедиста по (л: —10) километров в час, отставание, образовавшееся за 1 ч и, следовательно, равное 10 км, третий велосипедист покроет за ч. Аналогично второго велосипедиста третий догонит за -—т ч. (4 км — расстояние между третьим и вторым велосипедистами в момент старта третьего.) Таким образом, получаем следующее уравнение: ю *-10 х-8 = 4. Из этого уравнения находим *i = 12, *2 = 7,5. По смыслу задачи скорость третьего велосипедиста должна быть больше, чем скорости первого и второго велосипедистов, т. е. х>10. Из найденных решений этому условию удовлетворяет только х=12. Итак, скорость третьего велосипедиста равна 12 км/ч. Пример 14. Из пункта А в пункт В выехал грузовой автомобиль, через час из А в В выехал легковой автомобиль. В пункт В машины прибыли одновременно. Если бы из пунктов А и В машины выехали одновременно навстречу друг другу, то встреча произошла бы через 1 ч 12 мин после их выезда. Найдем время, за которое проедет путь от Л до Б грузовик. Решение. Пусть грузовик проезжает путь от А до В за х ч. Тогда легковой автомобиль проедет этот путь за (х— 1) ч. Принимая путь АВ равным у км, найдем, что скорость грузового автомобиля равна -£- км/ч, а легкового -^-т- км/ч. Так как при движении навстречу автомобили сближаются со скоростью ( ——|-—^—) км/ч, а весь путь они проезжают за -|-ч, то получаем следующее уравнение: 5 \ х ' jc— 1 / У' 84
В этом уравнении неизвестных величин две. Однако после деления обеих частей уравнения на уФО получим уравнение 5 содержащее уже только искомую неизвестную величину х. Решая это уравнение, находим *i=3, л:2 = 0,4. По смыслу задачи х > 1. Более того, ясно, что грузовик проедет путь АВ за время, большее чем — ч, т. е. х>™. Из найденных значений х, таким образом, следует "отобрать только х = 3. В итоге приходим к выводу, что время, за которое грузовик проезжает путь от А до В, равно 3 ч. Пример 15. Путь от пункта А до пункта В, по которому едет велосипедист, состоит из трех участков, причем длина первого в 6 раз больше длины третьего участка. Найдем среднюю скорость движения на всем пути АВ, если известно, что она равна скорости движения на втором участке, на 2 км/ч меньше скорости движения на первом участке и на 10 км/ч больше половины скорости движения на третьем участке. Решение. Пусть искомая средняя скорость велосипедиста равна х км/ч, расстояние АВ равно S км и длина третьего участка у км. Тогда длина первого участка равна 6у км, а длина второго равна (S — Ту) км. Время, затраченное велосипедистом на весь путь, равно —ч, а с другой стороны на прохождение первого участка пути он затратил j^ ч, второго —^-^- ч, третьего —2 (jc—Ю) ч* (скорость v велосипедиста на третьем участке связана со средней скоростью х условием х=у-+Ю). Сумма этих дробей и состав- А 6у им z км у км В Рис. 3 ляет —. На рисунке 3 представлена схема движения велосипедиста. Итак, мы получаем уравнение: 5 —7у | у _S Решим составленное уравнение: 6j/ ■ S _7g , у _S "Г" "*" х ' JE ISL+ х + 2 л- ' 2х^20
По смыслу задачи уФО, поэтому при делении обеих частей последнего уравнения на у потери решений не будет. Итак, 7Т2-7+2Г^=0' откУда *' = 14' * —20. По смыслу задачи все скорости положительны, т. е. лг>0, ,v + 2>0 и 2х — 20>0. Значит, х>10. Из найденных корней этому условию удовлетворяет только х=14. Таким образом, средняя скорость велосипедиста на всем пути АВ равна 14 км/ч. Пример 16. В реку впадает приток. Катер отходит от пункта Л, находящегося на притоке, идет по течению 80 км до впадения притока в реку в пункте В, а затем идет вверх по реке до пункта С. На путь от Л до С он затратил 18 ч, на обратный путь — 15 ч. Найдем расстояние от пункта В до пункта С, если известно, что скорость течения реки 3 км/ч, а собственная скорость катера 18 км/ч. Решение. Пусть расстояние от пункта В до пункта С равно х км. Выразим время, за которое катер проходит путь от А до С. Для этого придется ввести еще одну переменную, положим скорость течения в притоке равной у км/ч. Тогда скорость катера при движении в притоке от Л до В (по течению) равна (18 + 1/) км/ч, а при движении от В к Л равна (18— у) км/ч. Время, затраченное на путь от 80 80 Л к В, равно ч, а на путь от В к А — ч. При движении катера по реке от В к С (против течения) скорость его равна 18 — 3=15 (км/ч), а при движении от СкВ равна 18 + 3 = 21 (км/ч). Следовательно, время, затраченное на путь от В к С, равно -~ч, 1 о а на путь от С к В-~ ч. Таким образом, получаем следующую систему уравнений: ,21 Из этой системы находим: jci=210, По смыслу задачи х>0 и 0<у<18. Из найденных решений этим условиям удовлетворяет только первое. Это значит, что расстояние от пункта В до пункта С равно 210 км. Пример 17. По окружности, длина которой 100 м, движутся равномерно две точки. Они встречаются через каждые 4 с, двигаясь в противоположных направлениях, и через каждые 20 с, двигаясь в одном направлении. Найдем скорости этих точек. 86
Решение. Пусть скорость первой точки х —, а второй у —, причем пусть х>у. За 4 с первая точка проходит путь 4х м, а вторая — 4у м. Так как при движении в противоположных направлениях каждые 4 с происходит встреча этих точек, т. е. суммарно за 4 с они проходят 100 м, то первое уравнение будет таким: 4л: + 4г/ = 100. При движении в одном направлении первая точка догоняет вторую каждые 20 с. Это значит, что за 20 с первая точка проходит путь на один оборот (т. е. на 100 м) больше второй. За 20 с первая точка проходит путь 20* м, а вторая — 20у м. Таким образом получаем второе уравнение: 20jc — 20г/= 100. В итоге приходим к системе уравне- ний: /4х + 4*/=100, I 20лг —20г/= 100, из которой находим лг= 15, #=10. По смыслу задачи х>у>0. Ясно также, что 4Ж100, 4у<\00 и 20а:>100. Таким образом, 5<л:<:25, 0<*/<:25. Найденные значения х и у этим условиям удовлетворяют. Значит, скорость первой точки 15 м/с, а второй — 10 м/с. Прежде чем переходить к следующему примеру, заметим, что к задачам на движение можно отнести (условно) задачи на любые равномерные процессы (шлифовка деталей, заполнение резервуаров, перепечатка рукописей и т. д.). Пример 18. В резервуар поступает вода из двух труб различных диаметров. В первый день обе трубы, работая одновременно, подали 14 м3 воды. Во второй день была включена лишь малая труба. Она подала 14 м3 воды, проработав на 5 ч больше, чем в первый день. В третий день работа продолжалась столько же времени, сколько во второй, но сначала работали обе трубы, подав 21 м3 воды, а затем работала лишь большая труба, подавшая еще 20 м3 воды. Найдем производительность каждой трубы. Решение. Пусть х м3/ч — производительность большой трубы, у м3/ч — производительность малой трубы, t ч — время работы обеих труб в первый день. Тогда в первый день трубы подали (x+y)t м3 воды, что по условию составляет 14 м3. Получим первое уравнение: (x+y)t=\4. Во второй день малая труба работала (/ + 5) ч, подала у (£ + 5) м3 воды, что по условию составляет 14 м3. Получим второе уравнение: (/ + 5)14 ) В третий день сначала работали обе трубы, подавшие 21 м3 21 воды, значит, их совместная работа продолжалась ——ч. Затем работала одна большая труба, подавшая 20 м3 воды, значит, ее 20 работа продолжалась — ч. Так как работа в третий день длилась столько же времени, сколько во второй день, то получим третье 21 20 уравнение: ——| = /-|-5. Мы пришли к системе уравнений:
(x + y)t=l49 21 x+y Из второго уравнения системы находим / + 5 = —, тогда первое уравнение системы можно переписать в виде -—i-=——5, а тре- 21 20 14 * + У У тье — в виде ——|—=—. Получим систему из двух уравне- х~тУ х у НИИ 14 14 х+у* х у ix+y ' х у Освободившись в обоих уравнениях от знаменателей, получим: у + у, \4х2-27ху-20у2 = 0. Второе уравнение системы однородное. Разделив обе его части почленно на у2 и положив z=—, получим квадратное уравнение Ц I4z2 — 27z — 20 = 0, корни которого z\=—, 22= —4-- Так как по смыслу задачи х>0 и у>0, то из найденных значений оставляем — только zi=—. Теперь осталось решить систему уравнений Из нее находим х=5, у =2. Как уже отмечалось, по смыслу задачи х>0, у>0. Найденные значения х и у этим условиям удовлетворяют. Таким образом, производительность большой трубы 5 м3/ч, а малой — 2 м3/ч. Упражнения 480. Сумма квадратов цифр двузначного числа равна 10. Если от искомого числа отнять 18, то получится число, записанное теми же цифрами, но в обратном порядке. Найдите исходное число. 481. Какое двузначное число в 4 раза больше суммы своих цифр и в 3 раза больше произведения цифр? 482. Найдите два целых числа, сумма которых равна 1244. Если к первому числу приписать справа цифру 3, а во втором числе отбросить последнюю цифру 2, то полученные числа будут равны. 483. Трехзначное число оканчивается цифрой 3. Если эту цифру перенести в начало числа, то новое число будет больше утроенного первоначального числа на 1. Найдите исходное число. 484. Запись шестизначного числа начинается цифрой 2. Если эту цифру перенести с первого места на последнее, сохранив порядок остальных цифр, то вновь 88
полученное число будет втрое больше первоначального. Найдите первоначальное число. 485. Сумму всех четных двузначных чисел разделили без остатка на одно из них. Найдите делитель, если известно, что сумма его цифр равна 9 и что частное отличается от делителя только порядком цифр. 486. Если двузначное число разделить на сумму его цифр, то в частном получится 7 и в остатке 6. Если же это двузначное число разделить на произведение его цифр, то в частном получится 3, а в остатке число, равное сумме цифр исходного числа. Найдите исходное число. 487. Сумма двух трехзначных чисел, записанных одинаковыми цифрами, но в обратном порядке, равна 1252. Найдите эти числа, если сумма цифр каждого равна 14, а сумма квадратов цифр равна 84. 488. Однозначное число увеличили на 10. Если теперь полученное число увеличить на столько же процентов, как и в первый раз, то получится 72. Найдите исходное однозначное число. 489. Сплав весит 2 кг и состоит из серебра и меди, причем масса серебра 2 составляет 14-=- % массы меди. Сколько серебра в сплаве? 490. Два завода по плану должны были выпустить за месяц 360 станков. Первый завод выполнил план на 112%, а второй — на 110%, вместе заводы выпустили за месяц 400 станков. Сколько станков сверх плана выпустил каждый завод в отдельности? 491. Для выпечки пшеничного хлеба взято столько килограммов муки, сколько процентов составляет припек на эту муку. Для выпечки ржаного хлеба взято на 10 кг муки больше, а именно столько килограммов, сколько процентов составляет припек на ржаную муку. Сколько взято той и другой муки, если выпечено всего 112,5 кг хлеба? 492. Бригада по плану должна выпустить 360 деталей. Первые восемь дней она перевыполняла дневной план на 20%. Оставшиеся дни она перевыполняла план на 25%. В результате бригада сделала на 82 детали больше, чем требовалось по плану. Сколько дней работала бригада? 493. В начале года на сберегательную книжку было положено 1600 р. и в конце года взято 848 р. В конце второго года на книжке оказалось 824 р. Сколько процентов начисляет сберкасса в год? 494. В конце года вкладчику на его сбережения сберкасса начислила проценты, что составило 6 р. Добавив 44 р., вкладчик оставил деньги еще на год. По истечении года вновь были начислены проценты, и теперь вклад вместе с процентами составил 257 р. 50 к. Какая сумма первоначально была положена на книжку? 495. Количество студентов в институте, увеличиваясь на одно и то же число процентов ежегодно, возросло за три года с 5000 до 6655 человек. На сколько процентов увеличивалось число студентов ежегодно? 496. Объем вещества А составляет половину суммы объемов веществ В и С, а объем вещества В составляет 20% суммы объемов веществ А и С. Найдите отношение объема вещества С к сумме объемов веществ А и В. 497. В результате реконструкции цеха число высвободившихся рабочих заключено в пределах от 1,7 до 2,3% от общего числа рабочих цеха. Найдите минимальное число рабочих, которое могло быть занято в цехе до реконструкции. 498. Число студентов курса, успешно сдавших все зачеты, заключено в пределах от 96,8 до 97,2% от общего числа студентов. Найдите минимальное число студентов, которое может быть на таком курсе. 499. Сколько членов геометрической прогрессии нужно сложить, чтобы получить сумму 3069, если ^+«5 = 51; U2 + и6=Ю2? 500. При делении девятого члена арифметической прогрессии на второй ее член в частном получается 5, а при делении тринадцатого члена этой прогрессии на ее шестой член в частном получается 2 и в остатке 5. Найдите сумму 20 членов этой прогрессии. 501. Сумма бесконечно убывающей геометрической прогрессии равна 4, а сумма кубоз ее членов равна 192. Найдите первый член и знаменатель прогрессии 502. Найдите четыре числа, первые три из которых образуют арифметическую 89
прогрессию, а последние три — геометрическую, сумма крайних чисел равна 66, а сумма средних — 60. 503. Сумма первых трех членов геометрической прогрессии равна 91. Если к этим членам прибавить соответственно 25, 27 и 1, то получатся три числа, образующие арифметическую прогрессию. Найдите седьмой член геометрической прогрессии. 504. Найдите трехзначное число, цифры которого образуют геометрическую прогрессию. Если из этого числа вычесть 792, то получится число, записанное теми же цифрами, но в обратном порядке. Если из цифры, выражающей число сотен, вычесть 4, а остальные цифры искомого числа оставить без изменения, то получится число, цифры которого образуют арифметическую прогрессию. 505. Найдите четырехзначное число, первые три цифры которого образуют возрастающую арифметическую прогрессию, если известно, что оно делится на 225. 506. Три брата, числа лет которых образуют геометрическую прогрессию, делят между собой некоторую сумму денег пропорционально возрасту. Если бы ту же сумму денег они разделили пропорционально своему возрасту через три года, то младший брат получил бы на 105 р. больше, а средний — на 15 р. больше, чем теперь. Сколько лет каждому из братьев, если известно, что разница в возрасте между старшим и младшим равна 15 годам? 507. Найдите число членов арифметической прогрессии, у которой отношение суммы первых 13 членов к сумме последних 13 членов равно 1/2, а отношение суммы всех членов без первых трех к сумме членов без последних трех равно 4/3. 1 ft 508. Сумма бесконечно убывающей геометрической прогрессии равна —. Про- 1 ^ грессия содержит член, равный —. Отношение суммы всех членов прогрессии, 1 предшествующих члену, равному —, к сумме членов, следующих за ним, равно 30. Определите номер члена, равного — о 509. Турист, поднимаясь в гору, в первый час достиг высоты 800 м, а в каждый следующий час поднимался на высоту, на 25 м меньшую, чем в предыдущий. За сколько часов он достигнет высоты 5700 м? 510. Алеша, Боря и Вася покупали блокноты и карандаши. Алеша купил 4 карандаша и 2 блокнота, Боря — 6 карандашей и блокнот, Вася — 3 карандаша и блокнот. Сколько стоит блокнот, если известно, что карандаш стоит 3 к, а суммы денег, потраченные Алешей, Борей и Васей, образуют в указанном порядке геометрическую прогрессию? 511. Коля, Петя, Миша и Ваня ловили рыбу. Оказалось, что количество рыб, пойманных Колей, Петей и Мишей, образует в указанном порядке геометрическую прогрессию. Если бы Коля поймал на 2 рыбы меньше, а Ваня — на 12 меньше, чем на самом деле, то количества рыб, пойманных Колей, Петей, Мишей и Ваней образовали бы в указанном порядке арифметическую прогрессию. Сколько рыб поймал Миша, если известно, что он поймал на 18 рыб меньше Вани? 512. Два насоса различной мощности, работая вместе, наполняют бассейн за 4 ч. Для заполнения половины бассейна первому насосу требуется времени на 4 ч больше, чем второму для заполнения трех четвертей бассейна. За какое время может наполнить бассейн каждый насос в отдельности? 513. Двое рабочих выполнили вместе некоторую работу за 12 ч. Если бы сначала первый рабочий сделал половину этой работы, а затем другой остальную часть, то вся работа была бы выполнена за 25 ч. За какое время мог бы выполнить эту работу каждый рабочий в отдельности? 514. Двое рабочих выполняют некоторую работу. После 45 мин совместной работы первый рабочий был переведен на другую работу, и второй рабочий закончил оставшуюся часть работы за 2 ч 15 мин. За какое время мог бы выполнить всю работу каждый рабочий в отдельности, если известно, что второму на это понадобится на 1 ч больше, чем первому? 515. Два токаря должны были изготовить определенное число деталей. После трехчасовой совместной работы продолжал работать только второй токарь, который проработал еще 4 ч. После этого задание оказалось перевыполненным на 12,5%. За 90
какое время мог бы выполнить задание каждый токарь, если известно, что второму на это понадобится на 4 ч меньше, чем первому? 516. Бассейн может наполниться водой из двух кранов. Если первый кран открыть на 10 мин, а второй — на 20 мин, то бассейн будет наполнен. Если первый кран открыть о на 5 мин, а второй — на 15 мин, то заполнится — бассейна За какое время из каждого о крана в отдельности может заполниться весь бассейн? 517. Две бригады работали вместе 15 дней, а затем к ним присоединилась третья бригада, и через 5 дней после этого вся работа была закончена. Известно, что вторая бригада вырабатывает за день на 20% больше первой. Вторая и третья бригады 9 вместе могли бы выполнить всю работу за т^ того времени, которое требуется для выполнения всей работы первой и третьей бригадами при их совместней работе. За какое время могли бы выполнить всю работу все три бригады, работая совместно? 518. Для разгрузки баржи выделены бригады грузчиков. Если ко времени, за которое первая бригада может разгрузить баржу, прибавить время, за которое вторая бригада может разгрузить баржу, то получится 12 ч. За сколько часов каждая бригада может разгрузить баржу, если разность этих часов составляет 45% всего времени, за которое обе бригады могут разгрузить баржу, работая совместно? 519. Для прокладки траншеи выделены два экскаватора разных типов. Время, необходимое первому экскаватору для прокладки траншеи, на 3 ч меньше времени, необходимого второму экскаватору для прокладки этой траншеи. Сколько часов требуется каждому экскаватору для прокладки траншеи, если сумма этих часов в -»=- раза больше времени, необходимого для прокладки траншеи при совместной работе? 520. Теплоход загружается подъемными кранами. Сначала в течение 2 ч работали четыре крана одинаковой мощности, затем к ним присоединились еще два крана, но меньшей мощности, и через 3 ч после этого погрузка была закончена. Если бы все краны начали работать одновременно, то погрузка была бы закончена за 4,5 ч. За сколько времени выполнят погрузку один кран большей и один кран меньшей мощности при совместной работе? 521. В котлован равномерно поступает вода. 10 одинаковых насосов, действуя одновременно, могут откачать воду из заполненного котлована за 12 ч, а 15 таких насосов — за 6 ч. За сколько времени могут откачать воду из заполненного котлована 25 таких насосов при совместной работе? 522. Две фабрики должны совместно переработать некоторое количество сырья. Если бы производительность второй фабрики повысилась вдвое, то время, необходимое фабрикам для выполнения работы, уменьшилось бы на — времени, необ- 15 ходимого для выполнения работы одной первой фабрикой. На какой фабрике производительность выше и во сколько раз, если известно, что каждая фабрика переработала не менее ~ всего объема сырья? о 523. Две бригады, работая совместно, вырыли траншею за 2 дня. После этого они начали рыть траншею той же глубины и ширины, но длиннее первой в 5 раз. Сначала работала только первая бригада, а затем только вторая бригада, выполнив в полтора раза меньший объем работы, чем первая бригада. Рытье второй траншеи было закончено за 21 день. За сколько дней вторая бригада смогла бы вырыть первую траншею, если известно, что объем работы, выполняемый первой бригадой за один день, больше объема работы, выполняемого за один день второй бригадой? 524. Резервуар снабжается водой по пяти трубам. Через первую трубу резервуар наполняется водой за 40 мин, через вторую, третью и четвертую трубы, работающие одновременно,— за 10 мин, через вторую, третью и пятую вместе — за 20 мин и, наконец, через пятую и четвертую — за 30 мин. За какое время наполнится резервуар при одновременной работе всех пяти труб? 525. Три автоматические линии выпускают одинаковую продукцию, но имеют разную производительность. Производительность всех трех одновременно действующих линий в 1,5 раза выше производительности первой и второй линий, работаю- 91
щих одновременно. Сменное задание для первой линии вторая и третья линии, работая одновременно, могут выполнить на 4 ч 48 мин быстрее, чем его выполняет первая линия; это же задание вторая линия выполняет на 2 ч быстрее по сравнению с первой линией. Найдите время выполнения сменного задания первой линией. 526. Два трактора вспахивают поле, разделенное на две равные части. Оба трактора начали работать одновременно, причем каждый на своей половине. Через 5 ч после того момента, когда они совместно вспахали половину всего поля, выяснилось, что первому трактору осталось вспахать — часть своего участка, а вто- 3 рому своего. Сколько времени понадобится второму трактору, чтобы вспахать о поле? 527. Три тракторные бригады вместе вспахивают поле за 4 дня. Это же поле первая и вторая бригады вместе могут вспахать за 6 дней, а первая и третья вместе — за 8 дней. Во сколько раз площадь, вспахиваемая за день второй бригадой, больше, чем площадь, вспахиваемая за день третьей бригадой? 528. Танкер заполняется нефтью при работе двух труб, причем каждая из них заполнила более 0,25 его объема. Если бы количество нефти, поступающее в час через первую трубу, было в 1,5 раза больше, а количество нефти, поступающее через вторую трубу, было бы в 4 раза меньше, то время, необходимое для заполнения танкера, увеличилось бы на — часть того времени, которое необходимо для заполнения танкера через одну первую трубу. Через какую трубу поступает нефти больше и во сколько раз? 529. Три экскаватора заняты на рытье котлована. Разность производительности первого и третьего экскаваторов в 3 раза больше разности производительности третье- 4 го и второго экскаваторов. Первый экскаватор выполняет — всей работы за не- о которое время. Такое же время потребуется, если сначала второй экскаватор выпол- 1 9 нит — всей работы, а затем третий экскаватор ^ оставшейся работы. Во сколь- 10 Zo ко раз производительность первого экскаватора больше производительности второго? 530. Одну и ту же работу могут выполнить три бригады. Первая бригада выпол- 2 няет — всей работы за некоторое время. Такое же время потребуется, если сна- о чала третья бригада выполнит — всей работы, а затем вторая бригада выполнит «3 9 jt оставшейся работы. Производительность третьей бригады равна полусумме производительностей первой и второй бригад. Во сколько раз производительность второй бригады больше производительности третьей бригады? 531. Две бригады штукатуров, работая совместно, оштукатурили жилой дом за 6 дней. В другой раз они оштукатурили клуб и выполнили втрое больший объем работы, чем на штукатурке жилого дома. В клубе сначала работала первая бригада, а затем ее сменила вторая бригада и довела работу до конца, причем первая бригада выполнила объем работы, вдвое больший, чем вторая. Клуб они оштукатурили за 35 дней. За сколько дней первая бригада смогла бы оштукатурить жилой дом, если известно, что вторая бригада потратила бы на это более 14 дней? 532. На угольной шахте сначала работали два участка, а через некоторое время вступил в строй третий участок, в результате чего производительность шахты увеличилась в полтора раза. Сколько процентов составляет производительность второго участка от производительности первого, если за 4 месяца первый и третий участки вместе выдают столько же угля, сколько выдает второй участок за весь год? 533. Нефть в резервуар поступает по трем трубам и выкачивается по четвертой. В первый день третья и четвертая трубы работали по 6 ч, вторая — 5 ч, первая — 2 ч. В результате уровень нефти повысился на 4 м. Во второй день первая и вторая трубы работали по 3 ч, третья — 9 ч, четвертая — 4 ч. В результате уро- 92
вень поднялся еще на 6 м. В третий день в течение 6 ч работали вторая и четвертая трубы. Поднялся или опустился уровень нефти в третий день? 534. При смешивании 40%-ного раствора кислоты с 10%-ным раствором кислоты получили 800 г 21,25%-ного раствора. Сколько граммов каждого раствора было для этого взято? 535. Имеется 735 г 16%-ного раствора йода в спирте. Нужно получить 10%-ный раствор йода. Сколько граммов спирта нужно прибавить к имеющемуся раствору? 536. Имеется сталь двух сортов, один из которых содержит 5%, а другой 10% никеля. Сколько тонн каждого из этих сортов нужно взять, чтобы получить сплав, содержащий 8% никеля, если во втором куске никеля на 4 т больше, чем в первом? 537. В 500 кг руды содержится некоторое количество железа. После удаления из руды 200 кг примесей, содержащих в среднем 12,5% железа, содержание железа в оставшейся руде повысилось на 20%. Сколько железа осталось в руде? 538. Свежие огурцы, содержащие 98% воды, весили 100 кг. Когда огурцы немного усохли, то воды в них стало 96%. Сколько стали весить огурцы после усыхания? 539. Из 40 т руды выплавляют 20 т металла, содержащего 6% примесей. Каков процент примесей в руде? 540. Из 38 т сырья второго сорта, содержащего 25% примесей, после переработки получается 30 т сырья первого сорта. Каков процент примесей в сырье первого сорта? 541. Свежие грибы содержат 90% воды, а сушеные— 12%. Сколько получится сушеных грибов из 88 кг свежих? 542. Пчелы, перерабатывая цветочный нектар в мед, освобождают его от значительной части воды. Сколько килограммов нектара приходится перерабатывать пчелам для получения 1 кг меда, если известно, что нектар содержит 70% воды, а полученный из него мед — 17% воды? 543. Два металла содержатся в каждом из двух взятых сплавов. В первом сплаве металлы находятся в отношении 1:2, а во втором — в отношении 3:2. В каком отношении нужно взять части этих сплавов, чтобы получился новый сплав с отношением металлов 8:7? 544. Имеются два раствора кислоты разной концентрации. Объем одного раствора — 4 л, другого — 6 л. Если нх слить вместе, то получится 35%-ный раствор кислоты. Если же слить равные объемы этих растворов, то получится 36%-ный раствор кислоты. Сколько литров кислоты содержится в каждом из первоначальных растворов? 545. 40 кг раствора соли разлили в два сосуда так, что во втором сосуде чистой соли оказалось на 2 кг больше, чем в первом сосуде. Если во второй сосуд добавить 1 кг смеси, то количество соли в нем будет в 2 раза больше, чем в первом сосуде. Найдите массу раствора, находящегося в первом сосуде. 546. Имеются два раствора соли в воде, первый 40%-ный, второй 60%-ный. Их смешали, добавили 5 кг воды и получили 20%-ный раствор. Если бы вместо 5 кг воды добавили 5 кг 80%-ного раствора, то получился бы 70%-ный раствор. Сколько было 40%-ного и 60%-ного растворов? 547. Имеются три слитка. Масса первого 5 кг, второго — 3 кг, и каждый из них содержит 30% меди. Если первый слиток сплавить с третьим, то получится слиток, содержащий 56% меди, а если второй слиток сплавить с третьим, то получится слиток, содержащий 60% меди. Найдите массу третьего слитка и процентное содержание меди в нем. 548. Имеются два слитка золота с серебром. Процентное содержание золота в первом слитке в 2,5 раза больше, чем во втором слитке. Если сплавить оба слитка вместе, то получится слиток, в котором будет 40% золота. Во сколько раз первый слиток тяжелее второго, если известно, что при сплаве равных по массе частей первого и второго слитков получится слиток, в котором содержится 35% золота? 549. Сплав меди с серебром содержит меди на 2 кг больше, чем серебра. Если g к сплаву добавить -г^ того количества серебра, которое в нем содержится, то 1Ь процентное содержание серебра в новом сплаве будет равно процентному содержанию меди в первоначальном сплаве. Найдите массу первоначального сплава. 550. Из двух жидкостей первая имеет температуру а°, а вторая — Ь°. Если смешать некоторое количество первой и второй жидкостей, то полученная смесь будет 93
иметь температуру с0. Какова будет температура смеси, если взять первой жидкости столько, сколько предполагалось взять второй, а второй — столько, сколько предполагалось взять первой? 551. Сплавляя два одинаковых по массе куска чугуна с разным содержанием хрома, получили сплав, в котором содержалось 12 кг хрома. Если бы масса первого куска была бы в 2 раза больше, то в сплаве содержалось бы 16 кг хрома. Известно, что содержание хрома в первом куске на 5% меньше, чем во втором. Найдите процентное содержание хрома в каждом куске чугуна. 552. Имеются два слитка, представляющие собой сплавы цинка с медью. Масса первого слитка 2 кг, второго — 3 кг. Их сплавили вместе с 5 кг сплава цинка с медью, в котором цинка было 45%, и получили сплав цинка с медью, в котором цинка стало 50%. Если бы процентное содержание цинка в первом сплаве было таким, как процентное содержание цинка во втором, и, наоборот, процентное содержание цинка во втором — таким, как оно было в первом, то, сплавив эти два слитка с 5 кг сплава, в котором содержание цинка 60%, мы бы получили сплав, в котором цинка содержится 55%. Каково процентное содержание цинка в первом и во втором слитках? 553. Имеются два слитка, состоящие из цинка, меди и олова. Известно, что первый слиток содержит 40% олова, а второй — 26% меди. Процентное содержание цинка в первом и втором слитках одинаково. Сплавив 150 кг первого слитка и 250 кг второго, получим сплав, в котором оказалось 30% цинка. Сколько килограммов олова содержится в получившемся новом сплаве? 554. Сосуд емкостью 12 л наполнен кислотой. Из него выливают некоторое количество кислоты во второй сосуд такой же емкости и второй сосуд дополняют водой. Теперь смесью из второго сосуда дополняют первый сосуд. Затем из первого сосуда переливают 4 л во второй, после чего в обоих сосудах количество чистой кислоты (в растворах) оказывается одинаковым. Сколько кислоты первоначально перелито из первого сосуда во второй? 555. В сосуд с водой налили 6 л 64%-ного раствора спирта, а затем после полного перемешивания вылили 6 л получившегося раствора. Такую операцию повторили 3 раза. Сколько воды было первоначально в сосуде, если известно, что объем выражается целым числом литров, не превышающим 10 л, и что окончательная концентрация спирта в нем стала равной 56%? 556. В куске сплава массой 6 кг содержится медь. В куске другого сплава массой 8 кг содержится медь в процентном отношении, вдвое меньшем, чем в первом куске. От первого куска отделили некоторую часть, а от второго куска отделили часть по массе, вдвое большую, чем от первого куска. Каждую из этих частей сплавили с остатком другого куска, посте чего получилось два новых сплава с одинаковым процентным содержанием меди. Какова масса каждой из частей, отделенных от кусков первоначально? 557. Из сосуда, наполненного глицерином, отлили 2 л глицерина, а к оставшемуся глицерину долили 2 л воды. После перемешивания отлили 2 л смеси и долили 2 л воды. Наконец, опять перемешали смесь и отлили от нее 2 л, а долили 2 л воды. В результате этих операций объем воды в сосуде стал на 3 л больше объема оставшегося в нем глицерина. Сколько литров глицерина и воды оказалось в сосуде в результате всех переливаний? 558. Из двух баков первый заполнен чистым глицерином, а второй — водой. Взяли два трехлитровых ковша. Первым ковшом зачерпнули содержимое первого бака, а вторым ковшом — содержимое второго бака, после чего первый ковш влили во второй бак, а второй ковш — в первый бак. Затем после перемешивания такую операцию повторили еще раз, в результате чего чистый глицерин занял половину объема первого бака. Найдите объемы баков, если известно, что их суммарный объем в 10 раз больше объема первого бака. 559. Туристу надо пройти расстояние от деревни до станции. Пройдя 3 км за час, он понял, что опоздает на поезд, и пошел со скоростью 4 км/ч. На станцию он пришел за 45 мин до отхода поезда. Если бы он шел с первоначальной скоростью, то опоздал бы на поезд на 40 мин. Определите расстояние от деревни до станции. 560. Пассажир, сидящий в поезде, который идет со скоростью 40 км/ч, заметил, что мимо окна в противоположном направлении в течение 3 с прошел встречный поезд, длина которого 75 м. Какова скорость встречного поезда? 94
561. Велосипедист должен был проехать 48 км с определенной средней скоростью. Но по некоторым причинам первую половину пути он ехал со скоростью, на 20% меньшей, а вторую половину пути — на 2 км большей, чем ему полагалось. На весь путь велосипедист затратил 5 ч. Найдите предполагаемую вначале скорость. 562. Три тела движутся по прямой линии от точки Л к точке В. Второе тело начало двигаться на 5 с, а третье — на 8 с позже первого. Скорость первого тела меньше скорости второго на 6 см/с. Скорость третьего тела равна 30 см/с. Найдите расстояние АВ и скорость первого тела, если известно, что все три тела достигают точки В в один и тот же момент. 563. Самолет летел сначала со скоростью 220 км/ч. Когда ему осталось лететь на 385 км меньше, чем он пролетел, скорость его стала равной 330 км/ч. Средняя скорость самолета на всем пути равна 250 км/ч. Какое расстояние пролетел самолет? 564. Из пунктов А и В одновременно навстречу друг другу вышли два поезда. Скорость первого поезда на 10 км/ч больше скорости второго. Поезда встретились в 28 км от середины пути АВ. Если бы первый поезд отправился из А на 45 мин позже второго, то поезда встретились бы в середине пути АВ. Найдите расстояние АВ и скорости обоих поездов. 565. Два школьника вышли одновременно из дома в школу с одинаковой скоростью. Через 3 мин один из товарищей вспомнил, что забыл дома нужную книгу, и побежал обратно со скоростью, большей первоначальной на 60 м/мин. Взяв книгу, он побежал обратно с такой же скоростью и догнал товарища, который шел с постоянной скоростью, уже у дверей школы. Найдите скорости учеников, если расстояние от школы до дома равно 280 м 566. Два пешехода, находящиеся в пунктах А и В, расстояние между которыми равно 27 км, выходят из этих пунктов одновременно. Они встречаются через 3 ч, если идут навстречу друг другу, и один догоняет другого через 9 ч, если они идут в одном направлении. Найдите скорость каждого пешехода. 567. По двум сторонам прямого угла по направлению к его вершине движутся два тела. В начальный момент тело А отстояло от вершины прямого угла на 60 м, а тело В — на 80 м. Через 3 с расстояние между А и В стало равным 70 м, а еще через 2 с — 50 м. Найдите скорости каждого тела. 568. Из пунктов Л и Б, находящихся друг от друга на расстоянии 120 км, к пункту С ведут две дороги, причем Z-ЛСВ =60°. Из Л в С выехал грузовик со скоростью 40 км/ч, одновременно с ним из В в С выехал автобус со скоростью 60 км/ч. Найдите время движения обеих машин до пункта С, если известно, что автобус прибыл в С на час раньше грузовика. 569. Расстояние между двумя городами по реке равно 80 км. Катер проходит это расстояние дважды (вверх и вниз) за 8 ч 20 мин. Определите скорость катера в стоячей воде, если скорость течения реки равна 4 км/ч. 570. Катер прошел против течения реки 8 км, повернул обратно и прошел по течению 36 км. Весь рейс длился 2 ч. Потом катер прошел против течения реки 6 км и по течению 33 км, затратив на этот второй рейс 1 ч 45 мин. Найдите скорость катера в стоячей воде. 571. В озеро впадают две реки. Катер отплывает от пристани Л, находящейся на первой реке, плывет 24 км до озера, далее плывет 2 ч по озеру и затем 32 км по второй реке до пристани В, затратив 8 ч на весь путь от Л до В. Если бы катер проплыл по озеру еще дополнительно 18 км, то на весь путь от Л до В он затратил бы 10 ч. Найдите скорость течения каждой реки, если известно, что скорость течения первой реки на 2 км/ч больше скорости течения второй реки. 572. Два пешехода вышли одновременно из пунктов А и В навстречу друг другу. Когда первый прошел половину пути, второму до конца пути оставалось еще 24 км. Когда второй прошел половину пути, первому до конца оставалось еще 15 км. Сколько километров останется пройти второму пешеходу до Л после того, как первый закончит переход из пункта Л в пункт В? 573. Из пунктов Л и В вышли навстречу друг другу два поезда, причем второй поезд вышел на полчаса позже первого. Через 2 ч после выхода первого поезда расстояние между ними составляло 19/30 расстояния между Л и В. Поезда встретились на середине АВ. За сколько часов каждый поезд пройдет путь А В? 574. Расстояние между городами Л и В равно 60 км. Два поезда выходят одновременно: один из Л в В, другой из В в Л Пройдя 20 км, поезд, идущий из Л в В, оста- 95
навливается на полчаса, затем, пройдя 4 мин, встречает поезд, идущий из В Оба поезда прибывают к месту назначения одновременно Найдите скорости поездов 575. Два велосипедиста выехали одновременно из пунктов Л и В навстречу друг другу. Велосипедист, ехавший из Л, прибыл в В через 4 ч после встречи, а велосипедист, ехавший из В, прибыл в А через 9 ч после встречи Сколько часов был в пути каждый велосипедист? 576. Из пункта А вверх по реке отправилась моторная лодка, а из пункта В, находящегося выше пункта А, одновременно вышел плот. Через а ч они встретились и далее двигались без остановок. Дойдя до В, лодка, не задерживаясь, повернула обратно и догнала плот в пункте Л. Сколько времени плыли плот и лодка до встречи в пункте Л? 577. Расстояние между двумя городами скорый поезд проходит на 4 ч быстрее товарного и на 1 ч быстрее пассажирского. Известно, что скорость товарного поезда составляет — скорости пассажирского и на 50 км/ч меньше скорости скоро- о го поезда. Найдите скорости товарного и скорого поездов. 578. Из двух пунктов, расстояние между которыми равно 2400 км, навстречу друг другу вышли одновременно пассажирский и скорый поезда. Каждый из них идет с постоянной скоростью, и в некоторый момент времени они встречаются. Если бы оба поезда шли со скоростью скорого поезда, то их встреча произошла бы на 3 ч раньше фактического момента встречи. Если бы оба поезда шли со скоростью пассажирского поезда, то их встреча произошла бы на 5 ч позже фактического момента встречи. Найдите скорости поездов 579. Теплоход отплыл из порта А в порт Б, и через 7,5 ч вслед за ним из А вышел катер. В середине пути из А в В катер догнал теплоход. Когда катер прибыл в В, теплоходу оставалось плыть 3/10 всего пути. Сколько времени требуется теплоходу на прохождение расстояния от А до В? 580. Из пункта А в пункт В вышел пассажирский поезд. Через 3 ч вслед за ним из А вышел скорый поезд. Скорый поезд догнал пассажирский в середине пути из Л в В. В момент прибытия скорого поезда в В пассажирский поезд прошел 13/16 всего пути. Сколько времени требуется пассажирскому поезду на прохождение расстояния от Л до В? 581. Из п>нкта Л в пункт В вышел пешеход. Через 3/4 ч вслед за ним выехал велосипедист. Когда велосипедист прибыл в пункт В, пешеходу оставалось пройти — всего пути. Сколько времени потратил пешеход на весь путь, если известно, что о велосипедист догнал пешехода в середине пути из Л в В? 582. Из пункта Л в пункт В, расстояние между которыми равно 70 км, выехал велосипедист, а через некоторое время — мотоциклист, скорость которого 50 км/ч. Мотоциклист догнал велосипедиста в 20 км от пункта Л. Прибыв в В, мотоциклист через 48 мин выехал обратно в Л и встретился с велосипедистом спустя 2 ч 40 мин после выезда велосипедиста из Л. Найдите скорость велосипедиста. 583. От пристани Л вниз по течению реки одновременно отошли катер и плот. Катер, доплыв до пристани В, расположенной в 324 км от Л, после 18-часовой стоянки отправился обратно в Л. В тот момент, когда он находился в 180 км от пристани Л, второй катер, отплывший из Л на 40 ч позднее первого, догнал плот, успевший к этому времени проплыть 144 км. Найдите скорости обоих катеров, если известно, что они равны, и скорость течения реки. 584. В реку впадает приток. Катер отходит от пристани Л, расположенной на притоке, идет вниз по течению 60 км до реки, далее по реке вниз по течению 65 км до пристани В, Затем по тому же маршруту катер возвращается, затратив на обратный путь 10 ч. Найдите собственную скорость катера, если известно, что на путь от А до реки катер тратит 3 ч 45 мин и скорость течения реки на 1 км/ч меньше скорости течения притока. 585. Два пловца стартовали один за другим в пятидесятиметровом бассейне на дистанцию 100 м. Второй пловец, скорость которого 1,5 м/с, догнал первого на отметке 21 м, затем, доплыв до противоположной стенки бассейна, повернул обратно и встретил первого пловца через 2/3 с после поворота. Найдите интервал времени между моментами старта пловцов. 96
586. Из пункта Л в одном и том же направлении вышли два лыжника, причем второй стартовал на 6 мин позже первого и догнал первого в 2 км от старта Дойдя до отметки 5 км, второй лыжник повернул обратно и встретил первого в 4 км от старта. Найдите скорость второго лыжника. 587. Два велосипедиста стартовали один за другим с интервалом в 2 мин. Второй велосипедист догнал первого на расстоянии 1 км от старта. Если бы, проехав от старта 5 км, он повернул обратно, то встретился бы с первым велосипедистом через 20 мин после его старта. Найдите скорость второго велосипедиста. 588. Одновременно из Л в Б выезжает велосипедист и выходит пешеход. Скорость велосипедиста вдвое больше скорости пешехода. Одновременно навстречу им из В в Л выходит второй пешеход. Время между встречами второго пешехода с велосипедистом и первым пешеходом составляет 2/15 от времени его перехода из В в А. Какой из пешеходов и во сколько раз шел быстрее, если оба они прошли до встречи больше !Д всего пути из Л в В? 589. Из пункта А в пункт В отправился теплоход. В 8 ч он догнал движущуюся по тому же маршруту лодку, скорость которой равна 3 км/ч. Возвращаясь в А из В после 10-минутной стоянки там, теплоход встречается с той же лодкой в 8 ч 20 мин. В пункт А он прибывает в то время, когда лодка прибывает в пункт В. Определите время прибытия лодки в пункт В, если известно, что в 8 ч 10 мин она находилась в 1,5 км от пункта А. 590. Из пункта А в пункт В вышел пешеход. Одновременно с ним из В в А выехал велосипедист, встретивший пешехода через 50 мин после своего выезда из В. Сколько времени потребовалось бы пешеходу для того, чтобы пройти весь путь из А в В, если известно, что велосипедист проделал бы тот же путь на 4 ч быстрее пешехода? 591. Из пункта А в пункт В одновременно отправляются пешеход и велосипедист. Доехав до В, велосипедист поворачивает обратно и через 1 ч после начала движения встречает пешехода. После встречи пешеход продолжает идти в В, а велосипедист поворачивает и едет тоже в В. Доехав до В, велосипедист снова поворачивает обратно и встречает пешехода через 40 мин после первой встречи. Определите, за какое время пешеход пройдет расстояние от Л до В. 592. Из пункта А выехали три велосипедиста. Первый выехал на 1 ч раньше двух других, стартовавших одновременно. Через некоторое время третий велосипедист догнал первого, а второй догнал первого на 2 ч позже, чем третий. Определите отношение скоростей первого и третьего велосипедистов, если отношение скорости второго к скорости третьего велосипедиста равно 2:3. 593. Два пешехода вышли одновременно навстречу друг другу из пунктов А и В. Каждый идет с постоянной скоростью без остановок и, придя в конечный пункт, тут же поворачивает обратно. Когда они встретились во второй раз, оказалось, что первый прошел на 4 км больше, чем второй. После второй встречи первый прибыл в А через час, а второй — в В через 2,5 ч. Найдите скорости пешеходов. 594. Два приятеля собрались на охоту. Один из них живет в 46 км от охотничьей базы, а другой, имеющий машину, в 30 км от базы между базой и домом приятеля (на одной прямой). Они отправились в путь одновременно, причем владелец машины поехал навстречу приятелю, посадил его в машину и с той же скоростью поехал на базу; вся поездка заняла 1 ч. Если бы пешеход вышел из дома на 2 ч 40 мин раньше, то встреча произошла бы в 11 км от дома пешехода. С какой скоростью едет автомобиль и движется пешеход? 595. Из пункта А в пункт В вышел пешеход. Одновременно из В в А навстречу ему выехал мотоциклист. Встретив пешехода, мотоциклист посадил его на мотоцикл, привез в В, высадил и тут же снова поехал в Л. В итоге пешеход добрался до В в 4 раза быстрее, чем намечал. Во сколько раз быстрее прибыл бы мотоциклист в пункт Л, если бы ему не пришлось возвращаться? 596. Из пункта Л в пункт В доставлен груз. Из Л его везли сначала в автофургоне, а потом на грузовике. Расстояние от места перегрузки до пункта В в 3 раза меньше, чем от места перегрузки до пункта Л. Для доставки груза из Л в В потребовалось время, равное времени проезда из Л в В со скоростью 64 км/ч. С какой скоростью ехал грузовик, если известно, что скорость автофургона не более 75 км/ч, а также что если бы автофургон и грузовик выехали навстречу друг другу из пунктов Л и В, то они встретились бы через промежуток времени, необходимый для проезда из Л в В со скоростью 120 км/ч? 97
597. Два велосипедиста выехали одновременно навстречу друг другу из пунктов Л и Б и встретились через 2,4 ч. Если бы первый велосипедист увеличил скорость на 50%, а второй — на 20%, то на преодоление расстояния между Л и В первому велосипедисту понадобилось бы на 2/3 ч больше, чем второму. За какое время проходит расстояние между А и В каждый велосипедист? 598. Из пункта А в пункт В выехал мотоциклист. Через 2 ч вслед за ним выехал автомобиль, который прибыл в В одновременно с мотоциклистом. Если бы автомобиль и мотоциклист одновременно выехали из Л и В навстречу друг другу, то они встретились бы через 1 ч 20 мин после выезда. За какое время проходит путь из Л в В мотоциклист? 599. Из пункта А в пункт В выехал велосипедист. Одновременно с ним из В в Л выехал мотороллер и встретил велосипедиста через 45 мин после выезда. За какое время проходит путь из Л в В велосипедист, если известно, что мотороллер проходит тот же путь на 2 ч быстрее? 600. На путь из Л в В теплоход затрачивает 3 ч, на обратный путь — 4 ч. Сколько времени будет плыть плот из Л в В? 601. Монтер сбежал по ленте движущегося эскалатора за 30 с. Второй раз он спустился по неподвижной ленте за 45 с. За сколько времени он спустился бы, стоя на ступеньке движущегося эскалатора? 602. Из двух пунктов Л и В одновременно навстречу друг другу выехали велосипедист и автобус. На путь из Л в В велосипедист тратит на 2 ч 40 мин больше, чем тратит автобус на проезд из В в Л, а сумма этих часов в 16/3 раза больше времени, прошедшего от начала движения велосипедиста и автобуса до момента их встречи. Какое время велосипедист затрачивает на проезд из Л в В, а автобус — на проезд из В в Л? 603. Из пункта Л в пункт В доставлена почта. Сначала ее вез мотоциклист, 2 который, проехав — расстояния от Л до В, передал ее ожидавшему его велоси- 6 педисту. Почта была доставлена из Л в В за время, которое необходимо, чтобы проехать из Л в В со скоростью 40 км/ч. Известно, что если бы мотоциклист и велосипедист выехали из Л в В одновременно навстречу друг другу, то они встретились бы через промежуток времени, необходимый для проезда из Л в В со скоростью 100 км/ч. Найдите скорость мотоциклиста, счиая, что она больше скорости велосипедиста. 604. По окружности длиной 360 м движутся две точки, причем первая проходит окружность на 1 с быстрее. Найдите скорость каждой точки, если известно, что первая проходит за 1 с на 4 м больше, чем вторая. 605. Две точки, двигаясь по окружности в одном направлении, встречаются через каждые 56 мин, двигаясь в противоположных направлениях — через каждые 8 мин. Найдите скорость каждой точки и длину окружности, если известно, что за 1 с первая точка проходит на -рг м больше, чем вторая. 606. Две точки, двигаясь по окружности в одном направлении, встречаются через каждые 12 мин, причем первая обходит окружность на 10 с быстрее, чем вторая. Какую часть окружности проходит за 1 с каждая точка? 607. Два тела, двигаясь по окружности в одном направлении, встречаются через каждые 112 мин, а двигаясь в нротивоположных направлениях — через каждые 16 мин. Во втором случае расстояние между телами уменьшилось с 40 м до 26 м за 12 с. Сколько метров в минуту проходит каждое тело и какова длина окружности? 608. Три велосипедиста, стартовав одновременно с одного места и в одном направлении, едут по кругу длиной 1 км. Через некоторое время второй догоняет первого, проехав на один круг больше. 4 мин спустя в ту же точку прибывает третий, проехав такое же расстояние, какое проехал первый к моменту встречи со вторым. Скорости велосипедистов в некотором порядке образуют арифметическую прогрессию с разностью 5 км/ч. Найдите эти скорости. 609. На беговой дорожке состязались два спортсмена на дистанции 2 км. Когда победитель финишировал, второму оставалось пробежать еще целый круг. Какова длина беговой дорожки, если первый проходит круг на 5 с быстрее второго и заканчивает дистанцию за 3 мин? 610. Из пункта Л в пункт В выходит поезд. В момент прибытия его в В оттуда 98
выходит второй поезд, который следует в Л. Время, которое прошло от выхода первого поезда из Л до прибытия туда второго поезда, в 4— раза превышает время, которое затратили бы поезда до момента встречи, если бы вышли из А в В одновременно навстречу друг другу. Чему равна скорость каждого поезда, если скорость второго на 20 км/ч больше скорости первого? 611. Если пассажир из пункта А поедет поездом, то в пункт В он доберется за 20 ч. Если он полетит самолетом, которого ему придется ждать более 2 ч, то в пункт В он прибудет через 10 ч после выхода поезда. Во сколько раз скорость 8 самолета больше скорости поезда, если известно, что самолет через — ч после вылета окажется на таком же расстоянии от пункта Л, что и поезд? 612. Пункты Л, В, С удалены от пункта М соответственно на 60, 55 й 56 км. Одновременно из этих пунктов в пункт М вышли три пешехода: первый из Л, второй из В, третий из С. Первый прошел весь путь с постоянной скоростью и прибыл в М на 2 ч раньше второго и третьего, прибывших одновременно. Второй пешеход первые 40 км шел с той же скоростью, что первый, потом на час остановился, а после остановки шел со скоростью, которая меньше скорости третьего пешехода на столько же, на сколько скорость третьего меньше скорости первого. Третий пешеход весь путь шел с постоянной скоростью. Определите скорости первого и третьего пешеходов. 613. Две бригады начали работу в 8 ч. Сделав вместе 72 детали, они стали работать раздельно. В 15 ч выяснилось, что за время раздельной работы первая бригада сделала на 8 деталей больше, чем вторая. На другой день первая бригада делала за 1 ч на одну деталь больше, а вторая бригада за I ч на одну деталь меньше, чем в первый день. Работу бригады начали вместе в 8 ч и, сделав 72 детали, снова стали работать раздельно. Теперь за время раздельной работы первая бригада сделала на 8 деталей больше, чем вторая, уже к 13 ч. Сколько деталей в час делала каждая бригада? 614. Трое рабочих должны сделать 80 одинаковых деталей. Известно, что все трое вместе делают за час 20 деталей. К работе приступил сначала первый рабочий. Он сделал 20 деталей, затратив на их изготовление более 3 ч. Оставшуюся часть работы выполняли вместе второй и третий рабочие. На всю работу ушло 8 ч. Сколько часов потребовалось бы первому рабочему на изготовление всех 80 деталей? 615. Бассейн заполняется водой через первую трубу на 5 ч быстрее, чем через вторую трубу, и на 30 ч быстрее, чем через третью трубу. Известно, что пропускная способность третьей трубы в 2,5 раза меньше пропускной способности первой трубы и на 24 м3/ч меньше пропускной способности второй трубы. Найдите пропускную способность первой и третьей труб. 616. Два экскаватора, из которых первый имеет меньшую производительность, вырыли при совместной работе котлован объемом 240 м3. Потом первый стал рыть второй котлован, а второй продолжал рыть первый. Через 7 ч после начала их работы объем первого котлована оказался на 480 м3 больше объема второго котлована. На другой день второй экскаватор увеличил свою производительность на 10 мэ/ч, а первый уменьшил на 10 м3/ч. Сначала они вместе вырыли котлован в 240 м3, после чего первый стал рыть другой котлован, а второй продолжал рыть первый. Теперь объем первого котлована стал на 480 м3 больше объема второго котлована уже через 5 ч после начала работы экскаваторов. Сколько грунта в час вынимали экскаваторы в первый день работы? 617. Три сенокосилки участвовали в покосе травы с поля площадью 25 га. За час первая скашивает 3 га, вторая — на а га меньше первой, а треть"я — на 2а га больше первой. Сначала работали вместе первая и вторая сенокосилки и скосили 11 га. Оставшуюся часть поля скосили вместе первая и третья. При каком значении параметра а (0<а<1) все поле будет скошено за 4 ч, если работу вести без перерыва? 618. Три экскаватора роют котлован объемом 340 м3. Первый вынимает за час 40 м3 грунта, второй — на а м3 меньше первого, а третий — на 2а м3 больше первого. Сначала работали вместе первый и второй и выкопали 140 м3 грунта. Оставшуюся часть котлована выкопали вместе первый и третий экскаваторы. При каком значении параметра а (0<а< 15) работа была закончена за 4 ч, если известно, что она велась без перерыва? 619. Три бригады работают с постоянной производительностью каждая, прокла- 99
дывая рельсовые пути. Первая и третья бригады совместно укладывают в месяц 15 км путей. Три бригады в месяц уложат путей в 2 раза больше, чем первая и вторая при совместной работе. Сколько километров путей укладывает в месяц третья бригада, если известно, что она совместно со второй бригадой уложила некоторый участок пути в 4 раза быстрее, чем его уложила бы одна вторая бригада? 620. Три автомашины перевозят зерно, загружаясь в каждом рейсе полностью. За один рейс первая и вторая машины перевозят вместе 6 т зерна, а первая и третья вместе за 2 рейса перевозят столько же зерна, сколько вторая за 3 рейса. Какое количество зерна перевозит за один рейс вторая автомашина, если известно, что некоторое количество зерна вторая и третья перевозят вместе, совершая в 3 раза меньше рейсов, чем потребовалось бы третьей автомашине для перевозки того же количества зерна? 621. На заводе сначала работало 2 цеха, затем был пущен третий цех, в результате чего завод увеличил выпуск ежемесячной продукции в 1,6 раза. Во сколько раз больше продукции дает третий цех по сравнению со вторым, если известно, что за 2 месяца первый и третий цеха вместе выпускают столько же продукции, сколько второй за полгода? 622. Из города Л в город В выехал автомобиль. Одновременно с ним из города С, расположенного между Л и В, в город А выехал второй автомобиль. Первый в В и второй в А приехали одновременно. Затем оба одновременно выехали навстречу друг другу, встретились в пункте D и одновременно прибыли первый в Л, второй в В. Первый сделал остановку на пути от В к £>, а второй — остановку той же длительности, но на пути от С к Л. Найдите расстояние между С и D, если известно, что расстояние от А до С равно 270 км, а расстояние от С до В равно 180 км. 623. На полпути между городами М и N, расстояние между которыми равно 280 км, расположен поселок Р. Из N я Р одновременно выехали навстречу друг другу автобус и грузовик: автобус из М, грузовик из Р. Автобус прибыл в N тогда, когда грузовик прибыл в М. Затем обе машины одновременно выехали навстречу друг другу, встретились в пункте В и прибыли одновременно автобус в М\ грузовик в N. Найдите расстояние от М до В, если известно, что автобус и грузовик сделали остановки одинаковой длительности: автобус на пути от N к В, а грузовик на пути от Р к М. 624. Города А, В, С, D, расположенные так, что четырехугольник ABCD выпуклый, соединены прямолинейными дорогами Л В, ВС, CD, DA и АС. Их длины соответственно равны 6, 14, 5, 15 и 15 км. Из одного из этих городов одновременно вышли три туриста, идущие без остановок с постоянными скоростями. Маршруты всех туристов различны, причем каждый из них состоит из трех дорог и проходит через все города. Первый и второй туристы перед прохождением третьих дорог своих маршрутов встретились в одном городе, а третий закончил маршрут на час раньше туриста, закончившего маршрут последним. Найдите скорости туристов, если известно, что скорость третьего больше скорости второго и на 0,5 км/ч меньше скорости первого, причем скорости всех туристов заключены в интервале от 5 до 8 км/ч. 625. Города Л, В, С, D, расположенные так, что город С находится внутри треугольника ABD, соединены прямолинейными дорогами АВ, BD, DA, АС, ВС. Их длины соответственно равны 400, 600, 300, 80 и 400 км. Из одного города одновременно выехали три автомобиля, едущие без остановок с постоянными скоростями, заключенными в интервале от 95 до 125 км/ч, причем скорость второго на 10 км/ч больше скорости первого. Маршруты всех автомобилей различны, причем каждый состоит из трех дорог и проходит через все города. Второй автомобиль перед проездом третьей дороги своего маршрута встретился с третьим в одном городе, из которого они выехали по общей дороге. Первый и второй автомобили закончили свои маршруты в одном городе, причем первый закончил маршрут на час позже автомобиля, закончившего маршрут раньше других. Найдите скорости автомобилей. 626. На факультет от школьников подано на 600 заявлений больше, чем от производственников. Девушек среди школьников в 5 раз больше, чем девушек среди производственников, а юношей среди школьников больше, чем юношей среди производственников, в п раз (6^/1^12, n£N). Определите общее число заявлений, если среди производственников юношей на 20 больше, чем девушек. 627. Стройотряд состоит из 32 бойцов, каждый из которых владеет одной или двумя строительными профессиями: каменщик, бетонщик, плотник. Плотников в 100
2 раза больше, чем бетонщиков, и в я раз меньше, чем каменщиков (3<л<20, n£N). Сколько бойцов в отряде владеет только одной профессией, если число бойцов, владеющих двумя профессиями, на 2 больше, чем число бойцов, владеющих профессией плотника? § 12. ИРРАЦИОНАЛЬНЫЕ УРАВНЕНИЯ И СИСТЕМЫ УРАВНЕНИЙ Иррациональными называются уравнения, в которых переменная содержится под знаком корня или под знаком операции возведения в дробную степень. Основными методами решения иррациональных уравнений являются следующие: 1) метод возведения обеих частей уравнения в одну и ту же степень; 2) метод введения новых переменных. В некоторых случаях оказывается целесообразным применение различных искусственных приемов. Появление посторонних корней может произойти за счет того, что при возведении обеих частей заданного уравнения f (x)=g (x) в четную степень мы получаем уравнение, являющееся следствием не только этого уравнения, но и уравнения f(x)=—g (х). Действительно, (g (x))2 = ( — g{x))2. Если уравнение / (х) = — g (x) имеет корни, то именно они являются посторонними корнями заданного уравнения / (x)=g(x). Так, если заданным является уравнение х—1=3, то при возведении обеих частей уравнения х—1=3 в квадрат мы получаем уравнение (jc— 1)2 = 32, т. е. а:2 — 2л: — 8=0, корнями которого являются и корень заданного уравнения лг=4, и значение х= —2, являющееся корнем уравнения л:— 1 = —3, но не удовлетворяющее заданному уравнению. Еще пример. Дано уравнение У1— х= —х— 1. Возведя обе части уравнения в квадрат, мы получаем уравнение 1 — х=х2-\-2х+1, т. е. jc2 + 3jc=0. Это уравнение является следствием заданного уравнения. Его корнями будут jcj = —3 и ^2 = 0. Нетрудно убедиться, что х\ = — 3 является корнем заданного уравнения, а хз=0 — посторонний корень (это корень уравнения VI — х=х+1). Напомним, что если обе части уравнения f(x)=g(x) неотрицательны, то уравнения f (x) = g(x) и {f (x))n=(g (х))п равносильны (см. § 7, теорема 3). Отметим еще, что уравнения f (x) = g (х) и / (х)= — g (x) имеют одну и ту же область определения. Поэтому, решив заданное уравнение методом возведения обеих его частей в четную степень и даже убедившись затем, что найденный корень х=х0 принадлежит его области определения, еще нельзя утверждать, что х=хо является корнем заданного уравнения. Однако если х=хо не принадлежит области определения заданного уравнения, то это точно посторонний корень, который получен за счет расширения области определения заданного уравнения в результате использования формулы (2V/ (x))2n===f (*)• Рассмотрим уравнение У* — 2+У*+ 1 =У2г+3. Его областью определения является луч [2; оо). После возведения обеих частей это- го уравнения в квадрат и уединения радикала получим уравнение У*2 — х — 2 = 2. Областью определения этого уравнения является множество 101
(—оо, — 1]U[2; оо). Корнями уравнения У*2 —х —2 = 2 являются значения х\=3 и лг2= —2. Первый корень принадлежит области определения заданного уравнения, т. е. может являться его корнем. Второй же корень не принадлежит области определения заданного уравнения, т. е. является посторонним корнем. Вместе с тем второй корень принадлежит области определения уравнения У*2 —л:—2 = 2. Таким образом, посторонний корень появился за счет расширения области определения заданного уравнения. Причиной появления посторонних корней могут быть также некоторые замены, выполняемые в ходе решения иррационального уравнения (см. ниже пример 5). Мы назвали основные причины появления посторонних корней при решении иррациональных уравнений (возведение обеих частей уравнения в одну и ту же четную степень, расширение области определения и др.)- По этим причинам необходимой частью решения иррационального уравнения является проверка. В зависимости от вида корней (простые или громоздкие), от их количества (один, два или бесконечное множество), а иногда и в зависимости от выбранного способа решения эти корни проверяются либо подстановкой в заданное уравнение, либо путем доказательства равносильности уравнений, получаемых на всех этапах решения, либо каким-то другим путем (с использованием области определения заданного уравнения, с обращением к промежуточным уравнениям и т. д.). 1. Решение иррациональных уравнений методом возведения обеих частей уравнения в одну и ту же степень. Пример 1. Решим уравнение 7^1+^+6 = 6. (1) Решение. Возведем обе части уравнения в квадрат: х- 1 +2 У(*— и далее 2 У2х2 + 4л: — 6= — 3*+31. После возведения в квадрат последнего уравнения получим: 8*2+16jt—24 = 9х2—186х + 961, и далее лг2 — 202* + 985 = 0, откуда находим *i = 5, *2=197. Проверка. Найденные корни несложно проверить непосредственно подстановкой в уравнение (1). 1) V^T^+V^i + 6=V5:::T+-V2-5 + 6 = 6. Таким образом, *i=5 является корнем заданного уравнения. 2) У*2- 1 + л/2*2 + 6=У 197— 1 +л/2-197 + 6=5^=6, т. е. х2= 197-^ посторонний корень. Таким образом, только х=5 является корнем заданного уравнения. 102
Замечание. Это уравнение допускает следующее изящное решение. Имеем V2*-i-6=6 —У*"-*- Несложно подбором найти корень уравнения аг= 5. Так как далее функция i/=V2jc-f 6 возрастает, а функция y=6—^Jx— l убывает, то других корней уравнение не имеет. Более детально такой метод решения уравнений рассматривается ниже в § 2 части III. Пример 2. Решим уравнение Решение. Возведем обе части уравнения (2) в квадрат и уединим затем полученный радикал: (3) После возведения в квадрат обеих частей уравнения (3) и последующего приведения подобных членов получим квадратное уравнение корнями которого являются значения ;n = i+2^13 и Х2 = 1 Проверка. Проверять найденные корни подстановкой в уравнение (2) явно нецелесообразно. Поступим следующим образом. Найдем область определения уравнения (2). Из системы неравенств J 1 находим, что этой областью является луч [2; оо). Выясним, принадлежат ли найденные корни этому лучу. Имеем: о_ 1+2 УГЗ р_ 1+2 УГЗ —6 _ У52-У25 1 3 3 3 Таким образом, *i>2, .т. е. х\ принадлежит лучу [2; сю), и, значит, х\ может являться корнем уравнения (2). Далее, Таким образом, лгг<2, т. е. х2 не принадлежит [2; сю), и, значит, х2 не является корнем уравнения (2). Вернемся теперь к х\. Выясним знак разности, находящейся в правой части уравнения (3). Имеем: _„ +5= -l±ML+5= -2УШ+14 >Q О О Таким образом, х\ является корнем уравнения (3). А так как уравнение (3) равносильно уравнению (2), то корнями уравнения (3) могут являться только корни уравнения (2). Итак, корнем уравнения (2) является *=1+2O^ . з 103
Пример 3. Решим уравнение У#2 + *—5+У*2 + 8л:—-4 = 5. (4) Решение. Преобразуем уравнение (4) к виду —5 = 5—^х* + 8х—4 и возведем обе части полученного уравнения в квадрат: 2-5 = 25-10 л/ 2 Уединим корень и приведем подобные члены: (5) возведем обе части уравнения (5) в квадрат: 8*-4)=(7*+26)2, или 5\х2 + Шх-1076=0. Из последнего уравнения находим х\=2, *2=— тт-- Э1 Проверка. Первый из найденных корней нетрудно проверить подстановкой в исходное уравнение. Такая проверка показывает, что х\=2 — корень уравнения (4). Попытка проверить таким же способом второй корень приводит к громоздким вычислениям. Можно, ГОЛ однако, поступить по-другому. Выясним, является ли *2= —гг реше- нием уравнения (5). Замечаем, что при этом значении левая-часть уравнения (5) неотрицательна, а правая отрицательна. Значит, х2= —— не является корнем уравнения (5). Но уравнение (5) — следствие уравнения (4). Тогда тем более хг не является корнем уравнения (4). Итак, корнем уравнения (4) является х = 2. Пример 4. Решим уравнение Решение. Уединив \J2x-6, получим \/2х — 6=У*+ 1 — 2. Возведем в куб обе части этого уравнения: 2л: —6=(jc+1) V^+T—6(jc+1)+12 V^+~T—8- После приведения подобных членов и уединения корня получим уравнение откуда (х+13)2 (х+1)=64 (х+1)2, и далее (х+ l)((x+13)2 -64(х+1))=0, или (jc+1)(jc2-38a:+105)=0. Таким образом, задача сводится к решению совокупности: х+1=0; х2 —38х+105 = 0, откуда находим *| = —1, лг2 = 3, хз = 35. 104
Проверка. Подстановкой найденных значений х в заданное уравнение убеждаемся, что все они являются его корнями. Пример 5. Решим уравнение Решение. Возведем обе части уравнения (6) в куб. Получим: 2х+ 1 +3 У(2х+ 1)2-Убх+ 1 +3 У2х+ 1 -У(6*+ 1)2 + 6х + 1 =2х— 1, и далее 3 ^2х+ 1 >Убх+ 1 {^2х+ 1 +У&Х+ 1)= — 6х —3. Воспользовавшись уравнением (6), заменим выражение У2х+Т+Уб*+1 выражением или (7) У(2х+ 1) (6*+ 1) (2х— 1)= —2л:— 1. Возведем обе части последнего уравнения в куб: (2*+1) (6х+1) (2х-1)= -(2х+1)3, и далее (2л: + 1) ((6л: +1) (2х — 1) + (2х + 1 )2)=0, откуда находим Л1 — "~~U,O, Л2,3 —— U. Проверка. Подстановкой найденных значений х в заданное уравнение (6) убеждаемся, что его корнем является только х= —0,5. Замечание. При возведении обеих частей уравнения (6) в куб мы в соответствии с теоремой 3 из § 7 (с. 43) получили уравнение, равносильное уравнению (6). Однако дальнейшая замена выражения У2х+\ +Убх+\ на выражение \j2x—1 могла привести (и, как показала проверка, привела) к появлению постороннего корня. 2. Метод введения новых переменных. Пример 6. Решим уравнение Решение. Уединение корня и возведение обеих частей уравнения (8) в квадрат привели бы к громоздкому уравнению. В то же время, если проявить некоторую наблюдательность, можно заметить, что уравнение (8) легко сводится к квадратному. Действительно, умножив обе его части на 2, получим: = Зх+12, — 2 V2*2 — Зх+2 — 8 = 0. и далее 2х2 — Положив у=У2лс2-—Зд:-(-2, получим у2 — 2у — 8 = 0, откуда ух =4, у2= — 2. Значит, уравнение (8) равносильно следующей совокупности уравнений: 105
Из первого уравнения этой совокупности находим хх=—, х2= — 2. 2 f Второе уравнение корней не имеет. Проверка. Так как совокупность уравнений равносильна уравнению (8), причем второе уравнение этой совокупности корней не имеет, то найденные корни можно проверить подстановкой в уравнение У2а:2 — Зх+2 = 4. Эта подстановка показывает, что оба найденных значения х являются корнями этого уравнения, а значит, и заданного уравнения (8). Пример 7. Решим уравнение Решение. Областью определения уравнения (9) является луч [5; оо). В этой области выражение У*2 — Ъх можно представить следующим образом: Ух2 — 5х=УхУ5—х. Так как 2х=х+х, то уравнение (9) далее можно переписать так: х + х — 5 + 2 Ух Ух — 5 + 2У* + 5 + 2 Ух — 48 = 0, или т. е. Положив у = у* — 5 + У*, получим квадратное уравнение у2 + 2у—48=0, из которого находим у\ = 6, #2 = — 8. Таким образом, задача свелась к решению совокупности уравнений: У* — 5+У*=6; Ух —5 У^=—8. Из первого уравнения совокупности находим х=Г^) , второе уравнение совокупности решений не имеет. Проверка. Легко показать, что Ar=ryM является корнем уравнения Ух — 5+-фс=6. Но это уравнение равносильно уравнению (9), значит, ;с=Г^- j является корнем и уравнения (9). Иногда при решении иррациональных уравнений оказывается удобным ввести две новые вспомогательные переменные. Пример 8. Решим уравнение Vl--*+Vl5 + Jt = 2. (10) Решение. Положим f u=^\ —х, \ Тогда уравнение (10) примет вид u+v = 2. Но для нахождения значений новых переменных одного уравнения недостаточно. Возведя в 106
четвертую степень обе части каждого из уравнений системы, получим: ( иА= 1 — х, {4 { Сложим уравнения последней системы: uA-\-v4=l6. Таким образом, для нахождения иу v мы имеем следующую симметрическую систему уравнений: Решив ее (см. § 10), находим: ( Mi = 0, Таким образом, решение уравнения (10) свелось к решению следующей совокупности систем уравнений: Решив эту совокупность, находим #i = l, jc2= —15. Проверка. Проще всего проверить найденные корни подстановкой их непосредственно в заданное уравнение. Проделав это, убеждаемся, что оба найденных значения х являются корнями заданного уравнения. Замечание. Прием, примененный при решении уравнения (10), мог быть использован и при решении некоторых других уравнений из рассмотренных в этом параграфе. Так, решая уравнение У*+1 — М2х— 6 =2 (см. пример 4), можно было положить: Тогда мы пришли бы к системе уравнений {и — и = 2, 2и2-у3 = 8. Этот прием можно было применить и к решению уравнения из примера 1. Пример 9. Решим уравнение -*2 = 3. (11) Решение. Положив придем к уравнению и—о = 3. Перемножим правые части уравнений системы (12): 107
Полученный результат приводит к другому уравнению относительно новых переменных: ми = 28. Решив систему находим: 2=-4, и, =4; iu2=— 7. Таким образом, мы приходим к совокупности систем уравнений. Из этой совокупности выпишем только систему, соответствующую положительным значениям щ и v\ (система, соответствующая отрицательным значениям w«, u2, заведомо не имеет решений, поэтому мы ее опускаем): (13) 4. Решим второе уравнение системы (13). Возведем обе его части в квадрат: х У*2 + 282 —д;2= 16, и далее 5. (14) Теперь возведем в квадрат обе части уравнения (14): х2(х2 + 282)=(х2+16)2, (15) и далее 752х2 —256 = 0. 4V47 4-V47 Из последнего уравнения находим х\ = , х2= ^—. Проверка. Ясно, что х* не удовлетворяет уравнению (14), а значит, и второму уравнению системы (13). Проверим х\. Так как при х>0 уравнения (15), (14) и второе уравнение системы (13) равносильны, то х^^т корень второго уравнения системы (13). Теперь мы должны убедиться в том, что найденное значение х\ удовлетворяет и первому уравнению системы (13) (только в этом случае мы сможем считать это значение решением системы (13)). Сведем это уравнение к равносильному, но более простому. Имеем: х2 + 282 = 482jc2, (482 -1) х2=282. 108
Значение х\ удовлетворяет последнему уравнению, а вместе с тем и первому уравнению системы (13). Итак, х=4 V47— решение системы (13), а значит, и уравнения (11). Пример 10. Решим уравнение V(at —2)(jc —32) —VU— l)(x —33)=1. (16) Решение. Положим Тогда уравнение (16) примет вид ы — и = 1. Для получения второго уравнения относительно новых переменных и, v возведем обе части первого уравнения системы (17) в пятую степень, а второго — в четвертую. Получим: t,4 = jc2 — 34jc + 33, откуда и5 —-и4 = 31. Таким образом, для нахождения u, v мы имеем следующую систему уравнений: , и — 0=1, 1ав-»4 = 31, откуда v Из второго уравнения системы (18) находим и\ =2. Разделив многочлен иъ — и4 + 4и3 — би2 + 4м — 32 на двучлен м — 2, получим в частном и* + и* + 6и2 + 6и+16. Таким образом, система (18) равносильна совокупности систем + 16 = 0. Из первой системы находим Hi = 2, vi = l. Вторая система более сложная. При ее решении необходимо учесть следующее. Так как \1(х— 1)(х—33)= у, то Поскольку а — у = 1, то ы = с;+1, и, следовательно, м^1. Ясно, что уравнение w4 + u34-6a2 + 6w-j-16 = 0 не имеет положительных корней и тем более таких корней, которые удовлетворяли бы неравенству и^ 1. Итак, единственное решение системы (18) Wi=2, i>i = l, и нам остается решить такую систему: I 4 // 109
Имеем: У*2 — 34jc+64 = Положим у—х2 — 34л:+ 33, тогда система примет вид: Из этой системы находим у=1. Тогда х2 — 34л:+ 33=1, откуда 17±/257 Проверка. Проверить такие значения х подстановкой в заданное уравнение (16), конечно, сложно. Можно попытаться выполнить проверку доказательством равносильности уравнений, полученных на всех этапах решения уравнения (16). Анализируя эти уравнения (убедитесь самостоятельно в их равносильности), приходим к выводу, что найденные значения х являются корнями уравнения (16). 3. Искусственные приемы решения иррациональных уравнений. Пример 11. Решим уравнение Зл:. (19) Решение. Умножим обе части заданного уравнения на выражение ф (х)=У2*2 + Зл: + 5—у2х2 — Зх+5, сопряженное выражению Так как = (2х2 + Зх + 5) - (2л:2 - Зх + 5) = бл:, то уравнение (19) примет вид: 6л: = Зл: {^2х2 + Зх + 5 - ^2х2 - Зл: + 5), или Как легко видеть, х\ =0 является корнем этого уравнения. Остается решить уравнение Сложив уравнения (19) и (20), придем к уравнению (21) Решая уравнение (21) методом возведения в квадрат, получим: и далее *2=16, откуда *2 = 4, х3=—4. Проверка. Поочередно подставляя найденные значения л:1=0, х2 = 4, л:3= —4 в уравнение (19), убеждаемся, что ему удов- 110
летворяет только значение х2 = 4. Таким образом, х=4 — единственный корень уравнения (19). 4. Системы иррациональных уравнений. Пример 12. Решим систему уравнений 2х __ (22) 13х—2у 2х = 2, {4у2—1=Зу(х-1). Решение. Положим и = 2х .Тогда первое уравнение системы (22) примет вид и-\ =2, откуда находим и=1. Таким образом, решение системы (22) сводится к решению следующей системы: 4</2-1=Зу(*-1). (23) Возведя в квадрат обе части первого уравнения системы (23) и освободившись от знаменателя, приходим к системе 3*—2y = 2xt -1), (24) из которой находим: i =2, Проверка. При условии, что хфО и Ъхф2у, система (24) равносильна системе (23). В свою очередь система (23) равносильна системе (22). Таким образом, решения системы (24) являются также решениями системы (22). Итак, решениями системы (22) являются пары (2; 1) и (l; у). Пример 13. Решим систему уравнений (25) Решение. Так как У(х+у)3 (х—у)2 = У(х+у)3 • У(х — уг, а л/(*+1/)3=л/*+У и V(A:"~y)2==VlJC""f/l» T0 система (25) примет вид: Эта система равносильна следующей совокупности систем: {л — у ^ 0, (х — у<0, л/х+У + ух — у = 6% < л!*+У + Ух—j/ = 6, ill
получим совокупность систем \v=yx-~yy {x—y>ot fx—y<:o. и+1>=6, Jw+y=6, wz> = 8; yuv=— 8. (27) Решение первой системы совокупности (27) не вызывает затруднений. При решении второй системы этой совокупности следует учесть, что х —у<0, т. е. v<0. Таким образом, из совокупности (27) находим: {х>У, Mi = 2, откуда = 34, г х, = 34 {у, = - 30; х>у, 2 = 12, (*<У> _ «з = 3+л/Т7 y3 = 3-Vl7 = 103— 19 / *з= Ю \ у3= - 77 + 25V17. Проверка. Первые два решения легко проверить непосредственной подстановкой в систему (25). Однако проверить таким же способом третье решение непросто. Но, как нетрудно убедиться, совокупность систем (27) равносильна системе (26), а система (26) равносильна заданной системе (25). Поэтому решения совокупности (27) являются решениями и системы (25). Упражнения 628. 629. 630. 632. 633. 635. 637. 639. 640. 641. 642. 643. 644. 645. 646. 647. 648. 649. 112 Решите уравнения. УЗлг — 5 — У*—2=1. У*+ 1 — л/9 — х = У 2х + 5 + V^— У*+ x2 — Ax — 6=У2л:2 —8*+12.
— —"Л/* - = 0. 682. 683. 684. 686. 688. 690. 691. :V35-a:3(jc+V35-a:3)=30. Г^?==9. 685. \[х^1 ) —х=5. 687. 2. 689. У(х-2) (^_32)-V(*~l)(*- 113
Решите системы уравнений. 701. 703. ур I-VF-- 702. -\/—Л/^-т У х—у=5. У 65 х б f 704. (Ь\[х^ 13, 705. I + ' = тт^г , 706. I V2jc— I _^ 2* 182 ' < .V— 1 M2x~^ _ Угл7^ = J[__ l ^ ^ «/ 2л "" 182 * 707. 709. 711. 2 -у/х^ _i 1 i_ _ J*_ 4, 708. Г 7Ю. = 14, 712. ХлИТу '- (Уу + У^+Т=1, 716. Г2-д/^ iyiTT+y^i. < vr- 14Л/Й: 714. |\/LtL-2V^=l, + t/ + l+y^-i/+10 = 5. \/х + у—л1х—у. =9, 720. 717. 719. 721. fV-У — +Уу+У J 2У*—4~-V^— = 3, 718. f У*+У</+Уде—л^=2, \/«/+yjc—y#—yjc= 1. 722. 2, = — 12, 114
§ 13. ПОКАЗАТЕЛЬНЫЕ УРАВНЕНИЯ При решении показательных уравнений используются два основных метода: 1) переход от уравнения а^х)=а*кх) к уравнению f(x) = g(x)', 2) введение новых переменных. Иногда приходится применять искусственные приемы. Рассмотрим уравнения вида anx)=ag^x\ где а>0 и аф 1, и уравнения, сводящиеся к ним. Решение таких уравнений основано на следующей теореме: Теорема. Если а>0 и аф\, то уравнение аПх)=а8{х) равносильно уравнению f(x) = g(x). Пример 1. Решим уравнение 2*2~~2jc = 23x~"6 Решение. Заданное уравнение равносильно уравнению х2 — 2дг = 3*—6, а потому корни последнего уравнения *i=2, аг2 = 3 являются и корнями исходного уравнения. Пример 2. Решим уравнение $ =5- 0,04 х-1. = V5 Решение. Приведем все степени к одинаковому основанию, например к основанию 5: 50.5-х. g-0.5^5.5-2, + 2 Далее получаем уравнение 5"~*=5~2*+3, которое равносильно уравнению х=2х—3. Корнем этого уравнения, а значит, и корнем заданного уравнения является х=3. Пример 3. Решим уравнение 1 2 Щ^<-1=—. (1) Решение. Упростим выражение, находящееся в левой части уравнения (1): 3->/х+3 2 Тогда уравнение (1) примет вид: Это уравнение равносильно следующему уравнению: 10 ' 115
решение которого сводится к решению совокупности уравнений: Из этой совокупности находим л; = 25. Найденное значение х является корнем иррационального уравнения (2), значит, этот корень необходимо проверить. Проверка. Уравнение (1) равносильно уравнению (2). Подставляя х=25 в левую часть уравнения (2), убеждаемся в том, что х=25 является корнем этого уравнения и, значит, корнем заданного уравнения (1). Пример 4. Решим уравнение Решение. Воспользовавшись основным логарифмическим тождеством N = dogaN% приведем правую часть заданного уравнения к основанию 3. Тогда это уравнение примет следующий вид: Зх2-4=(31о*з5)2х. Полученное уравнение равносильно уравнению х2 —4 = 2jtlog35, или х2—(21og35)x — 4 = 0. Решая это квадратное уравнение, находим его корни: Xi,2 = log3 5± ±yiogi 5 + 4, которые являются корнями и заданного уравнения. Пример 5. Решим уравнение Решение. Так как 3Х+1=3.3Х; 3x+2=9-3x; 5х+2 = 25-5х, то заданное уравнение перепишем в виде 3Х + 3«3Х + 9*3Х = 5Х + 25«5Х, или 13.3Х = 26-5Х, т. е. 3Х = 2-5Х. Далее имеем: р-=2, (-§-)* = 2, g. Пример 6. Решим уравнение Решение. Так как 51+2х = 5-25*, 61+х = 6-6х и 150* = 6*.25\ то заданное уравнение можно преобразовать к виду и далее 5 (25х -6) -6х (25х -6) = 0, (25х - 6) (5-6х)=0. Решение последнего уравнения сводится к решению совокупности уравнений: 25х-6=0; 5-6х=0, 116
которая имеет такие корни: х\ = log2s 6, *2=log6 5. Найденные значения х являются корнями заданного уравнения. Пример 7. Решим уравнение Решение. Так как 4Х=(22)*=(2*)2 и 2*+| = 2-2*, то заданное уравнение можно переписать следующим образом: Введем новую переменную, положив и=2\ и решим полученное квадратное уравнение м2 + 2и— 24 = 0. Его корнями являются Mi=4, а2= —-6. Таким образом, решение заданного уравнения сводится к решению совокупности уравнений: 2* = 4; 2*=-6. Из первого уравнения этой совокупности получаем jc = 2, а второе уравнение корней не имеет, так как 2х >0, а — 6<0. Итак, х=2 — корень заданного уравнения. Пример 8. Решим уравнение ^{== 6. Решение. Перепишем заданное уравнение следующим образом: 22(*+V*2-2) 5^ ,2*+^**-* 6 = 0 Положив теперь u = 2x"w**~2, получим квадратное уравнение а2—у-и —6 = 0, откуда Mi=4, u2=—~. Таким образом, решение заданного уравнения сводится к решению совокупности уравнений: =2==4; 2х+л/р1Г2 = ^-. Из первого уравнения получаем jc+Vx2"~2 = 2, откуда х=-^-. Второе уравнение корней не имеет. о Проверка. Подставляя jc=— в уравнение убеждаемся, что значение ^=-9- является корнем этого уравнения. о Но тогда х=— является корнем и заданного уравнения. Пример 9. Решим уравнение +12==0. 117
Решение. Перепишем это уравнение следующим образом: 2 * _23+1з +12=0, или 2Т -8-2т +12=0. Положив далее и=2 х, получим квадратное уравнение м2 —8м + + 12 = 0, откуда Mi = 6, w2 = 2. Таким образом, решение заданного уравнения сводится к решению совокупности уравнений: 1 1 2х =2; 2* = 6. Из первого уравнения получаем *i=3, а из второго, переписав его в виде 2Т =2loga6, находим х2 = ^-^ . Проверка. Так как в заданном уравнении х является показателем радикала, то x£N, хФ 1. Этим условиям удовлетворяет только значение х=3. Итак, корнем заданного уравнения является 3 3. Пример 10. Решим уравнение 27*-2-9*-9= ур 27*-2-9*-9=0. Р е ш е н и е. Полагая м = 3*, перепишем заданное уравнение следующим образом: 32 — 9 = 0. Так как все коэффициенты многочлена м3 — 2м2 — 9 — целые числа и его старший коэффициент равен 1, то можно попытаться найти целый корень этого многочлена среди делителей его свободного члена. Этими делителями являются числа ±1; ±3; ±9. Пробуя эти значения м, находим, что м = 3 является корнем указанного многочлена. Разделив далее м3 —2м2 — 9 на и — &, получим в частном м2 + м4-3, и поэтому м3 — 2м2 — 9=(м—3)(м2 + м + 3), т. е. уравнение м3 — 2м — 9=0 равносильно следующей совокупности уравнений: и_3=0; м2 + м + 3=0. Второе уравнение этой совокупности действительных корней не имеет. Итак, остается решить уравнение 3* = 3, откуда и находим х=\ — корень заданного уравнения. Пример 11. Решим уравнение 2*+(0,5)2'-3-6(0,5)х=1. Решение. Так как (0,5)2х-3 = 23-2х=|7, и 6(0,5)*=-|-, то 2*+^—1—1-0. Положив м = 2х, получим м+Л~——1=0» и далее м3-м2-6м + 8 = 0, т. е. (м-2)(м2 + м-4)=0. 118
=2 и2=—lt 7 w3 = Последнее уравнение имеет три корня: Wi=2, и2=t 2 Теперь задача сводится к решению совокупности уравнений: о*-_9- о*_л/Т7-1 . Ох_-1-УГ7 Из первого уравнения находим jti = l, из второго Ar2 = log2 ~ , третье уравнение не имеет решений, так как ~~х~^17<0, а 2х>0 при хб/?. Значит, исходное уравнение имеет следующие корни: x,-l,x2-log,^bi. Пример 12. Решим уравнение Решение. Так как 6х = 3х«2х, то имеем: Положив ы = Зх, и = 2*, получим уравнение 2 2 = 0, (4) которое является однородным уравнением второй степени относительно переменных и и и. Так как и = 2* ни при каких значениях х не обращается в нуль, то, разделив обе части уравнения (4) на у2, получим уравнение, равносильное уравнению (4): Полагая z=—, получим: 6z2-13z + 6 = 0, откуда zi=-|- Учитывая, что 2=—=Г—J , запишем совокупность уравнений: (l-Y—JL. /"i-V—i- \2/""2'V2/""3> из которой находим х\ = \,х2= — 1. Значит, уравнение (3) имеет два корня: Jti = l, jc2 = — 1. В определенном смысле к показательным уравнениям примыкают так называемые показательно-степенные уравнения. Это уравнения вида (/ (x)f (J°=(/ (x))h {х\ Если известно, что / (*)>0 и f (х)Ф 1, то это 119
уравнение, как и показательное, решается с помощью приравнивания показателей: g(x)=h(x). Если условием не исключаются возможности /(х)<0, /(*)=1, приходится рассматривать несколько случаев, как это сделано в следующем примере. Пример 13. Решим уравнение (x2+*-57f2+3=(jt2+x--57)10\ (5) Решение. При решении этого показательно-степенного уравнения возможны пять случаев: 1) *2 + х-57=1; 2) х2+х-57=-1; 3) 4) (х2+х—57>0, 5) (х2+х— 57<0, |0* Рассмотрим эти случаи. Jt2-fx-57=l, т. е. ) f, е +*58 0. В этом случае уравнение (5) принимает вид 13*2+3=1|0*э т. е# 1 З 2 580 у ур () р э 1 = 1. Значит, корни уравнения х2 + х—58=0 являются и корнями уравнения (5). Из уравнения х2 + х—58 = 0 находим х\,2= 2 2) х2 + х — 57= — 1, т. е. х2+х — 56=0. В этом случае уравнение (5) принимает вид: (-1^2+3=(_1)10\ (6) Уравнению (6) могут удовлетворять только такие значения х9 при которых Зх2+3 и 10* — целые числа (поскольку отрицательное число (—1) можно возвести лишь в целую степень) одинаковой четности (т. е. либо оба четные, либо оба нечетные). Из уравнения х2+х—56 = 0 находим jci = — 8, *2 = 7. Значение х\ = — 8 не удовлетворяет уравнению (6), а значение *2 = 7 удовлетворяет. Значит, х=7 — корень уравнения (5). 3) х2+х—57=0. В этом случае уравнение (5) принимает вид: Уравнению (7) могут удовлетворять только такие значения х, при которых Зх2 + 3>0 (это верно при всех х) и 10jc>0, в этом случае уравнение (7) примет вид 0 = 0 (напомним, что выражение 0г имеет смысл лишь при г>0). Из уравнения х2+х — 57 = 0 находим *i,2 = ~~ldj^29 . Значение х\ = ~~1 ~V^ не удовлетворяет условию 10*>0, а *2 = =""1 ~^Ч 9 удовлетворяет этому условию. Значит, лс = "~" у* корень уравнения (5). 4) Если х2+х — 57>0 и х2+х — 57=И= 1, то из уравнения (5) заключаем, что Здс2 + 3=10х, откуда находим *i=3, хг = 120
=—. Ни один из этих корней уравнения Зл:2— 10x + 3 = Q не удов- о летворяет неравенству х2-\-х — 57 >0. 5) Если х2+х — 57<0 и х2+х — 57Ф — \, то опять от уравнения (5) переходим к уравнению-следствию Зх2 + 3=10х, откуда находим *i = 3, х2=4-- Обязательна проверка этих корней подстанов- кой в исходное уравнение (5). При *i = 3 получаем ( — 45)30 = = (—45)30 — верное равенство, при хг=— получаем ( —56 -jtj == / \10 =( — 56 — у —неверное равенство (возведение отрицательного числа в дробную степень не имеет смысла). Значит, лишь х=3— корень уравнения (5). Подводя итоги, приходим к выводу, что уравнение (5) имеет пять корней: Х1Л-=1Ш ^Ш Упражнения Решите уравнения. 1)=Е t 8 / 64- 724. 2* -5* =0,001.(Ю3-')2- 7. "Y2*3 V 4х-0,1257 727. "Y2*3 V 4х-0,1257 =4\^. 728. Ю^-б'-1.2Jf"2 = 729. 5х + 5х+2 + 5*+4 = 651. 730. 3JC + 3JC+I+3X+2 = 4X 731. 23х.Зх-2*х~1-Зх+* = -т. 732. 2*+10= * , 733. 2.73х — 5.493х + 3=0. 734. 3-52х~!— 2.5Х~1=О, 735. 9х1""1—36.3х2~3 + 3 = 0. 736. 33JC+I— 4.27х"1 +9|>5*-' -80=0. 737. (3>/5)х~(3А/5)2х4-20 = 0. 738. 4V5rFT-2^FT+2==O. 739. 2x+V7r=ri-5(V2)J:--2 + VF:ri--6==0. 740. (г+узг+сг—V3)*=4. 741. (2+V5)Jtt-&+I+(2-V5)JBi"to"1—^-p. 2—V5 742. 8*-4X+T-2x + 2 = 0. 743. 8x-2Jf+I-4=0. 744. 2x~(0,5)2x-(0,5)x+l=0. 745. 27.2-3x+9.2x-23x~27.2-x=8. 746. 52x—7х—35-52х + 35-7х=0. 747. 4х — 3*-o.5==3;f+o.s_22x-1. 748. 4X + 6X=2-9X. 121
749. 100* +25* =4,25-50 x. 750. 6V9—13V6 + 6V4=0. 751. 125V4-70Vi0 + 5V25=0. 752. 4*+ 10* = 2-25х. 753. 2.4* + 25x+I = 15.10*. 754. 16х + 36* = 2-81*. 755. 7-4*-2.72* + 5.14x = 0. 756. 22*.9*-2.63*-l+42x-I.92*-|=0. 757. ( 758. 759. {f — x—\f-x=\. 760. 761. (x-2f-x=(x-2)12. 762. (3jt-4)2*2 + 2=(3*--4)5x. 763. U|^~^=l. 764. 3*-8*+1=6. 765. 5X"2-2X+I=4. 766. |2*-1| + |2*-2|=1. 767. 2I*+2|-|2*+I-1|=2*+I + L 768. 3U"2|+3U+2I=3*. 769. 770. § 14. ЛОГАРИФМИЧЕСКИЕ УРАВНЕНИЯ При решении логарифмических уравнений во многих случаях приходится использовать свойства логарифма произведения, частного, степени, корня. Напомним эти свойства: 1°. lOga f W+lOgfl g W=lOga (/ (x).g (x)). 2°. ч nlogaf(x), если n£Ry пф2к, k£Z, п loga I / (jc)I, если п — четное число. Все логарифмы в формулах 1°—3° имеют одно и то же основание а>0, а Ф1. В тех случаях, когда в одном логарифмическом уравнении имеются логарифмы с различными основаниями, применение формул 1°—3° возможно лишь после перехода к логарифмам с равными основаниями. Этот переход выполняется с помощью формулы 4°. logaf(*)= iSfcf-^, где частным случаем которой является формула 5°. Отметим, что замена выражения loga / {x)-\-\oga g (x) выражением 1°В«(/(*)•£(*)) приводит, как правило, к расширению области 122
определения выражения \ogaf (x)+\oga g(x). Действительно, область определения выражения loga/ (x) + \ogag{x) задается системой неравенств а область определения выражения loga (/ (x)'g (*)) задается совокупностью систем неравенств UW>0; UW<0. Таким образом, если вторая система этой совокупности имеет решение, то область определения выражения \oga{f(x)*g(x)) будет шире, чем область определения выражения \ogaf{x) + \ogag{x). Аналогично может произойти расширение области определения выражения loga / (*)—• loga g (x) при замене его выражением 1. Решение уравнений вида loga / (x) = loga g (x) и уравнений, сводящихся к этому виду. Уравнения вида /(*)= lOga g(*) (I) и уравнения, сводящиеся к этому виду, можно решить одним из следующих способов: 1-й способ. Решить уравнение /(*)=*(*). (2) являющееся следствием уравнения (1), и выполнить проверку корней уравнения (2) подстановкой их в заданное уравнение (1). Если уравнение (1) само является следствием некоторого заданного логарифмического уравнения, то проверку найденных корней выполняют подстановкой их в заданное уравнение, а не в уравнение (1). 2-й способ. Решить уравнение (2) и проверить корни этого уравнения подстановкой их либо в систему неравенств f/W>o. UW>o, (6) которой задается область определения уравнения (1), либо в неравенства, являющиеся решением системы (3). Если же уравнение (1) само является следствием некоторого заданного логарифмического уравнения, то проверку найденных корней выполняют подстановкой их в неравенства, с помощью которых записывается область определения заданного уравнения, а не уравнения (1). 3-й способ. Этот способ отличается от 2-го способа практически лишь тем, что обращение к неравенствам, с помощью которых записывается область определения уравнения (1), происходит не в проверке, а уже непосредственно в процессе решения уравнения (1). Применяя этот способ, решают смешанную систему 123
f(x)>0, ё(х)>0, которая составлена из уравнения (2) и неравенств системы (3). Эта смешанная система равносильна уравнению (1). Если же уравнение (1) само является следствием некоторого заданного логарифмического уравнения, то в смешанную систему, равносильную этому уравнению, войдут, кроме уравнения (2), неравенства, с помощью которых записывается область определения заданного уравнения, а не уравнения (1). При решении логарифмических уравнений применяются три основных метода: метод потенцирования, т. е. переход от уравнения logo / (x) = \oga g {х) к уравнению-следствию / (x) = g(xyy метод введения новых переменных; метод логарифмирования, т. е. переход от, уравнения f(x) = g(x) к уравнению \oga f (x) = \oga g (х) (об этом методе идет речь ниже в п. 3). Перейдем к рассмотрению примеров. Пример 1. Решим уравнение log3 (7-2x)=log3 (jc2-3x — 5). Решение. Перейдем от этого уравнения к уравнению 7 Оу y2 Чу R / "~~ &Х Л —~" ОХ — О. Далее получаем уравнение х2 — лг —12 = 0, откуда xi=4, лг2= —3. Так как уравнение х2 — х—12 = 0 является следствием заданного уравнения, то найденные корни необходимо проверить. Проверка. Подставляя х\ =4 в левую часть заданного уравнения, получаем 7 — 2*i<0. Это значит, что log3 (7 — 2jci) не существует, т. е. Xi=4 — посторонний корень. Далее, Iog3(7 — 2x2)=log3 13 и log3 (4 — 3x2 — 5) = log3 13. Таким образом, х= — 3 является корнем заданного уравнения. Пример 2. Решим уравнение -2х). (4) Решение. Преобразуем уравнение (4) к следующему виду: От этого уравнения перейдем к уравнению (аг + 4)(2дс + 3)=1 —-2х и далее 2лг2+13х+11=0, откуда jci = — 1, Х2= — 5,5. Так как найденные значения х — это корни уравнения, являющегося следствием уравнения (4), то их необходимо проверить. Проверка. Область определения уравнения (4) задается системой неравенств U-2a:>0. 124
Подставляем х\= — 1, а затем х2 = — 5,5 в неравенства этой системы, убеждаемся, что при xi = —1 все неравенства выполняются, а При хо=—5,5 не выполняется, например, первое неравенство этой системы. Таким образом, только jc = — 1 является корнем уравнения (4). Пример 3. Решим уравнение Р е ш е н и е. Так как lg 77r = lg x—■ 1, то заданное уравнение можно переписать следующим образом: Положив u = \gx, получим уравнение #2 + м+1 = , откуда находим и = 2. Из уравнения lg л: = 2 находим #=100. Проверка. Подставив х= 100, например, в систему с помощью которой записывается область определения заданного уравнения, убеждаемся, что х=100 является корнем заданного уравнения. Пример 4. Решим уравнение lgV-lg(O,Lvlo) = O. Решение. Имеем: (lg*3)2-lgxl0-Ig 0,1=0, 9 1g2x-101g |*| + l=0. Рассматривая случаи, когда х>0 и когда х<0, можно свести решение этого уравнения к решению совокупности уравнений, однако можно поступить и по-другому. А именно, заметив, что областью определения заданного уравнения является множество значений х, удовлетворяющих неравенству х>0, а в этом случае \х\=х, перепишем последнее уравнение следующим образом: 9 lg2 х— 10 lg Аг+ 1 =0. Положив далее для краткости w = lgx, получим квадратное уравнение 9и2— 10и+ 1 =0, корнями которого являются Wi = l, w2=—. Таким образом, задача свелась к решению совокупности уравнений: \gx=U lg* = f. Из первого уравнения находим #, = 10, а из второго х2 = \[\0. 125
Проверка. Оба найденных корня принадлежат области определения заданного уравнения, т. е. являются и его корнями. Пример 5. Решим уравнение j.()g(+) (5) Решение. Прежде всего перейдем в уравнении (5) к логарифпо одному основанию, например по основанию —. еем: log! (4 —x)=log/± V (4-jt)3 = log , (4-х)3. V4 \V4/ "* Тогда заданное уравнение принимает вид: flog,, (* + 2)2-3 = log ,, (4-x)3 + log^, (x + 6)3. (6) Так как далее log i (x+2)2 = 2 log , |jc + 2|, то уравнение (6) равносильно уравнению 3 log^, U+ которое преобразуется в уравнение 3 log^, U+2| -3 = 3 log_, (4-x) + 3 log^ (x + 6), ^ ^ (x + 6) и затем в уравнение log^ 4 U + 2|=log^ (4-x)(jc + 6). Из этого уравнения получаем уравнение, не содержащее логарифмов: решение которого сводится к решению следующей совокупности: Из уравнения первой системы находим *i=2, и дг2= — 8. Однако неравенству этой системы удовлетворяет только значение х = 2. Из уравнения второй системы находим xi,2=l±V33- Неравенству же этой системы удовлетворяет только значение jc= 1 — V33. Итак, проверке подлежат корни лг=2 и х= 1 —\/33. Проверка. Найдем область определения уравнения (5), для чего решим систему (Х + 2Ф0, 126
Получаем — 6<*< — 2; — 2<x<4. Оба проверяемых корня принадлежат этой области, и, значит, корнями уравнения (5) являются х,=2 и *2=1-V33- 2. Решение уравнений вида loga(*) / (x)=\oga(x) g (x) и уравнений, сводящихся к этому виду. При решении уравнений вида W (7) и уравнений, сводящихся к этому виду, может применяться любой из тех трех способов, которые применяются при решении уравнений вида (1). Следует лишь учесть, что к неравенствам, с помощью которых записывается область определения уравнения (1), необходимо добавить еще условия а(лг)>0 и а(х)Ф\. Таким образом, при решении уравнений вида (7) вторым или третьим способом из корней уравнения (2) отбирают лишь те, которые удовлетворяют следующей системе: f/W>0. a(x)>0, а(х)Ф\. Пример 6. Решим уравнение log^t*2 — l) = log^+4(5 — х). Решение. Найдем корни уравнения х2—1=5 — х. Получаем х\=2, х2=—3. Проверим эти корни. Проверка. Значение х\ = 2 удовлетворяет одновременно условиям л:2— 1>0, 5 — х>0, * + 4>0 и х + 4ф\. Значит, х\=2 является корнем заданного уравнения. Значение хг=—3 не удовлетворяет условию х + 4Ф\. Таким образом, jc2=—3 не является корнем заданного уравнения. Итак, корнем заданного уравнения является только х=2. Пример 7. Решим уравнение log* x+log* x=0. (8) ГО "5 Решение. Перейдем во всех логарифмах к одному основанию, например х. Так как >*То з , * l-log.5- О то уравнение (8) примет следующий вид: l-logx10 ' 1 — logx 5 127
и далее 1 — logx 5+1 — logjc 10 = 0, откуда log* 50 = 2, и тогда Х\ = 5/2 5/2 /, 2 л/ Проверка. Нетрудно убедиться, что значение Х\ =5^/2 принадлежит области определения уравнения (8), т. е. является корнем этого уравнения, а #2=—5-\/2 не принадлежит этой области, т. е. является посторонним корнем. Однако при переходе в логарифмическом уравнении (8) к основанию х нас подстерегала другая, более «грозная* опасность, чем опасность приобретения посторонних корней. Так, подставив значение х=\ в левую часть уравнения (8), убеждаемся, что х=1 является корнем этого уравнения. Потеря корня произошла при переходе в логарифмическом уравнении к основанию х, что сужало область определения уравнения (8) и, следовательно, могло привести (и, как видим, привело) к потере корня. Итак, корнями уравнения (8) являются х\=Ъ^2 и Jt2=l. Замечание. При переходе в уравнении (8) к логарифмам по основанию, не содержащему х, например к основанию 10, мы получили бы уравнение I ^_ ^ +. xl_| 5=0' КОРНЯМИ которого являются jci = 1 и *2 = 5У2, т. е. потери решения при таком способе не происходит. 3. Разные логарифмические уравнения. Пример 8. Решим уравнение х>->«* = 0,01. (9) Решение. Область определения уравнения лг>0. В этой области выражения, содержащиеся в обеих частях уравнения (9), принимают только положительные значения, а тогда логарифмы этих выражений существуют. Взяв логарифмы от обеих частей уравнения по основанию 10, получим уравнение или (1 —lgx) lgjc= —2. Положив w = lgx, придем к уравнению и2 — и — 2 = 0, откуда Mi= — 1, ы2 = 2. Таким образом, задача свелась к решению следующей совокупности уравнений: lgjc=-l; lgx = 2. Из этой совокупности находим jci=0,1 и *2=100. Проверка. Оба найденных значения х принадлежат области определения уравнения (9) (этой областью является луч л:>0), и, таким образом, Х| =0,1 и лг2 = 100 — корни уравнения (9). Пример 9. Решим уравнение log, ( 3^°25* + 4) = 2 logs х. (10) 128
Решение. Воспользовавшись определением логарифма, преобразуем уравнение (10) к виду Положив u = dogiXy получим уравнение w2 — 3« — 4 = 0, корни ко торого Mi = — 1, М2 = 4. Теперь задача сводится к решению следующей совокупности уравнений: *Iog$*=-l; xlog5* = 4. Так как xlog5j:>0, a — КО, то первое уравнение совокупности не имеет решений. Взяв от обеих частей второго уравнения совокупности логарифмы по основанию 5, получим: logi x = log5 4, т. е. logs * = ±yi°g5 4, откуда находим xi,2 = 5 Проверка. Найдем область определения уравнения (10). Так как х — основание логарифма, то jc>0, хФ 1. Как нетрудно заметить, выражение 3^од5* + 4 положительно при всех значениях х>0. Таким образом, область определения уравнения (10) определяется условиями х>0, хф\. Этим условиям удовлетворяют оба найденных значения х. Итак, jci=5viog54 и jc2 = 5""viog54 — корни уравнения (10). Пример 10. Решим уравнение logs (5^ + 125)=log5 6+1 Н-^-. (11) Решение. Сначала будем рассматривать данное уравнение как логарифмическое. Так как 1 + ^-= logs 5 Тх, то уравнение (11) запишем в виде logs (5^ + 125)=log5 6 + logs 51+2^. Далее имеем: logs (5Т+125)= logs (6-5-5^), т. е. 5* + 125 = 30-52\ Это — показательное уравнение, которое можно решить методом введения новой переменной. Положив и = 5Гх, получим уравнение и2 —30ы+ 125 = 0, корни которого «i=5, ^2 = 25. Теперь задача свелась к решению совокупности двух уравнений: 5^=5; 5^=25. Из первого уравнения получаем «-=1, откуда Х|=—. Из второго уравнения получаем о"~"=2, откуда ^2 = т"- 129
Проверка показывает, что оба найденных значения *i=— и > = —являются корнями уравнения (1). Упражнения Решите уравнения. 771. log4j^Y=log4(4--x). 772. 773. Ig(4,5-x) = lg4,5-lg*. 774. loss ((x— 1) (2jc— 1»—0. 775- Iogv2*2 777. log , logs 5~=0. 778. log4 log2 log3 (2x— l)=-r-. "2 Л — О L 779. Iog2(9 — 2X)=25] log5 ^ 78il. 781. 782. lgV^^+lg V2^—3+1 =lg 30. 783. lg 5 + lg (x+10)= 1 — lg (2x— l)+lg (21a:— 20). 784. log5 (3x- 1 l) + log5 (x-27)=3 + logs 8. 785. lg(5 —л:) g^ 787. 0,1 lg4JC-lg2*+0,9 = 0. 788 1I5 789' + 790. 4 —l 791. lg2 (100*) +ig2 (10x)= 14 lg jc+ 15. 792. log, 5V5--j-= log? V5. 793. 3 794. g g 795. Io24jc+log 797. Ig (*3+27)-0,5 ig (a:24-6jc+9)=3 lg 4?. 798. lo 800. 130
801. 802. 803. 804. 805. 806. 807. 808. 809. 810. 811. 812. 813. 814. 815. 816. 817. 818. 819. 820. 821. 822. 823. 824. 825. 826. 827. 828. 829. 830. 831. 832. 833. 834. 835. 836. 837. 838. 839. *+10)P-2!gll. V2 logs ( — *)—lo у log2 (jc+3)2-5=log2 (3-x)5+log2(* + 5)5. log3 (-**-»*-14) 1<*я1+4х+4 9=1. i (\-x*)2+2. log2 s/T+x+3\og± (1—Jc)= (l-2lg2)log5JC=lg3-lg(jc- log2log3(A:2-16)-log, log, —2|-|iog3*—2|=2. Ilog2 (За:— 1)— log2 3| = I log2 (5—2д:)— 11. 1(^-2(2* — 9)=log,_2 (23— 6x). ,6 (4x+1)- logx4. 10g4 X 7 + log9*7=0. log9x3 (x+1). 131
840. 841. 5lgJf = 50-*Ig5. 842. jtIog255.14log27=l. 843 jc(log3Jc)1~3 1обз^_з ^ x«oBlx»-logf2x-2 + ( 845. I 846. lg(3x —24"x) = 2+0t25 1g 16 —0,5jc Ig 4. 847. 3 lg 2+ Ig (2^1ГТ-l - 1)= lg (p, 848. l(1T l 849. 850. x2\og3fi(5x2-2x-3)-x\og1 851. ^±£ § 15. СИСТЕМЫ ПОКАЗАТЕЛЬНЫХ И ЛОГАРИФМИЧЕСКИХ УРАВНЕНИЙ При решении систем показательных и логарифмических уравнений применяются те же приемы, что при решении систем алгебраических уравнений. Следует лишь подчеркнуть, что во многих случаях, прежде чем применить тот или иной метод решения систем, следует преобразовать каждое уравнение системы к возможно более простому виду. Пример 1. Решим систему уравнений 25 + 25 30, 25*+* = 5 л/5. (1) Решение. Положив а = 25*, v = 25*, получим систему уравнений имеющую четыре решения: «2=л/5' Г "з=— 5, <аА=— -\[ЪУ у2 = 5; \у3=— л/5; \г>4=— 5. Но ы = 25*, у = 25у, значит, w>0, v>0y т. е. из найденных четырех решений надо взять лишь первые два. Таким образом, задача сводится к решению следующей совокупности Систем уравнений: 25* = 5, /25*=У5, =V5; 125*=5. Из первой системы находим: х\=—, f/i=—, из второй: *2 = -т"• 132
Итак, система (1) имеет два решения: f-|-; ~ч , f-^-; -£Л . Пример 2. Решим систему 2*. 3х =18. (2) Решение. Перемножив почленно уравнения системы (2), получим уравнение ++ , или 6*+* = 63, откуда х+у=3. Разделив почленно первое уравнение системы (2) на второе, получим уравнение 2»-'.y-*-f. или (•§-)*"'«•§-. откуда х—у=1. Таким образом, решение системы (2) сводится к решению следующей системы уравнений: Пара (2; 1) является решением этой системы и, следовательно, системы (2). Пример 3. Решим систему Решение. Взяв логарифмы по основанию 2 от обеих частей каждого из уравнений системы (3), получим следующую систему уравнений: / log2 *"-*+9-3, / (2у2 2 = 2, или \(i/2^ Разделим первое уравнение этой системы на второе (это деление не приведет к потере решений, так как ясно, что у2 — 5+60 и хф\, т. е. log2jc=^0): 2j/2-9j/+9_ 3 2Ь+Ъ 2 э откуда t/i = 3, i/2=0. Таким образом, решение системы (3) мы свели к решению совокупности: У==3, гу==О, (25 6)l2 l(2 Первая система этой совокупности решений не имеет, а пара (V2; 0) — решение второй системы — является решением и системы (3). 133
Пример 4. Решим систему уравнений y-3x)=l. (4) Решение. Приведем первое уравнение системы (4) к более простому виду. Для этого возьмем от обеих частей уравнения логарифм по основанию у: и далее log^x^' + logy y = -|-log<, х, \0g2yX+l=-y\0gyX. Положив u = logyx, получим квадратное относительно и уравнение и2 — -|-и+1=0, корни которого Mi=2, u2=—- Значит, либо \ogyx = 2t тогда х = у2, либо log1/x=—, тогда х=-\[у, т. е. у = х2. Итак, следствием первого уравнения системы (4) является совокупность уравнений: Приведем теперь второе уравнение системы (4) к более простому виду. Для этого перейдем от логарифма по основанию у к логарифму по основанию 4: и далее log4 (у — Злг)= 1, откуда у —Зд: = 4. Таким образом, решение системы (4) мы свели к решению следующей совокупности систем уравнений: Первая система не имеет решений, вторая имеет два решения: (4; 16), (-1; 1). Проверка. Решения системы (4) должны удовлетворять следующим условиям: f/>0, УФ\. Пара (4; 16) этой системе удовлетворяет, а пара (—1; 1) — нет. Значит, (4; 16) — единственное решение системы (4). 134
Упражнения Решите системы уравнений: U+j/=5. 854. 855. f Sx=l0yt 856. f f 2X Зх 857. ( 3х — 22у = 77, 858. (xy+l=27, Ту | 2t/-5 1 859. 861. 863. {*2*-3=64. 860. 64. 862. 864. 865. 867. (lg(^ +У )— 1 = lg 13 866. f 5 (logy a; lg(*+</)-lg(*-i/)=31g2. Jxt/=64. I ig (jc—1/)2=2 Ig 2. ( — 869. J 2 2я -(у2)х-*'= 12 870. | 3X 871. 866. f 5 (logi/ a:-f- logx 1/) = 26 j64 868. 872. I 3(2 log.» Jt—log, £/)= 10 873. f 875. f 877. ~ 874. f (x + t/).3*-x = -! < 27 |^ 3 logs (x + £/) = JC — ^ 876. f lQg4^+3r^lf=0 log, log4 |.l iJ/=0. 9. Г lgx- \ lg«/' -Ig ( \ »g i/ 880. 881. f *.2х+|— 2- 878. Г log2 (10 —2^)=4 —^/ I jc + 3i/—1 lg (x-t J-jr). 135
§ 16. РАЦИОНАЛЬНЫЕ НЕРАВЕНСТВА 1. Основные понятия. Областью определения неравенства f(x)>g(x) называется множество таких значений х, при которых и функция / (х), и функция g (x) определены. Иными словами, область определения неравенства f(x)>g(x) — это пересечение областей определения функций f (х) и g(x). Частным решением неравенства f(x)>g(x) называется всякое удовлетворяющее ему значение переменной х. Решением неравенства называется множество всех его частных решений. Два неравенства с одной переменной х называются равносильными, если их решения совпадают (в частности, если оба неравенства не имеют решений). Если каждое частное решение неравенства f\(x)>g\(x) является в то же время частным решением неравенства М*)>&2(*)» полученного после преобразований неравенства f\ (x)>g\ (*), то неравенство M*)>g2(*) называется след- ствием неравенства f\ (x)>g\ (х). В следующих теоремах речь идет о преобразованиях, приводящих к равносильным неравенствам. Теорема 1. Если к обеим частям неравенства прибавить одну и ту же функцию <р (х), которая определена при всех значениях х из области определения исходного неравенства, и при этом оставить без изменения знак неравенства, то получится неравенство, равносильное исходному. Таким образом, неравенства f(x)>g(x) и / равносильны, если <р (х) удовлетворяет условию теоремы. Следствие. Неравенства /М + Ф (*)>*(*) и f равносильны. Теорема 2. Если обе части неравенства умножить (или разделить) на одну и ту же функцию ср (х), которая при всех значениях х из области определения исходного неравенства принимает только положительные значения, и при этом оставить без изменения знак исходного неравенства, то получится неравенство, равносильное исходному. Таким образом, если ф(х)>0, то неравенства f{x)>g(x)*f(x)4p(x)>g(x)-v{x) (или -Ц4->^44") равносильны. v фМ ф(*) / v фМ ф() Следствие. Если обе части неравенства умножить (или разделить) на одно и то же положительное число, сохраняя знак неравенства, то получится неравенство, равносильное данному. Теорема 3. Если обе части неравенства умножить (или разделить) на одну и ту же функцию ф (*), которая при всех значениях х из области определения исходного неравенства принимает только от- 136
рицательные значения, и при этом изменить на противоположный знак неравенства, то получится неравенство, равносильное исходному. Таким образом, если <р(лг)<0, то неравенства /(*)>*(*) и f(x)-<p{x)<g(x)-<f(x) (или iw"<fw") Равносильны. Следствие. Если обе части неравенства умножить (или разделить) на одно и то же отрицательное число, изменив знак неравенства на противоположный, то получится неравенство, равносильное данному. Теорема 4. Пусть дано неравенство f(x)>g (x), причем f (*)>0 и g(x)^0 при всех х из области определения неравенства. Если обе части неравенства возвести в одну и ту же натуральную степень п и при этом знак неравенства оставить без изменения, то получится неравенство (f(x))n>(g(x))\ равносильное данному. Замечание. Выше (§ 7) мы уже отмечали, что при выполнении тождественных преобразований возможно изменение области определения выражения; например, при приведении подобных членов, при сокращении дроби может произойти расширение области определения. При решении неравенства в результате тождественных преобразований может получиться неравносильное неравенство. Рассмотрим для примера неравенство V*+*-l>V*-5. (1) Прибавив к обеим частям неравенства (1) одну и ту же функцию ф(*)=—V*» получим неравенство V*+x-1 - V?>Vr-5—\fx, (2) равносильное (по теореме 1) неравенству (1). Далее имеем: *-1>-5, (3) откуда х> — 4. Но неравенство (1) имеет решение х^О, т. е. неравенства (1) и (3) неравносильны (неравенство (3) есть следствие неравенства (1)). Дело в том, что неравенство *— 1 > — 5 имеет более широкую область определения, чем неравенство (1); это расширение произошло в результате приведения подобных членов в неравенстве (2). Поэтому после выполнения тождественных преобразований, которые привели к расширению области определения неравенства, из найденных решений нужно отобрать те, которые принадлежат области определения исходного неравенства. 2. Рациональные неравенства. Рассмотрим функцию f (x)= (*-<*.Г (*-д>У*-...-(*-д»)<" /4) (x-bi)mi(x-b2)m2'...-(x-bp)m> ' где пи /*2, ..., nky гп\9 т>2> ..., гпр — натуральные числа и где Ъфаь Ъ1фЪ] при %ф\\ п1фЪ\§=\% 2, ..., k; /=1, 2 р). В точках х = а, функция обращается в нуль (эти точки называют нулями функции), точки x=bj — точки разрыва функции f(x). Если 137
все нули функции и точки разрыва отметить на числовой прямой, то они разобьют ее на k + p+ 1 промежутков. Из курса математического анализа известно, что внутри каждого из этих промежутков функция / (х) непрерывна и сохраняет постоянный знак. Для установления этого знака достаточно взять любую точку из интересующего нас промежутка и определить знак функции в этой точке. Пример 1. Решим неравенство (х-4)7 Решение. Функция / (х) = у _Л обращается в нуль в точках Jti=O, *2 = 2, *з=— 3 и претерпевает разрыв в точке *4 = 4. Эти четыре точки разбивают числовую прямую на пять промежутков (рис. 4): (-оо; -3), (-3; 0), (0; 2), (2; 4), (4; оо). Определим знак функции / (х) в каждом из этих промежутков. -о о о о ■ *~ Рис. 4 В промежутке ( — оо; —3) возьмем точку х=—-4. Имеем /( — 4)<0, значит, в (— оо; —3) f (х)<0. В промежутке ( — 3; 0) возьмем точку х= — 1. Имеем / (—1)>0, значит, в ( — 3; 0) f(x)>0. В промежутке (0; 2) возьмем точку х= 1. Имеем /(1)>0, значит, в(0; 2)/(х)>0. В промежутке (2; 4) возьмем точку л; = 3. Имеем /(3)<0, значит, в (2; 4) f(x)<0. В промежутке (4; оо) возьмем точку х = 5. Имеем /(5)>0, значит, в (4; оо) /(л:)>0. Нам надо было решить неравенство /(*)>0. Из проведенного рассуждения ясно, что это неравенство выполняется в промежутках ( — 3;0), (0; 2) и (4; оо). Объединение этих промежутков и представляет собой решение данного неравенства. Ответ можно записать двумя способами: 1) (-3; 0)U(0; 2)U(4; oo); 2) -3<х<0; 0<*<2; 4<л:<оо. На практике для решения неравенства f(x)>0 (соответственно <, >, О, где f(x) — функция вида (4), применяют так называемый метод интервалов — геометрический метод решения, основанный на следующих трех достаточно очевидных утверждениях: 1) Если с — наибольшее из чисел a/, bh то в промежутке (с, оо) функция f (х) положительна. 2) Если сц (соответственно bj) — такая точка, что показатель степени hi выражения (л: — а/У" есть число нечетное, то справа и слева от щ (или bj), т. е. в смежных промежутках, функция / (х) имеет противоположные знаки. Такую точку щ (соответственно Ь,) будем называть простой. Вы- 138
сказанное выше утверждение означает, что при переходе через простую точку функция / (х) меняет знак на противоположный. 3) Если щ (соответственно 6у) — такая точка, что показатель степени Л, выражения (jc —а,/1' есть число четное, то справа и слева от а,, т. е. в смежных промежутках, функция имеет одинаковые знаки. Такую точку а, (соответственно Ь\) будем называть двойной. Высказанное выше утверждение означает, что при переходе через двойную точку функция / (х) не меняет знака. Так, в примере 1 точки х = 2, л:=— 3, х = 4 простые, а точка jt = O двойная. Знаки функции f(x) в промежутках представлены на рисунке 5. -3 0 2 4 х Рис. 5 Значит, / (лг)>0 в промежутках ( — 3; 0), (0; 2), (4; оо). То же было получено нами выше при решении примера 1. Метод интервалов, основанный на сформулированных выше трех утверждениях, применяется для решения неравенства вида Он заключается в следующем: 1) Отмечают все нули и точки разрыва функции f (х), содержащейся в левой части неравенства (5), незакрашенными кружками на числовой прямой. 2) Проводят волнообразную кривую, которую начинают всегда в какой-нибудь точке, находящейся над числовой прямой, правее наибольшего из чисел щ и bj. При этом проводят ее так, чтобы она проходила все отмеченные точки с учетом того, что при переходе через простые точки кривая пересекает числовую прямую, а при переходе через двойные точки она остается по ту же сторону от числовой прямой. Эту волнообразную кривую называют кривой знаков. 3) Выбирают промежутки числовой прямой в соответствии со знаком неравенства (5) (а именно /(*)>0 там, где кривая знаков располагается над числовой прямой, и /(х)<0 там, где кривая располагается под числовой прямой). Объединение отобранных промежутков представляет собой решение неравенства (5). Условимся двойные точки показывать на рисунках подчеркнутыми числами. На рисунке 6 проведена кривая знаков, иллюстрирующая решение неравенства из примера 1. Точка 0 двойная, поэтому она подчеркнута, и кривая знаков справа и слева от этой точки расположена по одну сторону от числовой прямой. Пример 2. Решим неравенство 139
Решение. Отметим на числовой прямой (рис. 7) точки —2 и 1 — нули функции / (*)=!*~li*+?v и точки 4 и 5 — точки ее разры- ва. Проведем через эти точки кривую знаков, начиная ее правее и выше точки 5, с учетом того, что все отмеченные точки простые. Рис. 6 Выберем на числовой прямой те промежутки (на рис. 7 они заштрихованы), где кривая знаков проходит над числовой прямой. Это промежутки (— оо; —2), (1; 4) и (5; оо). Их объединение и является решением заданного неравенства. Рис. 7 Пример 3. Решим неравенство (2лг—3)(4лг+5) Решение. Заданное неравенство необходимо сначала преобразовать к виду, удобному для применения метода интервалов. Получаем г~ г~ 5 3 Далее наносим на числовую прямую точки —5, —у2, УЗ и ——, — (рис. 8) —нули и точки разрыва функции f(x)=(x.'{'5^~f^x+'f>. (х-т){х+т) Затем проводим кривую знаков, учитывая, что все указанные точки простые. Выбирая те промежутки числовой прямой (на рис. 8 они заштрихованы), где кривая знаков проходит под числовой прямой, получаем промежутки ( — оо; —5)Л — л/2; —-|-) и (-|-; V5 ) . Их объединение и представляет собой решение заданного неравенства. 140
Пример 4. Решим неравенство x2(x-7f Решение. Отметим на числовой прямой (рис. 9) закрашенными кружками нули функции / (*) = - **»(* —7)э " » т. е. точки —6, —2, 1 и 3, и незакрашенными кружками точки разрыва этой функции, т. е. точки 0 и 7. Отметим, что точки —2 и О являются двойными, и проведем кривую знаков. Отбирая промежутки, где /(*Х0 (кривая знаков под числовой прямой), получаем следующее решение заданного неравенства: [—6; 0)U(0; l]U[3; 7). Пример 5. Решим неравенство г— *2 — 42 Решение. Приведем сначала неравенство к виду, позволяющему применить метод интервалов. Получаем последовательно: ИЛИ Сократив дробь на а: — 6 при условии, что хФб, получим: аг —7 откуда (см. рис. 10) находим — 3<дс<7. Исключив из этого промежутка точку х=6, получим решение заданного неравенства: — 3<х<6; 6<jc<7. Пример 6. Решим неравенство Рис. 9 ///////////////////////// Рис. 11 141
Решение. Так как дискриминант знаменателя D = 9 — 20<0и коэффициент при х2 а=1>0, то при всех х выполняется неравенство а:2 — Зл: + 5>0. Тогда, умножив обе части заданного неравенства на <р(х)=х2 — Здс+5, получим неравенство (Зл: + 4)Х Х(х2 — 1)<0, равносильное заданному. Решением этого неравенства является объединение промежутков: (-—оо; ——^ U( — 1; 1). Пример 7. Решим неравенство *'— ] Решение. Преобразуем заданное неравенство к виду, позволяющему применить метод интервалов. Получаем последовательно: Решение этого неравенства находим с помощью кривой знаков (рис. И). Получаем ( — 5; —1)11(1; °°)- 3. Системы и совокупности неравенств с одной переменной. Несколько неравенств с одной переменной образуют систему неравенств в том случае, если ставится задача об отыскании всех тех значений переменной, которые удовлетворяют одновременно каждому из этих неравенств. Несколько неравенств с одной переменной образуют совокупность неравенств в том случае, если ставится задача об отыскании всех тех значений переменной, каждое из которых удовлетворяет по крайней мере одному из этих неравенств. Из сказанного следует, что решением системы неравенств является пересечение решений неравенств, образующих систему, а решением совокупности неравенств является объединение решений неравенств, образующих совокупность (здесь, как и выше, под решением понимается общее решение, т. е. множество всех частных решений). Если неравенства f\ {x)>g{ (x) и /2 {x)>g2 (х) образуют систему неравенств, то их записывают в столбик с помощью фигурной скобки: В некоторых случаях неравенства, образующие систему, можно записать и в строчку. Так, если неравенства / {x)>g\ (х) и / (*)<#2 (х) образуют систему, то эту систему можно записать в виде так называемого двойного неравенства: gi(x)<f(x)<g2(x). Из определения системы неравенств следует, что если неравенство f(x)>g(x) является следствием неравенств f\(x)>g\(x) и 142
(или следствием только одного из этих неравенств), то система неравенств •fi(x)>gi(x). равносильна следующей системе: Иными словами, если к заданной системе неравенств приписать неравенство-следствие или, наоборот, из заданной системы неравенств исключить неравенство-следствие, то получится система неравенств, равносильная заданной. Так, равносильны системы неравенств 'fi(x)>gi{x)9 (h(x)>gi(x)9 и (из первой системы исключено неравенство /i () f () >gi(x)-\-g2(x), являющееся следствием неравенств f\(x)>g\ (х) и f2(x)>g2(x)). Если неравенства f\ (x)>g\ (x) и f2 (x)>g2 (x) образуют совокупность неравенств, то их записывают либо в столбик с помощью квадратной скобки: \h(x)>g\(x)9 М)Ы) либо в строчку с помощью знака «;»: f\(x)>g\(x)9 f2(x)>g2(x). Всякое нестрогое неравенство f (x)^g (x) является совокупностью строгого неравенства f(x)>g(x) и уравнения f(x) = g(x) и поэтому может быть записано так, как обычно записывается совокупность: ЛИб° [f$if$; -ибо f(x)>g(xy, f(x) = Всякое «не равенство» f (x)Фg (х) можно записать и в виде совокупности двух строгих неравенств: М или f М>*<*>; f(x)<g(x). Несколько систем неравенств с одной переменной образуют совокупность систем неравенств в том случае, если ставится задача об отыскании всех тех значений переменной, каждое из которых удовлетворяет по крайней мере одной из этих систем. 143
Рис. 12 sl///////////////////////////////////////////////-. Рис. 13 '//////////У//////////////////- -8 -2 0 2 8 Рис. 14 Рис. 15 Пример 8. Решим систему неравенств (*2+*-4 « *2<64. Решение. Рассмотрим сначала первое неравенство. Имеем: С помощью кривой знаков (рис. 12) находим решение этого неравенства: (—оо; — 2)(J(0; 2). Решим второе неравенство заданной системы. Имеем х2 — 64<О, или (х—8)(дс+8)<0. С помощью кривой знаков (рис. 13) находим решение этого неравенства: (—8; 8). Показав решения первого и второго неравенств штриховками, имеющими разный угол наклона относительно числовой прямой, или, как на рисунке 14, штриховками, расположенными выше и ниже числовой прямой, найдем пересечение решений неравенств, образующих заданную систему: ( — 8; — 2)U(0; 2). Пример 9. Найдем область определения функции х+2 Решение. Задача сводится к решению следующей системы неравенств: 144
Преобразуем первое неравенство системы к виду jtj^O и с помощью кривой знаков (рис. 15) найдем решение этого неравенства: (— оо; — 2)U[2; оо). Преобразуем второе неравенство системы к виду С помощью кривой знаков (рис. 16) найдем решение этого неравенства: [-1; 1]U[2; 3]. Показав штриховками найденные решения первого и второго неравенств заданной системы на числовой прямой (рис. 17), найдем пересечение решений: [2; 3]. Пример 10. Решим совокупность неравенств Решение. Преобразуем первое неравенство совокупности к виду *3 (*— 10) (л:+ 10)>0. С помощью кривой знаков (рис. 18) найдем решение этого неравенства: [—10; 0] (J [Ю; оо). Рассмотрим второе неравенство совокупности. Имеем: Так как дискриминант квадратного трехчлена л:2 — 5а: +18 отрицателен, а старший коэффициент положителен, то *2 —5х+18>0 при всех значениях х, и, следовательно, разделив обе части неравенства на х2 — 5лс+18 и сохранив знак неравенства, получим равносильное неравенство х+9 -2 -7 Рис. 17 Рис. 18 145
У///////////////У///У Рис. 21 -J -2 Рис. 22 С помощью кривой знаков (рис. 19) находим решение последнего неравенства: ( — сю; —9] U (3; 15). Объединив найденные решения каждого из неравенств совокупности (рис. 20), получим (— оо; 0] U (3; + оо) — решение исходной совокупности. Пример 11. Решим совокупность систем неравенств гх —2>0, гЗл: — 9<0, \х2<16; Решением первой системы является числовой промежуток (2; 4). Решением второй — числовой промежуток [—10; 3). С помощью числовой прямой (рис. 21) получим объединение решений первой и второй систем: [—10; 4), т. е. решение заданной совокупности систем. Пример 12. Решим систему совокупностей неравенств гЗх-2>0; *2-4<0, *2-9<0. Решение. Перепишем заданную систему, решив ее отдельные неравенства: /*>|-; -2<*<2, 1*> — 2; — 3<л:<3. Покажем теперь на числовой прямой (рис. 22) решение первой совокупности (х^ — 2) штриховкой над числовой прямой, а решение второй совокупности (х> — 3) штриховкой под числовой прямой. Промежуток числовой прямой, заштрихованный дважды, является решением заданной системы. Итак, [—2; оо) — ее решение. Пример 13. Выясним, при каких значениях а оба корня квадратного трехчлена (а—2)х2 — 2алс + а + 3 положительны. 146
Решение. Так как по условию трехчлен имеет действительные корни, то его дискриминант D^O, т. е. должно выполняться неравенство 4а2 — 4 (а — 2)(а + 3)>0. По теореме Виета имеем a-2f где х\, хч — корни заданного квадратного трехчлена. По условию оба корня положительны, значит, xiJt2>0, j В итоге мы приходим к системе неравенств решая которую получаем i —3; а>2, ( а>2, откуда находим а<—3, 2<а<6. Пример 14. Выясним, при каких значениях а неравенство *2-8*+20 ах2+2(а+\)х+9а+4 выполняется при любых значениях х. Решение. Трехчлен jc2 —8*+20 имеет положительный старший коэффициент и отрицательный дискриминант, значит, л:2 — 8х+ + 20>0 при всех х, а потому знаменатель заданной дроби, т. е. ш:2 + 2 (а+ l)x+9a-\-4t должен быть отрицателен при всех х. А это возможно, если а<0 и D<0, где D — дискриминант трехчлена ах2 + 2 (а-\-1) х+9а-\-4. Значит, задача сводится к решению системы неравенств /а<0, \4(а+1)2-4а из которой получаем а<——. 4. Неравенства, содержащие переменную под знаком модуля. При решении неравенств, содержащих переменную под знаком модуля, иногда бывает полезна теорема 4 о равносильности неравенств (с. 137). Пусть, например, нужно решить неравенство \f{x)\ > \g(x)\. Воспользуемся тем, что если р (х) — некоторая функция, то \р(х)\>0и \р(х)\2=(р(х))2. Это значит, что по теореме 4 неравенство l/(x)l>lg(x)| равносильно неравенству (/ (x))2>(g(x)f. Кроме того, иногда полезно вос- 147
пользоваться геометрической интерпретацией модуля действительного числа. Дело в том, что геометрически \а\ означает расстояние от точки а числовой прямой до начала координат, а |а — Ь\ означает расстояние между точками а и Ь. Пример 15. Решим неравенство U-1K2. Решение. 1-й способ. Так как по определению х—1, если х— 1 ^0, —(jc — 1 )э если х— 1<0, то заданное неравенство равносильно совокупности двух систем: х-КО, ( х — 1^0, Г а:— К U-l<2;\ -(x- Из первой системы получаем 1^jc<3, из второй — 1<дс<1. Объединив эти решения, находим решение заданного неравенства: (-1:3). 2-й способ. Поскольку обе части неравенства неотрицательны при всех х и так как (| х — 1 |)2=(л:— I)2, то после возведения в квадрат обеих частей неравенства получим неравенство (лс—1)2<4, равносильное заданному. Далее имеем (х+1)(х — 3)<0, откуда находим, что промежуток (—1; 3) является решением заданного неравенства. 3-й способ. Известно, что если х\ и Х2 — точки числовой прямой, то расстояние между ними с/ = | jci — jcaI - Таким образом, U— 11 можно рассматривать как расстояние между точками 1 и х. Значит, решением заданного неравенства будет множество всех тех точек, которые удалены от точки 1 на расстояние, меньшее, чем на две единицы. На числовой прямой есть две точки, удаленные от точки 1 на две единицы. Это точки — 1 и 3. Значит, искомое решение — это промежуток (—1; 3). Пример 16. Решим неравенство Решение. После возведения в квадрат обеих частей неравенства получим: (2х-1)2<(Зл:+1)2, и далее jc откуда находим (— оо; — 2] U [0; +<»). Пример 17. Решим неравенство Решение. Это неравенство равносильно неравенству (2х+3^2 148
-f ?////////Z////////////////////////////////////////fs Рис. 23 которое можно переписать следующим образом: 4*+12*+9 ,^Q -5*г+24*+5 9ж2-12*+4 I>U> ИЛИ (Зх-2)2 ( откуда / о v <°- () Методом интервалов (рис. 23) находим решение последнего, а вместе с ним и заданного неравенства: -7"=f)u(f:»> О О / \ о / Пример 18. Решим неравенство U2 — Здс+2| ^2дг—х2. Решение. Неравенство равносильно следующей совокупности систем: (х2 — Зх + 2>0, (х2_3х+2<0, |x2_3x + 2<2jc-jc2; \-(jc2- решая которую находим последовательно: откуда y*<jc<1; x = 2; 1<х<2. Объединяя найденные решения, получим [у-; 2]. Пример 19. Решим неравенство U-4|>10. Решение. Отметим на числовой прямой точки, в которых выражения, находящиеся под знаками модулей, обращаются в нуль. Это точки — 3 и 4. Числовая прямая разбивается этими точками на три промежутка: (оо; —3], [—3; 4] и [4; оо). Рассматривая заданное неравенство на каждом из этих трех промежутков, получим совокупность трех систем: 149
Из первой системы находим лг<—4, из второй 0<х^4, из третьей jc^4. Объединяя найденные решения, получим: (_оо; -4)U(0; оо). Пример 20. Решим неравенство Решение. Так как по определению х—3, если —(л: —3), если х<3, то заданное неравенство равносильно следующей совокупности систем: Решим первую систему. Получаем: Ясно, что так как *>3, то2лг + 8>0, т. е. |2л: + 8| =2jc + 8, и поэтому система примет вид: откуда Решим вторую систему. Получаем: или откуда х< —-|-; — -~ Объединяя теперь решения первой и второй систем, получим f — оо; — — j U (— —; оо ) — решение заданного неравенства. 5. Задачи на составление неравенств Пример 21. У школьника было некоторое количество марок. Ему подарили альбом для марок. Если он наклеит по 20 марок на лист, то ему не хватит альбома, а если он наклеит по 23 марки на лист, то, по крайней мере, один лист останется пустым. Если школьнику подарить точно такой же альбом, на каждом листе которого наклеено по 21 марке, то всего у него будет 500 марок. Сколько листов в альбоме? Решение. Введем две переменные: х — число листов в альбоме, у — число марок, которые были у школьника. 150
Если школьник наклеит по 20 марок на лист, то расклеенными окажутся 20л: марок, что по условию меньше числа марок, которые были у школьника, т. е. 20* < у. Если он будет наклеивать по 23 марки на лист, то для расклейки достаточно использовать (х— 1) лист, на которых поместится 23 (х— 1) марок. По условию это число не меньше числа марок, имевшихся у школьника, т. е. 23 (х—1)^у. Наконец, в задаче сказано, что если школьнику подарить альбом, в котором наклеено 2\х марок, то всего марок у него станет 500, т. е. у + 21* = 500. Таким образом, можно записать следующую систему: (20х<у 23*—23 [ i Выразив у из уравнения системы и подставив результат в оба неравенства системы, получим систему неравенств г20*<500-21х 523 500 решив которую находим 74"^л:<тг- По условию х — целое число. Но в указанном промежутке содержится лишь одно целое число — 12. Значит, в альбоме было 12 листов. Пример 22. Путь из А в В плот проплывает за 24 ч, а катер тратит на путь из А в В и обратно не менее 10 ч. Если собственную скорость катера увеличить на 40%, то путь из А в В и обратно займет не более 7 ч. Сколько времени плывет катер из А в В и сколько времени плывет он из В в А? Решение. Пусть х км/ч — скорость течения реки (а следовательно, и скорость движения плота), у км/ч — собственная скорость катера. Тогда путь из А в В составляет 24* км, а время движения катера от А до В и обратно составляет (^г'+^Л ч. Если г г \у+х ' (/—х) собственная скорость катера станет равной 1,4у км/ч, то путь из А в В и обратно займет ( *—Ьгт~~) ч- По условию первое время не менее 10 ч, а второе — не более 7 ч. Таким образом, приходим к системе неравенств \Ау — В каждой из четырех имеющихся дробей разделим числитель и знаменатель почленно на х и введем новую переменную t = ^ * 151
Так как по смыслу задачи у>х, то /> 1. Таким образом, получаем систему неравенств относительно переменной / 24 24 Так как t> 1, то знаменатели дробей во втором и третьем неравенствах этой системы положительны, поэтому, освобождаясь в этих неравенствах от знаменателей и выполнив необходимые преобразования, получим 5/2 — 24*-5 <0, |49/2 — 240* — 25 > 0, и далее Решением последней системы является значение / = 5. В задаче требуется найти время движения катера от Л до В и от В до Л. Время движения от Л до В выражается дробью ~г, а значит, равно 4 ч. Время движения от В до Л выражается дробью -2£-, а значит, равно 6 ч. Упражнения Решите неравенства. 882. *(х-1)2>0. 883. (2 —дг)(3х+1)(2х—3)>0. 884. (3jt-2)(jt--3)3(*+l)3(x+2)4<0. 885. x3-64x>0. 886. *2+10<7лг. 887. x2-7x<z3. 888. — х2— 889. *2 + 5*+8>0. 890. 89!. (а:— 1)(дг2 —3jc + 8)<0. 892. (jc- 152
910. -H-r 9 £±i>_ 1 ' jc-2^^-2 2* 3. 911. x-i jc-t- 1 3x-2-x2-"7x-4-3x2- 25x-473 2-х 1-2* 9l4# _ c-15 3x+4* "We F+P^*3-**2" ^ 915 3(jc-4)""T'F=I^ jc-2 ' Решите системы неравенств. ) —Зх^ 2х+7 148 "*" 3 ~* 21 ' 13-х 2 * /Зх + 5 10 —Зх 2х+7 1 7х 11 (х+1) Зх— 1 \Т б ^'""з 917. | 3-- 918. 920. 922. 924. 925. 926. 10 — *)> 11 -5 (лг-3). £, 919. Гх2- 921. jc2>7. 923. 4х-2<х2+\<4х+6.. 3jc-2 И7. <0, i x3<16x, kr4-2x-i 928. 5£=7<4-^+^х^<4> х-5 ^ 5-х^х2-25 929. Г^-~Ш ^2х2 + х-1 V x2-f-x 930 930. -<1. Найдите области определения функций. f 2 с2—12х+11 */*2-49* 153
те. Решите совокупности неравенств и систем неравенств. 936. (х— 1)(л; — 2)(л:— 3)<0; *2<1. 937. *~ >0; g-^- 938. *2 —5*+8<0; x2 — 3jc+6<0; x2<\. 939. 5л:-20<л:2<8л:; 1<3^71^+8<2- 3 Решите системы совокупностей неравенств. 941. ((х+\)(х—3)>0; 1 л>25; 942. jc—3 х 5 ji_ я — 4 х— л- + 2< а: ' je — jc2~5x-f 6>0; a: —S 3-7л:+6>0. 946. При каких значениях а квадратный трехчлен х2+2(а+1)*+9а-5 а) не имеет действительных корней; б) имеет только отрицательные корни; в) име ет только положительные корни? 947. При каких значениях а квадратный трехчлен (а2 _ а _ 2) х2 + 2ах+ а3 — 27 имеет корни противоположных знаков? 154
948. При каких значениях а неравенство х2 + ах— 1 222 + Ъ< выполняется при любых х? 949. При каких значениях а система неравенств выполняется при любых х? Решите неравенства. 950. а) |х+5|>11; б) |2х-5|<3. 951. а) |3*-1|>5; б) |2х-4|<1. 952. а) |2х-1|<|4х+1|; б) |1—Эис| > |2х+3|. |1-2х|>3--х; б) |х+8|<Зх-1. |4-3*|>2-х; б) |2х-3|>х+4. |2*-3|>2х-3; б) |Зх+1|<Зх+1. ^2+1|1 959. \-2я*+Зх+Ь\>2. 97!- х*5х+б>2' 972# ^- 975. U| + |jc-1|<5. 976. U+ 978. |3x-l| + |2x-3|-|x+5|<2. 979. |x-l| + |2-x|>3 + x. 980. J±±±^<\ 981. ^zJgEZ^L 982. ||2x+ll-5|>2. 983. ||x-3| + l|>2. 984. Mx-l|+x|<3. 985. ||x-2|-x+3|<5. 986. |2x-|3-x|-2|<4. 987. |2x-|x+3| -»<•■ Решите задачи. 989. В двух ящиках находится более 29 одинаковых деталей. Число деталей в первом ящике, уменьшенное на 2, более чем в 3 раза превышает число деталей во втором ящике. Утроенное число деталей в первом ящике превышает удвоенное число деталей во втором ящике, но менее чем на 60. Сколько деталей в каждом ящике? 990. В двух бригадах вместе более 27 человек. Число членов первой бригады более чем в 2 раза превышает число членов второй бригады, уменьшенное на 12. 155
Число членов второй бригады более чем в 9 раз превышает число членов первой бригады, уменьшенное на 10. Сколько человек в каждой бригаде? 991. Если пионеров лагеря построить в колонну по 8 человек в ряду, то один ряд окажется неполным. Если построить по 7 человек в ряду, то рядов будет на 2 больше и все они будут полными. Если же выполнить построение по 5 человек в ряду, то рядов будет еще на 7 больше, но один ряд будет заполнен не весь. Сколько пионеров в этом лагере? 992. Имеется некоторое количество проволоки. Если ее намотать на катушки, вмещающие по 800 м проволоки, то одна катушка будет намотана не полностью. То же самое произойдет, если пользоваться только катушками, вмещающими по 900 м проволоки, причем таких катушек потребуется на 3 штуки меньше. Если же проволоку намотать только на катушки, вмещающие по 1100 м, то потребуется еще на 6 катушек меньше, но все катушки будут намотаны полностью. Сколько метров проволоки было? 993. Если жидкость разлить в бутыли емкостью 40 л, то при этом одна бутыль окажется не совсем полной. Если ту же жидкость разлить в бутыли емкостью 50 л, то бутылей понадобится на 5 меньше и все они будут заполнены. Если жидкость разлить по бутылям емкостью 70 л, то понадобится еще на 4 бутыли меньше, но опять одна бутыль будет неполной. Сколько было литров жидкости? 994. Двум бригадам общей численностью 18 человек было поручено организовать в течение трех суток непрерывное круглосуточное дежурство по одному человеку. Первые двое суток дежурили члены первой бригады, распределив между собой это время поровну. Известно, что во второй бригаде 3 девушки, а остальные — юноши, причем девушки дежурили по одному часу, а юноши распределили между собой остаток времени поровну. При подсчете оказалось, что сумма часов дежурства каждого юноши второй бригады и любого члена первой бригады меньше 9 ч. Сколько человек в каждой бригаде? 995. При покупке нескольких одинаковых книг и одинаковых тетрадей за книги уплатили 10 р. 56 к., а за тетради — 56 к. Книг купили на 6 штук больше, чем тетрадей. Сколько купили книг, если цена одной книги больше чем на 1 р. превосходит цену одной тетради? 996. Группа студентов, состоящая из 30 человек, сдавала экзамен. При этом выставлялись оценки 2, 3, 4, 5. Сумма полученных оценок равна 93, причем троек было больше, чем пятерок, и меньше, чем четверок. Кроме того, число четверок делилось на 10, число пятерок было четным. Сколько каких оценок получила группа? 997. Группа студентов решила купить магнитофон ценой от 170 до 195 р. Однако в последний момент двое отказались участвовать в покупке, и поэтому каждому из оставшихся пришлось внести на 1 р. больше. Сколько стоил магнитофон? 998. Изделие высшего сорта на столько же дороже изделия первого сорта, на сколько изделие первого сорта дороже изделия второго сорта, но эта разница в цене не превышает 40% от цены изделия первого сорта. Предприятие заплатило 9600 р. за изделия высшего сорта и столько же за изделия второго сорта. Общее количество всех этих купленных изделий составило 1400 штук. Известно, что цены изделий каждого сорта выражаются целыми числами рублей. Сколько стоит изделие первого сорта? 999. Рота солдат прибыла на парад в полном составе прямоугольным строем по 24 человека в ряду. По прибытии оказалось, что не все солдаты могут участвовать в параде. Оставшийся для парада состав роты перестроили так, что число рядов стало на 2 меньше прежнего, а солдат в каждом ряду стало на 26 больше числа новых рядов. Известно, что если бы все солдаты участвовали в параде, то роту можно было бы выстроить так, чтобы число солдат в каждом ряду равнялось числу рядов. Сколько солдат было в роте? 1000. Бригады, состоящие из одинакового числа рабочих, получили на складе спецодежду. Каждый рабочий получил по два комплекта спецодежды, а каждой бригаде выдали на 20 комплектов больше, чем было бригад. Если бы бригад было на 4 больше и каждой бригаде выдавали бы по 12 комплектов, то спецодежды на складе не хватило бы. Сколько комплектов спецодежды было на складе? 1001. Расстояние между пунктами А и В равно 105 км. Из Л в В выехал автобус, а через 30 мин вслед за ним выехал автомобиль, скорость которого 40 км/ч. Догнав автобус, автомобиль с той же скоростью возвращается в А. При каких значениях скорости автобуса он приедет в В раньше, чем автомобиль в Л? 156
1002. В магазине продаются красные и синие карандаши. Красный карандаш стоит 17 к., синий — 13 к. На покупку карандашей можно затратить не более 4 р. 95 к., причем число синих не должно отличаться от числа красных карандашей более чем на 5. Необходимо купить максимально возможное суммарное число красных и синих карандашей, причем красных нужно купить как можно меньше. Сколько тех и других карандашей можно купить при указанных условиях? 1003. На покупку роз и гвоздик можно затратить не более 30 р. 50 к. Роза стоит 2 р., а гвоздика — 1 р. 50 к. Число гвоздик не должно отличаться от числа роз более чем на 6. Необходимо купить максимально возможное суммарное число цветов, причем гвоздик купить как можно меньше. Сколько роз и сколько гвоздик можно купить при указанных условиях? 1004. Трое мальчиков вместе хотели купить две одинаковые игрушки, но общего количества их денег не хватило даже на одну. Если бы у первого мальчика денег было вдвое больше, то на одну игрушку денег хватило бы, а на покупку двух игрушек им не хватило бы 34 к. Если бы у третьего мальчика денег было втрое больше, то после покупки двух игрушек у мальчиков осталось бы 6 к. Сколько стоит одна игрушка, если известно, что у второго мальчика было на 3 к. больше, чем у первого? § 17. ИРРАЦИОНАЛЬНЫЕ НЕРАВЕНСТВА При решении иррациональных неравенств используются те же приемы, что и при решении иррациональных уравнений: возведение обеих частей неравенства в одну и ту же натуральную степень, введение новых (вспомогательных) переменных и т. д. Осуществлять решение можно, придерживаясь, например, следующего плана: 1) Найти область определения заданного неравенства. 2) Руководствуясь предложениями о равносильности неравенств (§ 16), решить заданное неравенство. 3) Из найденных решений отобрать значения переменной, принадлежащие области определения заданного неравенства. Пример 1. Решим неравенство — 4<х. Решение. 1) Область определения неравенства (1): дг>— о 2) Так как на множестве (1) — о — обе части неравенства (1) неотрицательны, то при возведении обеих частей неравенства (1) в квадрат получим 5л: — 4<x2, или х2 — 5х + 4>0. Это неравенство равносильно неравенству (1) в области его определения. Из последнего неравенства находим лг<1; х>4. 3) Из найденной совокупности х< 1; лс>4 решениями неравенства (1) будут лишь те значения х, которые принадлежат области определения неравенства (1), т. е. значения х, являющиеся решением следующей системы: Из этой системы находим — ва (1). 5 ; лг>4 — решение неравенст157
Пример 2. Решим неравенство ± (2) Решение. 1) Область определения неравенства (2): х^—2. 2) На множестве х> — 2 левая часть неравенства (2) неотрицательна, а правая часть может принимать как неотрицательные, так и отрицательные значения. Поэтому необходимо рассмотреть два случая: х+-~>0 и *+у-<0. В первом случае можно обе части >0 и *+у неравенства (2) возвести в квадрат (можно в том смысле, что получится равносильное неравенство), а во втором этого делать нельзя, да и не нужно, так как ясно, что при *+-т-<0 получается, что левая часть неравенства (2) неотрицательна, а Правая отрицательна, а это противоречит смыслу неравенства (2). Значит, во втором случае неравенство (2) не имеет решений. Итак, неравенство (2) равносильно в своей области определения следующей системе: r*+4->o, Из этой системы находим 3) Остается из решени жащие множеству х^ — 2, т. е. решить систему неравенств 3) Остается из решений х>^2. отобрать значения х, принадле- Получаем (jr-; + оо j — решение неравенства (2). Пример 3. Решим неравенство j.. (3) Решение. 1) Область определения неравенства (3): х> — 2. 2) Как и в предыдущем примере, надо рассмотреть два возможных случая: x-fy-^О и х+^-<0. Однако теперь во втором случае неравенство (3) выполняется при всех х из области определения (неотрицательное число в левой части неравенства (3) больше отрицательного числа в правой части неравенства (3)). Таким образом, неравенство (3) равносильно в своей области определения следующей совокупности: 158
Из первой системы находим ——^х<^- , а из неравенства х + — <0 получаем *< ——. Объединяя эти значения х, получим 3) Остается решить систему неравенств 2 ' -2. Получаем Г— 2; ^-J — решение неравенства (3). Замечание. При получении определенных навыков решения иррациональных неравенств можно не расчленять решение на три этапа, а сразу сводить данное неравенство к системе или совокупности систем более простых неравенств. Так, неравенство У*+2<х+—- (пример 2) можно заменить равносильной ему системой х+2< (*+у) а неравенство V*+2>*+— (пример 3) —совокупностью систем (*+2>0, Так мы будем поступать при решении следующих примеров. Пример 4. Решим неравенство УЗх—V2x+l>l. (4) Решение. Найдем область определения неравенства (4). Из системы / t получаем дг^О. Прежде чем возводить обе части неравенства (4) в квадрат, перепишем его следующим образом: 159
На множестве х^О обе части этого неравенства неотрицательны, значит, возведение их в квадрат есть равносильное преобразование. Имеем (л/3х)2>(1 +У2*+ О2» или 2У2х+1<х—2. Это неравенство (а с ним и неравенство (4) с учетом области определения равносильно системе неравенств k (2 л^чРТ)2 < (л:—2)2. Решение этой системы: Итак, [12; +оо) — решение неравенства (4). Пример 5. Решим неравенство V2xT5+V*^>8. (5) Решение. Найдем область определения неравенства (5). Из системы получаем х^1. На множестве х^1 обе части неравенства (5) неотрицательны. Поэтому, возведя в квадрат обе его части, получим равносильное неравенство 2х+5 + 2 У2х2 + 3х — 5 + *-1 >64, или -5> -3*+60. Так как на множестве х^ 1 правая часть этого неравенства может принимать как неотрицательные, так и отрицательные значения, то придется рассмотреть два случая, т. е. решить совокупность систем неравенств -Зх+60>0, {х> -З (2 У Г -5)2>(-Зх + 60)2; \- После упрощений получим: (х-10) (*-362)<0; Из этой совокупности находим 10<л:<20; х>20. Объединяя эти решения, получаем лг>10. (10; +оо) — решение неравенства (5). Пример 6. Решим неравенство 160
Решение. Положим для краткости у=—. Тогда неравенство (6) примет вид: Это неравенство равносильно системе неравенств решив которую находим —<1, откуда находим 2<л:< X о Осталось решить систему не- 4V3 равенств -^ Пример 7. Решим неравенство 4 Решение. Областью определения неравенства (7) является, как нетрудно найти, множество —l^x^l. Попытка решить неравенство (7), как в предыдущих примерах, возведением обеих его частей в квадрат наталкиваемся на значительные затруднения. Поступим по-другому. Рассмотрим выражение <f(x)=^Jl+x+\. Так как <р{х)>0 при любых допустимых значениях х, то, если обе части неравенства (7) умножить на <р(х) и сохранить знак неравенства (7), получится равносильное ему неравенство Далее имеем: Это неравенство равносильно совокупности систем неравенств: Г —1 ^х^ 1, ( — 1 < х>0, J*<0, 161
которая, в свою очередь, равносильна совокупности: , ( — \л/1 +х>4 л/1-х + З; \ Vl +x<4 уГ=^ Ясно, что при 0<л:^1 второе неравенство первой системы не имеет решений, значит, первая система не имеет решений. Ясно также, что при — 1<!х<;0 второе неравенство зторой системы верно, значит, вторая система выполняется при —1<а:<0. В результате получаем [—1;0) — решение неравенства (7). Пример 8. Решим неравенство У* — 2+уЗ — х>У*--1— уб—х. (8) Решение. Область определения неравенства (8): 2<ХЗ. Прежде чем возводить в квадрат обе части неравенства (8), необходимо убедиться в том, что обе его части неотрицательны. Однако оказывается это не так. Действительно, так как 2<л^3, то 1 <х— 1 <2 и 3<6 —х<4. А это значит, что У*— 1<уб —х, или У*— 1— уб—л;<0. Но Уде—2+УЗ—jc>0. Таким образом, при всех значениях х из отрезка 2^л:^3 неравенство (8) выполняется. Итак, 2<;*<3 — решение неравенства (8). В заключение рассмотрим пример уравнения, сводящегося в процессе решения к иррациональному неравенству. Пример 9. Решим уравнение 1. (9) Решение. Положим у=-\/х — 2. Тогда у2=х — 2, откуда х=у2 + 2. Таким образом, уравнение (9) можно переписать в следующем виде: или |у-2| + |у-3| = 1. Это уравнение равносильно следующей совокупности смешанных систем: —(У—2)—(t/—3)=1; \(y-2)-(y-3)= решением которой является множество. у^ Итак, решение уравнения (10) свелось к решению системы неравенств 2^Ух—2^3. Решением этой несложной системы, а с нею и уравнения (9) является отрезок [6; 11]. Проверка. Вполне понятно, что от проверки всех значений х из множества 2<х<3 подстановкой в уравнение (9) придется отказаться. Вместе с тем так как все преобразования, выполненные в процессе решения уравнения (9), были равносильными, то множество 2<л:<3 является решением уравнения (9). 162
Упражнения Решите неравенства. 1005. У3х— 2>1. 1006. 1010. У 1012. Ух2—х—12<х. 1014. У9х-20<л\ — 22лг>2лг—7. 1016. 1018. 1020. У*+3 —У*^4>2. 1031. jc-4 1032. 1033. 1034. (х+5)(лг— 2) + ЗУ*(*+3)>0. 1035. У*2 —3jc+"5 + *2<3* + 7. 1036. 2JC2—\/(jc — 3)(2jc — 7)< 13jc+9. 1037. 1038. 1039. 1041. \[х~^ 1043. у*«-2л:2+1>1-л:. 1045. , 4 — У2^с<2. 1040. 1042. У4^ 1044. 1046. (х- >0. —9. 1049.
1050. Z? у*—з 1051. У**+3*+4+уГ+1 > 1,4. 1052. 1053. Решите уравнения. 1054. f 18. ПОКАЗАТЕЛЬНЫЕ НЕРАВЕНСТВА Решение показательных неравенств вида a! W>ag(*x\ где г а>0, 1 основано на следующих двух теоремах: Теорема 1. £ели а>1, го неравенство а/(х)>а*(дс) равносильно неравенству f(x)>g(х). Теорема 2. Если 0<а<1, то неравенство а'(дс)>а*(х) равносильно неравенству f(x)<g(x). Пример 1. Решим неравенство х-3 Решение. Преобразуем неравенство (1) к виду Зх-1 З(х-З) По теореме 1 неравенство (1) равносильно неравенству Здг— 1 ^ З(х-З) ,ov 3(х-1) ^ Зл:-7 У ' (неравенства (1) и (2) одинакового смысла). Из неравенства (2) последовательно получаем неравенства 5 Из последнего неравенства, применяя метод интервалов (рис. 24), получаем (•— оо; 1)u(t*; у) —решение неравенства (1). Пример 2. Решим неравенство (0,04)5*-*'-8<625. (3) 164
Решение. Так как 0,04=77-77 =^ = 25"1» то неравенство (3) 1UU Zo можно переписать следующим образом: (25-1)5х-х2-8<625, или 25х2~5х+8<252. По теореме 1 неравенство (3) равносильно следующему неравенству: *2_5х+8<2 (4) (неравенства (3) и (4) одинакового смысла). Далее получаем (jc — 2) (jc—3)<I 0, откуда находим (2; 3) — решение неравенства (3). Пример 3. Решим неравенство 2*+2_2*+3-. 2X+4>5X+I — 5Х+2. (5) Решение. Последовательно получаем: 4.2х — 8-2х — 16-2х>5.5х —25-5х, (-20).2х>(-20).5х, 2Х<5Х. Разделив обе части последнего неравенства на 5х > 0, получим (2 \х / 2 \х / 2 \^ —) <1, или l-jr) <("57 " Здесь основание -§- удовлетворяет двойному неравенству 0<—<1. Значит, о о по теореме 2 неравенство равносильно неравенству противоположного смысла лг>0. Итак, промежуток (0; оо) является решением неравенства (5). Пример 4. Решим неравенство _J L_>n (0,5)х—1 1—(0,5)x+1 ^ Решение. Положим у=(0,5)х. Тогда заданное неравенство примет вид: !L откуда после преобразований получим неравенство У to—i)to—2) 165
Методом интервалов (рис. 25) находим решение последнего неравенства: Ку<-|-; у>2. Таким образом, задача свелась к решению следующей совокупности: 1<(0,5)*<-|-; (0,5)*>2, или 2°<2-*<21О82Т; 2~*>2. Из этой совокупности по теореме 1 получим 0< — Jt<log2 —; о — х>1, откуда ( — оо; — l)u[—log2 -|-; 0} — решение заданного неравенства. Пример 5. Решим неравенство i_52^-1 —20>0. Решение. Перепишем это неравенство следующим образом: 5.54 5 После замены у=5^* получим неравенство и далее у2 — 25у+ 100<0, или (у — 5) (у—20)<0, откуда 5<у<20. Таким образом, решение заданного неравенства сводится к решению системы неравенств 5<5^<20, или 5<5^<5log520# Эта система равносильна (так как основание 5>1) системе l<V^<'°g5 20, откуда l<jc<log|20. Итак, промежуток (l;log§20) является решением заданного неравенства. Пример 6. Решим неравенство 166
Решение. Перепишем это неравенство в следующем 4-32*-4.3'>^- + З.Зх. о После замены у=Ъх получим неравенство Умножая обе части этого неравенства на у>0, придем к неравенству 4у3-4у2>45 + 3у2, или 4у3 - 7у2 - 45 > 0. Найдя, что у = 3 — целочисленный корень многочлена 4у3 — 7у2 —45 и разложив затем этот многочлен на множители (с помощью деления его на у —3), получим неравенство (у — 3)(4t/2 + 5y+15)>0, откуда У>3. Таким образом, решение заданного неравенства сводится к решению неравенства 3* >3, откуда х> 1. Итак, промежуток (1; оо) является решением заданного неравенства. Пример 7. Решим неравенство 8х+18* — 2.27*>0. (6) Решение. Перепишем неравенство (6) следующим образом: Разделив обе части этого неравенства на 33*>0, получим равносильное ему неравенство Полагая далее У=("37 » придем к неравенству у3+у —2>0, или (у— 1)(у2+у + 2)>0, откуда у>1. Таким образом, решение неравенства (6) сводится к решению (2 \х / 2 \х / 2 \^ —J >1, или (—) >("о~) » откуда (так как основа- ние 0<—<1) по теореме 2 получим х<0. Итак, промежуток (— оо; 0) — решение неравенства (6). Пример 8. Решим неравенство (х2 + х+\)х<1. (7) Решение. Так как дискриминант квадратного трехчлена + х+1 отрицателен, а коэффициент при х положителен, то *2 + ;с+1 >0 при всех действительных значениях х. Поэтому правую часть неравенства (7) можно представить как (*2 + х+1) и переписать неравенство (7) следующим образом: 167
Так как относительно основания *2+*+1 неизвестно, больше оно единицы или меньше, то следует рассмотреть обе эти возможности. Если х2 + х+1>1, то к неравенству (8) применима теорема 1. Если же х2 + *+К1» то к неравенству (8) применима теорема 2. Таким образом, неравенство (8) равносильно следующей совокупности систем неравенств: \ или ( Первая система решений не имеет, а из второй системы найдем промежуток (— оо; —1), являющийся решением неравенства (7). Упражнения Решите неравенства. 1058. 63-*<216. 1059. юоо-о,зл/5+1<27. 1060. 35x+VFZrr.132VF:rf>35x-Vi7Z:r. 1061. (Ig3)3*-7>(log3 10)7х+3. 1062. (log2 3-log3 4-log4 3)x>(log3 4-log3 5-logs 6)2дс. 1063. 2х-5х5 1064. 3-х- 2. 1071. (3,2)2x+l<6,4. 1072. (0,(4))Jf2""1>(0,(6))Jf2+6. Ux-2) Ю73. 4х—22(ж-|)+8 >52. jj +53^+I>-625 < 15 749. 3x + 5 3X+1 —1 * 1076. V^51— 7Л/3Г1Г5*<162. 1077. 0,32+4+б+-+2х>0,372 (х 168
1079. 1 14-1 !+ +Г-1Г- 1080. 0,021"Т+4"8+"+с и 2" 1081. 52х+1>5х + 4. 1082. в^-в2'"1 >30. 1083. 52Vi + 5<5Vi+l + 5^. 1084. 42x+I+22x+6<4.8x+1. 1085. 22+х-22~х> 15. 1087. 52х(5х-2)>5(5х+Ю). 1088. 73х+1>7х-1(55.7х-41). 1089. 24х—23x+1—22x —2X+1—2<0. 1090. 36Х-2.18Х—8-9х>0. 1091. 4х+|«5 + 1092. 22х+2+6х-2.32'+2>0. 1093. 0,008х+ 5!-3х +0,04 <30,04. 1094. V9X —3Х+2>3Х—9. 1095. 25.2х— 10х + 5х>25. f 2 1097. \х-3\2х ~7 § 19. ЛОГАРИФМИЧЕСКИЕ НЕРАВЕНСТВА Решение логарифмических неравенств вида g(*)> (1) где ( a^Lx основано на следующих двух теоремах: Теорема 1. Если а>\, то неравенство (1) равносильно системе неравенств (f(x)>0, »0, (2) J(x)>g(x). Теорема 2. Если 0<а< 1, то неравенство (1) равносильно системе неравенств (f(x)>0, g(x)>0, (3) J(x)<g(x). Замечания. 1) В системе (2) можно опустить первое неравенство, так как оно следует из второго и третьего неравенств. Аналогично в системе (3) можно опустить второе неравенство. 2) Первыми двумя неравенствами систем (2) и (3) задается область определения неравенства (1). 169
3) При решении логарифмических уравнений область определения уравнений, как правило, не находилась (выписывались лишь условия, задающие эту область). И при решении логарифмических неравенств, как показывает практика, нахождение области определения заданного неравенства в большинстве случаев является нецелесообразным. Обычно условия, задающие область определения неравенства, подключают к тому неравенству, которое является следствием заданного логарифмического неравенства, и решают затем полученную систему. Пример 1. Решим неравенство Решение. Так как основание логарифмов равно 10, т. е. больше 1, то к этому неравенству применима теорема 1. В соответствии с ней получаем систему неравенств Ю>0, Опуская первое неравенство (как следствие второго и третьего неравенств) и выполняя упрощения в третьем неравенстве, получим систему р2 4* + з>0, (*2 + 8х + 7>0, идалее откуда находим (— оо; — 7)U(— 1; 1)11(3; «>) — решение заданного неравенства. Пример 2. Решим неравенство Решение. Так как — l=log1 2, то это неравенство можно переписать так: Т Здесь основание логарифмов равно —, т. е. к неравенству применима теорема 2. В соответствии с ней получаем следующую систему неравенств: 4лг— 1 4л:— 11 Эта система равносильна неравенству 4*—1 170
из которого находим [2; 2,75)U[4; оо)— решение заданного неравенства. Пример 3. Решим неравенство Решение. По теореме 1 заданное неравенство равносильно следующей системе неравенств: лг + 3 2-*>0, гг, >2~лг, x+3 откуда получаем: !(jc+2)(x—1) и далее ( — 3; —2)U(1; 2) — решение заданного неравенства. Пример 4. Решим неравенство log о,з (*3+8)-logo,3 (*2 + 4x + 4)<log0,3 (х + 58). (4) Решение. Перепишем неравенство (4) следующим образом: или logo,3 Так как основание логарифмов 0<0,3<1, то к этому неравенству применима теорема 2. Однако применение формулы loge/(x) — — \ogag(x)=\oga'^' могло привести к расширению области определения неравенства (4). Это значит, что к неравенству являющемуся следствием неравенства (4), нужно присоединить условия, которыми задается область определения неравенства (4), а не неравенства (5). Таким образом, неравенство (4) равносильно следующей системе неравенств: ? + 8>0, . . (*+2)2>0, *+58>0, ^ *+58>0, или х+2 171
Из первого неравенства системы находим, что дс+2>0. Это позволяет в последнем неравенстве системы освободиться от знаменателя, не изменяя знака этого неравенства. Получим систему х+2>0, 2 откуда затем найдем (—2; — -Ц) —решение неравенства (4). Пример 5. Решим неравенство log! (х-1)2 + 5 logo.5 (х-1)> -1. (6) Решение. Заметим, что Iog2 (х—1)2=2 Iog2 |х— 11. При этом так как областью определения неравенства (6) является множество значений х, удовлетворяющих неравенству х>1, заключаем, что | дг— 11 == jc — 1. Таким образом, Iog2 (jc— 1)2 = 2 Iog2 (л:— 1). Далее, logo.,(x-l)=!a^Ji = _log2(*-l). Итак, неравенство (6) можно переписать так: 4 log! (х- 1)-5 log2 (х- 1)+1 >0. Полагая y=log2 (х — 1), получим неравенство 4(/2-5t/+l>0, откуда У<-т-; У>1. Таким образом, решение неравенства (6) сводится к следующей совокупности неравенств: или 0<x-l<V2;x-l откуда находим (1; 1+V2)U(3; с») —решение неравенства (6). Пример 6. Решим неравенство log,_2 (2x-3)>logx-2 (24-бх). (7) Решение. Если основание логарифмов х—2> 1, то к неравенству (7) применима теорема 1, если же 0<х—2<1, то к нему применима теорема 2. Таким образом, решение неравенства (7) сводится к решению следующей совокупности систем неравенств: х—2>1, [ 0<х—2<1, 2х-3>0, I 2х-3>0, 24-6х>0, | 24-6х>0, 2х—3>24 —6х; \ 2х—3<24 — 6х. 172
97 97 Из первой системы этой совокупности получаем — <х<4, а из о второй 2<х<3. Итак, (2; 3)U(-g- ; 4 ) — решение неравенства (7). Пример 7. Решим неравенство Решение. Перепишем неравенство следующим образом: Так как относительно основания логарифмов х + — возможны два предположения, то, как и в предыдущем примере, приходим к выводу, что заданное неравенство равносильно совокупности систем неравенств: 5^, 2*-3 (££)<" с-5 \2 2jc-3 ИЛИ Г — 2,5<х< —1,5, Решение этой совокупности — ( — 2; — 1,5)и("з~; ^)U(5; °°) — является решением и заданного неравенства. Пример 8. Решим неравенство xlgx>10. (8) Решение. Неравенство (8) можно назвать показательно-логарифмическим. Выше (см. с. 128) мы отмечали, что при решении показательно-логарифмических уравнений целесообразно взять логарифмы от обеих частей уравнения по одному и тому же основанию. Этот же прием можно применить и при решении показательно-логарифмических неравенств. Естественно, при взятии логарифмов от обеих частей неравенства (как и уравнения) предварительно следует убедиться, что эти логарифмы существуют. Знак полученного логарифмического неравенства останется таким же, каким он был до логарифмирования, если логарифмирование выполнялось по основанию а> 1; если же логарифмирование выпол- 173
нялось по основанию 0< а< 1, то знак неравенства изменится на противоположный. Вернемся к неравенству (8). Обе его части принимают только положительные значения, и поэтому логарифмы этих частей существуют. Возьмем логарифмы по основанию 10. Так как 10>1, то получим неравенство lgjclgx>l (того же знака, что и неравенство (8)), равносильное неравенству (8). После преобразований получим неравенство lgx«lgx>l, т. е. lg2*— 1>0, откуда lgjt< — 1; lgjc>l. Из первого неравенства полученной совокупности найдем 0<х<0,1, а из второго *>10. Таким образом, (0; 0,1)11(10; оо) — решение неравенства (8). Упражнения Решите неравенства. x\ 1102. logs -JL->log3 (5-4 ПОЗ. log, (2-*)>log, x\ 1104. logj_(5+4A:-*2)>-3. 2 1105. logo,, (*2 + 75)-log0,, (*-4)< -2. 1106. 1(2+1)1(16*)1 (jc+2)>log1 4. 1114. (\ Т V 1125. log5V3Jt+4-logx5>l. 1126. log,(*3+l)-logx+,x>2. 1127. log,(jc+1X^(2-4 1128. Iogu_4,(2jt2-9jt+4)>l. 1129. log|jc+6|2.1og2(x2-JC-2)>i. ИЗО. log§,5 *+log0,5 л:- 1131. \+{££*x <{■ 1*32. 174
(г3 \2 ^2 log± i- j +9 log2 ^<4 (log± x)2. 1134. log^A:+log4A:>l. 1135. log, 5 V§- l,25>(log, л/5)2. 2Iog04X*logM2'5jf>l. 1138. 6 — 0,2 log4 x> 0,4 log3T-log33x 1136. 1137. 1139. 1140. 1141. 1143. 1144. 1146. 1147. log2(log3(2-log4A:))<l. 1148. 1149. 1150. 1152. 1153. >6,25log3*+2. *>2,5. 1142. 3lgx+2<3lg*' + 5-2. 8.5log2(x-l)"2>9Iog2(jc""1)~16.5log g2X<17. П45. log3 (4JC x"1 —2 )<х4-3,5. logj ^+-y 1-4 log2! *<1. 1151. -Wl-91og2, jc>l-41og1 log2 (jc- 1)- log2 T 1154. 1155. 1157. 1158. 1159. 1160. 1162. log. log2 0,3° log3 lOg5 log2 log3 2.1og2,2.1og24*>l. log! (*2-2)<l. 1156. 2 3x+6 з >*• (Iog2(2-log4*)-l)<l / i y°g3 \T/ log3log2(22*-3.2*+10)>0. (1 + logi jc—log9jc)<l T logx2 log,* xA > 0. 1163. 4 \ . 1161. log! log2logx-i9>0. logx l0g2 2 :(4"-12)<1. „68. 1170 1172. ) log. I 2 2 • lg V-«—40 625. 175
1174. log2 х-log3 2л'-Ь log3 *• log2 1175. \ogQ5(x + 2)\og2{x+\) + \ogx 1176. log,' (6*+1-36*)>-2. 1177. 1178. 25logI4x!og5X<30. 1179. 1180. U=<; Ut-гт. П81. 1185. Известно, что х=-^6 — одно из решений неравенства loga(jc2 — x — 2)> l(x2 + 2jc-f3). Найдите все решения этого неравенства. -у* —2 § 20. УРАВНЕНИЯ, СИСТЕМЫ УРАВНЕНИЙ И НЕРАВЕНСТВА С ПАРАМЕТРАМИ. Пусть дано уравнение F(x;a) = 0. (1) Если ставится задача отыскать все такие пары (х\ а), которые удовлетворяют данному уравнению, то уравнение (1)—это уравнение с двумя переменными х и а. Однако относительно уравнения (1) можно поставить и другую задачу. Дело в том, что если придать а какое-либо фиксированное значение, то уравнение (1) можно рассматривать как уравнение с одной переменной х. Решения этого уравнения, естественно, определяются выбранным значением а. Если ставится задана для каждого значения а из некоторого числового множества А решить уравнение (1) относительно х, то уравнение (1) называют уравнением с переменной х и параметром а, а множество А — областью изменения параметра. Условимся всюду в настоящем параграфе уравнение (1) понимать как уравнение с переменной х и параметром а. Уравнение (1)—это, по существу, краткая запись семейства уравнений. Уравнения этого семейства получаются из уравнения (1) при различных конкретных значениях параметра а. Так, уравнение 2а(а—2)л:=а—2, у которого областью изменения параметра а является множество Л={—1; 0; 1; 2; 3}, есть краткая запись следующего семейства уравнений: 176
6л:=—3 при а= — 1 0-х=— 2 при а=0 — 2х= — 1 при а=1 0-* = 0 при а=2 6лг= 1 при а = 3 Условимся всюду в дальнейшем под областью изменения параметра подразумевать (если не сделано специальных оговорок) множество всех действительных чисел, а задачу решения уравнения с параметром формулировать следующим образом: решить уравнение (1) (с переменной х и параметром а)—это значит на множестве действительных чисел решить семейство уравнений, получающихся из уравнения (1) при всех действительных значениях параметра. Ясно, что выписать каждое уравнение из бесконечного семейства уравнений невозможно. Тем не менее каждое уравнение семейства должно быть решено. Сделать это можно, если, например, по некоторому целесообразному признаку разбить множество всех значений параметра на подмножества и решить затем заданное уравнение на каждом из этих подмножеств. Для разбиения множества значений параметра на подмножества удобно воспользоваться теми значениями параметра, при которых или при переходе через которые происходят качественные изменения уравнения. Такие значения параметра будем называть контрольными. Не давая строгого определения контрольного значения параметра, покажем на примерах, как эти значения обнаруживаются, как с их помощью множество значений параметра разбивается на подмножества и как затем на каждом из подмножеств решается заданное уравнение (система уравнений, неравенство). Пример 1. Решим уравнение 2а (а—2)* = а —2. (2) Решение. Здесь контрольными будут те значения параметра, при которых коэффициент при х обращается в 0. Такими значениями являются а = 0 и а=2. При этих значениях а невозможно деление обеих частей уравнения на коэффициент при х. В то же время при значениях параметра аФО, аф2 это деление возможно. Таким образом, целесообразно множество всех действительных значений параметра разбить на подмножества и решить уравнение (2) на каждом из этих подмножеств, т. е. решить уравнение (2) как семейство уравнений, получающихся из него при следующих значениях параметра: 1) а = 0; 2) а=2; 3) Рассмотрим эти случаи. 177
1) При а=0 уравнение (2) принимает вид 0-х= — 2. Это уравнение не имеет корней. 2) При а=2 уравнение (2) принимает вид 0 •*=(). Корнем этого уравнения является любое действительное число. 3) При аФО и аФ2 из уравнения (2) получаем х=2а(а—2\' откуда *=^-. Ответ: 1) если а=0, то корней нет; 2) если а=2, то х — любое действительное число; 3) если [ аФО, то х=^-. [2 2а Пример 2. Решим уравнение (в_ 1) £ + 2 (2а+1) *+<4а+3)-0. (3) Решение. В данном случае контрольным является значение а=\. Дело в том, что при а=1 уравнение (3) является линейным, а при аф\ оно квадратное (в этом и состоит качественное изменение уравнения). Значит, целесообразно рассмотреть уравнение (3) как семейство уравнений, получающихся из него при следующих значениях параметра: 1) а=1; 2) аф\. Рассмотрим эти случаи. 1) При а=\ уравнение (3) примет вид 6х+7=0. Из этого уравнения находим #= — -g-. 2) Из множества значений параметра аФ 1 выделим те значения, при которых дискриминант уравнения (3) обращается в 0. Дело в том, что если дискриминант D=0 при а=а09 то при переходе значения D через точку ао дискриминант может изменить знак (например, при а<ао D<0, a при а>по D>0). Вместе с этим при переходе через точку ао меняется и число действительных корней квадратного уравнения (в нашем примере при а<ао корней нет, так как D<0, а при а>ао уравнение имеет два корня). Значит, можно говорить о качественном изменении уравнения. Поэтому значения параметра, при которых обращается в 0 дискриминант квадратного уравнения, также относят к контрольным значениям. Составим дискриминант уравнения (3): — =(2а+1)2 — (а— 1)Х Х(4а + 3). После упрощений получаем —=5а+4. Из уравнения —=0 находим а= —\ второе контрольное зна- 4 О чение параметра а. При этом если а<—у-, то D<0; если Таким образом, осталось решить уравнение (3) в случае, когда 178
4 (А а< — —» и в случае, когда I а> ——, \ \ Если а< ——, то уравнение (3) не имеет действительных корней; о о если же | а^——, то находим __(2g+ 1)±л/5а + 4 Ответ: 1) если а<—4"» то корней нет; 2) если а=1, то о х =--Ь 3) если о Замечание. При а= —■[- имеем *| =дг2= ——. о 3 Пример 3. Решим уравнение х 2 з-а2 а(др+1) (4) Решение. Значение а=0 является контрольным. При а=0 уравнение (4) теряет смысл и, следовательно, не имеет корней. Если же аФО, то после преобразований уравнение (4) примет вид: 2а~3 = 0. (5) Найдем дискриминант уравнения (5): -£-=(1 _a)2-(a2-2a-3)=4. Находим корни уравнения (5): Xi = a+1, х2 = а—3. При переходе от уравнения (4) к уравнению (5) расширилась область определения уравнения (4), что могло привести к появлению посторонних корней. Поэтому необходима проверка. Проверка. Исключим из найденных значений х такие, при которых jti-H=0, jd+2 = 0, *2+1=0, аг2 + 2=0. Если jci -4-1 =0, т. е. (a-fl)-f 1 =0, то а=— 2. Таким образом, при a=— 2xi—посторонний корень уравнения (4). Если jti+2=0, т. е. (а-|-1)-|-2=0, то а=—3. Таким образом, при а= —-3 х\ — посторонний корень уравнения (4). Если *2+1=0, т. е. (а—3)+1 =0, то а=2. Таким образом, при a=2 X2 — посторонний корень уравнения (4). Если дс2 + 2=0, т. е. (а—3) + 2=0, то а= 1. Таким образом, при a=l X2 — посторонний корень уравнения (4). 179
^ 42 ^ -2 Рис. 26 Для облегчения выписывания ответа сведем полученные результаты на рисунке 26. В соответствии с этой иллюстрацией при а= —-3 получаем *=-3-3=-6; при а=-2 х=-2-3=-5; при а=\ Итак, можно записать ответ: 1) если а=— 3, то х=— 6; 2) если а=—2, то jc=— 5; 3) если а=0, то корней нет; 4) если а=1, то аг=2; 5) если а=2, то лг=3; 6) если аф — 2у то лг,=а-И, Пример 4. Решим уравнение —(дс + а). (6) Решение. Здесь а = 0 — контрольное значение параметра (при а<0 левая часть уравнения не определена, а при а^О определена). Поэтому при решении уравнения (6) целесообразно рассмотреть следующие случаи: 1) а<0; 2) а^О. 1) Ясно, что при а<0 уравнение (6) не имеет корней. 2) Если а^О, то, выполнив возведение обеих частей уравнения (6) в квадрат и последующие упрощения, придем к уравнению 2^ах=\-2х-2а. (7) Здесь мы не обнаруживаем никаких новых контрольных значений параметра. Снова выполнив возведение обеих частей уравнения в квадрат и последующие упрощения, получим уравнение 2 (а— 1) х+4а2 —4а+1 =0, (8) у которого -J-= 4 (а -1 )2 — 4 (4а2 - 4а + П = 4 (2а — За2). Полагая D=0, находим ai=0, a2=-r вторые контроль- о 9 ные значения параметра. Заметим, что D<0, если а>— (на- о помним, что мы рассматриваем случай а^О). Таким образом, целе- 180
2 2 сообразно рассмотреть следующие случаи: а>—; О^а^—. В первом случае уравнение (8) не имеет корней, во втором получаем: Xl2—. Выше мы отмечали, что при а<0 уравнение (6) не имеет корней. Итак, решая уравнение (6), мы пришли к следующему результату: если a<0; a>-f-t то корней нет; если О^а^—, то корнями урав- «5 «5 нения (6) могут быть значения Эта осторожная формулировка связана с тем, что при решении уравнения (6) выполнялось возведение обеих частей в квадрат, что могло привести к появлению посторонних корней. Значит, найденные значения х\ и х2 необходимо проверить. Проверка этих значений подстановкой в уравнение (6) затруднительна, поэтому выберем другой путь. Отметим, что область определения уравнения (6) задается системой неравенств [\-(х+а)>0. Далее из уравнения (7) следует, что должно выполняться неравенство 1— 2jc—2a ^0. Значит, корни уравнения (6) должны удовлетворять системе неравенств 1— (*+а)>0, или J д:+а<1, 1— 2х—2а>0, 2 ' (9) Проверим, удовлетворяет ли системе (9) значение х\. Рассмотрим систему неравенств Второе неравенство системы (10) равносильно неравенству V2a—3a2<— a, которое в рассматриваемом случае имеет единственное решение: а=0. 181
Так как это значение а удовлетворяет и первому неравенству системы (10), то система (10) имеет единственное решение: а=0. Это значит, что х\ = 1ш~а+V2fl—3fl при а = 0 является корнем уравнения (при а=0 имеем jci=y-J, если же аФО, то х\ — посторонний корень. Проверим, удовлетворяет ли системе (9) значение *2. Рассмотрим систему неравенств '1— a—V2a—За Она равносильна следующей системе: / Х-^—^ \-\J2a—За и далее 4а2 — 4а+1>0, или Г(2а—1)2>0, 4220 | откуда 0<а<-|-. Итак, х2 = 1~~а~~^а~~3а—корень уравнения (6), если параметр а удовлетворяет следующей системе: откуда 0<a< щим образом: 1) если а<0; а>—, то корней нет; 2) если а = 0, то х= Таким образом, решение уравнения (6) можно записать следую- \_ 2 —kzt 3) если л» l_a_-J2a—За2 2 Заметим, что если а=0, то х\=х2. Это позволяет сделать запись ответа более короткой. Ответ: 1) если a<0; a>-i-, то корней нет; 2) если ~' — 2 Пример 5. Решим систему уравнений \г = ах+2ау. (11) 182
Решение. Заменим первое уравнение системы (И) суммой уравнений системы, а второе уравнение — разностью. Получим систему, равносильную исходной: *3+У3 = За(х+у), 3 — уъ = а{х—у), или ((х+у)(х2 — ху+у2 — За)=0, Последняя система, в свою очередь, равносильна следующей совокупности четырех систем: у х-у=0; (12) (13) 1 х2-ху+у2=3а, х-у=0; (14) г2 (хг—ху+у 3а, \х2+ху+у2 = а. (15) Из системы (12) находим: г jci =0, — решение системы (11) при любых значениях a£R. Из системы (13) получаем: {К.Г об) Здесь а=0— контрольное значение параметра. При а<С0 система не имеет действительных решений, если же а^О, то получаем: -у/а, Из системы (14) находим: (у=х, \2 3 \ Здесь, как и в предыдущем случае, а=0 — контрольное значение параметра. При а<0 система не имеет действительных решений, если а^О, то получаем: хъ =— л/За, V Система (15) симметрическая. Полагая г х+у = и> \xy = vy получаем г м2 — Зу = 3а, [2 откуда г и = 0, | и = — а. Таким образом, мы приходим к следующей системе уравнений: 183
J*+y=O, или г (/=--*, \xt/= — а, \х2 = а. Эта система совпадает с системой (16), которая выше уже решена. Ответ: 1) если а<0, то (0; 0); 2) если а>0, то (0; 0), (V5; -л/а)> (-л/а; л/а); (л/^а; л/3а)> (-л/За; -л/За). Пример 6. Решим неравенство Z£=iL>(i+3a)f. (17) Решение. Полагая а+3 = 0, находим а= — 3 — первое контрольное значение параметра. Значит, надо рассмотреть следующие случаи: 1) а<—3; 2) а= — 3; 3) а>—3. 1) Рассмотрим случай а<—3. В этом случае а+3<0 и неравенство (17) равносильно неравенству: 4(7х— 11)<(а+3)Х Х(1+За)х, т. е. неравенству: (3a2+10a-25)x>-44. (18) Полагая 3a2+ 10a—25=0, находим вторые контрольные значения параметра а: а=—; а=—5. о Решение неравенства (18) нужно, таким образом, рассмотреть в следующих случаях: (a<-5; a>f, (а=-5; \а<—3; \а<—3; с<—3, т. е. в случаях а<—5; а=— 5; — 5<а<— 3. В первом случае За2 + 10а—25 >0, и из неравенства (18) на- ходим: *>-3a4l_25- Во втором случае неравенство (18) принимает вид: 0-х> —44 — это верно при любых х. Если, наконец, — 5<а< — 3, то За2 + 10а— — 25<0, и из неравенства (18) находим, что *<""3a2-t-i0a--25' 2) Рассмотрим случай а=—3. В этом случае неравенство (17) не имеет решений. 3) Рассмотрим случай а> — 3. В этом случае а+3>0 и неравенство (17) равносильно неравенству: 4(7*-11)>(а+3)(1+За)х, или (3a2+10a-25)x<-44. (19) Как и для неравенства (18), контрольными значениями параметра а здесь являются значения а=— и а= — 5. Так как мы рассматри- о ваем сейчас случай, когда а> — 3, то из указанных двух контрольных значений параметра нам следует принять во внимание лишь одно: а^=—. Таким образом, при решении неравенства (19) должны быть 184
рассмотрены следующие случаи: а>—; а==~; "~3<а<"з"# В первом случае находим: *< ~~3а2+10а—25 * во ВТОРОМ — неравенство (19) не имеет решений, в третьем получаем: li 3a2+10a—25 ' Ответ: 1) если а= — 3; а = -|-, то неравенство не имеет реше- о ний; 2) если а<—5; —3<а<-|-, то *> ~За2+^а_25 ; 3) если -5<а<-3, а>-§-, то х< -За2+"а_25 ; 4) если а==~5' то — оо Пример 7. Решим неравенство ах2-2х+4>0. (20) Решение. Приравнивая к нулю коэффициент при х2 и дискриминант квадратного трехчлена ах2 — 2л:+4, находим первое контрольное значение параметра а=0 и второе контрольное значение а=— (причем если а>—, то D<0, если а^.—, то D^O). Решим неравенство (20) в каждом из следующих четырех случаев: -Ь 2) 0<а<-±-; 3) а=0; 4) а<0. 1) Если ^>4", то трехчлен ах? — 2х+4 имеет отрицательный дискриминант и положительный старший коэффициент. Значит, трехчлен положителен при любых х, т. е. решением неравенства (20) в этом случае является множество всех действительных чисел. 2) Если 0<а^—, то трехчлен ах2 — 2*+4 имеет следующие корни: х1.2=Ь*аЕ£, причем ^ a r a a Значит, решением неравенства (20) является следующая совокупность: х< '"У13*5 ; х> a 3) Если a=0, то неравенство (20) принимает вид: — 2х+4>0, откуда получаем лс<2. 4) Если а<0, то имеем Значит, в этом случае решением неравенства (20) является следующая система: 185
1+л/1-4а a Ответ: 1) если a>\-> то — oo<jc< + oo; 2) если 0<a< -J-, то х<{"-^=:Га ; *>1+тр« ; 3) если a=0, то х<2; 4) если a<0, то Пример 8. Решим неравенство * " ^ a2x-2a 2-а* a * Решение. Преобразуем неравенство (21) к виду а(ах — 2) ' ах—2 < и далее /1 «\ J I П.. I I I , ->0. (22) ■(-4) Неравенство (22) равносильно неравенству (21). Значение а = =0 — первое контрольное значение параметра. Приравнивая в числителе коэффициент при х2 к нулю, находим второе контрольное значение параметра: а=\. Наконец, дискриминант квадратного трехчлена (1 —а) х?-\-2х+1 +а равен a . Он обращается в нуль при уже отмеченном контрольном значении а=0. Значит, целесообразно рассмотреть следующие случаи: 1) а=1; 2) а=0; 3) / афО \аф\. Решим неравенство (22) в каждом из этих случаев: 1) При а=1 неравенство (22) примет вид: х* >0, откуда находим: х< — 1; х>2. 2) При а=0 неравенство (22) не имеет решений. 3) Если { ^5 J* то» выполнив в числителе левой части неравенства (22) разложение на множители, получим неравенство X а (23) равносильное неравенству (22), а значит, и неравенству (21). Неравенство (23), в свою очередь, надо рассмотреть в двух случаях: [па^х и а>1. 186
Рис. 29 В первом случае 1—а>0 и неравенство (23) принимает вид: (24) во втором случае 1— а<0 и неравенство (23) принимает вид: (25) Для решения неравенств (24) и (25) методом интервалов необходимо расположить на числовой прямой точки —1, а+1 , —в по- а— 1 а рядке возрастания. Для этого составим следующие разности: а—1 '* а а—1 а и выясним вопрос о знаке в каждом из полученных случаев. Рассмотрим разность Л i = ^ . Получаем (рис. 27), что если а<0, то ;4i>0; если 0<а<1, то 4i<0; если а>1, то Л1>0. Анализируя разность Л2==^£., получаем (рис. 28), что если а<—2, то Л2>0; если — 2<а<0, то Л2<0; если 0<а<1; а> 1, то Л2>0, наконец, если а= —2, то Л2 = 0. Рассмотрим теперь разность 187
Так как дискриминант квадратного трехчлена а2—а+2 отрицателен, а коэффициент при а2 положителен, то а2—а+2 > 0 при любых значениях а и знак разности Лз зависит лишь от знака знаменателя а (а— 1). Мы получаем (рис. 29), что если а<0, то Лз>0; если 0<а<1, то Л3<0; если с>1, то Аз>0. Проиллюстрируем теперь результаты исследования знаков разностей А\, А2, А3 (рис. 30). Неравенство (24) решается при условии 0фа<\ (на рисунке 30 эти значения а отмечены штриховкой), поэтому нужно рассмотреть это неравенство в каждом из следующих случаев: с< —2; —2<а<0, 0<а<1; а=—2. В первых трех случаях получаем соответственно: Решив неравенство (24) методом интервалов (рис. 31 а, б, в), находим: если а<— 2, то — ^ *± если —2<а<0, то •£- — 1; {А,>0 А2>0 А3>0 {А,>0 Аг<0 А3>0 [А,<0 [а3<о //7 -2 Рис. 30 б) *?/////////////// Рис. 31 77/////////////// 188
если 0<а<1, то ^Ц-<х< —1; х>—. а— 1 а Наконец, при а=— 2 неравенство (24) принимает вид: откуда находим х>—~. о При решении неравенства (25) нас интересуют знаки разностей Ль А2у Аз только на интервале а>\ (на рисунке 30 этот интервал не заштрихован). Значит, при а>\ имеем: С помощью метода интервалов находим решение неравенства (25): если а>1, то х< — 1; —^Ц а а а—1 Выпишем теперь окончательный ответ для неравенства (21): 1) если а<—2, то —Кх<—; х>^±±; а а—1 2) если а= —2, то х> —^-; о 3) если —2<а<0, то ^-<х< — 1; ^{ 4) если а=0, то неравенство не имеет решений; 5) если 0<о<1, то ^Ц-<ж< —1; х>—; 6) если а=1, то *<-— 1; 7) если а>1, то jc< —I; —<jc<^±1. а а— I Пример 9. Найдем все значения параметра а, при которых система уравнений г — 4х+ау=1+а \(6+а)х+2у=3 + а (26) не имеет решений. Решение. Данная система несовместна тогда и только тогда, когда ±- фИ±. (27) Из уравнения ^т~=-|-находим: ai = — 2; a2=—4. Из уравнения -|-= ]^а находим: аз = 1; а4=—2. 2 189
1////////////////////////////////////Л 2а-3 . 2-За 5) 2-Зп 2а-3 Рис. 32 Значит, условие -%-ф)^- выполняется, если (аф\\ 2 3+° \ф j=-2; а=-4 из системы I аф\ находим, что условие (27) равно- {а=-2; аф\ аФ-2 сильно равенству а= — 4. Итак, система (26) не имеет решений при а= — 4. Пример 10. Найдем все значения параметра а, при которых неравенство (*—2 +За) (ж—2а+3)<0 выполняется для всех х, принадлежащих отрезку [2; 3\ Решение. Данное неравенство имеет вид (х—х\) (х—х2)<0, где х\=2 — За, х2=2а—3. Решив его, получаем Х1<*<х2 (если Jti<*2) или Х2<х<х\ (если jc2<jci); если xi=x2, то решений нет. Таким образом, решением исходного неравенства служит либо интервал (2а—3; 2 —За), либо интервал (2—За; 2а—3) (рис. 32, а, б). Из условия задачи следует, что все точки из отрезка [2; 3] должны удовлетворять заданному неравенству, а это будет выполняться тогда и только тогда, когда точки с координатами 2 и 3 лежат внутри интервала (jci; х2) или (дс2; *i), т. е. когда 2а—3<2<3<2—За или когда 2—За<2<3<2а—3. Из системы неравенств 2а—3<2<3<2—За получаем си- Г2а-3<2, стему J2 —За>3, откуда находим, что а<—|-. Система неравенств 2 —За<2<3<2а—3 равносильна системе г 2 — За<2 |2а—3>3, откуда находим, что а>3. Итак, заданное неравенство выполняется для всех *6[2; 3] при а<—— или а>3. «5 Пример 11. Найдем все значения параметра а, при которых уравнение х2+4х-2 U-a| +2-a=0 (28) имеет два корня. Решение. Данное уравнение равносильно совокупности двух смешанных систем 190
х—а>0, -2 (jt-a)+2-a=0; (29) (30) Решим систему (29). Имеем \ x24-2x4-a+2 = Дискриминант уравнения x2 + 2x+a+2 = 0 D=—a—1. Если £><0, т. e. a> —1, то уравнение х2+2х + а+2=0 не имеет корней. Если D=0, т. е. а= — 1, то это уравнение имеет единственный корень л:= — 1; если Р>0, т. е. а< — 1, то уравнение имеет два корня: х\ = — 1 — V—а— 1, хг== — 1+У—а—1. Найденные корни должны удовлетворять неравенству х^а. Только в этом случае их можно считать решениями смешанной системы (29). Нам нужно рассмотреть два случая: 1) а= — 1; 2) а< — 1 (при а> — 1 уравнение системы (29), как мы отметили выше, не имеет корней, значит, и система (29) не имеет решений). 1) Если а= — 1, то ж= —1. Неравенство х>а в этом случае выполняется, значит, х= — 1 — решение системы (29). 2) Если а< —1, то xi = — 1 —\/—а— 1, аг2 В ) \/V Выясним, при каких значениях а выполняется неравенство ^ и при каких значениях а выполняется неравенство х2 ^ а. Обратимся сначала к неравенству х\ > а. Имеем последовательно: V—a—1<—a—1. (31) Разделив обе части неравенства (31) на выражение -у —а—1, принимающее при а< —1 только положительные значения, получим неравенство 1 ^-V—a—1, равносильное неравенству (31). Далее имеем: 1<—a—I, откуда а<— 2. Обратимся теперь к неравенству *2>а. Имеем: — 1+У—а— 1>а, У—а— Поскольку при а< —1 левая часть этого неравенства положительна, а правая — отрицательна, то неравенство выполняется при всех а< — 1. Окончательно получаем следующие решения системы (29): если а> — 1, то решений нет; если а= — 1,тох= — 1;если —2<а< — 1, то х= — 1+У— а— 1; если а<— 2, то jci = — 1— У— а— 1, х2= — 1+У—а— 1. Решим систему (30). Имеем г * ^а» 2 + 6 Из уравнения jc2 + 6jc+2 — 3a=0 находим: 191
Если а< ——, то действительных корней нет, значит, и система о (30) не имеет решений; если а=—^-, то х= — 3; если а>—-^-, Из найденных корней отберем те, которые удовлетворяют неравенству х^а. Если а=——t то х=— 3 и неравенство х^а вы- полняется. Значит, х= —-3 — решение системы (30). Пусть а>——. Выясним, при каких значениях а выполняется неравенство х3^а. Имеем: -3 — (32) Поскольку при а>—--левая часть неравенства (32) положи- «5 тельна, а правая — отрицательна, то неравенство (32) истинно. Значит, Хг — решение системы (30) при всех а>—~-. Обратимся теперь к неравенству х4^а. Имеем: (33) Поскольку при а> — ^-обе части неравенства (33) положительны, о то, возведя их в квадрат, получим равносильное неравенство 7 + За < ^(а + 3)2. Далее имеем: (a+l)(a+2)^0, откуда находим, что -—2 или а^ —1. Таким образом, х4 — решение системы (30), если —т"<а^—2 или а^—1. о Окончательно получаем следующие решения системы (30): если а< ——, то решений нет; если а= — —, то х= — 3; «5 о если —-|-<а<—2, то х3.4= — если —2<а< —1, то х= если а^ —1, то jc3t4= — Мы нашли решения систем (29) и (30). Решение уравнения (29) есть объединение найденных решений систем (29) и (30). Из приведенных выше рассуждений ясно, что это объединение нужно выполнить по отдельности при следующих значениях параметра: -1;4) а=-2;5) —1-< -2; 6) а=—Ь 7) а<-^-. 192
1) Если а> — 1, то уравнение имеет два корня: х3, х4, т. е. 3±л/7 + За. 2) Если а= — 1, то уравнение имеет два корня: — 1, —5. 3) Если — 2<а< —1, то уравнение имеет два корня: а:2, *з, т. е. — 1 -Ьл/—«— 1 и — 3—у/7 + За. 4) Если а=— 2, то уравнение имеет три корня: —2, 0, —4. 5) Если —^-<а<—2, то уравнение имеет четыре корня: з — ldhV—a—1; -: 7 2 6) Если а= —^-, то уравнение имеет три корня: —3; —1 ±—. 7) Если а<—г-, то уравнение имеет два корня: xi = — 1 -+- о 4-У—а—1, хг== — 1 — У—а—-1. Таким образом, уравнение (28) имеет ровно два корня при а> —2 или при а< ——. о Пример 12. Найдем все значения а, при которых уравнение -Agax (34) имеет единственный корень. Решение. Преобразуем уравнение к виду lg(x+3f = \g ах. Далее получаем (jt-f-3)2 = ax, откуда *2-(a-6)jt+9=0. (35) Уравнение (34) имеет единственный корень в следующих случаях: 1) уравнение (35) имеет единственный корень и этот корень удовлетворяет уравнению (34); 2) уравнение (35) имеет два корня, но из этих корней один является посторонним для уравнения (34). Рассмотрим первый случай. Уравнение (35) имеет один корень, если его дискриминант D равен нулю. Имеем D=(a-6)2-36 =a2-12a. D = 0 при a=0 или при a=12. Случай, когда а=0, отпадает, так как при а=0 правая часть уравнения (34) не определена. Если а=:12, то из уравнения (35) находим х=3 — единственный корень уравнения (35) и, как показывает проверка, удовлетворяющий и уравнению (34). Рассмотрим второй случай, когда D>0. В этом случае уравнение (35) имеет два корня: 193
Чтобы найденные корни были корнями уравнения (34), необходимо и достаточно, чтобы они удовлетворяли неравенству jc+3>0. Значит, из найденных корней уравнения (35) один будет корнем уравнения (34), а другой не будет корнем этого уравнения тогда и только тогда, когда где г *2>- Таким образом, задача сводится к решению совокупности двух систем неравенств {а —б + Уо — 12а^ о (а — 2 >-3> | — а — 6—Уа2— 12а ^ о 1 а — 2 <~3; I" Решим первую систему. Имеем: откуда а2—-12а>а2, т. е. а<0. Решим вторую систему. Имеем: 2—12а<-а. Эта система не имеет решений, так как либо а<0, либо —-а<0, т. е. либо первое, либо второе неравенство последней системы не имеет решений. Итак, второй случай имеет место при а<0. Окончательно получаем, что уравнение (34) имеет единственный корень, если а=12 или если а<0. Упражнения Решите уравнения с параметром а. 1189. (а2- 1190. (а3-а2- а 3 а+3 !. П92. 1193. 3*~2 + :;^+—= 0. 1194. д;2- а — za а—2 а 1195. а^-(1- 1197. (а2 + а 194 1196. (2а- 1)*2-(За + 1)х+а- 1 =0.
x—a 2 . 1212. log9 Ar+log9— 1214. =log9 log9 a. 1219. 1220. 1222. 1223. 1224. 2-loge2 (1 +д:) = 3 logfl V^T-loga* (*2- I)2. xXo*'x=a2x. 1221. log, 2 Решите системы уравнений с параметром a. 1226. f (3+a)*+2(/=3, 1227. Г (7- \ ax-^=3. \ 1228. Г x+ay= 1, 1229. ( x+y=a, \ax+y=a2. \xA+y4 = a\ 1230. /(Jc-t/)(x2-y2)=3a3f 1 1231. f x+y+z=l, 1232. ( I x+ay+z=a, I y+z*=3ax, 1 х+у+аг=а2. 1 i/3+z3=9aV. 1233. |x-(/=8a2, 1234. r^- 195
Решите неравенства. 1235. а2 + ах<\-х. 1236. 2*+3(а*-8)+-£<4 (*+-!-)-5. 1237. Зв+9 а+З + За-9* 1238. |^±|<4. 1239. 1240. 1241. ^[х~+2ах+Зх>0. 1242. 1250. logo(*- l)+loga — З. 1252. log, (х-а) > 2. 1254. e i±jaifL a 1-logaJC 0. 1256. 1257. При каких значениях а оба корня уравнения х2—6ах-\-(2 — больше 3? 1258. При каких значениях а оба корня уравнения х*—ах+2=0 принадлежат отрезку [0; 3]? 1259. При каких значениях а неравенство 4х — a-2* —a+3<0 имеет хотя бы одно решение? 1260. При каких значениях а неравенство ■— <0 выполняется при всех дг, принадлежащих отрезку [1; 2]? 1261. При каких значениях а неравенство (х—За) (х—а—3)<0 выполняется при всех х, принадлежащих отрезку [1; 3]? 1262. При каких значениях а уравнение х \х+2а\ +1— а=±0 имеет только один корень? 1263. При каких значениях а уравнение х \х—2а\ — 1— а±=0 имеет только один корень? 1264. При каких значениях а уравнение я2 — 4jc—2 \х—а\+а+2=0 имеет два корня? 1265. При каких значениях а система имеет хотя бы одно решение? 1266. При каких значениях а система \ах=\ имеет хотя бы одно решение? 196
1267. При каких значениях а система ( *2-(За+1)*+2а2 не имеет решений? 1268. При каких значениях а система *=а> ' не имеет решений? 1271. При каких значениях а система [х2-2(а+2)х+а(а+4)=0 имеет два решения? 1272. При каких значениях а уравнение единственный корень? 1269. При каких значениях а система *2-7*+6|+*2+5х+6-12 |*|=0, имеет ровно два решения? 127°- При каких значениях а система 1*'+5*+4| -9^+5^+4- Юх \х\ =0. -2(а + 1)ж+в(в+2)=0 имеет только одно решение? — lg(8x—6а—3)=0 имеет
Часть II. ТРИГОНОМЕТРИЯ Глава III. ТОЖДЕСТВЕННЫЕ ПРЕОБРАЗОВАНИЯ f 1. ТОЖДЕСТВЕННЫЕ ПРЕОБРАЗОВАНИЯ ТРИГОНОМЕТРИЧЕСКИХ ВЫРАЖЕНИЙ Напомним основные факты тригонометрии. I. Знаки тригонометрических функций по четвертям: Четверть I II III IV sin х + + — COS X + — — + + — + — ctg x + — + — II. Некоторые значения тригонометрических функций: X sin х cos х ctg x 0 0 1 0 — я ~6 1 2 л/з 2 л/з 3 л/з Т Л/2 2 л/2 2 1 1 л У л/з 2 1 2 л/з л/з 3 л Т 1 0 — 0 л 0 — 1 0 — Зл 2 -1 0 — 0 198
HI. Четность, периодичность. Функция t/=cos x является четной, остальные тригонометрические функции нечетные: sin (—х)= — sin дс, cos(—x)=cos jc, tg(-*)--tgx (хф^+яп). ctg (—x)= —ctg x (хфлп)*. Все тригонометрические функции являются периодическими. При этом Г=2л— основной период функций y=sinx, y=cosx, a Г = я — основной период функций t/=tg x, j/=ctg x. (Напомним, что основным периодом называется наименьший из множества всех положительных периодов периодической функции.) Таким образом, sin (x+2n)=sin (х — 2л)=sin x, cos (дс-f 2n)=cos (х-2л) = cos х, ctg (х + л)=ctg (х—л)=ctg х (х Ф лп). IV. Формулы сложения аргументов: sin (oc±P) = sin a cos P±sin Pcos a, (IV. 1) cos (a±0)=cos a cos pTsin a sin j3, (IV.2) ) (IV.3) ctg (a±Р)=^аС^^' V. Формулы, связывающие тригонометрические функции одного и того же аргумента: a + sin2a=l, (V.I) (V.3) (V.4) (V.5) VI. Формулы, связывающие тригонометрические функции аргументов, из которых один вдвое больше другого: sin 2a = 2 sin а cos а, (VI.1) * Всюду в дальнейшем, если нет специальных оговорок, подразумевается, что параметры п, k, if, m, ... принимают любые целые значения. 199
cos 2a = cos2 a —sin2 a, 1-f cos 2a=2 cos2 a, 1—cos 2a=2 sin2 a, 1 ±sin 2a=(cos a±sin a)2. VII. Формулы приведения: (VI.2) (VI.3) (VI.4) (VI.5) (VI.6) (VIJ) sin jc cos x tg* ctg jc я y-a cos a sin a ctg a tga T+« cos a — sin a — ctg a — tga я —a sin a — cos a — tg a — ctg a — sin a —cos a tg a ctg a Зя — cos a — sin a ctg a tga Зя , T+a — cos a sin a — ctg a — tga 2л —a — sin a cos a -tga — ctg a Для облегчения запоминания указанных в таблице формул приведения можно применять следующее мнемоническое правило: 1) если дуга а откладывается от горизонтального диаметра (я±а, 2л —а), то название функции сохраняется; если дуга а откладывается от вертикального диаметра (--pta, -£±а J , то название функции меняется (синус на косинус, косинус на синус, тангенс на котангенс, котангенс на тангенс); 2) перед полученной функцией ставят тот знак, который имела бы приводимая функция в случае, если 0<а<--р VIII. Формулы преобразования суммы тригонометрических функций в произведение: (VIII.1) sin a + sin p = 2 sin 2~L&- cos ^ sin a — sin p = 2 sin ^—^ cos cosa + cos p = cos a —cos p = 2 sin ^ (VIII.2) (VIII.3) (VIII.4) 200
ctg a±ctg P^£^g| (афлп, &фпк). (VII1.6) IX. Формулы преобразования произведения тригонометрических функций в сумму: COS a COS p=cos(<x-P) + cos(cx + P)> (K 2) sin a sin p=cos(«-p)-cos(«+p) (IX.3) Пример 1. Упростим выражение г/ ч sin3 (a -270°) cos (360° - a) /V ' tg3(a-90°)cos3(a-270°) # Решение. Так как функция y=cosx четная, а функции = sinx и y=tgjt нечетные, то получаем: с/ ч _(— sin (270° — a))3 cos (360° — a) f\a) (_tg (90°-a))3 cos3 (270°-a) ' Применяя затем формулы приведения, получим: f /Vi— (cos a)3 cos a / lub)-^- '. .o , . vo—COS ОС. (—ctg a)3 (—sin a)3 Итак, если sina=^=0, cosa^=0, т. е. аФ-^-п, то /(a) = cosa. Пример 2. Найдем sin a, cos a и ctg a, если tg a= —-|- и Решение. Применяя формулу (V. 4), получаем: 2a 25 cos2a= 1 Таким образом, либо cos a=4"» либ° cos a= — 4~- Так как по о о условию а принадлежит второй четверти, где косинус принимает только отрицательные значения, то cosa=—~. 5 Так как tga=^-^, то sin a = tg a-cos a, откуда sina=-|-. cos a 5 Осталось найти ctg a. Имеем ctg a=^—= — 4~- & Б tga 3 Итак, sin a=-f-, cos a= ~^-9 ctg a= —-|"- 20i
Пример 3. Вычислим tg-2-, если cos a=—0,6 и 180°<а< <270°. Решение. Воспользовавшись формулами VI.5 и VI.6, получим: 2 4 t . a 1+cos у Таким образом, задача свелась к нахождению cos -|-. Из формулы VI.5 следует Cos2-2-=1i^L«==1JzM==J-. Тогда 2 2 2 5 так как 180°<а<270°, то 90°<-|-<135о, и, следовательно, cosy<0. Значит, cos у-= — -у-^-= — g • Итак, tg2-j-= .Так как 45°<-f<67°30', то tg-f>0, т. е. 4 4 4 V 20 2 Пример 4. Вычислим sin a, cos а, tg а, ctg а, если а= 112°30/. Решение. Из формулы (VI.5) следует: Так как 90°<112°30/< 180°, то cosa<0. По условию 2а = 225°, значит, cos Аналогично, воспользовавшись формулой (VI.6) и учитывая, что по условию a — дуга второй четверти, получаем: sin tg 112° 30х = - ctg 112° 30'= —г-= 1 -л/2. Пример 5. Вычислим 16 sin -^-sin у-, если cos a=-|-. Решение. По формуле (IX.3), а затем (VI.5) получаем: h |.Л й_ re 202
=8 (cos a —cos 2a)=8 (cos a —2 cos2 a+1) = -8(f-2(f)'+'H- Пример 6. Докажем, что если tg a=i-, sin P=™, °<«<f и 0<P<f-> TO Решение. Вычислим tg (a+2p). Имеем: •-т Теперь нужно найти значение tg 2р. Для этого воспользуемся тем, что sin Р=-р=г, 0<р<-~. yio z Имеем: Значит, tg(a + 2p)= 7 4 =1. 1 L.± 7 4 По условию 0<а<-2- и 0<р<у-, значит, 0<2р<я. Но tg2p=-|->0, значит, 0<2р<--р а потому 0<а+2р<я. Но в интервале (0; я) функция tg x принимает значение 1 только в точке -J-. Значит, a+2p=-J-. Пример 7. Проверим равенство sin 47° + sin 61°-sin 1 Iе —sin 25°=cos 7°. Решение. Используем для преобразования левой части равенства формулы из группы VIII. Имеем: (sin 47° + sin 61е)—(sin ll° + sin 25°)=2 sin 54° cos 7°- -2 sin 18° cos 7°=2 cos 7° (sin 54°-sin 18°)= =2 cos 7°.2 sin 18° cos 36°. Если полученное выражение умножить и разделить на cos 18°, то можно воспользоваться тем, что 2 sin 18° cos 18°=sin 36°. 203
Тогда получим: Значит, исходное равенство верно. Замечание. Во многих случаях, когда имеется произведение вида sin a cos 2а cos 4а-...• cos 2яа или вида cos a cos 2а cos 4а«...«cos 2па, оказывается полезным прием, который мы применили в примере 5. Он заключается в том, что данное выражение умножают и делят на cos а или sina, с тем чтобы, использовав формулу 2 sin a cos a = sin 2a, затем формулу 2 sin 2a cos 2a=sin 4a и т. д., упростить заданное выражение. Проиллюстрируем сказанное на следующем примере. Пример 8. Вычислим cos ^- cos ^ cos ^ cos ^r cos -т£ cos -^. 00 ОО О5 00 ОО ЬЬ Решение. Обозначим заданное произведение через А. Умножим и разделим А на 2 sin ^-. Так как 2 sin ~-cos ^-=sin т-r , то DO DO DO DO . 2я 2я 4я 8л 16я 32я л^81П65СО865СО565СО565СО5бГСО865- Далее имеем: и т. д. В итоге получим: . 64я 8Ш /1 26 sin £=• 64 sin ^=- 64 sin zz DO DO DO Пример 9. Докажем тождество cos'2a lsin4a. ctga — tga 4 Доказательство. Преобразуем левую часть тождества: cos2 2a cos2 2a cos2 2a cos2 2a sin a cos a ctga—tga cos a sin a sin a cos a sin a cos a По формулам (VI. 1) и (VI.2) sin a cos a=—s'n 2a» cos2 a —sin2 a=cos 2a, Поэтому cos22asinacosa=scoS22aiHn2a==l sin 2a CQS 2a ■ sill 4a. J cos2 a—sin2 a 2 cos 2a 2 4 204
Итак, тождество доказано. При этом cosa=£0 и sin а Ф О, т. е. ^г+nk и афпк, а значит, аФ-^-k. Кроме того, cos2 a — sin2 афО, т. е. cos 2а^0, откуда 2аФ ф+пк,шиаф+к. Таким образом, приходим к выводу, что тождество выполняется при аф^. Пример 10. Докажем тождество Решение. Разложим на множители числитель и знаменатель выражения, содержащегося в левой части тождества: tg22a-tg2a (tg2a-tga)(tg2a + tga) = 1—tg2 2a tg2 a (1 -tg 2a tga)(l +tg 2a tga) tg2a+tga , tg2a—tga 1— tg2atga*l+tg2atga * Далее, использовав формулу (IV.3) (с. 199), получаем: *tg (2a—a)=tg 3a-tg a. Доказанное тождество выполняется при аф-^+лк, 2аФ-^--{- Ъ*Ф±+пт, т. е. при аф^+^f и аф^+^f (множество Р всех чисел вида -|-+л* содержится в множестве М всех чисел вида f +f). Пример 11. Докажем тождество 4 sin a sin (60° — a) sin (60° + a)=sin 3a. Решение. Здесь целесообразно применить для левой части тождества формулы из группы (IX) (с. 201). Имеем: 4 sin a sin (60° — a) sin (60° + a)= ==4 sjn acos(60o--a~60o~a)~cos(60o — = 2 sin a (cos ( — 2a)—cos 120°)=2 sin a(cos 2a+-£-) = = 2 Sin a COS a = 2 + 4-sin a= —sin a + sin 3a&-|-sin a = sin 3a. Таким образом, тождество выполняется при всех действительных значениях а. 205
Пример 12. Докажем, что если a + p+Y = -§-, то tg a tg p + tg p tg v + tg Y tg a= 1. (1) Доказательство. Преобразуем левую часть равенства, учитывая, что по условию 7=-?—(a + p): tg a tg p+tg P tg Y + tg Y tga = tg a tg P+tg y (tg P+tg a)= = tg a tg p + tg (-|—(a + p)) (tg a + tg P) = tg a tg p + ctg (a + + P) (tg a + tg p)=tg a tg p+j^L-^Ctg a+tg p) = = tg a tg Р + \^+а^рР(^ a + tg P) = tg a tg p+1 -tg a tg p= 1. Таким образом, тождество (1) доказано (a, p, у=-^~\-лп). Пример 13. Докажем, что если у-<а<я, то Доказательство. Имеем: V2 Ctg a+ii^=^2 ctg a+1 +ctg" a==^(1 +ctg a)2=='1 +ctg a|' Так как в интервале ^-<а<я выполняется неравенство ctga< —1, то в этом интервале l+ctga<0 и, следовательно, |l+ctga| = — 1— ctg a. Итак, если -^-<а<я, то тождество (2) доказано. Пример 14. Докажем, что если sin a+sin p =2 sin (a + p), где a + рт^яЛ, то tgftgf=i-. (3) Доказательство. Преобразуя левую и правую части равенства sin a + sin p=2sin(a + p) по формулам групп (VIII) и IV, получаем: 2 sin Ц&cos ^=4 sin «±£Cos £±^. (4) Так как а + р#=яЛ, т. е. 2^=^2* то cos^-±£-=^0 И i^i^O и тогда из равенства (4) следует: cos 2^=2 cos Ц±. (5) 206
Рассмотрим выражение tgy-tg -|-. Имеем: . а . В а — ft sin-o"sinlT cos—^ JL= ? 2_= 1 ё 2 ё 2 a p a-p , a + P cos у cos -|- cos —2—+ cos -y- (мы применили формулы (1Х.З) и (1Х.2). Воспользовавшись равенством (5), получим: a —p a-fP . а + р а + р cos -^— cos -^ 2 cos -тр— cos -^ j 1273. а —Р a-f р а-ЬР , а + р 3 " cos -у^+ cos -Х£ 2 cos -yil+ cos -^- Итак, равенство (3) доказано. Упражнения Упростите выражения. C0S (iF™ a ) Sin \Т+ « ) tg (я — a) sin 1274. ■ n — a) in(|+a)tg(|+p) Зя ft\ Y-p J я—a) 1275. = =-. 1276. -t4; ctg 2a. . a . a sin 4a ctgy-tgy 1-smg cos a О РПС2 Г§ _— 1 1279. — 2tg(~—с 1280. cos2 (a + p)+cos2 (a-p)—cos 2a cos 2p. sin a + sin 3a + sin 5a sin a + sin 3a + sin 5a + sin 7a 1282. cos a +cos 3a+cos 5a ' " cos a + cos 3a + cos 5a + cos 7a ' 1283. V2si" «-«*«. 1284. COs4«+4cos2«+3. sina-cosa 1 +cos 4a# 1 +cos 2a * sin2 2a+4 sin2 a-4 (-£+-£ Vsin2(~4V 1288.— 207
Вычислите без помощи таблиц. sin 10° cos 20** + cos 10° sin 20° sin 9° cos 39°-cos 9° sin 39° cos 19° cos 11°-sin 19° sin 11° ' Зя 5л , . Зя . 5я cosTcos-+smysm-. 1291. cos 15°. 1292. tg 15°. 1293. sin 285°. 1294. cos 165°. 1295. cos 292°30/. 1296. 2 sin 40° + 2 cos 130°—3 sin 160°—3 cos (—110°). 1297. cos 10° cos 30° cos 50° cos 70°. 1298. 16 sin 10° sin 30° sin 50° sin 70° sin 90°. 1299. tg9o-tg27°-tg63o 1300. Вычислите sin x, cos x, tgx, если ctgjt=— 2 и -^- о 1301. Вычислите sin x, tgx, ctg xt если cos*=—— и о Z 1302. Вычислите cos x, tg x, ctg х, если sin х= —— и -^<х<2я. lo Z 1303. Вычислите sin 2x, cos 2xf tg2x, ctg 2дг, если cosjt=-pr и lo z „ЛЛ- т> 5 sin x+7cos х . 4 1304. Вычислите ^ —--—, если tgJt=TT. 6 cos x—3 sin x ь 15 1305. Вычислите cos I ——а V если sina=—— и ~я<а<2я. 1306. Вычислите cos(a + p), если sina=—, sinp=-r=, 0<a<-^-, 0<p<-jr-. V5 лДО 2 2 1307. Вычислите, a) tg2a + ctg2a; б) tg3a + ctg3a; в) tga—ctg a, если tga + +ctga=3. 1306. Вычислите sin-«г, cos"o"» ^"o"» если а) cosa=0»S и 0<a<-^-; 6) tga = =3-|-и 180°<a<270°. a 336 1309. Вычислите sin — , если sin а=тгг и 450°<а<540°. 1310. Вычислите sin a, cos a, tg a, ctg а, если tg -^-=л/2. 1311. Вычислите tg-jr-, если sin a+cos а=^-г- и 0<а<-5~. Проверьте равенства. 1312. siny|cos^=—. 1313. tg55o-tg35°=2tg20°. -_._ л , Зя 1 1314. cos-^+cos-^^-^-. о о z «о.е 1—4 sin 10° sin 70° f !3!5- 27ПГТ05 =L 1316. 8 cos 10° cos 20° cos 40°=ctg 10°. -.-- я 2я 4я 1 1317. cos-y-cos-s-008-^-—— -g"- *o*o Я ЗЯ 1 1318. cos-=-cos-=-=—т. О Э 4 .лв_ я Зя 7я 9я я 2я 4я 8я 131». cos ^ cos ^cos 20 cos ^^ cos ш cos ^ cos ^ cos ^. 208
„_ . л . 2я. 4я гг 1320. tgytgytgy=V3. 2л , 4я , 6я 1 1321. cosy+cosy+cos-y^—у. 1322. tg 20° + tg 40° + tg 80° —tg 60°=8 sin 40°. 1323. tg 30° + tg 40° + tg 50° + tg 60° = 8л^ °°S 2°° . «3 1324. sin 70°+ 8 cos 20° cos 40° cos 80° = 2 cos2 10°. 1325. cos 24° +cos 48° — cos 84° — cos 12°=y. 1326. tg6 20° —33 tg4 20°+ 27 tg2 20°=3. ««-, • 2 n • 2 2я . 2 Зл 7 1327. sin*-=•sm ? sm ?~Щ. 1328. tg 55° tg 65° tg 75° = tg 85°. 1329. tg236°tg272° = 5. Докажите тождества. 1330. ctg (y-« ) sin (y+« ) sin (<*-y ) + tg (я + а) cos (я+а) cos (2я-а)=0. 1331. sin (a —270°) cos (a+90°) tg (3a- 180°) = cos (180° —a) sin (180° — a) ctg (90° — 3a). 1332. sin(p-y) sin(Y-a) sin(a-p)=() cos p cos y cos у cos a cos a cos 0 1OOO sin2 3a cos2 3a 1333. —г-ч 9—=8 cos 2a. sin** a cos'* a 1334. -д/cos2 a cos2 p— у sin 2a sin 2p + sin2 a sin2 p= |cos (a + p)|. 1335. 3 (sin4 a+cos4 a)—2 (sin6 a + cos6 a)= 1. 133e. ii^_^_! +^iL=tg3a+ctg3a. smJ a sin a cos a cos' a o cos3 a — cos 3a sin3 a + sin 3a cos a sin a 1338 cos2a 1~tga l+sin2a 1+tga* 1339. 1—sin 8a = 2 cos2 (45° +4a). a . a cos T-sin T 1340. = -=—i tga. a , . a cos a cos+sm 1341. (cos a + sin p)2+(sin a-cos p)2=4 cos2 Ю4О 1—2 cos2 a . . 1343. —: =tga — ctg a. sin a cos a 1344. sin2 a + cos (——a j cos (y+a J=-j-. 1345. 4 sin (a+y ) sin (a-y )= -3+4 sin2 a. 209
1346. 2 cos a cos P cos (a+ P) = cos2 а + cos2 р —shr (а + р). r i о 1347. cos а+cos p+cos v+cos (a + p + v)=4 cos- cos cos sin a + sin p sin (a + p) sin (a — P) 1349. cos a —£~cos Зое—^- cos 5a =8 sin2 a cos2 a. 2 sin a— sin 3a + sin 5a 2 cos 2a cos a — 2 cos 2a+cos За ~~ а 1351. cos a + cos (120° —a)+cos (120° + a)=0. л/3 1 -£-cos2a —— sin 2a 1354. 1355. 1356. 1357. 1358. 1359. 1360. 1361. tg 3a = tg a tg (60° +a) tg (60°— a). cos a cos 2a cos 4a cos 8a cos 16a =тпг—:— . 32 sin a 9 cos 15a + 3cos7a + 3cos 19a+9cos 11 a = 24 cos3 2a cos 13a. 3 — 4 cos 2a + cos 4a=8 sin4 a. "ч/ V "\/ V h т:hi 1+cosa 1— cos a sina=V2, если 0<а<я. v t4| 1 T— sin4* a cos2 a Дг"» если —5 sin 2a 2 2 a (1 +ctg a)+cos2 a (1 +tg a) a—-?- \ если —^- 1362. Vctg a + cos a + Vctg a —cos a = 2cos- 1363. 1IT- _,__.- иГ4;8Ъа- 2tga, если — у+2л£<а<у+2л/г, 1364. — 2tga, если у+2л/г<а<у+2л/г, » еСЛИ а, если —:—г-» если —s- sin 2a 2 1365 210 VI + cos 2а + VI ~ cos 2a + -у/2 (sin a+cos a)=
2^(sina+cosa), если 2л/г<а<у+2я£, sin а, если у+2л/г<а<я+2я£, О, если я4-2я/г<а<-х-я+2я£, 2-yJ2 cos а, если -£+2nk < а < 2я+2nk. 1366. У!+2 sin a cos а= [V^cosfa —j-Y если _£ —V5c —j), если ^ 1367. sin a+sin p+sin у—sin (a + p+Y)=4 sin ^-t& sin £i^ sin jjp 1368. tg 2a tg (30° — a) + tg 2a tg (60° — a)+tg (60° —a) tg (30° —a)= 1. 1369. cos a+cos p+cos v=l+4sin — sin-^-sin-|-, если а + р+у=я. 1370. sin a + sin p + sin y=4 cos—cos-~cos-|-, если а 1371. tga+tgP+tgY=tgatgptgv, если a 1372, sin a cos p cos y+cos a sin p cos y+cos a cos p sin Y = sin a sin p sin y» если + р+ 1373. sin2na+sin 2np+sin2/iY=(—ir+l'4sin/zasin nPsinny, если Докажите утверждения. 1374. Если tg a+tg P+tg Y=sstg a tg p tg y, to a+fi+y=nn. 1375. Если tg2a—ctg2p—ctg2Y=tg2actg2pctg2Y, то a 1376. Если tg(a + p)sin y=cos y, то a + p+Y=-o-+Jw. 1377. Если 3 sin a=sin (a+2p), то tg (a+P)=2 tg p. 1378. Если sin2 p=sin a cos a, to cos 2p=2 cos2 (-T'+a )• 1379. Если 2 sin a—3 sin (a + p)=2 sin p, то sin 5Lz£:sin ^dlft 1380. Если cos2a+cos2p=m, то cos (a+P) cos (a — p)=m — 1. ioo« с ^п P n i i i i tg(a—P) m+n 1381. Если . ,n , o4=— и m > \n , то -^-j ^=—•—. sin(2a + P) m * tg a m—n 1382. Если ( cosa+cosp=m, 2 . 2 , л I . . . Г и nr + ггфО, то lsin a+sin p=n ' sm(a+p)= 1383. Если Г m ctg a=a, . , 2 , 2ч л I . л * то Ь (а2+/п2)=2атя. ^nsm2a=6 v 7 1384. Если t sin a+cos a=m, I n^ a+cos3 a=n, то m3—3m+2n=0. 211
1385. Если (tga + ctga = m, 1 i -cos а=я, то ь cos a m3 1386. Если ( a cos3 а + За cos a sin2 a = m, | a sin3 а + За cos2 a sin а = л, то f 2. ТОЖДЕСТВЕННЫЕ ПРЕОБРАЗОВАНИЯ ВЫРАЖЕНИЙ, СОДЕРЖАЩИХ ОБРАТНЫЕ ТРИГОНОМЕТРИЧЕСКИЕ ФУНКЦИИ Напомним определения обратных тригонометрических функций. 1) у=arcs in x — это функция, определенная на отрезке [—1; 1], обратная функции * = siny, у£ [~""5"; Тт Таким обРа3(>м> ((/==arcsin x) о 1 V sin у=х. Для любого х из отрезка [—1; 1] имеем: -i<arc8in*<-|.t (1) sin(arcsinx)=jc. (2) 2) t/ = arccos x — это функция, определенная на отрезке [—1; 1\ обратная функции x=cosy, у 6 [0; я} Таким образом, (y=arccos x) о у cos y=x. Для любого х из отрезка [—1; 1] имеем: 0<arccos х^я, (3) cos (arccos x)=x. (4) 3) t/=arctgx — это функция, определенная на (— оо; оо), обратная функции x = tgy, у g (— -у; —-Y Таким образом, (y=arctgx) Для любого х имеем: -f-<arctg*<f-, (5) tg(arctgjc)=x. (6) 4) (/=arcctgx — это функция, определенная на (— оо; оо), обратная функции x=ctgy, у 6 (0; я). Таким образом, 212
\ctgy=x. Для любого х имеем: 0<arcctgx<n, (7) ctg(arcctg x)=x. (8) Функции у = arcsin x, у = arccos x, у = arctg x, y = arcctgx называются обратными тригонометрическими функциями или арк- функциями. Отметим некоторые важные тождества: arcsin (— х) = —arcsin х (— 1 ^х^2 1), arccos ( —х) = я — arccos х( — 1^11) arcsin arccos x=-£-( — 1 arctg (—-x) = — arctg x, arcc tg (—x) = я — arcctg x, arctg x+arccos *=-?-• Рассмотрим примеры. Пример 1. Упростим выражение cos (arcsin х), где — 1 <х<! 1. Решение. Положим y=arcsirt x. Тогда — -~^у^--ри sin y = =х. Чтобы найти cosy, воспользуемся соотношением cos2y = = 1 —sin2 £/. Получаем cos2 у = 1-х2. Но "~-|~^y^"f"- На этом отрезке косинус принимает только положительные значения. Таким образом, cos y=Vl—х2, т. е. cos (arcsin x)=yi — х2, где 1<1 Пример 2. Упростим выражение cos (2 arcsin x). Решение, cos (2 arcsin x)=cos2 (arcsin x) —sin2 (arcsin x) (l2)2122V Пример 3. Упростим выражение sin (arctg x). Решение. Положим у = arctg x. Тогда —-р<у<-|-и tg y = x. Найдем сначала cos у, для чего воспользуемся формулой 1 +tg2 y= =~2Т» откуда cos2 у = | . Так как —Y<y<^t' и на этом интервале косинус принимает только положительные значения, то cost/=— { ^ . Теперь найдем sin у. Имеем sin y=tg у cos у = —1=рг. Это значит, что sin (arctg х)=—=j. 213
Пример 4. Вычислим sin (— arcctg ( —т-))- Решение. Пусть а = arcctgf—-|-Y Тогда 0<а<я и ctga = — —-1-- Более того, так как —-|-<0, то -|-<а <я. Для вычисления sin-|- воспользуемся формулой 1— cos 2a=2 sin2 а. Из нее следует, что sin2-|-=1""^osa. Таким образом, необходимо сначала найти cos а. Так как l+tg2a= \ и ясно, что tga=— -^-, то cos' a cos2 a — . ZO I Так как в интервале -£-<а<я cos а<0, то cos а = — -л -£-= 2 V 25 1+4- = -|-. Итак, sin2f=—^-=±-, и так как -2-<-2-<-2-, а в этом интервале синус принимает только положительные значения, то sin ^_-^Z»M. Таким образом, sin (^-arcctg(—-|-))=^-. Пример 5. Вычислим arccos(cos( —<■""))• Решение. Положим y=arccosf cosf —j-я jY Тогда 0<«/<я и cost/=cos(—^-л). Но cos(—^.n) = cos(-4n+-|-n) = 3 3 3 = cos-7>-tt. Таким образом, cos у=cos — я, и, так как 0<—я<я, О и О 3 / / 17 \\ 3 получаем */=—я. Итак, arccos I cos (——я jj=—я. Замечание. Ясно, что равенство arccos ( cos ( —гя|)= —г- я ложно. Действительно, по определению значения косинуса принадлежат отрезку [0; я]. Зна- 17 чение же —г-я этому отрезку не принадлежит. Примерб. Проверим равенство arccos у~Ь arccos ( —j-j = arccos ( — -Ц-Y (9) Решение. Положим а = arccos—, p = arccos(——Y y = = arccos(— Щ-\ Тогда cosa=4", т. е. a=-£-; cosp=—4"» \ 14 / 1 6 7 причем у-<р<я; cos Y = — jf-» причем -2-<7<я. Докажем, что a + p=7- Для этого сначала докажем, что вы- 214
полняется равенство Т(<х + $)=Т (у), где Т — некоторая тригонометрическая функция. Заметим, что из равенства Г(а + Р)=Г(у) может не следовать равенство a + p=v. Действительно, например, равенство sin 30° = sin 150° истинно, а равенство 30° = 150° — ложно. Равенство a+p = Y будет выполняться, если Г(а + Р)=Г(7), и, кроме этого, а+ р и у принадлежат одному и тому же промежутку монотонности функции Т. В рассматриваемом примере a + Р принадлежит либо второй, либо третьей четверти, у принадлежит второй четверти. Таким образом, и a + р, и у принадлежат отрезку —-; -—■ . Поэтому в качестве функции Т целесообразно взять такую тригонометрическую функцию, которая монотонна на указанном отрезке. Такой функцией является, например, синус. Итак, вычислим sin(a + P), а загем sin у: sin (a + P)=sin a cos p+sin p cos a = sin ~-f—7~)~l~ Итак, sin (a + P)==sin у. И, кроме того, a + р и у принадлежат одному и тому же промежутку монотонности синуса, поэтому а + р=у. Тем самым равенство (9) доказано. П р и м е р 7. Докажем, что если — 1 <*< 1, то arcsin *=arctg —=j. (10) Доказательство. Вычислим значения тангенса от обеих частей равенства: tg (arcsin x)==sin(arcsinx) ^_ & v ' cos (arcsin x) -J\ZZ tg(arctg-^=i)=-^, \ -\/l—X2 / л/1—X1 т. е. тангенсы равны. Далее, так как —1<*<1, то ~-~<arcsin x<-j-. Но по определению и —~<arctg _*<—-, т. е. и arcsin х, и arctg—==г 2 лЛ— х2 -yj\-—x2 принадлежат одному и тому же промежутку монотонности тангенса. Тем самым тождество (10) доказано. 215
Упражнения Вычислите. 1387. 2 a resin ( —^- )+ arctg (—1)+arccos —-r--7rarccos(~~ 0- \ & / -^ 2. 1388. tg (б arctg ^~ arcsin ^ ). 1389. sin (3 arctg ->/3 + 2 arccos у Y 1390. cos (3 arcsin ^+ arccos ( — у ))* 1391. arccos (cos у). 1392. arctg (tg 0,3л). 1393. arcsin ( — sin -^- я J. 1394. arccos (—cos —J. 1395. arctg I — tg-^ V 1396. arcsin (sin -y- 1 +arccos f cos -^- V 1397. arctg(-tglf ) + arcctg(ctg(-lf )) . 1398. sin (i-arcsin (-^)) 1399. tg(y arcsin^)- 1400. ctgf —arccos f—7" ))• Ш)1- sin f arctg——arcsin — V (1 3 \ 2 arctg —+ arccos -r- )• 1403. sin (2 (arcsin ^- arccos ^ )Y Упростите выражения. 1404. cos (arccos x+arccos y). 1405. sin (arccos x+arcsin y). 1406. tg (arctg *+arctg y). 1407. tg (arcsin jc+arcsin #). 1408. sin (2arcsin x). 1409. tg (2arctg x). 1410. cos (2 arctg x). 1411. sin (2 arcctg x). 1412. cos (2 arcctg x). 1413. cos f у arccos x J . 1414. tg lу arctg * J . Проверьте равенства. 1415. a) arccos -^-=2 arcsin -j-; о 4 б) arccos 7^=2 arcsin -7-; 10 о в) arccos—=2 arcsin —. У о 1416. a) arctgy+arctg ^ =-2-; 6) arcctg у + arcctg "5-=-^- 1417. arcctg-=-+2 arcctg—=-£ . 1418. arcsin ——arecos --= arctg- 7 о 4 О -^5 1419. arcsin «R+y arccos ^=arccos-g-. 216
1420. arctg ^+arcsin ^H= arctg (3+2 -yft). 1 1 2 я 1421. arctg—+arctg—+arctg—«=—. о 4 У 4 1422. arcsin-=-+a res in -r^+arcsin^r—-^-. о \6 DO Z Докажите тождества: 1423. arctg x=arcsin 1424 .„, farccos ТП^если 0<х<1, 1425. «-лллв ^ i—».—.. v«— •* » ^wm O^x^l, я — arcsin VI— x2, если — 1<х<0. arccos --- , если jc>0, 1426. arctg x=| VI + ^ arccos -—===-, если х<0. ■[: [arcctg —, e<y] arcetg я, f " La [arcs r Г arctg , если 0<*<1, f-2-^—• если — ;—-, если х>0, 1428. arctg x m . l если jarcctg-i , если 0<*<1, 1429. arcsin Jt = I x |aroctg-= я, если — farcsin—=5, если arcsin — •-, если VT+7 —, если х>0, 1431. arcctg jc=-| |я +arctg —, если *<0. 1432. 2 arccos "Y-Ц-^=arccos x. 1433. ~ arccos (2x2— 1)=arccos лс, если дс>0. f 3. ДОКАЗАТЕЛЬСТВО НЕРАВЕНСТВ При доказательстве тригонометрических неравенств используются те же приемы, что и при доказательстве алгебраических неравенств (см. с. 27). Отметим лишь, что при доказательстве триго- 217
нометрических неравенств синтетическим методом в качестве опорных часто используются следующие неравенства: IsinxKl, IcosxKl, sin jc<x<tgjc, где 0<jc<-|-. Иногда в качестве опорных используют неравенства, вытекающие из монотонности тригонометрических функций. Так, в интервале (0; -|Л функции y=smx и y=tgx возрастают, а функции у= =cosjc и t/=ctgx убывают. Поэтому если 0<Xi<X2<-~, то sin x\ < sin *2, cos х\ > cos *2, tg Х\ < tg X2, ctg x\ > ctg дсг. Аналогичные неравенства могут быть получены для других промежутков монотонности тригонометрических функций. Рассмотрим примеры. Пример 1. Докажем неравенство a sin2 а Н—г4—> 2 л/а&, sin а если известно, что а>0, 6>0, аФпп. Доказательство. Воспользуемся неравенством, связывающим среднее арифметическое и среднее геометрическое двух положительных чисел а\ и а2: Положим в этом неравенстве asin2a = ai, -г-^—=аг. Получим: . 2 , Ь sin a - : Sin' а * . 2 sht a 2 1 Пример 2. Докажем, что если А, В, С — углы треугольника, откуда a sin2 aH—r^—^2-\/ab, что и требовалось доказать. sin сь ТО 2 Доказательство. Выполним некоторые преобразования левой части неравенства (1). Имеем: cos A + cos B+cos C=2 cos ^±£ cos ^^+cos С. Так как по условию Л-f В+С=180°, то ^±-£=90°— у- и, следовательно, cos ±±£- cos (90° —£-) = sin |-. Так как 0<cos^£<l, то cos ^±^ cos ^^< sin -^-. л, Z JL /> 218
Таким образом, 2 cos 4±1 cos 4y2-+cos C<2 sin -f--{-cos С. Рассмотрим выражение 2 sin —+cos С. Имеем: 2 sin -~+cos C=2 sin -|-+1 -2 sin2 -£-. Положим x=sin-j-. Тогда 2 sin -£-+1-2 sin21-= -2*2 + 2х+1. Если мы теперь докажем, что — 2х2 + 2*+ 1<-|-, то тем самым мы докажем и неравенство (1). Составим разность ( — —|- и выясним ее знак. Получаем Таким образом, — 2jt2 + 2x-f 1 ^—> т- е- неравенство (1) доказано. Пример 3. Докажем неравенство о sin a sin 2а sin За <-т-- (2) Доказательство. Выполним некоторые преобразования левой части неравенства (2). Имеем: (sin a sin 2a) sin 3a =cosa-cos3a sin 3a =2sin3«cos«-2sin3acos3« = sin 4a-f sin 2a —sin 6a "~~ 4 Так как sin 4a < 1, sin 2a ^ 1, — sin 6a ^ 1, то sin 4a +,sin 2a — — sin6a<3, причем знак равенства имеет место только для тех значений а, которые удовлетворяют системе уравнений 'sin 4a=l, sin 2a=l, sin 6a= — 1. Однако эта система не имеет решений. В самом деле, если sin 2a = 1, то cos 2a=О, и поэтому sin 4a = 2 sin 2a cos 2a=0. Таким образом, sin 4a + sin 2a —sin 6a <3, а значит, ;-|-, откуда и следует неравенство (2). 219
Пример 4. Докажем, что tgtt,< sin«,+sina,-|-...+sina. <t ь cosai+cosa2 + +cosan ь если 0<ai<a2<...<an<y-. Доказательство. Так как в интервале (0; -|-) функция y=sinx возрастает, а функция y=cosx убывает, то 0 <sin он <sin аг <... < sin а„, cos ai>cos a2>...>cos an>0. Значит, п sin ai<sin ai + sin аг + ... + sin an<n sin art, n cos ai >cos ai + cos аг +... + cos an >n cos ап, откуда n sin ai ^ sin a^sin a2-f -4-sin an ^n sin an n cos ai cos ai + cos аг + ... + cos аЛ я cos a« * * * ® <Sina,+Sina2 + ..-fsina.<t цто и требовалось доказать. cos ai+cos аг+ ...*+cos an Пример 5. Докажем, что tg a tg p+tg p tg Y + tg a tg y< 1, если a>0, p>0, v>0 и a + p+v<-~. Доказательство. Положим -2—a —p=yi- Тогда так как по условию Y<-^—a— p. Рассмотрим выражение tg a tg P + tg p tg Yi+tg a tg yi. Можно доказать (см. с. 206), что tg a tg p + tg p tg Yi +tg a tg Yi = 1. Ho y и Yi — аргументы из первой четверти, причем Значит, tgY<tgYi» а потому tg a tg p + tg p tg Y + tg a tg Y<tg a tg p + tg p tg y{ + + tgatgYi, т. e. tgg gg что и требовалось доказать. Пример 6. Докажем, что если Л, В, С — углы треугольника, то (sin A + sin В + sin С)2 ^ 9 sin A sin В sin С. Доказательство. Воспользуемся неравенством, связывающим среднее арифметическое и среднее геометрическое трех положительных чисел: о Положив в этом неравенстве sin A = a, sin В = ft, sin С=с, получим: sin у4 4- sin В + sin С • fSin A sin о : 220
и далее (sin A -fsin В + sin C)2>9 V(sinA sin В sin С)2, но V(sin A sin В sin Cf> V(sin Л sin В sin C)3 = sin Л sin В sin C. Таким образом, (sin Л -fsin В-fsin C)2>9 sin Л sin В sin С, что и требовалось доказать. Пример 7. Докажем неравенство а—^-<sin а, (3) где 0<а<у. Доказательство. Выберем в качестве опорного неравенство -§-<*6"|~' Последовательно преобразуя его, получим: a cos -f-<2 sin у-, а cos y-cos у-<2 sin -|-cos y-, occos2y-<sin a, a(l-sin2|-)<sina. (4) Воспользуемся еще одним опорным неравенством sinf<f. Так как по условию 0<a<y, то sin y->0 и у>0, поэтому неравенство sin -§-<-§- можно преобразовать и получить в результате следующее неравенство: sin2 у<^-» к далее 1 —sin2 y> 1 — ^-, 1 — sin2y-J>a—^-, или Сопоставляя неравенства (4) и (5), получим: a—j<a(l — sin2-|-j<sin a, 3 откуда a—— <sin a, что и требовалось доказать. Пример 8. Докажем неравенство tga-a<tgp-p, (6) если 0<а<р<-|-. Доказательство. 1-й способ. Воспользуемся неравенством 221
tg x>x> гдеО<х<-|-. Положим x = p—а. Тогда tg.(p—a)>p —a. Если мы докажем, что tgp-tga>tg(P-a), (7) то тем самым будет доказано неравенство tg р —tg a>p— а, а, следовательно, и неравенство (6). Итак, докажем неравенство (7). Для этого составим разность (tg Р — tg a)—tg (P — a) и преобразуем ее: tg p-tgc-tg (р-с0=lg P-tg «- Полученное выражение положительно, так как tg a>0, tg P>0 и tgP>tga. Таким образом, доказано неравенство (7), а значит, и неравенство (6). / . 2-й способ. Рассмотрим функцию у = tg *—х на интервале f 0; -|-V Имеем t//=—^r--—1>0, значит, функция возрастает на (О;—). Но тогда для 0<a<p<-^- будем иметь tga — a<tgP—P, что и требовалось доказать. Пример 9. Докажем неравенство cos a+3 cos 3a+6 cos 6a> —7-^. (8) Доказательство. Предположим противное, т. е. что для некоторого а выполняется неравенство cos a + 3 cos 3a + 6 cos 6a< — 7-^-. (9) Преобразуя неравенство (9), получим: cos a + 3 cos 3a+6 cos 6a + 6< — 1-^-, cos a+3 cos 3a+12 cos2 3a< — 1 -|-, cos a+3 (cos 3a+4 cos2 3a)< — 1-jjr, 3 (4 cos2 3a+cos 3a+-jL) + cos a—^-< -1-^-, 3(2 cos 3a+-j-)2 + cos a< — 1. (10) (l \2 2cos3a+—J >0, а cos a > — 1. Значит, наше предположение неверно, т. е. доказано неравенство (8). 222
Пример 10. Докажем неравенство cos36o>tg36°. (11) Доказательство. Предположим, что неравенство (11) ложно, т. е. cos 36°^ tg 36°. Тогда последовательно получим: cos2 36° < sin 36°, 1 + cos 72° < 2 sin 36°, 1 + cos (90°-18°)<2 sin (6°+30°), 1 + sin 18° < 2 sin 6° cos 30°+2 sin 30° cos 6°, 1 +2 sin 9° cos 9°<cos 6°+ 2 sin 6° cos 30°. (12) Так как I>cos6°, sin 9° > sin 6°, cos 9° > cos 30°, то неравенство (12) ложно. Значит, верно неравенство (11). Пример 11. Докажем, что если А, В, С — углы треугольника, то sin -fsin-f-sin !■<-!-. (13) Доказательство.- Предположим, что неравенство (13) A R Г* 1 ложно, т. е. sin -5"" sin у sin "9~>т-> ПРИ некотором наборе значений Л, В, С — углов треугольника. Тогда, преобразовав произведение sin— sin—, получим: и далее ;cos^-cos^L cos 4=5-.ь, f--cos Atfi-sin f sin s=fib. + sin -A+B+C-sin d±B±£+ 2^2 • Так как Л+Я + С=180°, то А — £ + С=180° — 2В, —А+В + С= 180° — 2А, А + В — С=180°—2С, а потому (И) sind±f±£=i, ., ^ Это позволяет переписать неравенство (14) следующим образом: 223
cos B-f cos A — 1 +cos C>~-, или cos Д+cosB+cos C>-|-. А это противоречит неравенству, доказанному в примере 2 (см. с. 218). Значит, наше предположение неверно, т. е. истинно неравенство (13). Пример 12. Докажем неравенство tgmx>/ttg<x, (15) если 0 < а <4 " , п — натуральное число, п Ф1. Доказательство. Применим метод математической индукции. 1) Проверим выполнимость неравенства (15) при и=2, т. е. убедимся, что tg2<x>2tg<x, (16) где 0<a<j-. В самом деле, tg 2a-2 tg a—ffi^ —2 tg a=2 tg a ^ °a . При 0<a<-^-имеем tga>0, 1—tg2a>0, а, значит, Отсюда и следует, что неравенство (16) выполняется. 2) Предположим, что неравенство (15) выполняется при n = ky т. е. tgfta>*tga, где 0<а< . " п. Докажем, что тогда нера- венство (15) выполняется при n—k+l, т. е. a>(*+l)tga, (17) где 0<a<^-. В самом деле, I— tgfeatga I— t По условию 0<а<тг> значит, tgfea<tg—=1 и tga<l. Но и, следовательно, неравенство (17) выполняется. По принципу математической индукции заключаем, что неравенство (15) верно для любых натуральных п>2. 224
Упражнения Докажите неравенства. 1434. Vcos~a<V2cos у, если —-jr<a<"a"- 1435. ctg — > 1+ctg a, если 0<а<л. 1436. tg a tg р<1, если а, p— величины острых углов тупоугольного треугольника. 1437. tg a tg р> 1, если a, p — величины углов остроугольного треугольника. 1438. cos a + cos р>cos у, если a>0, P>0, y>0> a + P+V = y- 1439. sin2 a + sin2 p + sin2 y<2, если a, p, у — величины углов нсостроугольного треугольника. 1440. sin~ a + sin2 p + sin2 y> 2, если a, p, у — величины углов остроугольного треугольника. 1441. соз* a4-cos2 р + cos2 y> 1, если a, p. у — величины углов неостроугольного треугольника. 1442. cos2 a + cos2 p+cos2 y< 1» если a, p, y — величины углов остроугольного треугольника. ,443. sin^±£>8hlg+>inP 4-- 1449- sin8 a + cos8 a>4" • 4 8 1448. sina + cosa>4- 1449- sin a + cos a>4" 4 8 1450. tg(a + P)>tga + tgp, если а>0, p>0, + p< 1451. sin4a-6sin2a 1452. cos2 2a — 1453. cos (sin a)>0. 1454. sin (2 + cos a)>0. 1455. cos(n-harcsina)<0. 1456. sin(4- + агс1& а) >°- 1457. tga + ctga>2, если 0<а<~. 1458. -V< + <V 1459. tg a-f ctg a>sin a + cos а, если 0<а<—. 1460. |tga + ctga|>|sina + cosa|. 1461. sin (a + ЭХ sin a + sin p, если 0<а<я, <Р< 1462. cos(a —pXcosa-bsin p, если 0<а<я, 0<р<я. 1463. sin(a-r-p-|-YXsin a + sin p + sin y, если 0<а<~, 1464. sin y cos a+ctg a 225
1465. /t , . 2 ч/1 , 7-T (1-fsin2 a)(l+cos2a) 1466. 3(tg2a + ctg2a)-8(tg a-fctg a)+10>0. sina —1 , 1 .^2 —sin a 1468. (1+ sin2 a) (tg2 a - 2) + ctg2 a + cos2 a > 0 1469. . 2 ,C°Sa—: г>8, если 0<a<~ sin a (cos a — sin a) 4 1470. sin 2a cos 2a cos 4a cos 8a cos 16a < —. lo 1471. --j-<sinasin(y-a) sin (y 1472. 0<cos2a-f cos2(a-f P) — 2 cos a cos 1473. tg a (ctg p-fctg v)+tg P (ctg a + ctg Y) + tg у (ctg a-fctg p)>6, если 0<a |у 1474. (ctg2 a-1)(3 ctg2 a- 1) (ctg 3a tg 2a— 1)< — 1. 1475. (1 -tg2 a) (1 —3 tg2 a) (1 +tg 2a tg 3a)>0. 1476. cos2a-f cos2 P<3 — 2 sin a—2 sin p-f 2 sin a sin p. 1477. sin a (sin a — 2) + cos P(cos p-f 2)>2sin a cos 0—1. ,478. sin(a-P)sin(a + p) >2 tg a tg p+2 tg a-2 tg p-1. cos2 a sin2 p 1479. cos a + 2 sin a > 1, если 0<a<— . 1480. a—sin a<p — sin p, если 0<a<p<~. 1481. sina<a<tga, если 0<а<у. 1482. cosa< S1^a <1, если 0<а<у. 1483. sina(sina-2)-fcos2(a-l)>0. 1484. (l-sina)2-sin2(a-l)>0. 1485. sin a — cos a tg y 1486. cos (a+ p) cos (a — p)<cos2 a. 1487. -r-\—| \—>Ъ. sm4 a cos4 a 1488. tg2a + tg2P + tg2Y>l, если а>0, p>0, y>0, a-fp-fY = y. 1489. si^y-jf sin2i--fsin2-|->-|, если a + P + Y = Ji. 1490. 4 sin За-f 5>4 cos 2a-f 5 sin a. 1491. 1 -f cos 3a>cos a (1 -f cos a)(2 — 5 cos a). 1492. sin a-f sin 2a + —-f sin яа<я. 1493. cosa<cos211^, если 0<а<у. 1494. tga,<sfalflt| + statti + - + sinct'<tga., если 0<а1<а2<...<«я< ь cosa+cosa2 + ... + cosan ь , если tga=/i tgp, где «>0.
Глава IV РЕШЕНИЕ УРАВНЕНИЙ, СИСТЕМ УРАВНЕНИЙ И НЕРАВЕНСТВ § 4. УРАВНЕНИЯ Напомним общие формулы решений простейших тригонометрических уравнений (если не сделано оговорок, то предполагается, что параметры /г, Л, /, т, ... принимают любые целые значения). Уравнение sin х = ау где |а| < 1 cos х—ау где |<з| <1 tg x = a ctg х—а Решение * = ( — \)к arcsin a + nk х = ± arccos a + 2nk x = arctg a-\-nk x = arcctg a + nk Отметим особо некоторые частные случаи простейших тригонометрических уравнений, когда решение может быть записано без применения общих формул: sin x=0o x=nky sin x=\ ox=-Y sin x= — 1 ox= cos x= 1 о x cos a:= —-1 о x = tg x=Qo x=nk. Проверка найденных решений необходима: 1) если в процессе решения произошло расширение области определения уравнения в результате некоторых преобразований (освобождение от знаменателей, сокращение дроби, приведение подобных членов), 2) если в процессе решения уравнения использовалось возведение обеих частей уравнения в одну и ту же четную степень, 3) если при решении применялись тригонометрические тождества, левая и правая части которых имеют неодинаковые области определения, например, 227
=sina, =cosa, = tga, tgactgo=l, Использование этих тождеств «слева направо» приводит к расширению области определения уравнения, а, значит, может привести к появлению посторонних корней; использование этих тождеств «справа налево» ведет к сужению области определения, что, вообще говоря, недопустимо, так как это может привести к потере корней. Так, решим уравнение (f) -l. (1) Так как 1 то уравнение (1) преобразуется к виду 1 —tg л: tgx Положив £/ = tg jc, получим: ^~—= 1, откуда находим у= =у", т. е. tg* = -i-, и, следовательно, x=arctgу-+л;п. Это семейство удовлетворяет уравнению (1). Однако нетрудно заметить, что значения х = у-+д/г также удовлетворяют уравнению (1). Причина потери решений — применение тождеств (2) и (3). Замена выражения tg (* + —-) выражением tg*+1, , так же как и за- мена выражения ctgx выражением -—, сужает область определе- tg х- ния уравнения (1), а именно из области определения «выпадают» значения х = -^~\-пк. Они-то и оказались в данном случае «потерянными» решениями уравнения (1). Основными методами, используемыми при решении тригонометрических уравнений, являются следующие методы: 1) разложение на множители (см. с. 47); 2) введение новых переменных. В результате разложения на множители решение заданного 228
уравнения сводится к решению совокупности уравнений. Это, в свою очередь, означает, что после решения всех уравнений совокупности найденные семейства (множества) решений следует объединить. Объединяя семейства решений, иногда добиваются более компактной записи ответа. Например, объединяя семейства x=nk и х = -~ получим В тех случаях, когда среди найденных значений х имеются повторяющиеся, объединяя семейства решений, ищут такую запись ответа, в которой повторяющихся значений х нет. Например, среди значений ху принадлежащих семействам x==^-k и х=-£-л, есть повторяющиеся значения дс, а именно х = пт. Исключая значения лг = лт, например, из первого семейства, можно записать ответ так: х= ±-?-+ nk\ х = -~л. Если же исключить зна- «3 2. чения х из второго семейства, то ответ можно записать по-другому: х = -^-/г, * = -£-+ пп. (Как видим, полученные записи одного и того же ответа оказываются неодинаковыми. Это следует иметь в виду, сверяя свое решение с ответом, имеющимся, например, в книге.) Объединение решений удобно выполнять с помощью окружности 2, на которую наносят семейства решений совокупности уравнений. Так, пусть требуется объединить семейства х = -~+-^-й их=лл. о о Отметим на окружности 2 (рис. 33) значения х из первого семейства кружками, а значения х из второго семейства квадратиками. Значение х = л, как мы видим, оказывается отмеченным дважды, т. е. это значение х (а точнее, значения х=п + 2лт) при записи объединения семейств лг = ~— о и х = пп следует оставить только в одном се- мействе. Ответ можно записать, например, так: jc = -^—f ^- 3 3 или так: х= ±-£- х = пп. о При решении уравнений методом введения новых переменных следует помнить, что важную роль играет выбор функции, через которую выражаются остальные функции. Может оказаться, что при одном выборе такой функции получается иррациональное уравнение, а при другом — Рациональное. Ясно, что второй выбор предпочтительнее. Если, напри- МеР> в уравнении 2 cos2 jt+4 cos x= Рис. 33 229
= 3 sin2 x положить y = sin x, то получится совокупность двух иррациональных уравнений: Если же положить y = cos дг, то получится рациональное уравнение 2(/2 + 4у = 3(1— у2). Мы будем обозначать через R (cos х\ sin х) рациональное выражение от cos х и sin jc, т. е. выражение, получающееся из cos x и sin х и постоянных с помощью сложения, умножения и деления Рассмотрим уравнение вида R (cos x\ sin jk) = O. В некоторых случаях удается свести такое уравнение к рациональному уравнению относительно sin x (или относительно cos x). Укажем некоторые правила, облегчающие выбор подстановки при решении тригонометрических уравнений. Если cos x входит в уравнение лишь в четных степенях, то, заменяя всюду cos2 x на 1 —sin2 х, получим рациональное уравнение относительно sin x. Точно так же если sin x входит в уравнение лишь в четных степенях, то замена sin2 х на 1 —cos2 x приводит уравнение к рациональному виду относительно cos x. Однородным тригонометрическим уравнением 1-й степени называется уравнение вида a sin х + 6 cos jt = O. Однородным тригонометрическим уравнением 2-й степени называется уравнение вида a sin2 x + b sin x cos x + c cos2 лг = О. Аналогично может быть определено однородное тригонометрическое уравнение любой натуральной степени п. Рассмотрим случай, когда аФО. Нетрудно видеть, что при афО однородному уравнению не удовлетворяют те значения х, при которых cos х = 0. Поэтому деление на cos x (на cos2 x) обеих частей однородного уравнения 1-й (2-й) степени в случае афО приводит к равносильному уравнению. Разделим обе части однородного уравнения 1-й степени на cos лг, а обе части однородного уравнения 2-й степени на cos2 x. В результате получим соответственно следующие уравнения, рациональные относительно tg x: = 0 и aig2 x+bigx+c=0. Эти уравнения решаются с помощью подстановки y=tgx. Рассмотрим теперь подстановку, позволяющую свести к рациональному любое уравнение вида R (cos x\ sin *) = 0. Эта подстановка Если х=5^=л + 2яЛ, то cosx= , sinjc = 230
Рис. 34 Поэтому подстановка sin х)=0в уравнение Рис. 35 w = tg-|- преобразует уравнение R (cos x\ Левая часть последнего уравнения является рациональным выражением. Значит, наша подстановка привела уравнение к рациональному виду. Подстановка w = tg ~- называется универсальной. Поскольку использование универсальной подстановки исключает из области определения уравнения множество значений вида x=n-\-2nk, т. е. может привести к потере решений, то после решения уравнения необходимо еще выяснить, не являются ли числа вида х=л+2лк решениями заданного уравнения. В настоящем параграфе, кроме тригонометрических уравнений с одной переменной, рассматриваются также уравнения, содержащие переменную под знаком обратной тригонометрической функции. Рассмотрим примеры. Пример 1. Объединим семейства значений Решение. Отметим на числовой окружности 2 значения х из первого семейства, например, кружками и значения х из второго семейства, например, квадратиками (рис. 34). Как видно из рисунка, на окружности 2 получилось шесть точек, которые делят окружность 2 на шесть равных частей. Это наблюдение позволяет более компактно записать все значения х, принадлежащие заданным семействам, а именно x = ^+^-k. о о Пример 2. Объединим семейства значений 231
Решение. Нанесем на окружность 2 (рис. 35) значения х из первого семейства, например, кружками, а значения х из второго семейства квадратиками. Значения х = лту как видно из рисунка 35, являются повторяющимися. Если исключить эти значения х из первого семейства, ответ можно записать так: х= ±^~-+nk\ x=-~n. Ее- 3 4 ли же исключить значения х=пп из второго семейства, то ответ будет выглядеть так: x=-%-k; *=-£-+ля; *=-7-+-тгт. Исключить повторяющиеся решения можно и не обращаясь к окружности 2, если решить уравнение -j-k=-j-n, например, от- носительно k. Получим k = —n. Ясно, что при n = 4m значения k будут целыми (£ = 3т). Таким образом, ответ можно записать, сохранив целиком семейство х = -£-Л, а из второго семейства исключить значения х, ко- о торые получаются при n = 4m. Тогда второе семейство будет таким: * = -j-/i, где Пример 3. Из семейства х=~-+-£-й исключим значения х% о о принадлежащие семейству х=-^-п. Решение. Отметим на окружности 2 (рис. 36) значения х, принадлежащие первому семейству, и вычеркнем значения ху принадлежащие второму семейству. Как нетрудно видеть, значения х, оставшиеся после вычеркивания, можно записать в виде семейства Пример 4. Решим уравнение (cos 2x+ l)(sin2 (*~ Рис. 36 Рис. 37 232
Решение. Это уравнение равносильно следующей совокупности уравнений: cos2x=-l; Sin(jc—J-)-±^. Решением первого уравнения является семейство лг=-|—fn&, а второго — семейство х = -^-л. Отметим на окружности 2 (рис. 37) первое семейство кружками, а второе семейство квадратиками. Как видно из этой иллюстрации, значения jt=-~+jife содержатся в каждом из семейств, т. е. являются повторяющимися. Чтобы в записи ответа не было повторяющихся значений х, целесообразно не включать в ответ первое семейство. Итак, получаем ответ: х=-^-п. Пример 5. Решим уравнение tg х sin 2jc=0. Решение. Следствием этого уравнения является совокупность уравнений tgjc=O; sin2jt=0. Решением первого уравнения этой совокупности является семейство х=nky а второго — семейство х=~-п. Ясно, что все значения х из первого семейства принадлежат второму семейству, т. е. из записи ответа их следует исключить. Кроме того, как мы отметили, совокупность уравнений tgjc=O; sin2*=0 является следствием заданного уравнения. (Действительно, не исключается возможность, что при некоторых значениях х, являющихся решениями уравнения sin2x=0, не существует tgx.) Это значит, что необходимо выполнить проверку. Проверка. Так как tg x не существует при х=-^--\-пт, то из найденного семейства решений (х=-^п) необходимо исключить значения х=-%-+пт. На рисунке 38 значения х=-^п показаны кружками на окружности S, а значения х=~-+ят зачеркнуты. Оставшиеся значения х можно записать в виде семейства x = nk. Это — решение заданного уравнения. Пример 6. Решим уравнение sin3 2x—sin 2x cos Зх 233
Рис. 38 Решение. Следствием этого уравнения является уравнение sin 2x(sin22x — l) = 0, которое, в свою очередь, равносильно совокупности уравнений: sin2x = 0; sin 2x=l; sin2x= —1. Решением первого уравнения этой совокупности является семейство х=-j-k, второго — семейство x=-^-+7in и третьего — семейство х=— j+ш. Объединением этих трех семейств является семейство X = -j-/. При переходе от заданного уравнения к уравнению sin 2л: X X(sin2 2лс—1)=0 область определения заданного уравнения расширялась. Это значит, что найденные значения х необходимо проверить. Проверка. Исключим из семейства *=-j-f значения х, при которых cos Зх=0, т. е. значения x=-~+-^-m. Сделаем это с помощью о о окружности 2 (рис. 39), на которой отметим кружками значения х% принадлежащие семейству х=-^-/, и вычеркнем значения х, принадлежащие семейству jt = -f-+-^-&n. Оставшиеся значения х можно и о записать как объединение двух семейств: x=nk\ х=-^+-^п. Пример 7. Решим уравнение sin 2* sin -=0. (4) Решение. Так как дробь равна нулю, то из уравнения (4) следует, что sin 2лг = 0, откуда jc = y-&. Ясно, что из этого семейства уравнению (4) удовлетворяют 234
те и только те значения х, которые принадлежат области определения уравнения (4). Но в процессе решения эта область расширялась. Таким образом, необходима проверка. Проверка. Область определения уравнения (4) найдем из условия sin 2x~^Jl ФО, откуда хФ3пп~л . Воспользоваться окружностью 2 для исключения из семейства x=-%-k значений х=3м""я не удается. Поступим следующим образом. Отметим решения из семейства x=-j-k точками на числовой прямой (рис. 40) и вычеркнем точки, удовлетворяющие условию х=ЪппГ~п . Оставшиеся значения х— это решения уравнения (4). Их можно записать как объединение двух семейств: Отметим, что эти два семейства решений можно записать и короче, например так: * = —-&; где кфЫ — 1, /£Z. Пример 8. Решим уравнение cos 15jc = sin 5jc. Решение. Так как cosoc = sin(-£—ом, т. е. cos 15х= = sin(-2—15jc), to перепишем заданное уравнение следующим образом: sin f-2—15дм —sin 5jc=0, 2 sin (^~ IOjc) cos (7—5л:) =a или Таким образом, заданное уравнение равносильно совокупности уравнений: sin (-J— IOjc) =0; cos (-J--5jc) =0. Из первого уравнения совокупности получаем IOjc—-^-=яй, откуда находим х=£г + 77г k\ из второго уравнения получаем 5л:— ~т=т+пп> откуда x=sw+fn- ж • •—ж—•—•—*—•—■ ж • • х > f te5lb2tl Рис. 40 235 -f _тг _ж 0 f
Рис. 41 Попытка объединить найденные семейства с помощью окружности 2 наталкивается на чисто технические затруднения (придется полуокружность делить на сорок равных, частей). Так как тем не менее повторяющиеся решения, если они имеются, желательно обнаружить, чтобы в записи ответа указать каждое решение лишь один раз, то положим 40" *W 20~ ' !Гп' Решим это уравнение, например, относительно к. Получаем & = 1,25+2я. Таким образом, ясно, что не существует целых значений л, при которых число к было бы целым. Другими словами, найденные семейства повторяющихся значений х не содержат. Итак, решение заданного уравнения таково: Xz=z— U— k' Х = — + — П Замечание. Преодолеть трудности, связанные с делением окружности 2 на очень мелкие доли, можно и по-другому. А именно «укрупним» решения — +тт> * и 4U 1U — +-£-л в 5 раз. Получим 5*=—+-«гЛ; 5*=—+яя, нанесем эти значения 5jc 2\j О о Z 4 на окружность 2 (рис. 41). Приходим к выводу, что в этих семействах повторяющихся решений нет. Пример 9. Решим уравнение sin х—Ъ cos x=0. Решение. Это — однородное уравнение первой степени. Деление обеих его частей на cos x приводит к равносильному уравнению tg x=3, откуда находим семейство x=arctg 3 + л/г, являющееся решением и заданного уравнения. Пример 10. Решим уравнение sin2 jc+2 sin x cos jc — 3 cos2 jc=O. Решение. Это — однородное уравнение второй степени. Деление обеих его частей на cos2 x приводит к уравнению tg2 x+2 tg x— — 3 = 0, равносильному заданному. Полагая u=tgx, приходим к уравнению и2-\-2и — 3 = 0, откуда ai = l, мг=—3. Из совокупности уравнений tgx=l, tgx=—3 получаем j и x=arctg( — решение заданного уравнения. Пример 11. Решим уравнение 5 sin2 х+3 sin х cos л:—3 cos2 x=2. 236
Решение. Это уравнение не является однородным, так как правая часть уравнения отлична от нуля. Однако оно может быть преобразовано в однородное уравнение. С этой целью используем тождество sin2 x+cos2 х=1. Тогда уравнение можно переписать следующим образом: 5 sin2 х+3 sin х cos jc — 3 cos2 x=2 (sin2 x + cos2 x), и далее 3 sin2 x+3 sin xcos x—5 cos2 x=0. Последнее уравнение представляет собой однородное уравнение 2-й степени. Разделив обе его части на cos2 х, получим квадратное уравнение 3 tg2 jc+3 tg x — 5=0, откуда получим Пример 12. Решим уравнение 5 sin2 Jt+УЗ sin x cos х+6 cos2 х=5. Решение. Имеем: 5 sin2 х +л/3 sin х cos х+6 cos2 х=5 (sin2 x-f-cos2 х), УЗ sin x cos x + cos2 x=0. (5) Ясно, что делить обе части этого уравнения на cos2 x нельзя, так как те значения х, при которых cos2x=0, удовлетворяют уравнению (5), а потому деление на cos2x приведет к потере корней. Мы поступим по-другому: разложим левую часть уравнения (5) на множители. Получим: cos х (УЗ sin х + cos x) =* 0. Теперь задача сводится к решению совокупности уравнений: cosx=0; УЗ Sin x+cos х=0. Из первого уравнения совокупности находим семейство х= = ~Ья£- Разделив далее обе части второго уравнения на cos х, получим уравнение tgx==—^ , откуда находим второе семейство о решений заданного уравнения: х=—^-+лп. Повторяющихся значений х эти семейства не содержат, и запись ответа в виде объединения этих семейств достаточно компактна. (Для читателя, желающего найти более компактную запись ответа, приведем один из вариантов ее: x=-%-+-~k, где кфЪп-\-1.) 237
Пример 13. Решим уравнение 2 cos2 х+4 cos jc=3 sin2 х. Решение. Так как в это уравнение sin x входит только в четной степени, a cos x содержится в уравнении в первой степени, то целесообразно заменить sin2 х на 1—cos2 jc и затем положить y=cosx. Тогда заданное уравнение примет вид: откуда t/i,2 = "^ Таким образом, заданное уравнение равносильно следующей совокупности уравнений: =; tosx=. О О Первое уравнение этой совокупности решений не имеет, так как — 2 —УГ9< —5, т. е. ~~ Т*— < — 1, а из второго уравнения нахо- О дим семейство jc=±arccos ~~2+У1* -\-2nn, являющееся решением о и заданного уравнения. Пример 14. Решим уравнение. 8 cos4 jc —8 cos2 jc—cos x+1 =0. (6) Решение. Положим для краткости y=cosx. Тогда уравнение (6) примет вид: 8у4-8у2-у+1=0, (7) и далее Таким образом, уравнение (7) равносильно следующей совокупности уравнений: у-1=0; 8£/3 + 8£/2—1=0. Из первого уравнения совокупности получаем у\ = \. Решим второе уравнение. Положим z=2y. Тогда получим z34-2z2—1=0, откуда находим (подбором) z\ = — 1, а затем, разделив 23 + 2z2— 1 на z+1, получим z3 + 2z2-l=(z+l)(*2 + z-l), откуда 22,з=^ Таким образом, у2== —-«", Уз,4=—-^-*-. Итак, заданное уравнение равносильно следующей совокупности уравнений: cosjc=1; cosjc=——; cos х=~~ т^ * . 238
Решениями уравнений этой совокупности являются семейства х = 2л&; jc= ±-~--Ь2лм; дг= ±arccos ~ ^ + 2лт. Первые два семейства можно объединить в одно: x = —k. Таким образом, решения уравнения (6) можно представить как объединение двух семейств: * = ~- k\ x= ±arccos zlL±^2. -|-2лт. о 4 Пример 15. Решим уравнение sin Ar-f-7 cos х = 5. (8) Решен и е. 1-й способ. Разделив обе части уравнения (8) на ->/12 +72 = -д/50, получим: -— sin x + -~rcos * = -j=r. (9) V50 л/50 л/50 V ' Так как (-~г) +(—|=г) == 1, то существует такое значение ф, что Н/50/ ^л/50^ =sin ф, — =cos ф, где ф = arcsin ( или ф = л/50 л/50 л/50 ^ вспомогательный угол. Теперь уравнение (9) можно переписать следующим образом: sin ф sin x-f cos х cos <p= 2 » или cos (* —ф) = 2 • откуда x —ф = ±^-+2л&. Так как ф= arcsin-— , то окончательно получаем следующее л/50 семейство, являющееся решением уравнения (8): я 1 / л 7 \ х= ±-М-arcsin -— +2л&( или х= ±-—|-arccos -—■ +2л/г ). 4 л/50 V 4 л/50 / 2-й способ. Решим уравнение (8) с помощью универсальной подстановки. Выразив sin x и cos x через tg-|-, и, положив w = tg-|-, приходим к рациональному уравнению 2ц , 7(1-ц2) _ с решив которое находим u\=—y U2=—г-. Z о Теперь нужно решить следующую совокупность уравнений: Z Z Z О 239
Из этих уравнений находим: ; х=2 arctg{ — -у) +2лп. Проверка показывает, что значения * = л+ 2я/п не удовлетворяют уравнению (8) (о необходимости проверки этих значений при использовании универсальной подстановки говорилось выше). Таким образом, уравнение (8) имеет следующие решения: |; х= — 2 arctgy- Пример 16. Решим уравнение 5 sin * — 12 cos x= —13 sin Зх. (10) Решение. Как и в примере 15, применим метод введения вспомогательного угла. Разделив обе части уравнения (6) на У52+122=13, получим: 5 12 — sin х—— cos jc= —sin 3jc. (11) Так как (jgj +("if) =1» то существует такое значение ф, что ^- = cos ф, a l|-=sin ф (или ^- = sin ф, a jf-=cos ф). Теперь уравнение (11) можно переписать следующим образом: sin х cos ф —cos x sin ф= —sin 3*, и далее sin (jc—ф)+8Ш 3*=0, Решив совокупность уравнений sin(2*—!)=0; cos(x+-f-)=0. получим: *=-f-+7"*; агс8Ш 1о 12 Учитывая, что ф=агс8Ш —, получим следующие два семейства 1о решений уравнения (10): х—i-arcsinj|-+-5-ft; л:=-i-arcsin ||-+^ Пример 17. Найдем корни уравнения V3 sin jc—cos x= —cos 3jc, (12) принадлежащие отрезку [—я; 2л} 240
Р е ш е н и е. По виду это уравнение похоже на уравнение, решенное в предыдущем примере. Однако попытка решить это уравнение приемом, описанным при решении примера 15 (см. 1-й способ), к успеху не приводит. Перепишем тогда уравнение (12) следующим образом: cos х — cos Зх=УЗ sin jc, и далее 2 sin 2x sin x—-\f3 sin x = 0, или sin x (2 sin 2jc — УЗ) = 0. Таким образом, уравнение (12) равносильно совокупности уравнений: sin x = 0; sin 2jc=^- . Из первого уравнения совокупности находим х = л£, а из второго х = (— IV— 4-—п К ' 6 ' 2 Итак, объединение семейств x=nk, х=(— 0Лт~+*тг'г — это Ре' шение уравнения (12). Осталось выделить из найденных значений те, которые принадлежат отрезку [—-я; 2л]. Рассмотрев значения х=nk при ft=0, ±1, ±2,..., замечаем, что отрезку [—л; 2л] принадлежат точки —л (при£= — 1),0 (при& = 0), л (при ft = l), 2л (при k=2). Рассмотрев значения jc=(— 1 )Л 4г-+^£: при л = 0, ±1, ±2, ..., f замечаем, что отрезку [—л; 2л] принадлежат точки -~(n = 0), f и о Итак, на отрезке [—л; 2л] заданное уравнение имеет следующие корни: -л; -|-; 0; f; f; л; 7f; 4f; 2л. Пример 18. Найдем корни уравнения cos Ax cos 8jc—cos Ъх cos 9лг=0, удовлетворяющие неравенству —>4~- Р е ш е н и е. Так как cos a cos p=cos(a+p)-f cos(a-p) ^ TQ задаНв ное уравнение можно переписать так: cos 12jc+cos4jc cos 14jc+cos4jc л 2 2 f 241
и далее y-(cos 12л:—cos 14х)=0, а затем sin 13jc«sin х=0. Таким образом, заданное уравнение равносильно совокупности уравнений: sin 13х = 0; sin * = 0, решая которую находим * = т^- &, x = nn. Ясно, что второе семейство 1«3 содержится в первом. Значит, решением заданного уравнения является семейство *=тт" k. 1(5 Осталось выделить из найденных значений те, которые удовлетворяют неравенству —>-~. Имеем -^—у*>°» 2"^~ >0» *~ <0- Из последнего неравенства находим 0<jc<2. Значит, придавая параметру k целые значения, нужно выбрать из них те, при которых т~6(0; 2). Из системы не- равенств 0<-^<2, находим, что * = 1, 2, 3, 4, 5, 6, 7, 8. lu Ответ*— — — — — — — — 13 * 13 ' 13' 13 ' 13 ' 13 ' 13 ' 13 * Пример 20. Решим уравнение 2 cos2 6jc+2 cos2 8л:+ 2 cos2 10jc = 3. Решение. Воспользуемся формулой 1 +cos 2a = 2 cos2 а. Получим: 1 +cos 12л:+ 1 +cos 16л:+ 1 +cos 20л;=3, или (cos 12x-f cos20jc)+cos 16jc=0, 2 cos I6x cos 4a:+cos 16jc=-0, t. e. cos 16a: (2 cos 4x+1)=0. Таким образом, заданное уравнение равносильно следующей совокупности уравнений: cos 16*=0; cos 4x= — -^-. Из первого уравнения этой совокупности находим *==;§"+^" *» 242
а из второго jc= ±~-+у-л. Пытаться искать среди этих семейств повторяющиеся значения jc с помощью окружности 2 затруднительно. Можно предложить такой путь поиска. Из уравнений полученной совокупности найти сначала 4jc. Получаем 4*=~-+-7-Л; 4х=±~-+2лп. о *г о Теперь с помощью окружности 2 (рис. 42) нетрудно увидеть, что повторяющихся значений х в этих семействах нет. Значит, их нет и в Рис. 42 полученной выше записи решений. Таким образом, решением заданного уравнения является объединение семейств: JL. ь* х= -+- — ' я -r в 7 я х—зГ Л' Пример 21. Решим уравнение sin 2jc+5 sin jc+5 cos jc+ 1 =0. Решение. Положим w=sin jc+cos jc, тогда u2=(sin jc+cos jc)2, или w2=l+sin2jc. Поэтому заданное уравнение примет вид w2 + 5w=0, откуда Теперь задача свелась к решению совокупности уравнений: sin jc+cos jc=O; sin jc+cos jc= —5. Первое уравнение совокупности (однородное уравнение 1-й степени) после деления обеих частей на cosjc преобразуется к виду tg х+ 1 =0, откуда jc= — -7-+ nk. 4 Второе уравнение совокупности не имеет решений, так как sin х^ — 1 и cos jc^ — 1, т. е. sin jc+cos jc^ —2. Однако —-5< -—2. Таким образом, решением заданного уравнения является семейство jc= л \ л£ 4 Пример 22. Решим уравнение jc. (13) Решение. Получаем последовательно: sin х cos a: sin 2x cos 2x sin (x+2*) cos x cos 2x * sin jc sin 2jc=sin jc cos 2jc+sin 2jc cos jc, sin jc (sin 2jc—-cos 2jc—2 cos2 jc)=O. 243
Это уравнение равносильно следующей совокупности уравнений: sin х=0; sin 2jc—cos 2jc—2 cos2 x=0. Из первого уравнения получаем x=nk. Второе уравнение приводится к виду sin2 х+2 sin x cos х—3 cos2 х=0, откуда после деления обеих частей на cos2 а: получим уравнение tg2 jc+2 tg х—3=0. Далее имеем tgx= — 3; tgx=l, откуда x=arctg( — 3)+пп; х=^-+пт. Проверка. Область определения уравнения (13) задается условиями cos хФО, cos 2x^=0. Ясно, что значения х из первого и второго семейств удовлетворяют этим условиям. Значения х из третьего семейства удовлетворяют условию cos хФ0% но не удовлетворяют условию cos 2хфО. В самом деле, cos 2 Cj-+ nm Wcos (—+ 2пт )=0. Таким образом, третье семейство состоит из посторонних решений. Итак, решением уравнения (13) является объединение двух семейств: x=nk; x=arctg( — 3) Пример 23. Решим уравнение sin х + 2 sin 2x=3+sin Зх. (14) Решение. Преобразуем уравнение (14) к виду (sin х—sin 3jc)+2 sin 2x=3, и далее 2 sin x cos 2x—2 sin 2jc+3=0. Дополним имеющиеся удвоенные произведения 2 sin x cos 2x и 2 sin 2x до полных квадратов: (sin2 х+2 sin х cos 2x+cos2 2x)+(sin2 2x—2 sin 2x+1)+3 = = sin2 x + cos2 2x+sin2 2x+1, т. e. (sin x+cos 2x)2 + (sin 2x- l)2+3 = sin2 x+2, откуда (sin x+cos 2x)2+(sin 2x- l)2+cos2 x=0. (15) Но сумма квадратов равна нулю тогда и только тогда, когда каждое слагаемое равно нулю. Поэтому уравнение (15) равносильно следующей системе уравнений: 'sin x+cos 2х=0, sin 2x—1=0, (16) |cosx=0. {i Решив третье (самое простое) уравнение системы (16), получим х=-^-+лк. Подставив эти значения х во второе уравнение системы, 244
будем иметь sin 2(-|-+л/м — 1 =sin (я+2лА)—1 = — 1 ФО, т. е. значения х=-^Н-л& не удовлетворяют второму уравнению системы (16). Но тогда система (16) несовместна; таким образом, уравнение (14) не имеет решений. Пример 24. Решим уравнение -V —3 —cos2jc+3sin 5х= 1 — sin x. (17) Решение. Возведя обе части уравнения (17) в квадрат и выполнив последующее приведение подобных членов, получим: 2 sin x+3sin Ъх = Ъ. (18) Так как sinx^l, sin5*<;i, то уравнению (18) удовлетворяют те и только те значения х, при которых одновременно sinx=l и sin 5jc= 1. Иными словами, уравнение (18) равносильно следующей системе: {sinx=l, sin5*=l. Решим ее. Из уравнения sin лг= 1 находим х = -^- Подставив эти значения х в левую часть второго уравнения системы (19) получим: sin 5(у-+2л*) = sin (y- Таким образом, x=~\-2nk — решение системы (19), а значит, и уравнения (18). Проверка. Выполним ее подстановкой в заданное уравнение (17). Получаем в левой части - з - cos2 (^-+ 2nk) + 3 sin 5 (-|-+ 2nk) = 0, а в правой части (|) =0. Таким образом, семейство х = -|-+2л& является решением уравнения (17). Пример 25. Решим уравнение Уx = V2 cos 2x. (20) Решение. Возведя обе части уравнения (20) в квадрат, получим l+sin2jt=2cos22x. Положим t/=sin2jc, тогда cos22x= s=s 1 —У2* и мы приходим к уравнению 1 +у = 2 (1 —у2), откуда нахо- 245
дим £/i == — 1; у2=—. Задача свелась к решению совокупности уравнений: sin 2х= — 1; sin 2х=—, из первого уравнения которой получаем х= —-2-+яй, а из второго Проверка. Так как выражение 1 -f-sin 2jc, находящееся под знаком радикала, неотрицательно при всех х, то выполним проверку, принимая во внимание, что по смыслу уравнения (20) cos2x>0. Проверим семейство х= — -^-+л£. Имеем cos 2х= = cos(—-j~\-2nk)=0. Это значит, что дг= — -|-+яя — решение уравнения (20). Проверим семейство х=( — 1)" ^--fy-n. Будем давать параметру я значения 0, 1,2, 3, ... . При /i = 0 Jt=~- и cos 2jc= =cos-2->0, при n=l x=="~7F+"7" и cos2x=cosf — — +п) <0, при /1 = 2 х=^-~|-я и cos 2х=созГ-|-4-2я j >0, при /г = 3 х= = — ]|-+у- и cos2^=cos( —-2-+Зя) <0, ... . Можно заметить (да и нетрудно доказать), что при четном значении п cos 2jc>0, а при нечетном n cos 2jc<0. Таким образом, решениями уравнения (20) являются лишь те значения х, которые получаются из второго семейства при четных значениях п. Полагая п = 2ту получаем семейства х= — ~\-nk; *=^-+я/п — решения уравнения (20). Полученная запись ответа достаточно компактна, однако при желании можно дать и другую запись, например такую: х= =75—Ь"о~^» где т=7==3/г + 1. Пример 26. Решим уравнение arccos х— arcsin *=-£*-. (21) о Решение. Взяв синус от обеих частей уравнения (21), получим уравнение sin (arccos jc—arcsin x)=-^-, являющееся следствием уравнения (21). Далее получаем: sin (arccos x) cos (arcsin дс)—cos (arccos x) sin (arcsin x)=—, 246
откуда ^l—y» *2=—-у. Проверка. Выполним ее подстановкой. Получаем: arccos x\ — — arcsin x\ = arccos -|—arcsin у=-^—"If^lf • Таким образом, *!=- корень уравнения (21). Далее, arccos лгг — arcsin X2 = = arccos( -I) -arcsin( --L) =|--( _■£.) «|-*f. Таким об- разом, лг2= —- посторонний корень. Итак, корнем уравнения (21) является х = — . Пример 27. Решим уравнение arcsin 2лг + arcsin jc = -~. (22) Решение. Возьмем косинус от обеих частей уравнения (22): cos (arcsin 2л;-f- arcsin jc) = cos —-, тогда откуда 7jc2 = — ,т. е. х\=- Проверка. Положим ос = arcsin 2х{ + arcsin x\. Тогда (/"з" /1 Лз"\\ arcsin-д/— + arcsin ( — -у —\\ =cos a, откуда т. е. cos а = у-. Так как далее 0<^/|- <^ и Ъ<т~л[т <^ • то 0<arcsin-y-|-<-j-H 0<arcsin y"y|-<7"- Но тогда 0<arcsin у-уЧ- arcsin (ууу) <у» т- е- а принадлежит первой четверти. 247
Итак, cos а=—иО<а<-£-,нов таком случае а=•—-, а значит, 2 2 о Х1=-|--д/-|—корень уравнения (22). 1 / о Проверим теперь значение Хг=— —"Д/у • Положим 0 = = arcsin 2x2 + arcsin *2, тогда arcsin ( —"Д/у) + arcsin Г —~X X V7") =р. Так как — 1 < — "д/у <0 и — К —у уу <0' то — я<arcsin (—~\/у) + arcsin (—у V7"} <° или ~"Jl<P<°- 1 / о Значит, р=^:—, откуда следует, что X2=— -"у-—посторонний корень. Итак, уравнение (22) имеет единственный корень хь=1-\/|-. Упражнения 14%. Объедините семейства. . л , , а) jc=±"t--t-"^ и б) х= ±--~|-л/г и \ л . /лк« ■ «\ л I 2 л г) *=--^+я(2/г+1) и х = ~+у . Л|2Л. л2л . л _ л е) х—~Л и л~у«; ж) *=(—1)*-т—Ьл^ и х = -7г + я«; 6 2 ч Л . «. Л Л з) JC= —f-лЛ И JC=-j-±-7T 1497. Из семейства jci исключите значения х, принадлежащие семейству х2. б) в) г) д) 248 л л +
я . я . я е) *|=-т-л. *2=—б""'""2"/1; я . / 1 \я 2я ж) Х1=уЛ, *2=(— 1) -g- Решите уравнения. 1498. a) (sin*--—j(sin х+1)=0; б) (cosx+-i-)(cos*-l)=0; B)(cos,-f)(sin,+f)=0. 1499. a) cosxtg3x=0; б) sin Ах cos х tg-2x = 0. 1500. 1501. (l+cosx)tgy cosx+cosx = 1505 2s»n»*-3»in*+l =() smjc cos'x—cosjc 1506. sin 3jc=cos 2x. 1507. sin (x— l)=cos (x+2). 1508. cos 5x=sin \5x. 1509. sin(5.n — x)=cos(2jt-f 7л). 1510. 2 cos2 Jt-fcos x— 1 =--=0. 1511. sin23jt-5sin3*-M=0. 1512. tg3x-ftg2jc-3tgjc=3. 1513. 4 cos4 x — 2 cos3 л —4 cos2 x + cos x+ 1 =0. 1514. 2 sin3 x—cos 2jc—-sin jc=O. 1515. 2cos2jH-5sin*-4 = 0. 1516. 3 sin- 2*-f 7 cos 2x = 3. 1517. 2cos2jt + smx=-2. 1518. V2 sin2 jc + cos *=0. 1519. sin 2*+cos 2* = sin a--|-cos a*. 1520. ^cos2*=cosx-fsinx. 1521. 4 sin2 x+ sin2 2x^.3. 1522. 4 cos2 2*+ 8 cos2 .v=7. 1523. sin 1524. 8 sin6 x-1-3 cos 2*4-2 cos 4x+ I =0. 1525. 3 (1 -sin *)= {-f cos 2x. 1526. sin x^-^-cos x. 1527. 3 sin * = 2 cos x. 4 1528. 2sinx-f ccs* = 0. 1529. sin x<:os * — 3 cos2 *=0. 1530. sirr jt-f sin xcosx —2cos2x=0. 1531. sin2x-}-3cos2* — 2r>in2*--=0 1532. 3 sin2 x+2 sin x cos x=2. 249
1533. 2 cos2 x—3 sin x cos x+5 sin2 x=3. 1534. sin6 x+sin4 x cos2 x=sin3 x cos3 x+sin x cos5 x. 1535. sin2 x cos2 x— 10 sin x cos3 x-f-21 cos4 x=0. 1536. ^- sinx—^- cosx=l. 1537. УЗ sin 2x + cos 2х=У2. 1538. -7j-sin 3x+^- cos 3x=sin 5x. 1539. 2cos3jc+V3sin x+cos x = 0. 1540. sin 5x-t-cos Ъх=ф cos 13*. 1541. 3 sin х — 5 sin (7x+-^\ =4 cos x. 1542. sin 3x sin 6x = sin Sx sin 5x. 1543. cos 4x cos 2x = sin 3x sin 5x. 1544. sin 5x cos 3x = sin 9x cos 7x. 1545. sin 6x cos 2x = sin bx cos 3x —sin 2x. 1546. sin6x+cos6x = T7r . lb 1547. 2cos2x+cos5x=l. 1548. sinx+sin 2x + sin 3x = 0. 1549. sinx + sin Зх + cosx-f cos 3x=0. 1550. sin5x+sinx+2sin2x=l. 1551. sin2x—cos2x = 2 —sin 2x. 1552. 8sin2y--3sinx-4 = 0. 1553. cos4 x+sin4 x—sin 2x+-|-sin2 2x=0. 1554. 2sinx—3cosx=3. 1555. 3sin2x+cos2x=2. 1556. cos 4x+2 sin 4x=l. 1557. sin x cos x—6 sin x+6 cos x+6=0. 1558. 4—4(cosx—sin x)-— sin2x=0. 1559. 5sin2x—ll(sinx-fcosx) + 7=0. 1560. cos x cos 2x cos 4x cos 8x=— . lo 1561. 3tg-£- 2 ' & sin x' 1 1562. cos2x—3cosx+l (ctg 2x —ctg x) sin (x—л)' 1565. sin 1566. ctg (x+л)—tg(x-*)=6tg2x. 1567. tg(x-15°)ctg(x+15°)=y. 1568. tg(120° + 3x)—tg(140° — x) = 2sin(80° + 2x). 250
2 я , . я 1 Зя 1570. sinxcosy+cosx siny==y , где —у 1571. sin4jc + cos4x=cos4jf, где — 1572. 2 sin4 2x— sin2 2x sin 4x=2 sin2 2x — sin 4x, где Зя ^ я <574. 6 sin2 * — 6 sin jc + cos 2*-f-1 1575- ГгРЗвдТ+л1 e0- 1576. V25 — 4jc2 (3 sin 2ях + 8 sin ях)=0. 1577. V49 — 4** (sin n* + 3cosy J =0. 1578. л/9 — ** (sin 2л:—3 cos x) = 0. 1579. U + 31 sinx = ^+3. 1580. 2U — 6| cosjc=jc—6. 1581. Найдите сумму корней уравнения cos 4x-fcos 2x+\ =0 на отрезке [0; 10я]. 1582. Найдите корни уравнения sin x tg 2х+л/3 (sin х—л/3 tg 2x)=3 л/3, удовлетворяющие неравенству 2-f- log j x^0. Решите уравнения. 1583. 2 sin 17х + л/3 cos 5x + sin 5jc=0. 1584. 4cos34+3V2sin^ = 8cos~. 1585. — cos -j-=cos3-j--f sin у . 1586. 4 sin 2x-tg2 (^—J") ==4* 1587. (sin 2jc+л/3 cos 2x)2 — 5 = cos (-i—2jc V 1588 1 +sin jc+.^-fsin" x+... "~ 1+cos2jc* 4jc 1589. cos — =cos2 x. о 1590. sin x + 2 cos jc = cos 2x—sin 2x. 1591. 32 cos6*—cos6x=l. 1592. tgx+ctgjt—cos4a: = 3. 1593. 2(1— sinx—cosA:)-T-tgjcH-ctgx=0. 1594. sin5x~cos5jc=—l- Д-. cos x sin x 1595. sin82jt+cos82x=i^ . 1596. sin10 x+cosie x=^r . lzo O4 1597. sinlox + cosI0x=?| cos42x. lo 1598. |cosjc|=cosjt-2sinx. 251
1599. 1600. V5~2lin~Jt = 6 sin л — 1. 1601. V2-f4 cos ~x=y+3 cos x. 1602. 1603. У— 3sin Ъх — cos2 x — 3 + sin jc= 1. 1604. tgx+~ 1605. 1606. "\/cos2 * + y+ "\/sin2 x + y = 2. 1607. Vl —2 tg x—VI +2 ctg x = 2. 1608. УЗ sin x—У2 sin2x —sin 2x + 3 cos2 x=0. 1609. cos x + ysin2 x —2 sin 2x-|-4 cos2 x=0. 1610. ycos 2x+yi +sin 2x = 2 ysin x + cos x. 1611. -yJ2 sin (x+2)—У2 cos2 x = ysin x(2 cos 2 —cos x). 1612. 2ctg2x—с 1613. 1614. 2 (ctg x— 1) cos 2x= 1 +ctg x. 1615. 3 tg 2x—4 tg 3x=tg2 3xtg 2x. 1616. 4 tg -f+2 tg -f-f 8 ctg x=tg ~ -tg -£-. Z 4 1Z О 1617. ™2 Wl ^~*л 1618. sin x —cosx 4 cos3 x . tg(*—^)tgxtg(*+«j cos20jc + cos2jc + 2sin2 x ^ 1619. , _. = —~v/o. cos lOx v 1620. sin2 jc+-t- sin2 3* = sin jc sin Ъх. 4 1621. l+23 2 1622. sin 5x + sin x = 2 + cos2 x. 1623. 24 1625. sin 4" + 2 cos X~~ =3. 4 3 1626. (sin x + V3 cos a:) sin 3* = 2. 1627. (sin 2jc + V3 cos 2л:)2-2 = cos C^- 252
1628. cos (я V*) cos (л -^=4)= 1. 1629. sin 18x+sin 10jt+sin 2jc=3 + cos 2 2x. 1630 ( . cos2x(l—^sin22x j=l. 1631. s 1632. cos6 2*= 1+sin4*. 1633. ctg ( у cos 2nx J =V3. 1634. 2 sin2 (y cos2 * J = 1 -cos (л sin 2x). 1635. 4arctg(x2-3*+3)=n. 1636. arcsin ^tg ~j—arcsm"Y у—-^-=0. 1637. arctg 3x—arcctg 3* = -j-. 1638. 2 arcsin2 x — 5 arcsin x-f 2 = 0. 1639. 4 arctg x — 6 arcctg лс = л. 1640. arcsin jt-farccos (1 —*)== arcsin (— x). 1641. 2 arcsin *=arccos 2дс. 1642. arcsin ~ 1643. arccos x = arctg x. 2 1 1644. arcsin —■ —arcsin -J\ —jc= arcsin -r-. 3 -\fx 3 -~- J =-^-« § 5. СИСТЕМЫ УРАВНЕНИЙ При решении систем тригонометрических уравнений используются те же приемы, что и при решении систем алгебраических уравнений. Часто бывает удобнее вместо общих формул, по которым решаются уравнения вида sin x = a, cosx=a, записывать решения этих уравнений в виде совокупности двух семейств. Пусть, например, нужно решить систему уравнений Если воспользоваться общими формулами, то придем к системе -У=: откуда находим: 253
— решение системы (1). Если решение первого уравнения системы (1) записать в виде совокупности x+y=-%--\-2nk\ х+у=Щ-+ о о +2я&, а решение второго уравнения системы (1) записать в виде совокупности х—у=-2-+2яп; х—у= — —\-2лп, то получим совокупность четырех систем: я I o_t. £ ** i -. 5я (x+y=f+2nk, (x+y=5jL+2nk, 4 V 4 (4) дс-у=—2-+2яп; \x-t/=-^-+2яп, откуда 1«/з = |-+: ~ ~ Эта совокупность семейств представляет собой решение системы (1). Конечно, такая запись не столь компактна, как запись решения в виде системы (3), но более наглядна, поэтому часто подобной записи отдают предпочтение. Обратим внимание читателя еще на одно обстоятельство: при переходе от системы (1) к системе (2) или к совокупности систем (4) мы использовали для записи решений первого уравнения системы (1) параметр &, а для записи решений второго уравнения системы — другой параметр — п. Употребление только одного параметра, например /?, привело бы нас к потере решений: так, в этом случае из первой системы совокупности (4) мы получили бы а множество 1\ пар вида (х{; у\) представляет собой собственное подмножество множества Z\ пар вида (х\; у\\ 254
где Итак, Z' c= Zi, 1'Ф1\У поэтому все пары (х\ у) такие, что (х\ £Z\\Z' оказываются «потерянными» решениями. Рассмотрим примеры. Пример 1. Решим систему уравнений ( sin х sin y = 0,75, Решение. Разделив левую и правую части первого уравнения системы (5) соответственно на левую и правую части второго уравнения системы, получим уравнение cos atcos {/=—. Заменив этим уравнением второе уравнение системы (5), получим систему sin х sin y=—, \ (6) ^COS XCOS У = Т", равносильную системе (5). Заменим теперь первое уравнение системы (6) суммой уравнений этой системы, а второе уравнение — разностью второго и первого уравнений. Получим новую систему: (cos xcos y + sin x sin y= 1, cos x cos y —sin x sin y= —\-* (7) ( cos (*-*/)=!, равносильную системе (6). Из первого уравнения системы (7) находим х—y=2nk, второе уравнение системы (7) равносильно совокупности уравнений х+у=Щ-+2пп\ х+у= — Щ-+2лп. о о Таким образом, от системы (7) мы перешли к совокупности систем (x—y=2nk, (x — y=2nk, [x+y=f+2nn; [x+y=-f+2nn, (8) которая равносильна системе (7). Из первой системы совокупности (8) находим семейство решений: 255
Из второй системы совокупности (8) находим семейство решений: •== — "з—^ я (п ~^~ ^' Проверка. Так как в процессе решения выполнялись только равносильные преобразования (это отмечалось в ходе решения), то совокупность семейств является решением системы (5). Пример 2. Решим систему уравнений (9) I cos'x=-5-cos у. s3*=4-< Решение. Возведем в квадрат обе части каждого уравнения системы (9) и сложим почленно уравнения, полученные в результате этого преобразования. Будем иметь sin6x+cos6 *=— и от системы (9) перейдем к новой системе: cos6jc=4-> <10) {sin6 jc+cos •з l sin3x=-£-s Решим уравнение sin6 x+cos6 *=-т-- Последовательно получаем: Таким образом, решение системы (10) мы свели к решению системы -4^2 *' [sin3 x*«i-sin y. Имеем: < i _._ „ / , V2\3 или % /о \.ln jr— ±^.. 256
Рис. 43 откуда (12) Переход от системы (9) к системе (10), возможно, не был равносильным преобразованием (возведение в квадрат), поэтому необходима проверка. Проверка. Изобразим значения хну, содержащиеся в системе (12), точками двух окружностей (рис. 43). В точке А\ имеем simtX), cosjoO. Тогда из системы (9) заключаем, что siny>0 и cos(/>0. Но из точек Вь В2, В3, В4 только точка В\ имеет положительные абсциссу и ординату. Значит, (А\\ В\) — геометрическое решение системы (9), т. е. я 4 •2л*, — решение системы (9). Рассуждая аналогично, получаем (А2; В2), (Аз; Вз), (А*; В4) — геометрические решения системы (9), т. е. — решения системы (9). Итак, решением системы (9) является следующая совокупность семейств: 257
Пример 3. Решим систему уравнений (x+y+z=n, Jtg*tgZ=:2, (13) |tg«/tgz=18. Решение. Так как x+y+z=n, то tg(x+y)=tg(я—г), Этим уравнением заменим первое уравнение системы (13) и рассмотрим новую систему: 1-tgxtgy lgZ> tgxtgz=2, (14) tgi/tgz=18. Введем новые переменные: (u = igxy \w = Тогда система (14) примет вид: \w=tgz. (15) или ■ • - - (16) fuvw = u-\~v + w9 law=2, \vw = \%. Разделив почленно первое уравнение системы (16) на второе, получим: у=ц+1;2+ц; , откуда v = u+w. Заменим этим уравнением первое уравнение системы (16). Получим: (17) и далее ( l l (18) Система (18) имеет следующие решения: {"1=0,5, (u2=— 0,5, fi=4,5, <u2=—4,5, t»i =4; (ш2=—4, Возвращаясь к старым переменным, получаем: 258
{jti = arctg 0,5 + nk, (X2= — arctg 0,5 + я£, yx = arctg 4,5 + лл, I y2 = — arctg 4,5 + nny (19) Zi = arctg 4+ ят; \z2= — arctg4+nm. Проверка. В процессе решения было три преобразования, каждое из которых могло привести к неравносильной системе: «взятие тангенса» при переходе от системы (13) к системе (14), освобождение от знаменателя при переходе от системы (15) к системе (16) и деление при переходе от системы (16) к системе (17). К потере решений могло привести только деление, но в данном случае этого не произошло, так как правая часть «уравнения-делителя» равна 2, т. е. отлична от нуля. Остальные преобразования могли привести к появлению посторонних решений: «отсев» посторонних решений можно осуществить с помощью непосредственной подстановки значений, содержащихся в найденной выше совокупности (19), в исходную систему. Легко убедиться в том, что совокупность (19) удовлетворяет второму и третьему уравнениям системы (13). Чтобы удовлетворялось и первое уравнение этой системы, придется записать совокупность (19) так: (20) •лл + 2л (мы воспользовались тем, что arctg 0r5 + arctg 4,5 + arctg 4 = л — это следует из решения примера). Совокупность семейств (20) представляет собой решение системы (13). Пример 4. Решим систему уравнений 'sin jc=cos у, |V6sin(/=tgz, (21) е Решение. Возведем в квадрат обе части каждого из уравнений системы (21). Получим: sin2 х=cos2 у, 6sin2i/=tg2z, (22) L4sin2z=3ctg2jc. Введем новые переменные Тогда система (22) примет вид: <23> 259
откуда находим: 2 ' Т* .-з- " 4 — решения системы (23). Теперь задача сводится к решению следующей совокупности систем: f sin2*=4-. 'sin2jt=l, ;sin2y = 0, { sin*y=T' (24) = 0; {sin2 х-- sin2 у- sin2 г-- sin2 Из первой системы этой совокупности находим: 2,== ЯШ. Из второй системы совокупности (24) находим: (25) (26) 22 = л/л. Проверка. Подставив найденные решения (25) и (26) в исходную систему (21). Для этого воспользуемся тем же приемом, который был применен при решении примера 3, а именно изобразим значения х, t/, г% содержащиеся в системе (25), точками соответственно трех окружностей (рис. 44). Рис. 44 260
Рис. 45 Возьмем точку Л|. В ней sinx>0, а потому cost/>0 (см. первое уравнение системы (21)). Тогда из двух точек Ви В2 мы выберем точку с положительной абсциссой, т. е. В\. Заметим, что при этом из точек С|, С2 можно взять любую. Аналогично точке А2 соответствует точка В2. Итак, мы получили четыре геометрических решения: (Л,; В,; С,), (Ах; Вх\ С2), (Л2; В2\ С,), (Л2; В2\ С2). Таким образом, вместо семейства (25) мы получаем следующую совокупность семейств: ( X[=f+2nk, I \ у\=2ппу \ У2' Z2'- -пт (27) (остальные кортежи (х\ у\ z\ содержащиеся в семействе (25), являются посторонними решениями для исходной системы). Изобразим теперь точками окружностей значения х, у, г, содержащиеся в системе (26) (рис. 45). Рассмотрим точку А\. В ней sin*>0, ctgx>0, значит, cos у >0, sinz>0 (см. первое и третье уравнения системы (21). Так как cos t/>0, то на второй окружности выберем точки с положительными абсциссами В\ я В4. Так как sin z>0, то на третьей окружности выберем точки с положительными ординатами С\ и С2. Рассмотрим точку Si. В ней sint/>0, значит, tg 2>0 (ем. второе уравнение системы (21)), а потому из точек Си С2 выберем точку С\ (в ней tgz>0). Аналогично точке В* будет соответствовать точка С2. Итак, мы получили еще два геометрических решения (А\\ В\\ С\) и (Ли Ва\ С2) и соответственно следующую совокупность семейств решений системы (21): п 4 (28) Рассуждая аналогично, найдем еще шесть геометрических реше- 261
ний: (А2; Вц С3), (А2; В4; С4), (Л3; В2; Ci), (Л3; В3; С2), (Д4; В2; С3), (44; В3; С4) — и соответственно совокупность семейств решений: •2ят; (29) z*=T+2nm; Таким образом, совокупность семейств (27), (28) и (29) —решение системы (21). Упражнения Решите системы уравнений. 1645. 1647. sin sin (jc—у)=0. sin x+cos «/=0, sin2*-f-cos2 у=-£- 1649. ^ л . . 1 sin x cosy =0,5. =2, 1646. 1648. 1650. 1652. sin jc cos (/=0,25, sin у cos jc=0,75. sin xsin (/=0,25, 1651. Г sin Jt+sin r/=0, у cosx-f-cos y=0. 1653. ( sin2 x+sin2 (/=0,75, 1654. 1655. Г sin2 *+cos2 y=0,5, 1657. fcos jcsin (/=^-, 1 Зя 1 х-|-г/= — 1659. pgx+tgy-1, 3 1656. 1658. 1660. cos jc-f-cosy=0,5, sin2 x-f sin2 (/=1,75. x-(/=i-, cos2 лх—sin2 я!/=0Д ' cos2 jc+cos2 (/=0,25, —2 1661. Г cos (x—1/)=2 cos (x+y)t 1662. у cos jc cos 1/=0,75. 0,5, cos x cos у =0,25. 262
1663. (sin(x+y)sin(x-y)= —L, 1 cos2xcos2i/=—. 1664. rsinxsiny=j^, 1665. fx-t/=—-, \tg x tg y=^r. I tg x tg (/= 1. 1666. 1667. i 2x-f-sin 2y)=2 (1 + cos2 (x—( 5n sin x=2 sin y. ^cos 2x=2 cos 2y. 1670. V2 sin x—sin {/, V2 cos x=V5 cosy.. 1671. ( sin x cos (x-f y)+sin (x+#)=3 cos (x |4sinx=5ctg(x+y). 1672. Г ctg x+ sin 2«/=sin 2x, 1673. ( 4 tg 3i/=3 tg 2x, | 2 sin (/ sin (x+y)=cos x. \ 2 sin x cos (x—y)=sin i/. 1674. Г tg x+ctg y=*3, 1675. Г sin x = sin 2yt 1676. |x-fy=~, 1677. | sinx—siny=—, * J ^^ I " ^» Щ \.V/J W —|— VVSO ff —— —JT—. 1678. Г sin y=5 sin x, 1679. J cos x cos у= | 3cosx+cosw=2. 1 9. J 1 1680. i sin2 x=cos x cos y, 1681. Г cos2 j/+3 sin x sin i/=0, \ cos2 x=sin x sin y. \ 21 cos 2x—cos 2y= 10. 1682. Г 3 sin2 x—cos x cos y=0, | 11 cos 2x+cos 2t/=6. 1683. Г sin2 x=sin V, 1684. Гх+1/+z=я, | J tgxtgz = 3, ^6 (х+у+г=*п, 1686. Гх+у+г < tg x tg y=2, J sin x=2 1 tg x+tg y+tg z=6. 1V3 sin y 1685. (х+у+г=*п, t t 2 J sin x=2 sin y, V3 sin y=sin z. 263
1687. fsin2jt+sin2«/+sin2z=l, ) COS2 2 2 1688. Найдите решения системы уравнений Isin x| sin у» — —, I cos (х+y)+cos (х—у)=у, удовлетворяющие условиям: Г 0<х<2л, \ я<1/<2я. § 6. НЕРАВЕНСТВА Решение тригонометрических неравенств сводится, как правило, к решению простейших тригонометрических неравенств, т. е. неравенств вида sin x>a, cos х«х и т. д., а также к решению совокупностей, систем или совокупностей систем простейших тригонометрических неравенств. Для решения простейших тригонометрических неравенств во многих случаях удобно пользоваться окружностью, на которой множество значений переменной, удовлетворяющих заданному простейшему неравенству, изображается в виде одной или нескольких дуг. Аналогично тому, как с помощью неравенств задаются промежутки на числовой прямой, можно записывать и множество точек, принадлежащих той или иной дуге окружности 2. Условимся символом wM,M2 обозначать дугу, для которой точка М\ — начальная точка (в обозначении дуги она записывается первой), М2 — конечная точка пути, описываемого текущей точкой по окружности 2 в положительном направлении (против часовой стрелки). Так, пусть с помощью неравенств требуется записать следующие дуги окружности 2 (рис. 46): 1) weo6i; 2) ^в|в3; 3) v^6i0<>; 4) ^6261; 5) V-/60M; 6) v^M6o; 7) ч^взМ, где точка М — середина дуги в^г. 1) Точка в0 соответствует числу 0, точка в\ соответствует числу -~, поэтому текущая точка дуги 60в| соответствует числу х такому, что 0<х<-|-. Учитывая, однако, что если точка окружности соответствует числу х, то она соответствует и всем числам вида x+2nk (k — целое), получаем, что точки дуги во6| соответствуют числам х, удовлетворяющим следующей системе неравенств: -2-+2я*, или Это— аналитическая запись дуги 6о6|. 264
в. i г » ув* Рис. 46 Рис. 47 2) Для дуги 6163 получаем: -j~\-2nk^x^:-2-+2nk. 3) Как было отмечено выше, в этом случае под записью ^6i60 понимается дуга 61626360. При первом обходе окружности точка 6i соответствует числу -2-, а точка во — числу 2я (но не числу 0, так как обход окружности от в| к во идет в положительном направлении), значит, аналитически ^6i6o можно записать следующим образом: -2-+ 2л* < х < 2я + 2nk. 4) Дугу 626i можно записать двумя способами: или 5) 6) 7) :22 Замечание. Записывая дугу в виде необходимо следить за тем, чтобы выполнялось неравенство а<р, иначе система неравенств (1) окажется противоречивой. Пусть теперь каждая четверть окружности 2 разбита на три равные части (рис. 47). Найдем аналитические записи следующих дуг: 1) wBifl2; 2) w6i54; 3) ^В3А\\ 4) ^А2В\\ 5) ^Л402; 6) Л3В2. 1) Рассмотрим дугу В\В2- Так как каждая из дуг 60Ль А\Ви Bi6i, 61Л2, ..., В460 имеет длину -|-, то при первом положительном обходе окружности точка В\ соответствует числу -2-, точка В2 — о 265
числу —. Следовательно, аналитическая запись дуги В\В2 будет: 2) |^ 3) ^ВзЛ,:-^Ч-2я*<х<|-+2я* (или ^- Замечание. Еще раз обращаем внимание читателя на необходимость контроля при записи концов дуг. Так, при первом обходе окружности 2 точка В* соответствует числу — ; продолжая движение в направлении от точки Вз к Аи мы при переходе через точку во начинаем обходить окружность второй раз, т. е. точка А\ соответствует теперь числу -т-. Отсюда и получается вторая запись для дуги В*А\. 4) ^Л2В1:-^Ч-2л*<х<!-+2яА (или ^ + 2ял). 5) ^Л4в2:-|Н-2я*<х<я+2яЛ (или 2£ + 2ял) . 6) ^ЛзВ2:-^"+2яЛ<х<^-+2л* (или ^-^ +2яп) . Пример 1. Решим неравенство i-. (2) Решение. По определению, sin х — это ордината точки / окружности 2, соответствующей числу х. Отметим на окружности 2 точки, имеющие ординату, равную — (точки М и Р на рисунке 48). Тогда точки, ординаты которых больше ^-, заполняют открытую дугу МР. Эту дугу естественно назвать геометрическим решением неравенства (2). Составим аналитическую запись открытой дуги МР: -^ ^. Это и есть решение неравенства (2). Припер 2. Решим неравенство cos *<-£-. (3) Решение. По определению, cos x — это абсцисса точки /££, 266
Рис. 48 Рис. 49 соответствующей числу х. Отметим на окружности 2 точки, имеющие абсциссу, равную — (точки М и Р на рисунке 49). Тогда гео- метрическим решением неравенства (3) будет открытая дуга МР (каждая ее точка имеет абсциссу, меньшую —). Составим анали- тическую запись открытой дуги МР: arccos •arccos —\-2nk. о Пример 3. Решим неравенство (4) Решение, tgx не определен при х==-|-+яй. Этим числам соответствуют точки 0i и 6з окружности 2 (рис. 50). Отметим на полуокружности 6361 точку М = М(х) такую, что tgjc=~Y-. Так как на дуге 6361 (а точнее, на каждом из интервалов числовой прямой R, отображающихся на дугу 6361) функция y = \gx воз- ft, ^"Чм 267
растает, то неравенство tgx< — у- будет выполняться для всех точек дуги 6361, лежащих от точки М в отрицательном направлении, т. е. на полуоткрытой дуге в3М. Так как, далее, основной период тангенса равен я, то неравенство (4) будет выполняться и для всех точек дуги в|Р, отличающейся от дуги взМ на половину окружности. Итак, геометрическим решением неравенства (4) является объединение двух полуоткрытых дуг 6зМ и в\Р. Составим аналитические записи указанных дуг. Для дуги 63М имеем: ^ -arctg у а для дуги в1Р:у y Впрочем, решение неравенства (4) можно записать короче: —у+яя<х<— a Пример 4. Решим неравенство — arctg у + ля. ^. (5) Решение, ctgx не определен при x=nk. Этим числам соответствуют точки в0 и в2 окружности 2 (рис. 51). Отметим на полуокружности вовг точку Af = Af(x) такую, что ctgx=^-f для этого отложим дугу 6oAf, длина которой равна arcctg*p=--p. Так как на дуге в0в2 функция t/=ctg x убывает, то неравенство ctg х< будет выполняться для всех точек дуги вовг, лежащих от «5 точки М в положительном направлении, т. е. на открытой дуге Мв2 (при решении неравенства ctg x>\- пришлось бы взять дугу «3 в0М). Учитывая, что основной период котангенса равен я, отметим еще дугу Я60, на которой выполняется неравенство (5) (она получается из дуги Мв2 поворотом вокруг точки О на 180°). Итак, геометрическим решением неравенства (5) является объединение двух открытых дуг Мв2 и Рво. Аналитическая запись дуги М&2 такова: —+2nk<x<n + 2nk; аналитическая запись дуги Рв0 такова: ^ + 2яЛ<х<2я + 2я*. о Короче решение неравенства (5) можно записать следующим образом: -%-+nk<x<n+nk. 268
Пример 5. Решим систему неравенств (6) cos х> — Решение. Найдем геометрическое решение неравенства sin x<^~ (дуга МР окружности 2 отмечена на рисунке 52 внутренней штриховкой). На той же окружности найдем геометрическое решение неравенства cos x> > —-^ (соответствующая дуга ЕК отмечена на рисунке 52 внешней штриховкой). Тогда геометрическим решением системы (6) будет пересечение дуг МР и ЕК, т. е. объединение дуг МК и ЕР. Осталось лишь составить аналитическую запись каждой из этих дуг. Для дуги МК имеем: для дуги ЕР имеем: Зл Пример 6. Решим неравенство 2 sin2 (*4~f-)+л/3 cos 2jc>0. (7) Решение. Применив формулу 1—cos 2а = 2 sin2 а, преобразуем неравенство (7) к виду 1 —cos (2x+у-)+л/3 cos 2jc>0, и далее — cos(2x+y-)+V3cos2x> — I, sin 2х+л/3 cos 2х> — 1, i 2jc> — -5-, sin -£-sin 2x+cos --г-cos 2x> —4"» 4-sin (8) Решим неравенство (8). Положив /=(2х—т") » получим неравенство cos/>——, решение которого находим с помощью окружности 2 (рис. 53): —^+2nk<t<^-+2nk. Возвращаясь «5 о 269
к переменной х9 получаем: — %Ч-2яЛ<2х—■£-<%■+ 2лА, отку- о Do да —-~+я*<х<^.+я* — решение неравенства (8), т. е. и неравенства (7). Пример 7. Решим неравенство б sin2 х—sin jccos ж—cos2 x>2. (9) Решение. Так как 2 = 2 (sin2 jc+cos2 jc), to можно преобразовать неравенство (9) к виду 4 sin2 jc — sin x cos x—3 cos2 x>0. (10) Так как cos2jc>0, to неравенство (10) равносильно следующей совокупности систем: cos2x>0, 2 /cos2x=0, jcoe-*>ut \4sin2*>0; t4tg2*-tg*-3>0. (11) Первая система совокупности (11) имеет решение: х=-^-+пк. Вторая система этой совокупности равносильна следующей системе: 2 [(tg*-i)(tg*+-f-)>o, которая, в свою очередь, равносильна совокупности неравенств: tgx<—f-; tgx>l. (12) Найдем решение совокупности (12). Геометрическим решением неравенства tg*>l является объединение открытых дуг М9\ и М&з (на рисунке 54 отмечено внутренней штриховкой), геометрическим решением неравенства tg х<—— является объединение открытых дуг Q\L и &зК (отмечено внешней штриховкой). Геомет- М Рис 53 270
рическое решение совокупности (12) представляет собой объединение четырех дуг: Мви AJ63, &\L, вэ/С. Так как, далее, геометрическое решение первой системы совокупности (11) представляет собой двухэлементное множество [Q\t 63}, то геометрическое решение этой совокупности представляет собой объединение двух дуг ML и NK. Составим аналитическую запись дуги ML: Учитывая, что дуга NK получается из дуги ML поворотом вокруг точки О на 180°, мы можем не составлять аналитическую запись дуги NK, а сразу записать решение совокупности (12) в виде Это и есть решение неравенства (9). Пример 8. Решим неравенство sin je + cos*<-7^—. (13) 1 sin х у ' Решение. Имеем последовательно: Д<0, Д<0, sin х sin* sin x cos л—cos2 x ^q cos x(sin jc—cos *) ^ n (\4\ sin x * sin x ' ^ ' Воспользуемся тождеством sin *=tg xcos x. В данном случае это сужает область определения неравенства, но не приводит к потере решений, так как значения х% при которых cosx=0, не являются решениями неравенства (14). Неравенство (14) преобразуется к виду cos»,(tg*-i) 0 и 1&£^<0. (15) sm х sin x ' Полученное неравенство равносильно следующей совокупности систем неравенств: sinjt<0, (16) (17) Решим систему (16). На рисунке 55 показано объединение дуг и Мвь которое представляет собой геометрическое решение неравенства tgx>l, а дуга 6260 — геометрическое решение неравенства sin x<0. Геометрическим решением системы (16) является дуга Рвз, ее аналитическая запись имеет вид: 271
м Решим систему (17). Из рисунка 56 видно, что геометрическим решением этой системы является объединение дуг 60М и 6162. Аналитическая запись имеет вид: Значит, решение совокупности систем (16) и (17), а вместе с тем и решение неравенства (13) таково: Отметим в заключение, что не всегда можно решить систему или совокупность тригонометрических неравенств с помощью окружности. Так обстоит дело, например, в тех случаях, когда наименьшее общее кратное основных периодов всех функций, входящих в неравенства, составляющие данную систему или совокупность, больше длины окружности, т. е. больше 2л. В подобных случаях вместо окружности используется числовая прямая. Упражнения Решите неравенства. 1 .a) sinx> — у; в) л/3 6) cosjt<^; г) ctgx<-l. 1690. a) sinx<4-; в) tgx<5; О б) cos x> -0,7; г) 272
Решите системы неравенств. 1691. (s\nx<-j> 1692. ( sin x>-^, \ I I tgjc<0. COSJC<y. V ,693.fcos*<f. im-f^X<U^ 1695. TsinjOy. 1696. (cos *>—у, (cos *> 1 1 |tgx<3. I cos x<-=-. V V 5 . I sinx<-=-, 1698. ( tg I ct 2 ICt( 1697. I sinx<^-t 1698. ftgx>0,23. 1699. rtg2x<l, 1700. 1701. f sin3x>y, Решите совокупности неравенств. о 1702. sinx>~; cos*<0. о 1703. cosjc<y; tgjc>—3,5. 1704. sinx<— ^; ctgx<7. 1705. tgx<^ 1707. sin2jc<—; cos3x>—^-. Решите неравенства. 1708. cos^>y. 1709. 1710. cos3jc + V3sin3jc<— -yfi. 1711. cos 2x + cos jc>0. 1712. ! +с^2*<0> l7!3' Sin 3jc>cos 3jc' 1714. tgJc+3ctgjt-4>0. 1715. sin2 x—cos2 x-3 sin x-f2<0. 1716. 2sin2y+cos2x<0. x. 1718. ~ Sin OJC-f COS OX 273
1719. 5 sin2 x—3 sin x cos x—36 cos2 x>0. 1720. 2 sin2 x—4 sin x cos x+9 cos2 x>0. 1721. cos2 x+3 sin2 x+2 V5 sin x cos x< 1. 1722. 3 sin2 x+sin 2x—cos2 x>2. 1723. sin 4x+cos 4x ctg 2x> 1. 1724. 2 + tg 2x+ctg 2x<0. 1725. 2cosx(cosx—V8tgx)<5. 1726. sinx-f-cosx< . COS X 1727. sin*x+cos6x<.jL. 1728. ctgxH—SW* <0. 1729. cos22x+cos2x<l. 1730. 8 sin2 ~+3 sin x-4>0. 1731. sinx+cosx>V2cos2x. 1732. tgx+tg 2x—tg 3x>0. 1733. cos 2x cos 5x<cos 3x. 1734. sin 2x sin 3x— cos 2x cos 3x>sin Юх. 1735. ct 1736. 2 sin2 x— sin x+sin 3x< 1. 1737. 4 sin x sin 2xsin 3a:>sin 4x. 1738. Сс^2^>3 tg x. 1739. 3 cos2 x sin x-sin2 x< -1. 1740 cos x+2 cos2 x-4-cos 3x cosxH-2cos2x— 1 Решите системы неравенств. 1741. fcosx<0, 1742. fsin4<4*. I . 3 All 1 S!n-=-X>0. \ л 1 ^ 5 ^Cos2x>-y. 1743. (s'mYX>~~T* x ^ Ф 8^ 2" f 7. УРАВНЕНИЯ, СИСТЕМЫ УРАВНЕНИЯ И НЕРАВЕНСТВА С ПАРАМЕТРАМИ Пример 1. Решим уравнение sin4 x+cos4 x=a. (1) Решение. Применяя формулы понижения степени, получим: и далее Найдем контрольные значения параметра (см. с. 177). В данном случае это такие значения параметра, при которых правая часть уравнения 0 или 1 (если 2а—1<0 или 2а—1>1, то уравнение не имеет решений). Если 2а—1=0, то а—^\ если 2а—1 = 1, то а=1. 274
Итак, рассмотрим уравнение (2) в каждом из следующих пяти случаев: 1) а<-±-; 2) a=j-; 3) у-<а<1; 4) а=1; 5) а>1. 1) Если а<-£"» то 2а—КО и уравнение (2) не имеет корней. 2) Если в=у-> то уравнение (2) принимает вид cos22*=0, откуда находим дс=-^-+-|-Л. 3) Если -|-< а < 1, то 0 < 2а — К1. Преобразуем уравнение (2) к виду: i"t"c^)s4*=2a— 1, и далее cos4x=4a— 3. Так как в рассматриваемом случае ^-<а<1, то 2<4а<4, а тогда — 1<4а— — 3<1. Значит, уравнение cos 4*=4а — 3 имеет решение. Получим 4х= ± arccos (4а—3)+2яЛ, откуда х= ±-j- arccos (4а—3)+-|-Л. (3) 4) Если а=1, то уравнение (2) принимает вид cos22jc=1. Из этого уравнения находим х=у Л. 5) Если а>1, то 2а—1>1 и уравнение (2) не имеет корней. Заметим, что если а=у- или а=1, то решение тоже можно записать в виде (3). Ответ: 1) если а<-|-; а>1, то корней нет; 2) если -|-<а<1, то *= ±— arccos (4а—3)+-j-fe. Пример 2. Решим уравнение (а-1) sin2 х-2 (а+1) sin х+2а-1 =0. (4) Решение. Положим r/=sin jc, тогда уравнение (4) примет вид к-1=0. (5) Первым контрольным значением параметра а будет значение а=1, которое обращает в нуль коэффициент при #*. При а=\ уравнение (5) принимает вид — 4у+1=0, откуда находим !/=—, т- е- sinx=—, и, следовательно, %=( — 1)* arcsin -i- Рассмотрим теперь случай, когда а#1. Найдем дискриминант уравнения (5). Имеем: -J-=(a+l)2-(a—1)(2а-1>=—а2+5а. Вторыми контрольными значениями параметра а будут те значения, при которых D=0. Это будут значения а=0, а=5. Заме-
тим, что D<0, если а<0 или а>5, и D>0, если 0<а<5. Значит, нам нужно рассмотреть уравнение (5) в каждом из следующих случаев: а<0; j 0<а<5; а>5. \ Если а<0 или а>5, уравнение (5) не имеет корней. В случае г 0<а<5 уравнение (5) имеет два действительных \1 корня Так как t/=sinx, то должны выполняться следующие двойные неравенства: — l<t/i<l, — l^j/2^1. Нетрудно заметить, что у\= ^J.ta""— удовлетворяет двойному неравенству — 1<^/i<1 лишь при а=0. В самом деле, если а=0, то i/= —1; если а>0, то а+1>а—1 и тем более а+1 + +У5а — а2>а— 1, т. е. i/i>1. Если а=0, то уравнение sin jc=i/i, принимает вид sinx= — 1, откуда находим х=—-|-+2яЛ. Будем теперь искать значения параметра а (из рассматриваемого множества г 0<а<5,), которые удовлетворяют системе неравенств \аФ1 — 1 ^ У2 ^ 1, т. е. системе <а-И—У5а—g2^ _^ j -V55^?< j (6) Система (6), в свою очередь, равносильна следующей совокупности систем неравенств: 'а-КО, {а—1>0, a+1-VSa- Решим первую систему совокупности (7). Имеем: {а>1, Га>1, V5a-az<2a, и далее J 5a-a^<4a2 ^=7>2, \5a-a2>4, откуда находим 1<а<4. Решим вторую систему совокупности (7). Имеем: fa<\, \ 276
a>0) fa<l, 1 5а—а и далее (так как -а2<4, откуда находим 0 < а < 1. Итак, совокупность систем (7), а следовательно, и система (6) имеют решения: 0<а<1; 1<а<4. Это значит, что на множестве \аф\ уравнение имеет решение только в том случае, если Это решение таково: *==(— 1)* arcsin a+l~~V5g—fl -f»nfe. Заметим, что эта за- а— I пись включает в себя и рассмотренный выше случай, когда а=0. Если 4<а<5, то уравнение (8), а с ним и уравнение (4) не имеют корней. Ответ: 1) если а= 1, то х=(—1)* arcsin -^- 2) если /0<а<4, \ то x=(-lf arcsin 3) если а<0; а>4, то уравнение не имеет корней. Пример 3. Решим уравнение cos(a+x)=—. (9) COSJC Решение. Умножив обе части уравнения (9) на cos xy получим: cos х cos (a-|-x)=cos a, и далее t — a—x)=2cos a, т. e. cos (2x + a) = cos a. (10) Из уравнения (10) находим: x = nk\ x=— а+лк. (11) Проверка. В процессе решения уравнения (9) мы выполнили умножение обеих частей уравнения на cos x, что привело к расширению области определения уравнения, а значит, и могло привести к появлению посторонних корней. Отберем из найденной совокупности семейств решений уравнения (10) такие семейства, которые являются решениями уравнения (9). Для этого исключим из совокупности семейств (И) значения х, при которых cosjc=0, т. е. значения х= 277
=-|-+ял. Ясно, что семейства x=nk и х=-|-+ял не пересекаются. Полагая далее — а+яЛ=-~4-лл, находим a—f(-l- Это значит, что семейство jc= — a + nk является решением уравнения (9) лишь при значениях аФ-^-(2п — 2* — I), или, короче, при -=-(2/-1), где / = л-*(/=0; ±1; ±2; ...). Ответ: 1) если а=-|-(2/ — 1), то x=nk\ 2) если а=И=у-(2/— 1), то х=яЛ; *= — a+nk. Пример 4. Решим систему уравнений sin х cos у = а2, sin у cos х=а. Решение. Заменив первое уравнение системы (12) суммой, а второе — разностью первого и второго уравнений, получим систему, равносильную системе (12): sin x cos y+sin i/cos х=а2 + а, / sin (х+у)=а2 + а, (13) sin xcos у — sin i/cos jc=az — ar ^sin (jc—y)=az — a. Ясно, что система (13) имеет решения тогда и только тогда, когда параметр а удовлетворяет следующей системе неравенств: r|a2+a|<l, Ila2-a|<l. Система (14) равносильна такой системе: или 1а2 + а+1^0, а2 — а—1^0, а2 —а-+-1 ^0. Второе и четвертое неравенства системы (15) выполняются при любых а, так как квадратные трехчлены, содержащиеся в левых частях указанных неравенств, имеют отрицательный дискриминант и положительный старший коэффициент. Значит, система (15) равносильна следующей системе: (а2 + а —1^0, ^а2 — а—1^0, решив которую находим: Только при этих значениях параметра а имеет решение система (13). Итак, пусть — g"! ^fl^ 2 1 ' **3 системы (^) получаем: 278
(x+y=(-\)k arcsin (а2 + а)+яЛ, |х—1/=(—l)n arcsin (а2 — а)+пп, и далее г=-^-((—1)* arcsin (а2 + а)+(—l)n arcsin (а2 — а)+яЛ + яя), ^=4-((-1)*агС5|п(а2 + а)-(~1)пас51п(а2-а)+я*-ял). -, то решений нет; Ответ: 1) если д<— где а=(— 1)* arcsin (а2 + а\ р=( — l)n arcsin {a2 —a). Пример 5. Решим неравенство Решение. Преобразуем неравенство (16) к виду '•—I—:—^я, и далее : ^#» т. е. /i*?\ sin jc cos*sin x (17) 2 9 Положим t/=sin2x. Тогда неравенство (17) примет вид —^а, if и задача сведется к решению следующей системы неравенств: (18) Замечаем, что а=0 — контрольное значение параметра а. Значит, нам надо рассмотреть три случая: 1) а=0; 2) а>0; 3) а<0. • 2 1) Если а=0, то система (18) принимает вид I откуда находим — 2) Если а>0, то система (18) преобразуется откуда находим: к виду I — (19) 279
Здесь контрольным значением параметра является значение а = 2, поэтому надо рассмотреть три случая: а) 0<а<2; б) а = 2; в) а>2. а) Если 0<а<2, то — >1, и система (19) имеет следующее решение: — 1 ^ у < 0; б) если а=2, то система (19) имеет следующее решение: в) если а>2, то система (19) имеет следующее решение: 3) Если а<0, то система (18) преобразуется к виду 2 , и далее (20) Здесь контрольным значением параметра является а =—2. Поэтому надо рассмотреть три случая: а) а<— 2, б) а=—2; в) —2<а<0. а) В случае а< — 2 имеем —> — 1 и из системы (20) находим б) В случае а=—2 из системы (20) находим —1<(/<0. в) Наконец, в случае — 2<а<0 имеем —< — 1, и система (20) имеет следующее решение: —1<(/<0. Подводя итоги, получаем следующее решение системы (18): 1) если а<— 2, то -2- 2) если —2<а<2, то —l<t/<0; 3) если а=2, то — 1<у<0; t/=l; 4) если а>2, то —1<(/<0; -^- Так как y = s\n 2x, то получаем: 1. Если а<— 2, то —<sin 2x<0, откуда (рис. 57) [ 280
Рис. 58 а значит, nk + -£-< x < -£-- 4- arcsin -^- nk+±- arcsin -^- 2. Если —2<a<2, то — l<sin2x<0, откуда а значит, яЛ—-^- 3. Если a ==2, то из системы неравенств — l<sin2x<0 получаем: яД—~<х<я/?, а из уравнения sin 2x= 1 находим: 4. Если а>2, то из системы неравенств — l<sin2x<0 находим (как и выше) nk——<x<nk% а из системы — <sin2x<l имеем (рис. 58): 2я* + arcsin — а я—arcsin —+ 2лЛ, а а откуда nk+-|- arcsin "^^^^j 1-arcsin -j—1-я*. Ответ: 1) если а<— 2, то яй+-2-<дс<-2—a + я*; ; 2) если —2<а<2, то nk — -~<х<я*; 3) если а=2, то — -|-<*<я*; ; 4) если а>2, то nk—-j-<x< , где а =-|-arcsin-|-. 281
Упражнения Решите уравнения с параметром а. 1744. cos 2х—cos 4х=а sin х. 1745. 12 sin х+4 УЗ cos (л + х) = а УЗ. 1746. sin(jc—a)=sin x+sin а. 1747. sin (a-f-jc)-fsin x=cos -^-. 1748. (a-1) cos x+(a+ 1) sin x=2a. 1749. sin (x-f-a)-f-cos (x-f-a)=sin (x—a)+cos(x—a). 1750. 1 + sin2 ax=cosx. 1751. sin6 x+cose x=a. 1752. sin4x+cos4x-hsin2x+a=0. 1753. tg x + tg a+l^tgx tg a. 1754. sin 3x=a sin x. 1755. cos 3x=a cos x. 1756. 2 cos (a- cosx 1757. sin(x+a)=-^?5iL. 1758. cos x-sin a + 2cos 3xsin (a—3x)=0. sin x 1759. a2-2aH g !, , 4=0. 1760. sin x-+-2 cos ax=3. cos2 я (a-fx) 1761. sin2 x+4 sin x-f a=0. 1762. cos2 x — 3 cos x+a=0. 0. 1764. acosx+l a sin x4-1 1765. "-3"" „»-^* 1766. tgJ*-2tg a cos x—1 а sin x—1 1767. sinxtgx + 2cosx=a. 1768. sin a tg2 x —2cos a tg x+ 1=0. 1769» arctg a—arctg = arctg x. 1770. arctg — arctg ——=-arctg 1771. sin Зх-f-sin 2x=a sin x. 1772. (sin x-f cos x) sin 2x=a (sin3 x+cos3 x). 1773. sin2 x —sin x cos x —2 cos2 x=a. Решите системы уравнений с параметром a. 1774. ( sin x+sin у=a, 1775. ( cos x—cos y=at 1776. ( sin x sin y=a, я J , 2 1 ^+У=л. 1777. f sin x cos i/=af 1778. [ sin2 x — sin2 (/=a, 1779. f sin x sin y=*a% 3 { cosx cos y=3a. 1780. Г sin x cos y=2at 1781. f sin x cos 2^ = a2+ I, { cosx sin y=a. { cos xsin 2y=a. 1782. [jf-y=o, 1783. Г sin x+sin 1/= a, i 2 (cos 2x+cos 2(/)= 1+4 cos2 (x— y). | sin x sin i/= —2a2 Решите неравенства с параметром a. 1784. |±i!!l£+I^i!lL£<a. ,785. 1 cos x 1fcosx +^<a. ,785. |+| — cos x 1-f-cosx 1-f-cosx 1—cosx 1786. cosx <a. cosx
Часть III ДОПОЛНИТЕЛЬНЫЕ ЗАДАЧИ В этом разделе рассматриваются уравнения, системы уравнений, неравенства, которые либо не могут быть отнесены к рассмотренным выше типам (показательные, логарифмические, иррациональные, тригонометрические) — это так называемые комбинированные уравнения и неравенства (им посвящен § 1, хотя следует отметить, что некоторые сравнительно несложные комбинированные уравнения и неравенства встречались в предыдущих параграфах), либо решаются не элементарными приемами, а с использованием различных свойств функций — монотонность, выпуклость, ограниченность (этому посвящен § 2), либо содержат избыточное число переменных (нестандартные уравнения — § 3). § 1. КОМБИНИРОВАННЫЕ УРАВНЕНИЯ, СИСТЕМЫ УРАВНЕНИЯ, НЕРАВЕНСТВА Пример 1. Рещим уравнение Решение. Так как 16°S'^ =16 * то заданное уравнение можно переписать в виде На первом этапе рассмотрим это уравнение как показательное. Введем новую переменную и = 45|пх. Тогда уравнение примет вид ■^Ч-и2=^-, и Далее 2а2 — Зы+1=0, откуда wi = l, "2=-j-. Значит, либо 4s!nx=l, откуда sinx=0, либо 4sln*=—, откуда sinx=—±-. Теперь задача свелась к решению совокупности двух тригонометрических уравнений: 283
sin х=0; sin x= — —. Из первого уравнения находим x=nky из второго x=(-iy+l±+*n. Ответ: х=л*; *=(- 1)п+|-2-+лл. Пример 2. Решим уравнение lg cos x+logo.i sin 2x = lg 7. Решение. Сначала рассматриваем это уравнение как логарифмическое. Так как logo.i sin 2x=log(0 ,r, (sin 2х)~х = — log sin 2xf то заданное уравнение можно переписать в виде Igcos х—lg sin 2jc=lg7, и далее lgcos jc = lg(7 sin 2jc), cos x=7 sin 2x. Так как по смыслу заданного уравнение cosx#0, то получим: откуда jc==( — 1)* arcsin j^ Проверка. Ее в данном случае можно сделать с помощью области определения исходного уравнения: rcosxX), или [ sin2x>0, {cos x>0, откуда г cos x>0, Это значит, что корни уравне- 2 sin х cos x>0, \ sin x>0. ния должны принадлежать первой четверти числовой окружности. Таким образом, из двух точек, служащих на окружности образами решений уравнения sin х=тг> надо взять лишь ту, которая находится в первой четверти. Ответ: х=arcsin jr-+2nk. Пример 3. Решим уравнение 1 — sin 2х . . ^ I— cos 2х +l°g Решение. Выполним некоторые преобразования: (sin х—cos*)2 . ш^^ 2 sin2х Г + 1о& ^., tgx=3, 284
Положив w = loglgjr(tg x—1), получим: 2ii + -I—3. 2u2-3u+l=0, откуда mi = 1, «2=y-. Значит, либо logtgx(tgх—1)=1, либо log^^tg x--l)=-j-. Отсюда получаем совокупность уравнений: tg х— 1 =tg х; tg х— 1 =Ytg x, первое из которых не имеет решений. Во втором положим f=ytg х. Получим /2—1 = /, откуда /| = "*!"* , /2 = ~ ^ . По смыслу сделанной замены 1>0. Этому неравенству удовлетворяет лишь первый из полученных корней. Итак, ytg х = *"*1*5 , откуда tg jc Проверка. Как и в предыдущем примере, ее можно выполнить с помощью области определения исходного уравнения: tg*>0, tgjt-l>0, sin2x У нас tg x=3"^V5 t Ясно, что первые три условия системы выполняются. Ясно также, что при этих значениях х выполняются неравенства 1— sin2jc>0, l+cos2x>0, I—cos2jc>0. Осталось проверить выполнимость неравенства sin 2х>0. Имеем sin 2x== x_Jfx2 - Отсюда ясно, что из tgx>0 следует sin 2x>0. Итак, все значения х, при которых tg х=3"^5 , принадлежат области определения заданного уравнения. Поскольку при его решении, кроме расширения области определения, не было преобразований, которые могли привести к появлению посторонних корней, то х = arctg 3"*""V5 + nk — решения заданного уравнения. 285
Пример 4. Решим уравнение {tgxYmx=(ctgx)C0SX. Решение. Сначала рассмотрим это уравнение как показательно-степенное (см. с. 119). Имеем: (tg*f*=(tg *)-"**. (1) Теперь надо рассмотреть это уравнение в каждом из следующих случаев: tg*<0, tff -_.. tajc = O tlIg*~~ I ъ*-" Если tg x<0, но tg хФ — 1, то из уравнения (1), приравняв показатели, получим sin х= —cos х, откуда tg х= — 1. Это уравнение несовместимо с условием tg хФ — 1. Если tg x= — 1, то |sin x\ = |cos x\ == V* Это значит, что в уравнении (I) отрицательное число возводится в иррациональные степени, что не имеет смысла. Если tg х=0, т. е. х=ля, то и sin x=0. Значит, левая часть уравнения (1) принимает вид 0°, что не имеет смысла. Если tgx>0, но tg лг=й= 1, то из уравнения (1), приравняв показатели, получим sin х= — cos х, т. е. tg х= — 1, что противоречит условию tg х>0. Наконец, если tgx=l, то уравнение (1) примет вид: дим х=-2- Итак, уравнение (1) сводится к уравнению tg x= 1, откуда нахо- I Х= . | Jlk. 4 Пример 5. Решим уравнение I yilx—х1 —10 I 1 (2) sinT ~ f I yilx—Xs —10 i4—б+——\ =!• Решение. Пусть / = sin-|- и и= \ 10/ —6+—| . Тогда нам нужно рассмотреть уравнение <2) в каждом из следующих случаев: Пусть и=0, т. е. 1Ш—б+у=0 или 10/2—6/ +1=0. Это уравнение не имеет действительных корней. Пустьм=1,т. е. 10/—6+-р=1 или 10/—6 + -|-= — 1. Первое из этих уравнений преобразуется в уравнение 10/2 — 7/+1=0, а вто-
рое —в 10/2 —5*+1=0. Первое уравнение имеет корни *i=-j-. /2=—, второе не имеет действительных корней. о Мы пришли к совокупности тригонометрических уравнений: sin—=—; sin— =—. Из нее находим х=( — 1)* -£—Ь2яЛ; jc=(— 1 )п 2 arcsin 4ч~2ял. При о о V1 этих значениях уравнение (1) принимает вид iVii*-*1-»»--^ Это — верное равенство при условии Их—х2 —10 >0, т. е. 1<х<10. Значит, из найденных решений совокупности тригонометрических уравнений надо отобрать те, которые принадлежат отрезку [1; 10]. Рассмотрим сначала серию х=(— l)*-f-+2n£. Если Л=0, то х=-2-6[1; 10} Если k=\, то *=^€[1; Щ Если *=2, то дс—^ «5 о «5 ({[1; 10]. Аналогично не принадлежат отрезку [1; 10] те значения х% которые получаются при других значениях k. Рассмотрим серию *=( — 1)" 2 arcsin 4-+2лл. Для облегчения о последующих рассуждений вычислим sin (2 arcsin —) • Имеем sin (2 arcsin —\=2 sin (arcsin y- j cos (arcsin -|-) = Значит, 2 arcsin 4-== arcsin ^^-. Ho 4OY < о ■ Поэтому 2э arcsin -тг~<-т"< 1. Теперь, зная, что 2 arcsin -r-< 1, отберем корни из серии jc=(— 1 )л 2 arcsin 4~+2яп. Если /t=0, то х=2 arcsin 4-£[1; 10]- Если ^=1» то jc = —2arcsin~-)e[l; 10]. Если /г=2, то (2 arcsin-|- £[1; 10]. При других значениях параметра п получающиеся значения х не принадлежат отрезку [1; 10]. Итак, в случае, когда и=1, мы получили следующие корни: хх =--р *2=^, х3 = 2л-2 arcsin-^-. Осталось рассмотреть случай, когда ы>0, но иФ\. В этом случае уравнение (2) равносильно уравнению УН*—х2—10=0, откуда находим х4=\ и jc5=10 — еще два корня уравнения (2). 287
Таким образом, корнями уравнения (2) являются следующие значения х: Jti=-j-; *2=^S хз = 2л—2 arcsin-^-; *4=1; *5=Ю. Пример 6. Решим неравенство 4 logie cos 2x4-2 log4 sin *+Iog2 cos x+3<0. (3) Решение. Выполняя преобразования, получаем последовательно: 4 logie cos 2х-\-4 logie sin jc + 4 logi6 cos Jt+3<0, 4 (logie cos 2x+ logie sin x+ logie cos x)< — 3, logic (cos 2x sin x cos x)< logie -p Значит, неравенство (З), рассматриваемое на первом этапе решения как логарифмическое, равносильно такой системе неравенств: cos2*>0, sin jc>0, cos x>0, cos 2x sin x cos x<—. о Из второго и третьего неравенств этой системы получаем: Из неравенства cos 2* > 0 получаем — -|— откуда —т—ЬяЛ<Сх<-т—Ья&. 4 4 Последнее неравенство системы преобразуется к виду sin 4х<-|-, откуда находим — ^-+2яЛ<4х<-^-+2лй, т. е. — т£-+-§-* Решая далее систему например, с помощью окружности 2 (см. с. 264), получим 2nk<x<-2f+2nk; |j-+2я*<дс<-2-+ 2nk — решения неравенства (3). 288
Пример 7. Решим неравенство Решение. Рассмотрим это неравенство как показательное (о \и / о \0 о — ) >("т"/ • ^ак как основание 0<у<1, то получим и<0, т. е. Vlog^ ctg x< 1. Полученное иррациональное неравенство равносильно системе логарифмических неравенств г log^ ctg х^О, 1 l°gV3 ctg x< 1, из которой получим г ctgx>l, \ctgx<V3, откуда находим лЛ + -|-<х <—-+«* — решения заданного неравенства. Пример 8. Решим систему уравнений r—sin (1 — y + *2)cos 2*=cos (x2 — y+\) sin xcos xt Г 2 ' Решение. Перепишем первое уравнение системы (4) в следующем виде: sin (jc2~i/4-1)cos 2х—cos (х2 —y+\) sin 2х=0. В левой части этого уравнения мы получили синус разности аргументов jc2-— у+1 и 2х, т. е. sin (д:2 —£/+1—2лг) = О. Из второго уравнения системы (4) получим уравнение откуда у+2дс— 1 — х2 = 2х—х2, т. е. у=1. Значит, систему (4) мы свели к более простой системе sin (1 — Решая ее, получаем sin (л:2 — 2jc)=0, т. е. х2 — 2x = nkt откуда *=1±л/1—я£, где Л=0, — 1, —2, —3, ... . Из этих значений надо отбросить значение х=0, при котором основание логарифма во втором уравнении системы обращается в 1. Тогда оставшиеся значения х можно записать так: х=2 (при £=0), х==1 ±л/1 —лЛ, где Л= — 1, —2, —3, ... . :=-1, -2, -3 289
Пример 9. Решим смешанную систему f(V3+1) (1 +cos xy sin д«/)=(л/3+1) sin2 xy+cos 2xy, ЬУ-«/2+1=0, \ (5) \ Решение. Первое уравнение системы после преобразования сводится к однородному уравнению: (V3+1) (sin2 xy+cos2 xy + cos xy sin xy) = (-\/3+1) sin2 xy + cos2 xy — sin2 xy, J / (\/) y sin2 xy+(-yJ$+ 1) sin xy cos V cos2 xy=0, -л/з, кп. (6) Подставив эти значения ху во второе уравнение системы (5), получим: J/2=(—г*+л*) +1; #2==(—Т"+ЯЛ) + !• (7) Но из неравенства системы (5) следует, что t/2<6. Подставляя в равенства (7) значения параметров *=0, ±1. ±2, ..., п=0, ±1, ±2, ..., убеждаемся, что условие у2<6 выполняется лишь при fc = 0, п=0. Из равенств (7) находим (при fc=0, л=0): у2- = i6 А из уравнений (6) находим соответственно: xy=-f;xy=-f. В итоге система (5) сведена к более простой совокупности двух систем: Найдя для каждой из систем решения, определяемые первыми двумя уравнениями, и проверив результат по условию - получим следующие решения системы (5): 290
Пример 10. Решим уравнение с параметром а: lg2 cos x+2 lg cos х —(a2 + a — 3)=0. Решение. Положив w = lg cos х, придем к квадратному уравнению w2+2w—(а2+а—3)=0, откуда находим щ,2= — 1 ±Уа2+а —2. Если D = a2 + a — 2<0, т. е. —2<а<1, то решений нет. Если £)=0, т. е. а= — 2 или а=1, то а= — 1, т. е. lgcos;t= — I, a =^j-, откуда х= ±arccos ^- Осталось рассмотреть случай, когда D>0, т. е. а<—2; а>1. В этом случае заданное уравнение равносильно совокупности двух уравнений: lg cos x= — 1 — Va24-fl — 2; log cos x= — 1 Из первого уравнения находим cos x= ю~"1""^в1+в""2 и, поскольку 0<10"l~Vfll+e 2<1, находим, что х= dzarccos (Ю-1-^+о-2)+2ям. Второе уравнение совокупности отличается от первого тем, что правая его часть может быть положительной при некоторых а. Поскольку lgcosjc^O, то при таких значениях а уравнение не будет иметь решений. Найдем эти значения а. Для этого надо решить неравенство — 1 +л/а'2 + а—2>0. Имеем: илиГ а< -2; а> 1, или ( а< -2; а> 1, -1+уГз откуда а<—' v^; a> При этих значениях а уравнение lg cos х = — 1 +Vaz-f a—2 не имеет корней. Если же -— 14"*13^а< —2; 1<а<^ЬЁ то 2 2 уравнение имеет решение: x=±arccos 10~l +V«2+fl-2 ц_ 2ддх. Подводя итоги, получаем следующий ответ: 1) *=±arccos 2) если а<^Ь^Ш; e>=i±afilt TO *=±arccos lO-l 3) если — 2<а<1, то уравнение не имеет решений. 291
Пример 11. При каких значениях параметра а имеет решение система уравнений U-Vsln^-ft-1?. Решение. Рассмотрим первое уравнение системы. Положим Тогда это уравнение примет вид: |12/-5| — |12/-7| + |24/+131 = 11—и, (9) где (0</<1, \0<<1 Заметим, что I24/ +131 =24/+13. Разобьем числовую прямую на три промежутка точками — , — (при первом значении 12/— 5 = 0, а при втором 12/ —7 = 0) и рассмотрим уравнение (9) в каждом из следующих трех случаев: /<^- ; ^-<^<]у ; <>]у • Если ^7о"' то Уравнение (9) принимает вид: 7о (5-12/)+(12/-7)+(24/ +13)= 11 - и, и далее Поскольку / ^ 0 и и ^ 0, равенство 24/ + и = 0 выполняется лишь в случае, когда / = 0 и м = 0 одновременно, причем / = 0 удовлетворяет j. ^ 5 условию t^TcT • Если w<t<W 9 то УРавнение (9) принимает вид: (12/ —5)+(12/ —7)+(24/+13)= 11 - и, и далее Ho^-</<~-, 0<и<1. Тогда 20<48/ + ы<29, т. е. 48/ + w не может быть равным 10. Значит, уравнение (9) не имеет решений при ]Т<'<7тг • Если, наконец, />^- , то уравнение (9) принимает вид: 292
(12/ —5)—(12^ —7)+(24/+13)= 11—w, и далее = — 4. Это уравнение также не имеет решений при *>]£- и и>0. Итак, уравнение (9) имеет только одно решение: А это значит, что первое уравнение системы (8) равносильно системе уравнений Решая эту систему, последовательно находим: Теперь обратимся ко второму уравнению системы (8). Положив = x2+(y — а)2, перепишем это уравнение в виде 22- 1 = 2^2 —J-. Имеем (22—1)2 = 4^2——J , откуда 2=1, т. е. а)2=1. (11) Речь идет об отыскании целочисленных решений уравнения (11), задаваемых условиями (10). Из уравнения (11) следует, что *2<1, т. е. х может принять только три значения: 0, 1, —1. Пусть л:=0, т. е. 3 + 4я + 3& = 0. Это равенство возможно, лишь если л:3, т. е. я = 3т. Тогда уравнение (11) принимает вид 02 + + (1 +6m—af= 1, откуда 1 +6т—а= ± 1, т. е. ах =6mt a2=6m+2. Пусть х=1, т. е. 3 + 4/i + 3fc = l, или 4м + 3* + 2 = 0. Переписав последнее равенство в виде (Зл + 3£ + 3)+(я—1) = 0, замечаем, что оно возможно при целочисленных п и k лишь в случае, когда (/г —1):3, т. е. n = 3m+l. Тогда уравнение (11) принимает вид 1+(6т + 3 —а)2=1, откуда а=бт + 3. Пусть, наконец, х= — 1, т. е. 3 + 4п + ЗЛ= —1, или 4я + 3£ + + 4=0. Переписав последнее равенство в виде (Зл+3* + 3) + + (л+1)=0, замечаем, что оно возможно при целочисленных п и k лишь в случае, когда (п +1): 3, т. е. п = 3т — 1. Тогда уравнение (11) принимает вид 1+(бт —1—а)2=1, откуда а = 6т— 1. Итак, система (8) имеет решения при следующих значениях параметра а: а=6т—1, а=6т, а=6т + 2, а=6т + 3, где m£Z. Пример 12. При каких значениях параметра а имеет решения система уравнений 293
2 cos x+a sin y= 1, г Sin у = loge п • loga (2 — 3 COS (12) Решение. Начнем решение системы (12) с наиболее сложного уравнения системы — со второго. Имеем последовательно: loga Sin у lOga (2 — 3 COS x) lOgaZ ~~ l0geZ lOga Sin у = lOga (2 — 3 COS x\ sin y = 2 —3 cos x. Рассмотрев это уравнение совместно с первым уравнением системы (12), получим siny= . Из последнего урав- -—— , cos х=-^:1 Z — «За 2. — нения системы (12) находим z = ^ . Выясним теперь, при каких а можно найти х, у, г, определяемые указанными уравнениями. Так как sin y= , а по смыслу системы (12) sint/>0, должны выполняться неравенства — >0 и = cosXj а по смыслу системы (12) . Так как -т-, должны выполняться неравенства 1—2fl <c—- 1"""2fl > — 1 3 2-3a ^ 3 ' 2-3a ^ Наконец, по смыслу системы (12) должны выполняться условия: а>0, аФ\, ^ 1 >0, т. е. 0<а<-^-. Кроме того, должно выполняться условие 2=5^1, т. е. х^2а Ф1, откуда аФ-^-. Итак, система (12) имеет решение при значениях параметра а, удовлетворяющих следующей системе неравенств: Решив эту систему, находим 0<а<^-; 4"<а^4"« При этих 4 4 *j значениях параметра а система (12) имеет решения. 294
Упражнения Решите уравнения. i 1787. 8isin2x+81cos2x=30. 1788. 4lgf ЧЯ^^-вО-О. 1789. 2cos2x=3.2cos2x-4. I 1790. 5ctg2x(26-5iIi?T) = 5. 1791. 3cos2*(4.3sln2*-9)=l. 1792. (0,5)cos2x-4-sin2*=0,5. ! 1793. 1794. 1795. 1796. sin sin ctg (3X-1 sir 2X~2 2X = h2cos -f3x-")cos(3x-1 1 2X _o /q COS 2X~2 "^ tg2x + 2 tg 2X+I. . 25jc 5*+16SW 2 =9. 1797 3sin2 f(r*=3-4 V2cos* 1798. l+2Ig =3-4 4y2cosl ——x\ . i 1799.4-2 V4 7— 15,5-4СО8д:+8ШХ=- 1800. log . cosjc+log sinjc = 2. ъ sin x ' & cos x 1801. sin (л lg jc) + cos (л lg jc)= I. 1802. lg2 (sin jf+4) + 2 lg (sin x + 4)—j-=0. 1803. log. • (sinx—j-cosjc)=3. &sm x \ 4 / 1804. log 2 sinjc=4"- ь 8 cos2 x 2 1805. log9 sin 2jt = log3 1807. logs 1808. 3 logisin jc + log2(l—cos2jc)=2. 1809. log2 cos 2jc — Iog2 sin x — Iog2 cos jc= 1. 1810. lg sin 2jc—lg sin *=lg cos 2jc—lg cosx + 2 lg 2. 1811. lgsin—=lg(cos jc—sin jc) + lg(cos x+sin jc). 1812. log2 sin x—log2 cos jc—log2 (1 +tg x)== 1 + Iog2 (! —tg x). 1813. Iogstgx=logs4*log4(3sinx). 1814. , //4f.2 л =log sjn 2x 10. 295
1817. log2 |tgx|+log4inn^_=s0, если |<х 1818. 1 +log6 (4 cos2 x-cos x- l)=loge (4-7 cos x). 1819. log x (-T-3 cos 2*—sin 2x+sin x—cos x J 1820. logt (—!j--— sin2x+cos2x+V3cosx— 1821. logcosx(?^+cosjc-sin2x-.V2smjc) =2. 1822. log^ ^(sin 2jc—2 cos x+sin jc—cos2 x)—2. ((тт)()) +2togl (cos(x-f) +e«(*+f) -j« —COS X 1824. Iog2( cos 2jc+cos у J + iog0 5f sin x+cos у 1825. Iog5(cos|-+3tgx-b^) +iog JL(co 1826. log x (sin-|.+cos2x) +log3(sin -|~sin x j =0. 1827. log^(sin|-3tgx-3-^.) +loge(sin^-tg2x-.3^) =0. 1828. |log x (l+sin T T 1829. |log3 (1 + cos 2x)\ +1 log3 (1 - cos 2x) | ~о~~Т" »vea we ** -JT* -JT* -у* 1009 Sin X 1830. 32 +62=92 l l 1 V -r+logssinx —+logl5cosx 1831. 52+52 «152 1833. 4^ 1834. log ^ (27cw'*-38in'*)+2 cos' *=j^ +«<>g9 (1 +3-°'5+cos 1835. V*g x+sin x+ytg x—sin x=2 -yjtgx cos x. 1837. 2%
1838. Решите нергвенства. 1839. а) (л/х2— б) 1840. а) 4 5!п'ях+3-4СО8г>"<8; б) 91+sin'n430-9cosIlut<117. 1841. 0,2cos2*— ' - 4 25СО5 * 1842. |3*«"-3|-*«"|>2. 1843. log2cosx>log2tgx, если 1844. log 2sjnx(2 cos х)+2 log 2cosx(2 sin x)>3. 1845. (log sjn x 2)2 < log $jn x (4 sin3 x). 1846. (log ^V<iogtgS1"(3 tg2 x). 1847 loff -—->0 1848. log sinxtgx<2 log |gxsin x+\. I8S0- V5(sin,+c.sx)(V6 sin x+^cosx)> 1. 1852. log . (sin x—5cosx)>l. Ь SIP. X —CO5 JC X ' ,853. log, , i± 6clg'x 1— 1854. 1855. logt 1856. log |n 1857. log 1858. log 2-8x+ 23)> T' еСЛИ 297
Ig2 + lgcos(x+i) () 186°- lg(sinx+cosx) >-1' «*" -*<*<*. **±Т 1861. logt 2^(cosx—cos3x)—logt 2 (sin x + s|n 3x)> 0,45. 1862. log tgt u (sin.x+sin 3x)<0,55 + log tg8 2x (cos x+cos 34 y 1864. 2 1865. 10.0,3VlogV5ct**>3. -1 (Iogtgx(2-h4cos2jc)~2)>0. c-5 1869. 1870. V5—2sinx^6sinx—1. 1871. V2+4 cos x>y+3 cos x. 1873. cos (2--4x)+cos (2+4x)> V2 cos2 2x— 1 1874. Vl +2 cos x+ Vcos x> y-j—cos x . 1875. 1876. arctg -y[x>arccos (1 — x 1877. 1878. Решите системы уравнений. 1879. | 2 sin x+sin 2x=2 cos2 -y. 1880. { 1 +log*2j~J «log x_y 2 • log2 ( 1881. ( VI +sin x sin i/=cos x, \2sinxctgi/-f 1=0. 1882. Г V1 +sin x sin //=cos y, \2sinyctgx+V3==0.
3. | cos2 xy — 3 si 1*6 + 2ju/<5. Р 1883. | cos2 xy — 3 sin xy cos xy=2 cos t/ cos (2ju/ — y) — 2 cos2 (jo/ — * J3 Ы==0, Решите уравнения с параметром а. 1884. lg2sinx—2algsin*-a2 + 2=0. 1885. log . 2-log . f a+l=0. bsin x ь sin2 jc ' 1886. |cos*|ctg2x+actgx=l. 1887. /inx-a>l (a>0; 0<x<-j) . 1888. Для любого а>0 найдите решения неравенства *51П*~"а ^it принадлежащие интервалу (О;-^-). Решите системы уравнений с параметром а. 1889. (2^/х — 2 arccost/-f z=l, < 5 V^-f-arccos y+z=6a—14, I V^+arccos y-\-2z = 2a+1. 1890. fV — (/ — arcsinz=— 6, 2x+2t/~ 3arcsinz=7, 2х —t/H-arcsin z=6a+2. 1891. При каких значениях параметра а система уравнений 3+1 - имеет хотя бы одно решение? 1892. При каких значениях параметра а система уравнений r(a—2) sin x+cos у=\, loga (2 COS l/) = loga Z-\o& (1 +7 Sin *), имеет хотя бы одно решение? § 2. ИСПОЛЬЗОВАНИЕ СВОЙСТВ ФУНКЦИЙ ПРИ РЕШЕНИИ УРАВНЕНИЙ И НЕРАВЕНСТВ Не всякое уравнение / (x) = g(x) в результате преобразований или с помощью удачной замены переменной может быть сведено к уравнению того или иного стандартного вида, для которого существует определенный алгоритм решения. В таких случаях иногда оказывается полезным использовать некоторые свойства функций /(*), g{x). Так, если одна из функций убывает, а другая возрастает на промежутке X, то уравнение f(x)=g(x) либо имеет один корень (см. рис. 59, а) и тогда можно найти его хотя бы подбором, либо не имеет корней (см. рис. 59,6). Например, для решения уравнения 299
-у/7—х =дг— 1 нет надобности возводить обе части уравнения в квадрат. Достаточно заметить, что х=3 — корень уравнения и других корней нет, поскольку левая часть уравнения — убывающая, а правая — возрастающая функция. Аналогично обстоит дело при решении неравенства У*+18<2 — х. Здесь при х=—-2 левая и правая части равны, но поскольку левая часть — возрастающая, а правая — убывающая функция, то неравенству удовлетворяют значения х, которые меньше —2. С учетом области определения получаем ответ: — 18<х< —2. Если, далее, функция / (х) на промежутке X ограничена сверху, причем Sup f (x)=A, а функция g (x) ограничена снизу, хек причем infg(x)=A, то уравнение f(x)=g(x) равносильно системе х£ X уравнений г / (х)=Л, \В(х)=А. Иногда для решения уравнения f(x)=g (x) полезно построить графики функций у=/ (х\ y=zg(x)n определить абсциссы точек их пересечения. Используются и другие неэлементарные приемы решения уравнений и неравенств, иногда с привлечением производных. Обо всем этом и идет речь в настоящем параграфе. Пример 1. Решим уравнение |6аг —5|=4 sin^- . «5 Решение. Построив графики функций у=|6х — 5| и у = = 4 sin у- (рис. 60), находим два корня уравнения: Х|=—, х2=-^-. Пример 2. Решим уравнение Решение. Подбором находим, что уравнение имеет корень х=2. Так как в области определения уравнения, т. е. на отрезке [1;3], функция 1/=Ул:—1+2 УЗх + 2 возрастает, а функция у = = 4+73—х убывает, то других корней уравнение не имеет. Итак, х=2 — единственный корень уравнения. Рис. 59 300
П р и м е р 3. Решим уравнение уш\бх-5\ Решение. Положим / = = log3*. Тогда х=3\ Ух=(-\/3)', и заданное уравнение можно переписать в виде откуда 1+(V3)'=2*. Это уравнение имеет очевидный корень / = 2, но утверждать, как в предыдущем примере, что это единственный корень уравнения, мы не можем, ибо как левая, так и правая часть уравнения — возрастающая функция. Но если обе части уравнения разделить почленно на (V3)', то получим: Рис. 60 V3 Теперь левая часть уравнения, т. е. показательная функция у= =(-р) +1»убывает (основание -р < 1), а правая часть уравнения, \уЗ/ ^ -уЗ ' т. е. показательная функция (/=(—) , возрастает (основание— > 1). Значит, t=2 — единственный корень уравнения. Поскольку f=log3 х, то из уравнения Iog3 х=2 находим лг=9 — единственный корень заданного уравнения. Пример 4. Решим уравнение ^tl (1) Решение. Выполнив преобразования, получим (—) + ^—^- . Замечаем, что это уравнение имеет корень дс=7. Дока- жем, что других корней нет. —) +8 убывает. Если окажется, что функция (х— IY У=^х_£ возрастает в области определения заданного уравнения, т. е. на луче [5,3; + оо), то можно будет сделать вывод о том, что jc=7 — единственный корень уравнения (1). (х— П2 Найдем производную функции Получим ц'- получим у — Если (*-1)(*75) (JC_3)2 (jc_3)2 Х то у'>0, т. е. функция У=^г^ возрастает на лу- 301
че [5,3; + оо), что и требовалось установить. Итак, лг=7 — единственный корень уравнения (1). Пример 5. Решим уравнение (2) Решение. Замечаем, что х\ = 1 — корень уравнения (2). Но, как и в примере 3, утверждать, что это единственный корень уравнения, мы пока не можем, поскольку и функция у=\]2х— 1, и функция t/=—а:2+— возрастают в области определения уравнения (2), т. е. на луче I у-; оо V Если в примере 3 нам удалось преобразовать уравнение к такому виду, что одна часть представляла собой убывающую, а другая возрастающую функцию, то здесь нам этого не удается. Поступим по-другому. Найдем производные функций у,=^/2х—1 и у2=—х2+— и вычислим их в точке х= 1 (в точке пересечения графиков этих функ- t/i(l)=~-. Далее, ций). Имеем у\=±-(2х— 1)" У2=-у\ J/2 0)=^г- Так как У» (0=1/2 (1), то графики функций у\ (x)f у2 (х) имеют общую касательную в точке (1; 1). Но поскольку функция ух {х) выпукла вниз, а функция у2 (х) выпукла вверх, то их графики расположены по разные стороны от общей касательной (рис. 61), а потому уравнение ух {х)=у2(х) имеет только один корень. Итак, х= 1 — единственный корень уравнения (2). Пример 6. Решим уравнение Решение. Положим / = 2\ Тогда правая часть уравнения (3) примет вид *+-р Воспользуемся известным неравенством /+-|->2, если />0 (см. сГ~27). В то же время справедливо неравенство 2 cos2 £t£ <2. Значит, уравнение о у=±х*+% i (3) сводится к системе уравнений ;Cos2^=2, о Рис.61 Из второго уравнения системы находим х=0. Поскольку это значение удовлетворяет и первому уравнению системы, то х=0 — решение системы, а тем самым и корень уравнения (3). 302
Пример 7. Решим уравнение =л/з—т-1 1 4 Решение. Имеем tg 2 ■ ? ■ •» =*К Так как 0<^+*)i+3 <^-, а на промежутке ((>;•§-] функция tg* возрастает, то tgO<tg(^+^+3 <tg-|-, т. е. (5) Значит, правая часть уравнения (4) должна быть положительной. Более того, поскольку sin(n+2pj <1, получим: л/3 , ' >л/3- (6) Сопоставляя неравенства (5) и (6), приходим к системе Первое уравнение системы обращается в верное равенство только при х= — 2. Поскольку это значение удовлетворяет и второму уравнению системы, то х=—2 — единственный корень уравнения (4). Пример 8. Решим уравнение sin*+2sin2jt=4 + sin 17*. (7) Решение. Так как sin x^l, sin 2x^1, то sin x+2 sin ^ Более того, sin x+2 sin 2x<3. В самом деле, рассмотрим уравнение sin х+2 sin 2*=3. Такое равенство может иметь место тогда и только тогда, когда sinx=l и sin2x=l, что невозможно, ибо sinx=l при *=-^-+2л/г, а при этих значениях имеем: sin 2jt=sin 2(-2-+2я*) =sin (я+4яй)=0. Итак, sin jc+2 sin 2jc<3. В то же время правая часть уравнения (7) удовлетворяет неравенству 4 +sin 17x^3. Таким образом, можно сделать вывод, что уравнение (7) не имеет решений. Пример 9. Решим неравенство 2 + log,x 6 /о\ jc-1 -^2х-Г 1; 303
Решение. Чтобы преобразовать неравенство к более простому виду, рассмотрим два случая: х> 1; 0<х< 1. В первом случае неравенство преобразуется к виду и далее log3x> 2л-4 2х-4 Во втором случае получаем log3 x< , . Итак, неравенство (7) равносильно совокупности систем неравенств i*>l, /Q4 fO<JC<l, 2х-4 * М 1 2*~4 (9) 2х— 1 \ 2дс— 1 Построим в одной системе координат графики функций y=log3 x и у= *~ (рис. 62). Замечаем, что при х> 1 график логарифмической функции расположен выше графика дробно-линейной функции, т. е. х> \ — решение системы (8). Если же 0<х< 1, то график логарифмической функции расположен ниже графика дробно-линейной Рис. 62 Рис.63 304
функции при 0<х<у-, т. е. 0<х<-—решение системы (9). Объединяя решения систем (8) и (9), получаем (o.yyUO; °°) — решение неравенства (7). Пример 10. Решим неравенство -4. (10) Решение. Так как область определения неравенства задается неравенством 8—х>0, т. е. х<8, то, прологарифмировав обе части неравенства (10) по основанию 2, получим равносильное ему неравенство l0g2 (8-jt)logl(8-x)<log2 23x-\ т. е. logi(8 — jc)<3jc — 4. Равенство logi (8—х)=Зх—-4 выполняется при х=4. Поскольку функция t/=logi (8—х) убывает, а функция у = 3х — 4 возрастает, делаем вывод, что неравенство logi(8—*)<3л:—4 выполняется при >4 (рис. 63). Итак, [4; 8) — решение неравенства (10). Пример 11. Решим смешанную систему + ) i/ + i/+, (И) \У\<\. Решение. Преобразуем второе уравнение системы к виду Аг . 4 4 + где 2 = sin2x, т. е. 0<z<l. Рассмотрим функцию и = 4г-\-—на отрезке [0; 1], найдем для нее «на.,6 и иняии. Имеем и'= 4* In 4 — 4.4~г In 4 = In 4^4г— ^-) = Значит, w' = 0, если 42г = 4, т. е. 2=—, а потому свои наибольшее и наименьшее значения непрерывная функция и = 4г+— на отрезке [0; 1] может принимать только на концах отрезка или в точке Имеем м(0) = 5, и(-^-) = 4, м(1)=5. Итак, м„аиб = 5, инаим=4. Значит, и правая часть уравнения (12) должна удовлетворять системе неравенств 305
{ 25t/24-6j/+l Решив эту систему и учтя, что система (11) содержит условие |у|< 1, получаем у= —■ 1; -|-<t/< 1. Рас системы (11) в каждом из этих случаев. Начнем со случая, когда -~<у о системы (11) можно переписать так: у у у, () р у 1, получаем у= —■ 1; -|-<t/< 1. Рассмотрим первое уравнение Начнем со случая, когда -~<у<1. Тогда первое уравнение о |sin*|. (13) Рассмотрим функцию v = . ^ и найдем ииаиб и t/Haiw на отрез- 1-t-oy ке [4-; И- Имеем v'=tt * ^ , т. е. tf'>0, а потому функция ^=Г+% возрастает на [-|-; l]. Значит, yHaHll=t;(-^-)=-|-, a 0наиб=0 (0="j"« Но тогда функция Iog2j^3" пРинимает значения от log2 4"A° I°g2 4-, т. е. от — 3 до —2. Оценим теперь границы левой и правой частей уравнения (13). Так как-|-<|/<1, — l<sin х<1, то — l<t/sin Так как —3<log2j^< —2, log2|sinx|<0, то + log2 |sinx|<—2. Итак, правая часть уравнения (13) не меньше чем — 1, а левая часть не больше чем —2, значит, уравнение (13) не имеет решений. Рассмотрим теперь случай, когда (/= — 1. В этом случае второе уравнение системы (11) принимает вид 4sin2*+4cos8jc=5, откуда после несложных преобразований получаем sin2jt=0, что нам не подходит, или sin2 jc= 1, т. е. *=±уЧ-2ял. Первое уравнение системы (11) при у=—\ принимает вид: (14) Так как \Щ*-\ <1, то log2 \Ц*-\ <0. Значит, - sin x< О, т. е. sin х>0» а потому из найденных выше значений х= ±~{-2лп 2 .-L-Огтп Они чгпльа п «vrartrbarwr vrvanuaui возьмем только х=~-+2ял. Они удовлетворяют уравнению (14).
Итак, J х=-|-+2ял, ly= — 1 — решения системы (11). Пример 12. При каких значениях параметра а из интервала (2; 5) неравенство Iog2 13— |sin ax\ | =cosf пх——) (15) имеет решения на отрезке [2; 3]? Решение. Ясно, что Iog2 13— I sin ах\\ ^Iog2 2=1, а cos (лх— —§-)^1- Значит, уравнение (15) равносильно системе уравнений f log2 13— |sin ax| | = 1, (\sinax\ = lf (16) |3-|sinax|| = l, или flsi nx-f)=l> l*= Из чисел вида -g-+2fe отрезку [2; 3] принадлежит только т. е. *=-r-- При этом значении первое уравнение системы (16) I . 13a I i принимает вид | sin — | = 1, откуда 1 Зд я I Зл -f- олм 6 2 ' 13 Осталось среди этих значений отобрать те, которые принадлежат интервалу (2; 5). Таковыми будут ai=-^- (при п=\) и a2=-j~ (при ) Итак, условиям задачи удовлетворяют значения параметра а: 9л_. 15л 13 * 13 ' Упражнения Решите уравнения. 1893. (-| у+-~- = 2'. 1894. 3* + 4* = 5*. 1895. \oglX = x — 4. 1896. Зж = 4-х. "з 1897. 5х-f 2Ж+1.3Х=9Х+4Х. 1898. *.2*=х(3-х)+2(2*-1). 1899. log! jc+(jc— 1) log2 jc=6 —2лг. 1900. xx=\0x-x' 1901. 8 — *. 1902. l2 1903. 1904.
1905. 1906. 2 sin x=5^+2x4-3. 1907. 2cosnjc=2jc— 1. 1908. cosnx=*2 — 4x+5. 1909. 3 arcsin x+nx — n=0. 1910. 3arccosjc—njc—?f=0 1911. 3UI = cos4- «5 1912. 1914. — 2V3^sinjc=U+n| + U—2я|. 1916. у5=3-у*"»+у. +1 1918. 2sinJ-jSin2-g-=^ 1919. 3+(*-n)2=!-2cos*. 1920. 2cos4=5*+5~*. ,924.2^2 1. JT 1-Х 1925. V^=b^+V2~Jf= 1926. 3U"l+2=5+4sin2ji*. 1927. 5|1-4x '== sin я*. 1928. 21" "~ 1929. 21' 1930. 2sin(x+-^ ) = tgjc+ctgjc. 1931. V2+cosz2jc = sin 3jc—cos 3jc. 1932. cos2 (^"(sin jc+^cos2 x))~tg2 (*+% tg2 x)= 1. 1933. log3(8+2jc-jc2)=2Jt-|+21-x. 1934. log2(3+2jc-*2)=tg2^ + ctg2~. 1935. log, (3+lsin x|)=2UI —2. 1936. Iog23(3-|cosjc|)=2""|jt~x|. 308
1937. loga^y- |~- 1938. log3 (4- I cos у 1939. sin -7—5 = >°g3Ul-HogM3 *2 + 6*+13 2-v^ 1940. cos 16л l "tjpjii+xtjpjiJt' 1941. sin x — sin 15* cos jc=~. 1942. Vsin3 x + Vcos3 x=-y/2. 1943. Ig sin JC.-f-lg sin 5x=logo.i cos 4jc. 1944. arcsin(x2 —2x + 2)=y . 1945. arccos(6jt-*2-10)=— ~. «5 1946. 3 arcsin tg'y+ctg2 — 1947. cos4 (arcctg x)+sin4 (arcctg jc)=sin^2 (arcctg x). Решите неравенства. 1948. 2*^11-х. 1949. £*+»_7 ю x— 1 3 —2jc ' 1952. 2""U""21 Iog2(4jc—x2 — 1953. cos2(jc+l)lg(9 —2jc-j 1954. (4x-*2-3)log2(cos2 1955. lV2UI-H-log2(2 1956. cos"2 (jc+3 tg x)+(tg jc-tg2 jc)2< 1. 1957. cos (я (jt+y sin juA)+(sin! 1958. sin [^ I 1959. Решите смешанную систему a/'+%)(32+25in4*+32cos<j 1960. Решите систему неравенств 1961. При каких значениях параметра а из интервала (2; 7) уравнение Iog2(l + -f sin2f ^-f--5 X\ =-1 cos ax\ — 1 имеет решения на отрезке [1; 2]? 309
1962. При каких значениях параметра а из интервала (5; 16) уравнение 1 + (^^) (i)|cos-sin]"1 имеет решения на отрезке [1; 2f 1963. При каких значениях параметра а из интервала (1; 5) уравнение cos2f nx + )()1+Ы1 имеет решения на отрезке [2; 3]? f 3. НЕСТАНДАРТНЫЕ УРАВНЕНИЯ И НЕРАВЕНСТВА Термин «нестандартная задача» имеет в методике математики много толкований. В настоящем параграфе к нестандартным мы относим уравнения и неравенства с двумя, тремя переменными, а также системы уравнений, в которых число уравнений меньше числа переменных. Разумеется, не всякое уравнение с двумя переменными можно отнести к нестандартным, например уравнение jc-)-(/ = 5, имеющее своими решениями любые пары чисел, которые в сумме дают 5. Это уравнение настолько же просто, насколько неопределенно (бесконечное множество решений). Мы будем нестандартным считать такое уравнение с двумя, тремя переменными, которое после более или менее оригинальных рассуждений приводит к вполне определенным решениям. Пример 1. Решим уравнение Решение. Имеем Значит, заданное уравнение можно переписать в виде (х — 3)2 + + (л/У"~2)2=0. Но сумма двух неотрицательных чисел равна нулю тогда и только тогда, когда каждое из них равно нулю. Значит, х=3, а л/У=2, т. е. у=4. Ответ: (3; 4). Пример 2. Решим уравнение sin4 x+cos4 у+2=4 sin x cos у. Решение. Последовательно имеем: (sin4 х—2 sin2 x cos2 у+cos4 y)+2 sin2 xcos2 y+2 — 4 sin xcos t/=0, (sin2 x-cos2 yf+2 (sin x cos y-1)2=0. Рассуждая, как в предыдущем примере, приходим к системе тригонометрических уравнений {sin2 Jt=cos2 у, sin xcosf/=l. Положив u = sinx, v=cost/, получим систему \uv=\, 310
откуда находим / щ = 1, t и2 = — 1, ,И2=—1, \ £/2= —1, т. е. j sin х=\ \ cos у = 1; """ \ cos х= — 1. Из первой системы получим Гх= ~- второй Г х=—|- из Это решения заданного уравнения. Пример 3. Решим уравнение cos (* — (/)—2 sin х+2 sin t/ = 3. Решение. Выполним некоторые преобразования: — 2 (sin jc — sin y)=(l —cos (x—f/))+2, -4 sin i=*- cos £^-=2 sin2 £Y Положив f = sin x~y , получим уравнение которое рассмотрим как квадратное относительно /. Имеем: == -cos Отсюда следует, что cos2^—^—1>0. Так как, с другой стороны, cos2 х*у < 1, то делаем вывод, что cos2^i£-=l, и тогда В итоге, как в предыдущем примере, приходим к системе тригонометрических уравнений (Г'Л!-!',». И'" 2 C0S Эта система равносильна совокупности двух систем: I8'" 2 ~ Из первой системы получаем: 311
откуда _«+2я(*+л), Из второй получаем: ± Пример 4. Решим уравнение 3 Решение. Заметив, что -4—= 1 +tg2 xy, а у*—2у+Щ-= (*/—1)2+"^-> перепишем уравнение (1) в виде (2) Нетрудно показать, что 7|" ^ ХУ^2. Для этого достаточно пере- писать это неравенство в виде ! . +ltgx|>2 и воспользоваться известным неравенством /+у->2, если />0 (см. с. 27). В то же время logj^ | -^+(у—1)2)<2. В самом деле, уЧ"(у— 02>у-. а (тогда в силу убывания функции log /) log -^-=2 3 3 1 * Итак, левая часть уравнения (2) не меньше чем 2, а правая не больше чем 2, значит, каждая из них равна 2, т. е. мы приходим к системе уравнений {l+tg2**/ |tgj?f/l log i (- Из второго (более простого) уравнения системы получаем у=\. Тогда первое уравнение системы принимает вид |tgx| = l, откуда Пример 5. Решим уравнение 312
Решение. Выполним некоторые преобразования: -L+cos2 l^L-sin2 ^= +2 sin2 £=IL cos2 £=*.. (4) Положим f = cos2 x~y . Тогда sin2 *"ТУ = 1—/ и уравнение (4) принимает вид: ■%-+t—(l-t)=-yl3+2x-x*-t+2(l-t).t, и далее У3+2х—*2=2*+^-, где 0</<1. Так как уз+2*—JC2== =V4—(1—х)2<2, а 2/+—>2, то получаем систему откуда находим: Решив эту систему, получим следующие решения заданного уравнения: Пример 6. Решим уравнение (sin2 л:Ч—Л—У +(cos2H т-У = 12+4- sin У- \ ■ sin2 х / ' \ ' cos2 х / '2 и Решение. Имеем последовательно: sin4 х+г+^+соз4 ^ + 2+^= 12+-i-sin у, sin4 x + cos4 x+sin4/+cofx=8+-L sin у, 1 ' sin4* cos4 jc ' 2 ^ (Sin2 Ж + COS2 xf-2 Sin2 X COS2 x+(sin'*+cosW-2sin**cos'*= (5) 4 ' 7 ' sin4 x cos4 jc sin4 x cos4 jc Положив f = sin2*cos2 x, перепишем уравнение (5) в виде т. е. -i— j— 2/ = 7+-L sin «/. (6) 313
Рассмотрим функцию £=4 2—2t. Здесь / = sin2 xcos2 x, т. е. f=—sin22x. Значит, 0</^—. Найдем наименьшее значение 4 4 функции z=p—-|—2t на промежутке A); -j-J. Имеем г'=—jr+-p—2=~2**f~~2t . Ясно, что на рассматриваемом отрезке — 2+2/ — 2/3<0, т. е. z'<0, а потому функция z(t) убывает на (0; -^-1. Значит, zuaHH=z( —j=7,5, т. е. левая часть уравнения (6) удовлетворяет неравенству р—-|—2f>7,5. В то же время правая часть уравнения (6) удовлетворяет неравенству 7+^-sin у<7,5. Значит, каждая из частей уравнения (6) равна 7,5, т. е. мы приходим к системе уравнений откуда I *=—, или L7+i-sint/ = 7,5, {sin2 xcos2x=—, у=-£-+2лп. Из уравнения sin2 x cos2 x=— получаем: sin22jt:=l, 2x=f+nk, В итоге находим следующие решения заданного уравнения: Пример 7. Решим систему уравнений г 8 cos х cos у cos (jc—y)+1 =0, ,7v \ х+е/ = г. Решение. Выполним некоторые преобразования первого уравнения системы: 4 (cos (x+y)+cos (ж—у)) cos (х—у)+1 =0, (4 cos2 (x—y)+4 cos (х—у) cos (x+#))+(cos2 (x+y)+sin2 (x+y))=0, (2 cos(x—( 314
Это уравнение равносильно системе уравнений ( 2 cos (х —y)+cos (x+t/)=0, \sin(x+t/)=0. (8) Уравнение sin(x + (/)=0 сводится к совокупности уравнений: х+у = 2пп или лг+у = л + 2ля. В первом случае cos (х+у)=1, во втором cos (х+у)= — 1. Значит, система (8) равносильна совокупности двух систем: !-; (х + у = Из первой системы находим т. е. Из второй системы находим {х+у= х—у= 2£ : 3 откуда (10) или {*в^-+я(л + *), (11) Пары, задаваемые условиями (9), (10), (11)—решения системы (8). Воспользовавшись теперь тем, что z=x+f/, получаем решения системы (7): = :Р-7Г+я(/1 — k\ = я + 2яп; Пример 8. Решим неравенство -Щ-+я(п-к), г=л+2яп. Решение. Преобразуем неравенство к виду (12) (13) 315
и возведем обе его части в квадрат, учтя при этом, что х2+у2 — I >0 (область определения) и х— \у\ —1^0 (по смыслу неравенства (13)). Получим систему неравенств, равносильную неравенству (13): (х2+у2-1^0, (14) Из третьего неравенства последовательно получаем: х2+у2 +1 -2х\у\ -2х+2\у\ >х2+у2-\, \\ \\ Отсюда, в частности, следует, что jcj^I. Поскольку из второго неравенства системы (14) следует, что *>1, нам остается лишь сделать очевидный вывод: х=1. Итак, системе (14) может удовлетворять лишь значение х=1. При этом значении система (14) принимает вид: с -\у\>о, [у2>у\ что выполняется одновременно лишь при у=0. Итак, (1; 0) — единственное решение неравенства (12). Пример 9. Решим неравенство 1 _tg ^-+ arccos (x+ Isin у|)<0. (15) Решение. Так как функция и = arccos t определена лишь для /6[-1; 1], то -l<x+lsiny|<l,T. e. -1 - Isin у\ <*< 1 - Isin y\. (16) Так как 0<|siny|<l, то из двойного неравенства (16) следует, что -2<*<1. (17) На промежутке ( — 2; 1] функция y = tg-y возрастает, значит» t;Han6=^(0:=tg-f-==1> Т* е> На ЭТ°М 0ТРеЗКе *&Х^*' а ЗНа" чит, 1— tg^p>0. В то же время функция v = arccos / по определению принимает значения из отрезка [0; п\ т. е. arccos (jc+ Isin у 1)^0. Итак, в левой части неравенства (15) содержится сумма двух неотрицательных выражений 1— tg^- и arccos (jc+ Isin i/|). Значит, неравенство (15) может выполняться лишь в случае, когда каждое из указанных выражений обращается в нуль: 316
|arccos(jc+|sin, Решая эту систему, получаем: i (18) \ |sin i/l = —4Л. Последнее уравнение имеет решения лишь при k=0 (при других целочисленных значениях к правая часть уравнения будет по модулю больше 1). Значит, систему (18) можно переписать в виде {*—1. \ Isin i/l =0, откуда находим гх=1, \y = nk — решения неравенства (15). У п ражн е н и я Решите уравнения. 1964. 1965. + 1966. 2(х4 — 1967. (x2- 1968. ,Г_ c —2 1969. 1970. 1971. 1972. 1973. cos2 jt-4-cos jccos «/ + cos2 y—0. 1974. tg2jc + 2tgjc(sin(/ + cosi/) + 2 = 0. 1975. 2 + 2 sin *(sin t/+ cos |/) = cos 2^. 1976. 2 -^(sin x+cos a:) cos (/ = 3 + cos 2y. 1977. sinx1+sinx lyl 1978. logi (дс+у)-2 sin ^log2 (jf+y)+ |(/- 11 = -1 1979. cos x+cos (/—cos (x 2И-2' 317
1987. tg2 n(x+y)+dg2 я 1988. logs 1990. 4 sin2 xy+4si sin In 1991. 3 1992. 4 1993. sinx cos (x+y) ) = 13+4 cos2 +() 1994. (cos2 *+^7 ) (1 + tg2 2y) (3+sin 3z)-4. Решите системы уравнений. 1995. Г 10^ + 5^+ 13z2=12j«/+4xz + 6(/z, I x3 + «/3+23=288. 1996. j 8 sin x cos «/ sin (x+y)+1 =0, 1997. Г 4 (2cos jc cos у -(л/З + 1) cos z) cos (*+|/)+3=0, 1998. rtg2x+ctg2jc=2sin2^ | sin2 y+cos2 z==l. 1999. flog, |sinjc|4-!og|sinjc,~=2cos24f, V5 V log^ sinz + sin2i/=l. Решите неравенства. 2001. 2003. cosx—i/2—Vi/—*z-l>0. 2004 "1 2005. (3—cos2 x-2 sin x) (lg2 y+2 lg y+4)<3. 2006. (sin2 (x+y)+2 sin (x 2007. 2y-2 cos jg+Vy—>-1<0. 2009. 2010. 2011. arcs (l+jf)+arccos(x+^)<-l. 318
ОТВЕТЫ Часть I. Алгебра Глава I. Тождественные преобразования. § 1. Разложение многочленов на множители ). 2. (а-3)2 (а+3)2. 3. (а- 1)(а2 + 1)(а2 + а + 1). 4. а2(а (1)(+1)3. 6. (а-Ь + )( + Ь)(+Ь + )( ()()(+) Х(а-2)(а + 2). 5. (а-1 )( 7. (а2 + а6 + 62)(а2-а6 + &2). 8. (а2 + а+1)(а2 —а+1). 9. (а2+а+ \){а2 — а+ Х(а2 + аУЗ+1)(а2 — а-^ЗЦ-1). 10. (а2+6а+ 18)(а2 — 6а +18). П. (а2 + а6 л/2 + &2)Х 2/ 2 2 2-а+1). 13. (а2+1)(а2+ал/3+1)Х 2 2 2 Х(л/ + ) ()(+)( + + )(+) (+)(+л/+)Х Х(а2-а л/3+1). 14. (а-1)2 (а+1)2 (а2+а + 1)2(а2-а + I)2. 15. (а- 1) (а+1) (а2+ 5). 16. (а2+1)(4а2+1). 17. (а2+ 1)(2а2 + а + 2). 18. (а+У2)(а—У2)(а2+3а + 6). 19. (а-1)(а2 + а + 2). 20. (а+1)(2а2-3а+3). 21. (а-1) (а + 3)2. 22. (а-с)(а + 26)Х Х(26-с). 23. (а+6)(6 + с)(с+а). 24. (а+6)(а-2с)(6-2с). 25. а(а-1)(а+1)Х Х(а-2)(а+2)(а-3)(а+3). 26. ЬаЬ (а + 6) (а2 + а*> + Ь2). 27. (а-6)(б-с)(с-а)Х Х(а6 + ^ + са). 28. (6 + с)(2а-6)(2а+с)(2а + 6-с). 29. 3(a+b)(b + c)(c+a). 30. а (а+1) (а2+а-Ь7). 31. (а2 + За+1)2. 32. (а+2)(а+6)(а2+8а+ 10). 33. (За^+4а— -1)(За* + 2а+1). 34. (а-Ь)(Ь-с){а-с){а+Ь + с). 35. 3(а-6)(6-с)(с-а). 36. 3(а+с)(а-с)(а2 + 62)(62 + сЬ). 37. (a2-aft-62)(a2+3a6-}-62). 38. (a+b + c)X X(ab + bc + ac). 39. (а+6 + с)(а + 6 — с)(а — b + c)(a—6 — с). 40. (а+1)(а2+а+ 1) Х(а2-а+1). 41. (а2 + а+ П2. 42. (а+^(а2-4а6-62). 43. (а2 + /2+1)(2 + 2). 44. (a2+a-f 1)(а8 —а^+а5—а4 + а3 —а+1). 48. При а § 2. Тождественные преобразования рациональных выражений 5a2-36 62. 2a. а4-1 -. 64. 0. 65. 0. 66. —l—. 67. a 2fl|oZ?.68.0. 69. а + ^ + с. 70. loc а-f-c а —loo Х[Ь + с)(с+а). 76. 1. 97. Sa=-^—. 98. Sn=4^—. 99. S^-r^r-r-. 100. § 3. Тождественные преобразования иррациональных выражений 119. 6V5 + 2. 120.39. 121. о О если 6< —1, Ь>\. 125. а+6, если а>0 и 6>0; _а(а если а<0, 6<0. 126. 2+V5. 127. V2-1. 128. 3. 129. (V5+1)^/5. 130. 1+->/2. 131. V3-1. 132. V5-2. 133. V6Vg У^У^^У^ V225+VT05 + V49 g 161. 2. 162. Va—V^- ^3. -a2\fb. 164. 2. 165. — 1, если 0<а<1; если — 1<а<0. 166.
171. 1. 172. mMb. 173. 1. 174. x, если ^J^X); -*, если 175. Q .+ l . 176. 4 177, __ty£. 178. * 179. a-l, если -l<a<0 2 0<a<l, a>l; 1— а% если a< —1. 180. 2a. 181. ~\l—. f 4. Тождественные преобразования показательных и логарифмических выражений 182. а) 0; б) —1. 183. а) -^; б) —Lr. 184. а) 24; б) 890. 185. 0. 186. а) log3 12 у9 V125 у9 б) i-. 187. а) 0; б) 0. 188. 3,0970. 189. jggj. 190. ^. 191. ,92. 1^. Ш '.,«.«+»=«. 195. ^±i .196. «±». 197. 1&± a-f-6 6 I—a a—b-\-\ 2a+b pq 198. 5n~3 . 199. 1. 208. \ogab. 209. log« 6. 210. a + 6. 211. 6log'a. 212. -a, если 0<a<i; a—2, если a>l. 213. loga bt если a>l и 6>1 или если 0<а<1 и 0<6<l. 214. 0, если а>1 и 6>i или если0<а<1 и0<6<1; — 2 (log* a-j-loga b)t еслиа>1 и 0<b<\ или если 0<а<1 и Ь>\. 215. 2, если 1<а<6; 2 loga b, если 1<6<а § 6. Сравнение значений числовых выражений 269. а>Ь, 270. а<Ь. 271. а>6. 272. а<Ь. 273. а) а>6; б) а<Ь. 274. а) б) а>Ь. 275. а) а<6; б) а>Ь. 276. а) а<6; б) а>Ь. 277. а) а = 6; б) 278. а) а>6; б) а<Ь. 279. а) а>6; б) а<Ь. 280. а) а>6; б) а<Ь. 281. а) б) а>Ь. 282. а) a<fr; б) а<Ь. 283. а<6. 284. d<b<a<c. Глава II. Решение уравнений, систем уравнений и неравенств § 7. Равносильность уравнений 295. Да. 296. Нет. 297. Нет. 298. Да. 299. Да. 300. Нет. 301. Да. 302. Да. 303. Да. 304. Нет. 305. Да. 306. Да. 307. Нет. 308. Нет. 309. Да. 310. Нет. 311. Нет. 312. Нет. 313. Нет. 314. 4. 315. 0; —2. 316. 13. 317. 5: -2. 318. 9. 319. 8. 320. 6. 321. 6. 322. 2; 34. 323. 4. 324. 3. 325. 4. 326. 8. 327. 5. 328. ~. 329. 3; 4. 330. 3. «3 § 8. Рациональные уравнения 331. 1; —1. 332. 2; —2. 333. 1. 334. — 1; 2; 3. 335. — 1. —3, —5. 336. —f-, —у, 3. 337. 1; 2; 2,5; 5. 338. —1; 2. 339. 1; 2. 340. —4; -3; 3. 341. — I. 342. —1, у, 3. 343. 4. 344. -2, 2, -^Ц^-, ^f^. 345. 0. 346. 0. 347. -4; -72; 72. 348. 3. 349. -1. 350. -2; 2. 351. а) 2; 3; б) -1; 3; 1±лД0- 352. ~; 2. 353. -2, -1, 0, 1. 354. 0; 1. 355. 1; 2. 356. -1; 2. 357. 3±^. 358. -3; 2. 359. -5^Vl3 360 _ 5 _ 1 361 45. 55 362 _5 _3. 363. -4"- 364. — S«5. —1. Щ^ • Звв. —1. 367. ~. у, -|. 368. 0.3; 0.4; 0.5. 369. -i. -|. 1. 320
370. -3. 371. у. 372. L. 373. у. —l-f . 376. . 377. 2; 6. 378. . 374. у; 2. 375. y, 2. . 37». 0. 380. -1; 2; . 381. -I; -f 382. 0. 383. -.; 9; . 381. I; f 382. 0. 383. .; 9; § 9. Уравнения, содержащие переменную под знаком модуля 384. а) 0; -1; б) 0; у2". 385. a) f; -\±f; б) ~'+V33 386. а) ^; 6) {I; 3. 387. а) -4; 4; б) ^; l±f-. 388. а) (- со; L] б) [|; + „ ) 389. а) -1; б) —|-; 4. 390. а) -5;—; б) -у, -^-. 391. а) -5; у; .. б) _V* .-VS. 393. а) б) —|. -у. 392. а) 1; - б) 1±л/2. 394. а) 0; б) -3; 3. 395. а) -2; 0; б) —3; -2; 0; I. 396. а) [—I; 0]; 15 3 9 2 11 б) -у; -у. 397. а) [2; оо); б) у; у. 398. а) [1; 2]; б) -14; у. 399. а) 1; у; б) -у; 0.400. а) -|; б) -|-; 2. 401. а) -2; б) у; у. 402. а) у; 3; б) -g-. 403. а) -3; 2; ~! *^ ; б) - 1. 404. а) (- оо; -3]U[3; оо);б)[-3; -2]U U[2; 3]. 405. а) 2; б) ~1 + Л//5. 406. а) у; б) ~. 407. а) 0; б) 0; [у; оо ) . 408. a) [_1;-L]; б) ±; -2. 409. a) -i-; 1; 2; б) [0; 1]. 410. а) -1; 1;-|; б) -1. § 10. Системы рациональных уравнений 411. (-4; -4), (-6; -2). 412. (-1; -3), (у; в). 413. (-4; -5), (5; 4). 416. (I; -3). (-1; -3). 417. (-1; -3). (у% -|). 419. (4; .). 420. (2; 0). (О; —2). 421. (1; 3). 422. (1; —2), (—Зг\-2 т/2; 2—2 -\/2), ( — 3—2 т/2; 2+2 -^). 423.(1; -1; 0). 424. (1; I; I). (7; -3; -1). 425. (3; 1; 2), (у; 5|; jj). 426. (у; з), (-у; -з), (1; 2), (-1; -2). 427. (1; 2). 428. (I; 2). (— Щ-: - 429- О: 1; 0. (-2; -2; -2). 430. (0; 0). (4; 2), (-2; -4). 431. (1; 0). 432. (3; 1), (3; - 1). ( _|; 3^) . ( -|; -Щ . 433. (3^; (-3V5; -л^). (Зл^: -Л^). (-Зл/2; л^)- 434. (3; 2), (1; 4), (-3; -4), (-5; -2). 321
(О; -f).{U I), (-,; -,). 437. (3; .). (-3; -.), 43, 0. 44„. (f; _f ^) (^ ^) С- >. <= >. (; ). ("2; -О- 444. (2; 3), (-2; -3). (3; 2), (-3; -2). 445. (2; 3), (3; 2). 446. (О; 0), (т/7; т/7), (-л/7; -iff), (л/19; -VI9), (-л/19; л/19). (2; 3), (-2; -3), (3; 2), (-3; -2). 447. (2; 1), (-2; -!). 448.(3; 2), (—2; —3), (0; 0). 449. (3; 2), (-4; —12). (О; 0). 450. (3; 5), (5; 3). 451. (2; 3), (3; 2), (-2 + л/7; -2-V7), (-2-л/7; -2 + л/Т). 452. (3; 1), (1; 3), (-1; -3), (-3; -1). 453. (-2; 3). (3; -2). 454. (6; 6), . 455. (,; 4). (4; (1; 2), (-2; -1). (-1; -2). 461. (2; -1), (-2; 1), (1; -2), (-1; 2). 462. (3; 1), (-1; -3). (VI3; -У13). 463. (I; 2; -1), (-!; -2; 1). 464. (1; 2; -1), (-1; -2; 1), (3^;Ь£; _±). (-If: -5-f:f). 465. (2; ,; 0), (-2; -,; 0). (-1; -1; -1). 466. (0; 0; 0), (1; 1; 1), (0; -yj2\ л/2), (0; -л/2. -л/2), (л/2; 0; л^). (-Л^; 0; -л/2), (л/2; л/5; 0), (-л/5; -л/2; 0). 467. (1; 2; 3), (1; 4; 1), (5; 2; -1), (5; 4; -3). 468. (1; -2; 3), (3; -2; 1), (1; -3; 2). (2; -3; 1), (2; -1; 3), (3; -1; 2). 469. (0; 0; 0), (л/2; л/2; л/2), (~л/2; - V2; -л/2)- 470. (3; 2; 5), (3; -2; -5), (-3; -2; 5), (-3; 2; -5). 471. (9; 3; 1), (1; 3; 9). 472. (3; -2; 2), ^ ;l;l). 476. (l; 2; |), (2; ±; l). (i-; 1; 2), (l; ±; 2), /-7 + УПЗ. -7-УЙЗ. \ /-7-лД13. -7 + л/ЙЗ, V 2 * 2 • ~9У' V 2 ' 2 ' ~ (4; 3; 5), (-3; -4; -5), (-4; -3; -5). 478. (0; 0; 0), (1; 2; 1), (2; 1; 1), /З + л/6 З-л/6 2 \ /З-л/6 З+л/6 2 \ ^—3—; —3—; ТУ' \~ЗГ"; —Г~ ;tJ- 479. (13; 0; 13), (8; 2; 4). § 11. Задачи на составление уравнений и систем уравнений 480. 31. 48.1. 24. 482. 12 и 1232. 483. 103. 484. 285 714. 485. 54. 486. 83. 487. 428 и 824. 488. 2. 489. 0,25 кг серебра. 490. 24 и 16 станков сверх плана. 322
491. 35 кг пшеничной и 45 кг ржаной муки. 492. 18 дней. 493. 3%. 494. 200 495. На 10%. 496. 1. 497. 44 чел. 498. 32 чел. 499. 10. 500. 820. 501. 6; —1" • 7 502. 12; 24; 36; 54 или 52,5; 37,5; 22,5; 13,5. 503. 5103 или ^т. 504. 931. 505. 1350. 81 16 506. 12, 18, 27. 507. 20. 508. 5. 509. За 8 ч. 510. 18 к. 511. 18 рыб. 512. За 16 ч и у ч. 5 5 513. За 20 ч и 30 ч. 514. За 3 ч и 4 ч. 515. За 12 ч и 8 ч. 516. За т ч и - ч. о 1о 517.За 16 дней. 518. За~ч и-^ч. 519.7,5 ч и 10,5 ч. 520. За 14,4 ч. 521. За 3 ч. 522. Пре- 60 изводительность второй фабрики в 2 раза выше. 523. За 6 дней. 524. За — мин. 525.8 ч. 526.50 ч. 527. В 1,5 раза. 528. Через вторую трубу в 2 раза больше. 529. В 3 ра- за. 530. В — раза. 531. За 10 дней. 532. 60%. 533. Уровень нефти поднялся. 534. 300 г и 500 г. 535. 441 г. 536. 40 т и 60 т. 537. 187,5 кг. 538. 50 кг. 539. 53%. 540.5%. 541. 10 кг. 542. 2~ кг. 543. 1:3. 544. 1,64 л и 1,86 л. 545. 15 кг. 546. 1 кг и 2 кг. 547. 10 кг, 69%. 548. В 2 раза. 549. 18 кг. 550. (kz-H-c)°. 551. 5% и 10%. 552. 40% и 65%. 553. 170 кг. 554.6 л. 555. 6 л. 556. 2,4 кг и 4,8 кг. 557. 3,5 л глицерина и 0,5 л воды. 558. 10 л, 90 л. 559.20 км. 560.50 км/ч. 561. 10 км/ч. 562.360 см и 18 см/с или 60 см и 6 см/с. 563. 1375 км. 564. 840 км, 80 км/ч, 70 км/ч. 565. 40 м/мин. 566.6 км/ч и 3 км/ч. 567. 6 м/с и 8 м/с. 568. 3 ч, 2 ч. 569. 20 км/ч. 570.20 км/ч. 571. 3 км/ч и 1 км/ч. 572. 8 км. 573. 10 ч и 9 ч. 574. 60 км/ч и 40 км/ч. 575. 15 ч и 10 ч. 576. а(\ +ф) ч. 577.50 км/ч и 100 км/ч. 578.60 км/ч и 100 км/ч. 579.25 ч. 580.16 ч. 581.2 ч. 582.25 км/ч. 583. Скорость течения 3 км/ч, а катеров 15 км/ч. 584. 14 км/ч. 585. 1 с. 586. 20 км/ч. 587. 20 км/ч. 588. Скорость первого пешехода в 2 раза больше скорости второго. 589. В 8 ч 30 мин. 590.5 ч. 591. За 3 ч. 592.1:2. 593.4 км/ч; 3,2 км/ч. 594. 4 км/ч; 60 км/ч. 595. В 2,75 раза. 596. 48 км/ч. 597. За 6 ч и за 4 ч. 598. За 4 ч. 599. За 3 ч. 600.24 ч. 601. За 90 с. 602. 4 ч и 4~ ч. 603. 80 км/ч. 604. 40 м/с и 36 м/с. 605. 20 м/мин, о 15 м/мин, 280 м. 606. ~ и ^г. 607. 15 м/мин, 20 м/мин, 280 м. 608. 20 км/ч; 25 км/ч; 15 км/ч. 609. «306,7 м. 610. 40 км/ч и 60 км/ч. 611. В 10 раз. 612. 5 км/ч и 4 км/ч. 613. Первая— 13 деталей, вторая— 11 деталей. 614. 16 ч 615. 60 м3/ч и 24 м3/ч. 616. 100 м3 и 140 м3. 617. а=0,5. 618. с=10. 619. 9 км. 620. 3 т. 621. В 1,5 раза. 622. 20 км. 623. 160 км. 624. 7 км/ч, 6~ км/ч и 6,5 км/ч. 625. НО км/ч, 120 км/ч о н 100 км/ч. 626. 832 заявления. 627. 26 бойцов. § 12. Иррациональные уравнения и системы уравнений 628. 2; 3. 629. 7; 8. 630. 2. 631. Нет решений. 632. 0. 633. -2; ~. 634. ^Ь£. 635. 0; 2. 636. 0; 0,5. 637. 1,25. 638. -1; 1. 639. 64. 640. —\-; 1. Z 3 641. -~; 1. 642. 1; 2. 643. -1; 0. 644. -2; 1. 645. 2. 646. 1. 647. -4; 4. 648. -2; 6. 4602 — 1 — 649. -1; 4. 650. -7; 2. 651. 657. 1. 658. -1; 8; 27. 659. ™; 1. -^ 18 654. -0,5. 655. U+Vi3t ese. 660. 1. 661. 5. 662. -37; 6. 663. 2. 664. 15. 665. -2; 5. 666. 1. 667. 1. 668. -88; -24; 3. 669. -1; —1; 1; 2. 670. 2. 671. 1. 672. 1; 2; 10. 673. 1; 20. 674. -3; 3. 675. -2. 676, 1; 3. 677. 8— ^ ; 8+ * ; 8. 678. 0. 679. 1416. 680. 9. 681. 12. 682. I. 323
683. 2; 3. 684. 1; 4. 685. 2; 6. 686. —61; 4. 687. 0. 688. 2; 6.J689. 1; 32. 690. 17 — -v^57; 17+V257. 691. 6/.1Q19 692. 0. 693. 0,25. 694. 3; 5 + ~^297 . 695. 2; 3. 11У о 696. -6; 1. 697. 1. 698. 3. 699. 5. 700. ^-. 701. (1; 4). 702. (9; 4). 70зУ~; ~\ (3; 2). 704. (^; -1) . 705. (7; 13), (-7; -13), (^; u) , (_^; -u), 706. (3; 1). 707. (6; 5). 708. (2; 8), (8; 2). 709. (1; 9), (9; 1). 710. (1; 4), (4; 1). 711. (1; 8), (8; 1). 712. (1; 8), (8; 1). 713. (-8; -1), (-8; 1), (8; -1), (8; 1). 714. (1; 7), (7; -8), (g; y). 715. (0; 0). 716. (5; 4). 717. (2; 3), (y; -y). 718. (II; A). 719. (4; 9; 1), (-4; -9; -1). 720. (3; -2; 6). 721. (5; 4; 5). f 13. Показательные уравнения 723. 3. 724. -3; 1. 725. izA; ±HH. 72e. _|. 3. 727. -±; 3. 728. 3. 729. 0. 730. log 3 %. 731. 2. 732. 3. 733. 0. 734. 0. 735. -1; 1; —у/2; л/2- 736. 1. 737. 3. 738. 3. 739. 2,5. 740. -1; 1. 741. 1; 1 +л/2; 1 —Д 742. 0; 1. 743. 1. 744. 0. 745. 0. 746. 0. 747. 1,5. 748. 0. 749. —0,5; 0,5. 750. 0. 751. 0. 752. 0. 753. — 1; logo4 5. 754. 0. 755. 1. 756. 1. 757. 0; log,,5 3. 758. — 1; 1. 759. — 1; 1; 2. 760. —4; —2; 2. 761. —3; 1; 2; 3; 4. 762. у; у ; 2. 763. -1; 1; 2. 764. -(2 + log3 2); 1. 765. -(1 + log5 2); 2. 766. [0; 1} 767. -3; [-1; oo). 768. 0. 769. 9; 2. 770. 1; 4. f 14. Логарифмические уравнения 771. 2; 3. 772. 2. 773. 1,5; 3. 774. 0; 1,5. 775. 2-л/З; 2 + л/1 776. 5. 777. 4; 6. 778. 41. 779. 0. 780. ~. 781. 8. 782. 6. 783. 1,5; 10. 784. 37. 785. 2; 3. 786. 0. м -4+V5 -4-Уб 787. 10~3; ИГ1; 10; 103. 788. — 1 + лЛО; 9. 789. 10 5 ; 10 5 790. 10. 791. Ю-1; 105. 792. \£; 5. 793. 10. 794. л/2; 4. 795. 2~7; 2. 796. -8; -3. 797. 1; 2. 798. — 5; 3.799. КГ1; Vi<>-МО. —100.801. —11; 1; — 5+7*4; — 5 — Уп.802.3; 803. -64; -1. 804. ~5+^; ~~3^^. 805. -4. 806. 0,75. 807. 3. 808. -5; 5. 809. А. 810.9"'; 9. 811. 1; у; |1. 812. 0. 813. 2. 814. 1. 815. 4. 816. 2. 817. 2. 818. 2; 8. 819. 3~f ^; i±|^/?. 820. -^. 821. -!-=:;-!-. 822. 1. 823. 2. 824. 10; 105. 825. 0,5; S S 12 3^/3 ^ 32. 826. КГ1; 103. 827. 100. 828.1; 10""^^; 10V^1^. 829. 10"2; 102. 830. 0. 831. 2. 832.10~4; 10. 833.5"1; 52. 834. 3"1; 32. 835. V^6- 836. КГ1; 2; 103. 837.2~4; 2. 838. 2~l; 1; 16. 839. 3->/3. 840. 4"; "R- ***• юо- ! V i о lo 845, 2 9. 846. 3. 847. 17. 848. —2. 849. 1. 850. —1; 3; —^. 851. 1; 2. I 15. Системы показательных и логарифмических уравнений 852. С- Ю; -12), (12; 10). 853. (2; 3), (3; 2). 854. (-g-; -J-), (-J-; -i-). 3S4
855 . (у ; ^) . 856. (3; 2). 857. (4; 1). 858. (3; 2). 859. (4; 3). 860. 0. 861. (1; 1), (4; 2). 862. (1; 1), (2; 4), (-2; 4). 863. (1; 2), (2; 1). 864. (-|-; 64 ) , (8; 2). 865. (9; 7). 866. (2; 32), (32; 2). 867. (у; у)! (3; 1). 868. (7; 3). 869. (17; 9). 870. (2; 6). 871. (125; 4). (625; 3). 872. (3; 27), (27; 3). 873. (-^S у). (3; 4). 874. (4; 1). 875. (9; (V9; 9). 876. (1; 64) , (1; 1) . 877. (5,5; 2,5). 878. (^-; з) . 879. /^ \2^~) 880; (l0g4 12; log4 3)' 88!* (1; ~ !)* § 16. Рациональные неравенства 882.(0; 1)U(1; во). 883. (-оо; _1 )(j(y; 2 ) . 884. (- оо; -2)U(-2; - l)U и(у; з). 885. (-оо; оо). 886. [2; 5} 887. ( ?"""2 ; ? +2 )' 888# 4 889. (-8; 0)U(8; оо). 890. ( — оо; —6]U[ —2; оо). 891. ( — оо; 1). 892. — 1; 1 893. (-оо; -y)u(l; у). «И. (-4; -3)U(-2; - l)(j(y ; 3 ) . 895. (- оо; -4] U ^;JLb^]U[4;oo). 896. (_2; 2)и(2; 4). 897. [_ 1+ J}. 898. (-оо; 2)U(3; 5)U(7; оо). 899. (-оо; -_3)U(2--^; 3)U ). 900. (-у; о]и(у; оо). 901. (-1; 5). 902. (-8; 1]. U[l; oo).906.(—oo;0)U(3; оо).907.(-8; -1).908.(-3; — 2)U(—I; 1). 909. (-оо; 2)U U(2:oo).910.(J^;-l)U(,±±^L). ЭП. (-оо; 1)U(±; 2) 912. (-оо; -±)и(-.1|; у)и(2; оо). 913. (- ooj -7]U(-1; 0)U(0; 1]U(3; oo). 914. (-оо; — 1)U(— 1; 2]. 915. (-оо; 2)U[3,5; 4)U[7; оо). 916. (-оо; 5). 917. (0; 9). 918. (2,7; 6). 919. (1; 2). 920. [1; 2). 921. (-3; -л/7)и(л/7; 3). 922. (-оо; у ) и(~; 2). 923. (-1; 1)и(3; 5). 924. (-5; -|-)и(у; l)* 925' ["4; т] 926. (0; 1). 927. (-4; -3)U[-2; — 1]U[1; 2). 928. (-8; -6,5)11(0; 5). 929. (-4; —3)U U(-2; -l)u(o; y)u(l; 2)U(2; 3)U(3; 4). 930. (-oo; -7)U(-7; -2]U(1; 7)U U(7; 8]U(11; 00). 931. [-1; —i-]u[~; l] 932. (-00; 1)(J(1; 2)U(2; в.). 933. jH; б)и(6; 7]. 934. [у; 2ju(3; 00). 935. 0. 936. (-00; 1)(J(2; 3). 937. (-00; -|-)u(3; 00). 938. (-1; 1). 939. [0; 8]. 940. (-00; 2)U[2,5; 00). 941. (-00; — 5)U(-2; -1]U(5; 00). 942. (-1; 0)U(3; 5). 943. (0; 1)U(2; 3)U(5; 6); (6; 7). 944. (-1; 3). 945. (-3; l)U(2; 00). 946. a) (1; 6); б) (у ; l](J[6; 00); в) 0 325
947.(-oo; -l)U(2;3).94S.(-6;2).949.(-2;4).950.a) (-00; — I6*)U(6; 00); б) (1;4). 951. a) ( - 00; -y ]u[2; 00); 6) [1,5; 2,5]. 952. a) (- 00; - 1)U(O; 00); 6) (- 00; -0,4]U U[4; 00). 953. a) (-00; -5)U( — 1; 1)U(U <»); 6) (-00; — 4)U( — 4; 2)U(6; 00). 954. a) (-00; _2)(j(y; 00); 6) [4,5; 00). 955. a) (-00; 1]U[1,5; 00); 6) (-00; -y]u[7; 00). 956.a) (-00; oo);6)[-y; 00 ). 957.(0;0,4). 958. [o;i-J. 959. (-co; iz^L)u(iz^; l±^L)U(±b^; „ ). eeo. (_; l}u U(2,2; 00). 961. i"1^^7; —1 )u(— 1; l)u(l; "^t^"} 962. (-оо; -2)U U(-2; — 1)U(—1; 0). 963. (-00; —2]U[—1; 00). 964. (-00; — 2)U(—2; 0]U[l,6; 2)U U(2; 2,5]. 965. [-1; 1J 966. (-00; -3)U(3; 00). 967. (-00; —|)u(3; 00). 968. (-00; -4]U[1; 00). 96». (-00; —5)U(—1; 00). 970. (-00; 00). 971. [1,5; 2). 972.(l;3),973.(-5; -2)(J(2; 3)(J(3; 5). 974.(-00; -3). 975.(-2; 3). 976.(- 00; -2)U U(3; 00). 977. (- 00; 00). 978. (-0,5; 2,75). 979. (-00; 0)U(6; 00). 980. (-00; -0,5)(J U(0;4).98V.(-oo; -2)u[y; 00 ) . 982.(- 00; -4)U(-2; 1)U(3; 00). 983. (- 00; 2]U U[4; 00). 984. (-00; 2). 985. (0; 00). 986. Г~; з]. 987. (-oo; 0)U(4; 00). 988. (—00; 1,75)U(2,5; 00). 989. В первом — 24 детали, во втором — 7 деталей. 990. В первой — 11 человек, во второй — 17 человек. 991. 119 пионеров. 992.25 300 м. 993. 850 л. 994. По 9 человек. 995. 8 книг. 996. Двоек — И, троек — 7, четверок — 10, пятерок — 2. 997. 180 р. 998. 14 р. 999. 144. 1000. 44. 1001. 30<х<5(29—V§05). 1002. 14 красных, 19 синих. 1003. И гвоздик, 7 роз. 1004. 39 к. § 17. Иррациональные неравенства 1005. (1; оо). 1006. [2,6; 4). 1007. (~- оо; 0,5]U[0,68; 00). 1008. (3; 00). 1009. (—00; -1). 1010. [0,5; 00). 1011. (—оо; —2]U[5; 5^V 1012. [4; 00). 1013. ( — 3; 1). 1014. Г~; 4^U(5; 00). 1015. (— 00; 0]U(4,5; oq). 1016. (—00; 0} Ю17. [-{5; 2JU[3; 00). 1018. (-00; 00). 1019. [3; 00). 1020. ^4; 4yU. 1021. 0. 1022. [4; 5). 1023. [3; -^±{f^]. Ю24. 0. 1025. 0. 1026. [2,5; 1±^Ш). 1027. 0. 1028. (V§T; 2^} 1029. (-5; 5). 1030. (-•>& 0). 1031. (9; 00). 1032. (-00; -2)U[20,5; 00), 1033. (-9; 4). 1034. (- 00; — 4)U(1; «>)• 1035. [-1; 4\ 1036. (—1; 3]U[3,5; 7,5). 1037.(2; 00). 1038. (-00; 00). 1039. (-00; 00). 1040.(0; 00). 1041.[2;6} 1042.(-oo; V2)U(W; 00). 1043.(-00; -2)U(0; 1)U(1; «>). 1044.(-2; -1]U u[—|;y)- 1045. (-00; -4+2V5). 1046. (-00; ~y]u[3; 00). 1047. [-4; 16). 1048. (2; 8). 1049. [-2; 0)U(0; 2} 1050. (5; 00). 1051. [-1; 00). 1052. (-00; -2]U [j[-l; ^e""1 )♦ Ю53. (y; 00). 1054. [0; 3]. 1055. [2; 5]. 1056. [-1; 0\ 1057. [-3; 1} § 18. Показательные неравенства 1058.(0; оо). 1059.(8; оо). 1060. (-оо; -!)(J(1; 00). 1061. (-00; 0,4). 1062.(0; 00). 326
1063. (—оо; 1,5). 1064.Г—оо;~-J . 1065. (— сю; — 1)U(7; oo). 1066. (—оо; 0)U(6; с»). 1067. (—с»; — 6]U[2; оо). 1068. ( — 8; 14). 1069. (0; оо). 1070. (—оо; 1 — log2 3). 1071. (-оо; -Llog3.22j 1072. ( — 2 л/2; 2 -у/2). 1073. (3; оо). 1074. (-оо; 1). 1075. (-1; I). 1076. (-оо; 66]. 1077. 1; 2; 3; 4; 5; 6; 7. 1078. Г %=; -i/3 1 IU f-^r]. 1079. (-1; 0)U(0; 1)U(1; 2). 1080. (-2; —|-)u(o; - 1081. (0; оо). 1082. (-|-; log8 6o) . 1083. (0; 1). 1084. 0. 1085. (2; оо). 1086. (0; 1087. (1; оо). 1088. (-оо; 0)U(l; оо). 1089. (-оо; log2 (1 +•$)). 1090. (2; оо). 1091. (0; оо). 1092. (-оо, log, >5 0,5). 1093. (-4-; °° ) • ИЮ4. (2; оо). 1095. (0; 2). 1096.(1; 2). 1097.(-oo;0)U(2;3)U(3;3,5)U(4; оо). 1098. (^; 1 ). 1099. (-оо; -0,5)U U(l; оо). 1100. [logs 7; 2]. 1101. [log,3 5; 1]. § 19. Логарифмические неравенства 1102. (1;2)U(4; 5). 1103. (-1; 0)U(l; 2). 1104. (-1; 1)U(3; 5). 1105. (4; 5]U[95; oo) 1106. (1; 4). 1107. (3; 4,5). 1108. (-1; ~V 1109. (3; 4)(J(4; oo). 1110. (0; oo). 1111.(1; 1,O4)IJ(26; oo). 1112. (3; 7). 1113. (-2; -^ V 1114. [1; 4\ 1115. (-oo; — 2)U U(6; oo). 1116. (°> 3"2Л5)и(3"2 ; 3)' 1H7# 0' U18# (1; 3)* 1120. (0; l)u[~""7~tp ; 2^. 1121. (-2V3; -2)U(2; 2 ф). 1122. (1; oo). 1123. (0; 0,75)11(1,25; 2). 1124. [2; 3)U(3; 4]. 1125. (1; 4). 1126. (2; 00). 1127. (0; 1)(J u({~2 ; 2)' l128' (""°°; 0)u(5; °°^ ll29# (~°°; ИЗО. [0,5; 4\ 1131. (0; 0,5)Uh/2; 00). 1132. (-00; —5)U(3; 00). 1133. Гу ; -|" 1134. (4log0-e0>2; 00). 1135. (V5; 5). 1136. (logs (л/2 + 1); logs 3). П37. (0; 0,4)U(l; «>). 1138.(0; 0,25)U(4; oo). 1139.(1; 2)U(64; 00). 1140. (о; у V(243; 00). 1141.(0; 0,5)U(2; 00). 1142.(0,01; 00). 1143.(1; 5). 1144.(0,25; 1)U(1;4). 1145. (—00; log4 (- 1+V3))U(1.5; 00). 1146.^0; Iog3^)u(log3 4; 00). 1147. (4"7; 4). 1148.(1; 00). 1149. (log4.5^; 1,5). 1150. Г1; -V)u(l; л/2]- 1151. [0,5; 1). 1152. (3; oo). 1153. (0; 2)U(4; 00). LV2 \(i' 1154. (2-^; ±)u(l; 2^). Ц55. (-V5; -1.5)U(1.5; л[3). 1156. [-1; - i^; ij. И57. (-0,5; 2). 1158. (2"28; 1). 1159. (- 00; 0)U(l; <»)• n60(y; 3)* 1161.(4; 10). 1162.(-V2; - 1)U(1; \5). 1163.(log4 12; 2]. 1164.(0; 4). 1165. (- 00; -|- U(3; oo). 1166. (0; 0,5)U(2; 3). 1167. (-oo; 0)U(l; 2)U(2; 3)U(4; oo). 1168. (-2; - l)U U(l; V»). П69. (5; oo). 1170. (-2; 13). 1171. (13; 29). 1172. (40; 41)U(48; oo). 327
1173. [22; eo)U(-2,96; -2). 1174. (о; ~-]u[l; oo), 1175. (-1; -0,5)U(0; 1). 1176. (-oo; 0]U[log65; 1). 1177. (-oo; 0](J[log23; 2). 1178. [0,2; 5]. 1179. (3; oo). 1180. (-1; 0)U[l; oo). 1181. (0; l)U[2; oo). 1182. (2~15; 2"9]U[29; oo). 1183. (-1; 0)U U(l,5;2). 1184. (-3; - 1-V5)U(1; 5). 1185. ^2; у ). 1186.(1; 1,5). 1187. (-log* 3; 0). 1188.(1; 4). | 20. Уравнения, системы уравнений и неравенства с параметрами 1189. (— оо; оо) при а=1; » при аф\. 1190. (— оо; оо) при а=1; 0 при :—2, а=2; р—- при t аФ\, 1191. 0 при а=—3, а= —1,5, а=0; J аф2, \аф-2. при /аФ— 3, 1192. 0 при а= — 2, а= — 1; —(2а2 + 3а) при < вф —1»5, уаФО. (аФ—2% 1193. 0 при а=—3, а=0, а*=2; -^г при ( аФ — 3, 1194. а; За. 1 -а. 1 а-ро I _ I пФ2. 1195. -2 при а=0; 0 при а<-^-; -3 при а«—i-; 1Г2а=^У4а±1, при I, а>0. 1196. —^ при 0=—; 0 при —9—2 при а<— 9 — 2V2T, —9 + 2 V^ при а=— 2; 0 при а=1; *i= ~° , jc2= ._а при (аФ—2, 1198. 0 при 2_12а-48 1199. —1 при а=— 2, а=1; xi=a+l, x2=a—2 при (аф— 2, 1200. 0 при а=0; 1 2 —^-а при а^О. 1201. 0 при а=0; —1,5 при а = 1; 1 при а=— -g-, jci = 1, х2=—5^4^- при Г а^— -I-, 1202. 0 при а#:1; (— оо; — 2)U(—2; — 1)U(— 1; W z 3 6 I афО. К { U(l; 2)U(2; 00) при a=l. 1203. 0, a>— ->/3; Jfi=O, x2= ^ при а<—^. 1204. 0 при a<0, 0<a<l; 0 при a=0; ■"" ^ при а>1. 1205. 0 при а<0, 1 a2 1 . a2+24a+16 ~^a<l; — г- при 0<a<~, a^l. 1206. 0 при a>—4; r^ при a<—4. 1207. — ал/3 при а<0; ал/3 при а>0. 1208. 0 при а<0; —а при а>0. a 1 4a2 4-1 1 1209. 0 при а<1; -р^ при а>1. 1210. 0 при |а|>у; —^— при |а|<у. 12Ц. ±Д+У«-16а+60 при а<15 0 при fl> 1О~"~4(1 4 4 328
при 1<а<3. 1213. 0 при а>1; 0 при а=1; ± log12 (1 + V1 —а) приа<1. 1214. 0 приа<3, а>27, 2 + log4 ^—^-при 3<а<27. 1215. 0 при а<0; ю «j — о 2000а 3 при а>0. 1216. 0 при а<1, а> 100; 2d=v4-2 1ga при 1<а<100. 1217. 0 при а<0, а=1; а6 при 0<а<1, а> 1. 1218. 0 при а<0, а=1, а>2У2; 4 —а2 при 0<а<1, 1<а<2У2. 1219. 0 при а<0, а>1; у^з ПРИ 0<а<1 1220. 0 при а<0, а=1; *i=a2, *2=— при 0<а<1, а>1. 1221. 0 при а = 0; (0; 6) при а=1; 2 при а<0, 0<а<1, а>1. 1222. 0 при а=0; (0; оо) при а=1; 3м, где л=±1, ±3, ±5, ... при а= — 1; *i=3, *2=~ при а< — 1, — 1<а<0, 0<а<1, а>1. 1223. 0 приаб*. 1224. 0 при а<0, а=1, а = 2, а + 2 при0<а<1, 1<а<2, а = 3; х,=а — 2, х2 = а + 2 при2<а<3, а>3. 1225. 0 приа<1, а=->/2; 3 при а = 2; лг,=а— 1, х2 = а+1 при 1<а<-\^, -у/2<а<2, а>2. 1226. 0 при а= — 1; ПРИ в# — 1- ^227. 0 при а=~7; И; ^5^)» где /61?, при ^у ^7)~7' ~7<а<3, а>3. 1228. 0 приа=-1;(/; 1-/), где /6^ при а=1; fidbi±fL; i.^'j при а< - 1, — 1<а<1,в>1. 1229. (0; а), \ а+1 a-f 1 / (а; 0) при а£Д. 1230. (а; 2а), (2а; а) при а^О; (/, —/), где /61?, при а = 0. 1231. (/,; /2; 1 —/i —/2), где /|6Я, /26Л, при а=1; (-1-а; 1; а+1) при а^=1. 1232. (/; at; 2а/), где /61?, при абЯ. 1233. 0 при а<0; (9а2; а2) при а*>0. 1234. 0 при а<1; (^^ ; ^"4^ )приа>1- 1235. 0 при а= - 1; (-оо; 1—а)приа>-1; (1 —а; оо) при а< — 1. 1236. (— оо; оо) при а = — ; ( — оо; =- ) при а>— ; У \ Уа — о / У f Q _- ; 00 j при a<-q-• 1237. 0 при а=— 3, а=1, а = 3; (— оо; j при —3, 1<а<3, а>3; (^33; °° ) ПРИ —3<а<1. 1238. (— оо; 8) при а=10; 34-16а 4а \ ,л / 4а \ / 3+16а 1239. (-со;.) при а=^;(-^^) 1239. (-со;.) при а=^;(-^;^)приа>-3;(^;^)при / 1 \ 2 2 1 2 а<—3. 1240. ( — оо;~ ) при а=—^ ; 0 при я>у; —у при а = у;(—оо; оо) 2 2 2 при ~"5" 12оТзг) р ~I- 1242' 0 при а=2; [т; т) при а>2; ^-; -jl при а<2. 1243. 0 при а<0; (1 — 2-у/а; l+2-yfa) при 0<а<1; [—1; \+2^[a) при а>1. 1244. [1; оо) при а<0; П; ^^"^ ) при 0<а<1; 0 при а>1. 1245. 0 при а< — 1; ( ; —1 при — 1<а< —— ; (— оо; — 1]при <<0; (-оо; — 1]иГо; i^2a2 ) ПРИ Q>0- 124a 0 приа<0, а>4; (-2; 2) 329
при а = 2;.[-а; а] при 0<а<2; (-° ^ а . ау4а—2\„ри2<а<4. 1247. 0 JUL при Д<-1; 1-1; При - 1^ J |^ ^JL2 1 1при 1<а<У2; [—1; 1] при а>У2. 1248. [а; 0] при а<0; 0 при а=0; (0; а) при ^^); о] при а<0; [а(!-у); 2а ] ПРИ а>0. 1249. - 125°- ( l; при 1251.(1— л/9-а; 1 _^/i _a)u(l +V1 —а; при 1<а<9; 0 при а>9. 1252. ~; 0 при а=0, 2. Л; 9-Д) при а< 1; ( при а<0; при 1; (а; 1) при -т-<а<1. 1253. 0 при а<0, а=1; (2; 3) при 0<а<1, а>1. 1254. 0 при а<0, а=1; (a; l)lj(~; °° ) при 0<а<1; /1 \ \а / (—; 1J при а>1. 1255. Нет решений, если а<1; (2 — V4 —lga; 1)U U(24-V4 — lga; 4) при 1<а< 10000; (1; 2-74-lg a)U(2+V4-lg a; 4) при 1000<a<10 000; (1; 4) при а>10 000. 1256. 0 при а<0, «=y; (1-VTT^; \+-JT^a) при 0<a<~; (1 —VHKa; 1 -л/Г1:а)и(Ц-л/Ь=Га; l+ViTa) при при 1259. 1263. ~1 1266. - 1 1260. 4" . 1257. а>^. 1258. у 4". 1262. а<"~ . 1264. а<2; а>^-. 1265. ~ о 1 ; 1 <а< . 1267. -||; 0<a<V2. 1 / -2; а>0. 1268. а< —~ ; 1269. а=1; а = 2; 5<а<6. 1270. —6<а<—4; — 3<а<-1; а=1. 1271, - —5; а=—2; а= — 1. 1272. — у<а<~"^; а==1* Часть II. Тригонометрия Глава III. Тождественные преобразования § 1. Тождественные преобразования тригонометрических выражений . 1276. tg 2а. 1277. sin 2a. 1278. 1. 1273. cos a. 1274. —C0Sg2f . 1275. —!— sirr p cos a 1279. 1. 1280.1. 1281. tg За. 1282. tg 4а. 1283. tg (у—у ) • 1284. 8 cos4 a. 1285. tg a. 1286. tg'a. 1287. 2^L±u I288.cos (зОЧ-j) . 1289. f. 1290. -^. 1291.ЭЁ+Д 1292. 2-^ 1293. - . 1294. - 1296. 1298.1. 1299. 4. 1300. sin a=^, о . |29в.0.1297.^. 4 tga=y, 3 <ОЛЛ -j-- l302- 5 =-^pt tga = —i-. 1301. о 12 5 tOM . o 120 —^. 1303. sm 2а==-^ 330
119 ___J~2. 1306. -^?. 1307. a) 7; 6) -18; в) =ЬЛ . a 1 ^. . a . 4 a 3 *«Т"Т: б) »inT"T'c0STe"? а О £ О ^ U 119 125 5-12 V3 —Г" 5. 1308. a) siny=^, cos|-= tg -^-= —tt • 1309. -=- • 1310. sin a cosa==—£-, a=——-. 1311. § 2. Тождественные преобразования выражений, содержащих обратные тригонометрические функции 1387. -5-я. 1388. —1. 1389. -^. 1390. 4-. 1391. 4. 1392. 0,Зя. 1393. —J-. 2 2 4 1394. -5.. 1395. -!£-. 1396. 5?. 1397. я 1398. ~^. 1399. 0,2. 1400. ^. 1401. 0. 4 7 II 1402. ~. 1403. 1404. х(/~УГ= ,4„. § 4. Уравнения .... я . ^ч я . Ч2яс . я . 2я , . я . 2л, я,о ч я t 1496. а) у*; б) т^; в) т Л; г) ^+у Л; д) Т+Т k; ±у+2яп; е) т*; |+ш; ж) |+~Л;у+2ял;з)~+|Л;и) ±~+яЛ. 1497. а) 0; б) ^+ТЛ; В) у^(~1)п+1Х+ял; г) 'J+23lk; Д) ±У 1498. а) у+~Л; б) ~ Л; в) 4+4 4 2 1499. а) 4^ у*; -^+~п; ж) яЛ; ±у+ял; (-iy^+ши. 1498. а) у+~Л; б) ~ "ТЛ- 150°- 4+2л/г- !501- 2лЛ- 2 2 у 1506. ^Ч-у*- 1507. -4+Т+**- 1508. i+ifc J+-f я. 1509. -i+^*. 1510. -^+^*. .i+^ik. 1512. *-£+**; -~+яп. 1513. ^+|*; Ц п. 1514. ^+у*; 1502. 0. 1503. 0. 1504. у+л*. 1505. (-1)*-~+л£. 1515. (-\f± 1516. Y+T 1518. ±~ + 2яЛ. 1519. 2лЛ; 4+^Гл- 1520. (-1)*+1 4 4 о о о уЛ. 1522. ±-^ + nk. 1523. у by + 2nn. 1524. ~-+у*. (—1)"~ + ял. 1526. arctg-|~f-*fc. 1527. 1528. arctg Г— у 1529. у + лЛ; arctg ЗЧ-яп. 1530. ~+яЛ; -arctg 2 + пп. 1531.-Я-+яЛ; arctg 331
; arctg3+nm. 1536. ~ + 2л*. 1537. ^+nk; ^j+яя. 1538. ■£ +S+Trtl гдеф=агссо5Т- 1542> ТГ*: Т+яп- 1543' Т?+Т*' 1544- Т: ±^+2nn. 1549. y+nft; ^+y«- 1550. -J+yA; (~>Г jg+T«- ««• Т од л arctgy+ял. 1552. —arctgy+яЛ. 1553. 0. 1554. я+2яЛ; 2arctg4-+2n«. 1555. arctg^^+л* 1556. -j k; i-arctg2+y n. 1557. у+2я*; я+2яя. 1558. 2я*; ~+2яп. 1550. -J±arccos^+2«ife. 1560. ^(л#1|^); i . 1561. 0.1562. 0. 1563. 0. 1564.(-1)*£+яЛ. 1565.—+Я*. 1566. ±J1+-?-^. 1567. —+"*• 1568. -40°+60°Л. 1569. ±~ + 2яЛ. 1570. -^; я 17я --«- л , я <р-л л я я я 5я Зя ---л 7я я ~; —. 1571. 0; ±-; ±я. 1572. 0; -; т; у; ^; -; я. 1573. -^ ; и; 52. 1574. у+2яп, где п=±1; ±2; ±3; ...; (-1)*^.+яЛ. 1575. у+2яп, где л=±1; d=2; d=3; ...; (-l)*i+n/Sf, где *=±1; ±2; ±3; ... . 1576. 0; ±1; ±2. о 1577. ±1; ±3. 1578. ±у. 1579.у+2я*, где/г=:0; 1; 2; ...; -у+2ял, гдел=-1; -2; -3; ... . 1580. 6; у я 5я t ntnu. я_|_д« 1585. 2я+4я*; 1589. . 1588. (-1)*-|- Цп. 1590. ~-?- =hy arccos (—i- я «.-АЛ я , , я ул. 1593. — -j+nk; -j±arccos ^+^-Л. 1596. -£Ч"£-*. 1597. ~+-^-Л. 1598. . 1600. (-1)*4 . 1609. ^+2лЛ; arctg 3+я(2п + 1). 1610. 2я*; ~+яп. 1611. у+лЛ. 1612. 0. 1613. dby arccos у+яЛ; ±у arccos (-^ +пп. 1614. ~+у*.1615.я*; ±у arccos-J-+nn. 1616. *^+Ц?-к. 1617. ~+nk; 332
±4 + яя- 1618. 0. 1619. i-ii+4*- 1620. nk; ±4 + лл- |в21- л + 2яЛ. 1622. -£ + 2лЛ. 1623. ^ + 3л/г. 1624. 0. 1625. 2л + 24л*. 1626. ^r + nk. 1627. ~ z biz 1628. 4. 1629. 0. 1630. nk. 1631. ~ + 2л6. 1632. я*. 1633. ±-§-+*. где 1634. -£-+л£; ±arctg4-+«i. 1635. 1; 2. 1636. 4. 1637. *+^2. 1638. sin 4*. 1639. tg|5. 1640. 0. 1641. ""1|">^ . 1642. 0. 1643. -\/^=1. 1644. ~. § 5. Системы уравнений 1645. (у(* + «); y(ik-«)) 1646. (i+n(*-n); i+n(* + n)) (i+(ikn) 1647. ((-1)*-^ 1648. (-jr+nft; -f—я*). 1649. ((_!)* i+яА; 2яя). 1650. (у+лА; у 1651. (t; t + n(2k+l)), где /€«• 1652. (-1+Л; —g"+*) • где 1653. (y+nfe; -я*), (яЛ; у-я*)- «654. (у+л*; у-я*). (у+л*; у-яй). 1655. (я*; £-я*). (f+я*; -у-яй). 1656. (±у+4лй; 2я + 4лл\ (гя+4лй; ±у+4яя\ 1657. (як; у-л*), (i+«»; у-л*). 1658. (g+y*; ^+y*). 1659. 0. 1660. (j+nk; -y+nft). 1661. (_+я(л + *); -*-+*(/.-*)), (-у+я(п+А); -у+я(п-А)). 1662. (±у+2яА; ±у+2яп). 1663. (±у+яй; яп), (у+яА; ±у+яп). — у±у+я(А+я); —2.±у+я(*—я)У где a = arccos^-. 65. (у+лА; -у+л*), (у+лА; у+я*). 1666. (а+лА; -*--а-я* ) э+лА; ^~Р-яа), где « = arctg^Z±^, p=arctg ~7~ 1667. (^.+яА; -^-яа), (»§+**; ^-яа). 1668. (|^ 333
,e70 . (J-+2n*; т+2я«), (y+2nft; ^+2л« ), (-y+2*ft; -т (я*; -^+2яп). 1671. ((-1)*-*-+„*; (_!)"+• y+arctgy+*(ft-«)) 1672. (у+я(*-п); у-n), (_y+n(ft-2n); у+яп), (у*+я(А-2п); -у+ял), (y+*(ft-2n); „„ ). 1673. (у ft; я(я+*)). 1674.(-y+(-l)*a+yft;y+(-l)'a + yft). где e = i- arcsin 1675. (2л*; у+2яп), (я+2яй; _у+2яп), (-J -|+2яЛ; |+2яп). 1676. (у+я*; |~я*). 1677. (у+2я(« + Л); т+) (« + *); -у+2яп). 1678. (2refe, я + 2яп). 1679. (§ ^ 1680. (|-+у(*+„); i+y(n-3*)). 1681. (|- 1682. (у ) ( ^) (^ ). 1683. (л*; 2яп), (■j+nk; у+2яп\ 1684. (y+ji*; arctg 2+nn; ~arctg 2-я (k+n)\ (^~+як; -arctg 2+яп; ^+ arctg 2-я (ft -j-n)\ 1685. Cj+nk; arctg 2+яп; ^— arctg2—n(k+n)\ (arctg 2 + яй; -^-+ял; ^-arctg 2-я (ft+n)). 1686. (я*; яп; я-я(А+п)), у+2я*; -"-+2яп; у_2я(* + п)), (-у+2я*; --- ..„ /я,, я, я,я \/ я, .я. я.я \ 1687. ^у+я*; у+ял; -J+Ym )• \~Т+ ' T"^11"' T+Tm /* (я,. я. я.я\ /я.. я. я.я\ у+яй; -у+я«; Т+ут j, (^-y+«ft; -у+««; Т+Тт> ■«* f5n 7я^ /"" |ИВЛт:т/Лт: |lя^ (1я 7л\ (Пя |lя 334
§ 6. Неравенства 1689. а) —*L b \ б) -£- b в) — -z b 1690. а) л — arcsin— б) - в) —~ 1691. ^ О г) -p 4 arcs\n-£-+2nk; —0,7)+2л/г; г) 1692. —%- 5 1693. -£- 4 — 1696. —y 6 . 1695. ~+2л*. 1694. 4 —arcsin-г- о л + arccos ~+ 2nk < д: < я + arctg 3 -f- 2nk. 1697. -^/~у+2яЛ, где /г6^. 1709. у 1717. -у Э35
1718. -у£+у*<*<у£+у*- !719- 1720. -оо<х<оо. 1721. —j+nk<x<nk' 1722.-^ 1723.|*<*<£+|*. 1724. -£+|*<*<-21+|*; _£ 1725. --^-+2лЛ<*<у+2л*; у+2л*<*<5р+2л*. 1726. -у 5р+2л*. 1727. ^Ч-у *<<+ 1728. —£+2л*<х<2яЛ; -"-+2лЛ<х<л + 2л^ 1729. -£- 3 3 6 1730. 2arcctg2 + 2W*<x<2arcctg(-y) + 2n*. 1731. -^ ~Н-2лЛ. 1732. -JL 4 о £- + л*; ~+лЛ<х<-^-Н-лЛ. 1733. —^- 4 о z о ^^+2л.. 1734. -£+*!*<,<-£ + *; ^+ ^+уЛ. 1735. _-1+я*<х<--у+л*; лЛ<х<~+лЛ; у у+яЛ. 1736. —£11 JL^ *£ <^+2лЛ. 1737. -JL j + 1738. -1?+я*<*<-у+л*; -|-+яЛ<^<^ + лЛ. 1739. -у ; я + arcsin у+2лЛ<х<у+2лЛ. 1740. —j у+2лЛ. 1741. ±+\0k<x<+Wk ^+ ЮЛ<< ; В25 + ЮлА<;с<у^+1ОлЛ. 1742. -^-+2лЛ<х<Н2лЛ; . 1743. Зл+24лЛ<л:<~^-Н-5л + 24лЛ; 12л—~ 4 4 ^ ; 21л—£- f 7. Уравнения, системы уравнений и неравенства с параметрами 1744. nk при а< — 2, а>2; х, =лЛ, x2=i-(- 1)* arcsin -^+4-* ПРИ —2<а<2. «5 Z о 1745. 0 при а<-8. а>8; -^-+(— i)* arcsin 4~+nk ПРИ —8<а<8. 1746. x£R при о о а=2лЛ; Х|=л + 2ял, ^2=в + л + 2лт при аФ2пк. 1747. x£R при а 336
— -£-+(— l)"-£- + Ji/i при 2 о . 1748. 0 при а<0, а>1; arcsin -f (— 1)* arcsin j nfe при 0<а< 1. 1749. при а = лЛ; -г- 4 У2а + 2 1750. 2л& при а рациональном; 0 при а иррациональном. 1751. 0 при a<-j-, a>l; —arccos ^Ь у* при —<а<1. 1752. 0 при а<—у, а>у; ■^■ приа= — у; —-*- приа=у; у(—1)* arcsin (1— у£ при —у< <а<у. 1753. 0 при а=у ' 2 л а=——+я(Л — п)\ —а—j- при Зя " 4 1 п \ 1754. —fc при а= — 1; ли при а=3; лп=л£, х2.з= ±-s-arccos—^—[-я£ при —К <а<3. 1755. yfc при а=1; у+л* при а=— 3; *,=-—+л£, х2. з = я д при —3<а<1. 1756. 0 при а=-^- + л (2л —Л); ~-f- при а#у 1757. у при а=у =_а+|+лЛ ПрИ а^ 1758. -~^ при 1759. 0 при при а=1. 1760. 0 при = ; 5<а<3; у+2лЛ при а=—5, а=— 3. 1762. 0 при а<—4, а>2; л при а= . 1761. 0 при а<—5, а>3; (— 1)* arcsin ( — при а=— 4; 2л£ при а = 2; dzarccos " ° при —4<а<2. 1763. 0 при — 72, ; nk 1764. -j ==—\/2; -т- у arccos (3 — 2УЗ —а2) + л£ при — при 1765. ^+2л* при a=V2; ~+2лЛ при а=—>/2; -"-ЬяЛ при ( аф-у/2, 4 4 4 {а^- 1766. 0 при а=у -г--|-ял; ——\-лк при а=-г—нлл; —;—ь ^ t 4 4 4 4 при а=—-г-н-ял; arctg (tg a±Vtg2 а — 1)+л£ при —Л-Н-лл<а<—~-f -Л-+лл<а<у+лл. 1767. 0 при —2<а<2; л+2л£ при а=— 2; 2лй при 337
a=2; ifcarccos g ^2nk при а<—2; ±arccos-——^ |-2я£ При а>2. , л , ft . cos a ± -ylcos2 a — sin а , 1768. 0 при ф4-2ял<а<л —ф+2ял; arctg ^Ц \-nk при я — — ф+2ял<а<я + 2ля, л-Ь2ял<а<2л + 2л/1, 2ял<а<ф+2лл; arctg(ctga) при а = ф+2лл; arctgf ±-9-) +nk при а=пп (ф=агс5т ~" 7~ V 1769. 0 при а= —1; 1 при а# —1. 1770. 0 при а<0, а=2; ±~\/—при Га>0, V а \аФ2. 1771. я^ при а<—J-, а>5; ^1 = яЛ, дг2.з = ±arccosf——)+2nk при а=,—т-; *1 = я/г, Х2, э=±arccos "" ""; +2я& нри —т"<а<^ *1 = яЛ, Х2,з=±-оГ+ 4 4 о при а=1; ^1=яЛ, хг, з= ±arccos ~р—— -\-2nk при 1772. —j+nk при а<—|, а>2; ~j+nk при а=~у; ^i = —~ in5^+y/5 при -А<а<2; -^-+яЛ при а=2. 1773. 0 при n+nkt X2=-aTctg3+nk при а при ^Ш<а<1, 1<а<^±^. 1774. 0 при а<—V2, а>У2; I -~f 2яЛ; — ——2я£ ) при а=— V2; ( -г- ; ~-±arccos^-2я^) при -^<a<V5; (^-+2яЛ; ~-2ял) при 1775. 0 при a<-V5, a>V3; (~2яЛ; —2-+2яЛ^ при а=—>/3; (у ^ + ji*; у+(- 1)я arcsin ^-я*) при -V5<a<V3;(-~- ^^ при a=V3. 1776. 0 при а<0, а>1; (яЛ; л — nk) при а=0; ((-1)* arcsin (±VS)+лЛ; я- (-1)* arcsin (±ja)-nk) при 0<а< 1; (у+яЛ; у- — я/5 j при а=1. 1777. 0 при а<0, а>1; (яЛ; у—я/5 J при а=0; -1)* arcsin (±л/а)+**; у~(->)* arcsin (±^)-ял) при 0<а<1; (у+я^; яЛу при а=1. 1778. 0 при а< —1, а>1; Ink; у—яЛ J при а= —1; f ±y arccos (—а)+яЛ; y^Fy arccos (—а)—nk ) при —1<а<1; fy+яЛ; у— -яЛ при а=1. 1779. 0 при а<—^-, а>-^ хх „. arccos 4а+arccos 2а . arccos 4а—arccos 2а 1 ^ ^ 1 -л)), где а= ^ , р= g , при —<а<—. 338
1780. 0 при . -1Ь); л- _р + я(л-*)), где а» у. 1781. 0 при arcsin За-f-arcsin а arcsin За —arcsin а 1 , при -—< л); у(Л — яЛ при а=0. 1782. 0 при 1р^-я(л-^ при уТу+л(л-Ж при а=±у+2лл. 1783. 0 при а<—у, а>у ((—1)*+| arcsin a + nk; (— 1)" arcsin 2а + ял), ((— 1)* arcsin 2а-f я*; (— l)rt+l arcsin а 11 Ч \ Ч-лл) при —у<а<у- 1784. 0 при а<у; arcctg arccos 2я/! при а>0. Часть III Дополнительные задачи § 1. Комбинированные уравнения, системы уравнений, неравенства 1787. -£ о - !788« "Т 6 1790. -jH-y*-1791- у -1792- т+у*-|793- . —?■+«*• 1789' 6 *где =0, 1, 2. 3 1794. log2 (у+4я* Y где ft = 0, 1. 2 logj(-y+4n/t Y где л = 1, 2.3 1795. log2 (^-+i*), где*=0.1. 2 |0g2(-^-+y n ), где л = I. 2, 3 1796. ***, |=+^Л, -^+^ft. 1797. 4 у+2лл. 1800. -~ 1801. 102*+т, 102rt. 1802. (-1)* arcsin (л/lO-4)+я^. 1803. у£ + 2яЛ, -^Ч-2лл. 1804. (-1)* arctg -yfi+nk. 1805. arccos ]^+2я*. 339
1Мв.у+2я*. I807.-J.+ 2**. 18О8.(-1)"|-+яп. 1809.-£+2я*. 1810. -i- arccos -L+ +2як. 18U.^+4nfe. 1812. arctg ~1+~^+2як. 1813. arccos ±-+2nk. 1814.4+"* О 4 О О 1815. -£+2л*. 1816.-i-f яЛ; я-arcctg 5+ял. 1817. ~. 1818. ±arccos (~\ + + 2л*. 1819. -^+2яЛ. 1820. -^+2я*. i82i. _-?.+2яЛ. 1822. ^+2яЛ. о о 4 3 1823. ±arccos ~+2яЛ; ±arccos—^±^Ё+2ял. 1824.4+4лЛ,^+2ял. 1825. -£+ 4 Z ODD . 1826. -у+2л*. ^+4ял. 1827. ~+4я/г. 1828. ±±+^k. 1829. ±~+ +у. 1830. ~ + 2л*. 1831. ^-+2nk, 1832. (-1)*£+л*; (-1)*уarccos ^T^+ + яЛ. 1833.-£+2л*, ~+2ял. 1834. ±4+^- ^^5. л*, -£-+2лл. 1836. ±-^.+2л^ 4 4 О О О (— 1)*+I arcsin4-+ji*. 1837. ~+2ял. 1838. 2я —arctg — ; ^г; 2; 6. 1839. а) 0 4 z о ^ <jc<arcsin "*!2 3 . 1858. я<х<агс^ (V2— 340
<«- -* 1861. 2лЛ<х<-^+2лЛ, -Я-+2лл<х<у+2ял. 1862. I . 5я кхфт 1863. arctg2 + n/j; л* <*<-£-+л£; arctg 4 +ял <*<-£-+яп. 1864. arctg— 4 z z ^-+л*. 1865. y+n*<Jt<-*-+nfc. 1866. 2л*<х<л + 2лА: (* = 1, 2, 3, ... . 1867. -£+я*<*<-£-+я*. 1868. -.?р<л:<~4; Л+2лл<^<~ 4 о Z О Z ~+2я/ (/#=0; —1); (-l)m-*-+*m («^0; 1). Z D О 1869. -±+nk^x^-%-+nk. 1870. -~+2лЛ<л:<4+2л/г. 1871. 4- 4 4 DO о <~+л*. 1872. 2лЛ<^<~+2лЛ. 1873. 4+^Г- 1874- -arccos ±- о Z о 4 7 . 1875. ^^<х<у; -|-<х<^Ь^. 1876. 0. 1877. у , 1, 2, ...), 0<*<1. 1878. ( я + 2яА5<^<2я+2лЛ (Л=0, 1, 2, ...). . 1879. (я + 2л*; log3 (я + 2лЛ)) (k=0t 1, 2, 3, ...); ((- 1)я~+ (-1)"у+ял)) (п=0,1,2,3,...). 188О.^Зп-|-,Л+~);(бл—~ п=1, 2, 3, 4 1881. (+2 л+2л) ( ,882. (^+; ^+) . (^+; +) MV^ Р)0 при " -l)»arcsin lO^^'^+nft при -VS<a<-l; (-l)»arcsin a<-V2t a>V2. 1885. 0 при a<0, a>l; ( — 1)* arcsin 2 при 0<а<1. 1886. 0 при а<——; arcctgfdb . ) + nk при а>—я-. * N Vl+2a/ 1887. 0<*<arcsina, i<jc<~ при 0<a<sin 1; arcsin a<*<-jp 0<jc<1 при sin 1<а<1; 0<jc<1 при а>1.-1888. 1) arcsin а<*<1 при 0<a<sin 1; 2) 1< 341
<*<arcsina при sin 1<а<1; 3) 1<*<у при а>1. 1889. 0 при а<3, , ((а —З)2, cos (а —2), при 3<а<л + 2. 1890. 0 при а< — 1, (Iog2(a+1); 5; sin(a + 2))npn -1<а<^1. 1891. 126 + 9; 126+11. 1892. 2<а< <2,5; 2,5<а<5. § 2. Использование свойств функций при решении уравнений и неравенств 1893. 1. 1894. 2. 1895. 3. 1896. 1. 1897. 2. 1898. 0; 2. 1899. 2; 4". 1900. 1. 1901. 2. 4 1902. 10. 1903. 3. 1904. 0. 1905. 0. 1906. 0. 1907. 0,5. 1908. 2. 1909. 0,5. 1910. 0,5. 1911. 0. 1912. л. 1913. -1; 1. 1914. ~\ ~\ ~; ^. 1915. 9. 1916. 1. 1917. 2. о о о о 1918. 0. 1919. л. 1920. 0. 1921. 0. 1922. л. 1923. 0. 1924. 0. 1925. -1. 1926. ~. 4 1927. у. 1928. ^-. 1929. 1. 1930. -j . 1931. -|^ . 1932. --j- - 1933- 1934. 1. 1935. 0. 1936. л. 1937. ^. 1938. -^ + 6л£. 1939. -3. 1940. -^-. 1941. 0. 1942. 0. 1943. у+2л£. 1944. 1. 1945. 3. 1946. —у. 1947. 0. 1948. [3; оо). . (о; у Y 1950. (1; 1,5). 1951. (~у; о\ 1952. 2. 1953. -1. 1954. 2. 1955. 0. ^y^~. 1958. 0. 1959. (nk; 1). 1960. (y+ylog4 3; y- 1949 1956. nk. 1957. 1 i „\ <лл. 6л 12л <лл 9л 17л „ „ллп 12л 24л 36л -Tlog43). .961. т; —. 1962. т; ^-; 5п. 1963. -g-; ^; ^-. § 3. Нестандартные уравнения и неравенства 1964. (-1; 1). 1965. (-3; -3). 1966. (l; f), (l; -f), (-1; f), (-1; -у)- »967. (1; -3). 1968. (11; 5). 1969. (ft; 0), (ft; 2). 1970. (--^+|n; -J+-Jft). 1971. (-2; яй), (2; y+nft). 1972. (l; -j+2jlfe)' (~1; Y+2nftY 1973. ^y+nft; y+nftY 1974. (nft-arctg^; -j-+2nftV (лп + |). 1975. ((-1)*+' -J+nft; ^+2nn), ((_l)*A+^; ^+ . 1976. (^-+2nft; 2лп), Г^+2лй; л + 2лпУ 1977 (nft; 0). 1978. (1; 1). 1979. (у+2я(я + й); _£+2л(n-ft)); (±+2n(n + k); _|^+2л(п-Л)); ) ( ~2-+2л (n + ft); —J+2n(rt-ft)). 198O.(-y+ t+"*= t+ . 1982. (-1; 1+-~+^-Aj\ 1983. (1; 1+4Л), (-1; 1+4Л). 1984. (0; 0). 342
1985. (о; у+2яА:). 1986. (як; 1). 1987. (1; ^-?). 1988. (-| ; -5._2. 1992. (г: ±у 1993. (0; 2лк<у<л + 2лк), (»; -у+2яй<(/<у+2яй). 1994. У ( у; —J+y m) . 1995. (4; 6; 2). 1996. (т +я(пЛ); +2яй) (5+я(я + *): - -.+„(„_*); 2я*). (~+n(n + fe); _i 1997. (^ ; 2я*), (j£ . 1998. (i 35 ) ( у ) ( +уА; у+яп; у+я/nV 1999. ^+уй; лп; л/nY 2000. (nk; ял; (-l)my + *") 2Ш- 0: °)- 2002. (0; 1). 2003. (0; 1). 2004. (0; nk). 2005. f-2-+n*- J-) / я \ \2 ' 10/ 2006. ^0; -—+2nk). 2007. (0; 1). 2008. (л + 2я*; /). где /gj?. 2009. (0; 1) 2010. (0; 0). 2011. (1; 0).
Указания к решению задач части III. 1803. Преобразуйте уравнение к виду I sin х—— cos x J = sin3 x и далее ( sin х—— cos x\ ( sin2 x + cos2 x\ =sin3 x. Это однородное уравнение сводится к уравнению третьей степени относительно ctg*, откуда ctgx=0, ctg * —V3. 1836. Уравнение преобразуется к виду 17 + 8 sin*-16sin2х 4sin2x/llil . ч2 и далее (1+4 sin дг)2=4 sin2x(l-f 4 sin xft откуда *=(— l)n+I arcsin ~+ля, = ±-^-+лЛ. Проверка по условию 2 tg *(l+4 sin jc)>0 отсеивает во второй серии часть решений. 1839. Найдите область определения неравенства. 1855. Задача сводится к решению совокупности двух систем неравенств -~<ctg**; | sin2*-A Вторая система не имеет решений, так как при 0<tg*<l имеем ctg2*>l. 1862. Так как левая часть неравенства преобразуется к виду logte»«x *S 2x<0,55, 1 т. е. -^-<0,55, то достаточно найти область определения. 1883. Тригонометрическое уравнение преобразуется в однородное cos2 jc«/—3 sin xy cos xy + 2 sin2 xy=0t откуда igxy=\ или igxy=—. находим*в+2*2-3<0, ~3<^2< 1,т.е. -2<j«/<2. Значит, из решений уравнений tg xy= I, tg xy~— надо отобрать те, что принадлежат отрезку [—2; 2]. г 1897. Преобразуйте уравнение к виду 5* = (3ДГ —2х)2 и далее Ь1ЪУ = 3х—2х, 1899. Рассмотрите уравнение как квадратное относительно fog2 x. 1903. Преобразуйте уравнение к виду (х+ 1)2=(б — ->Jx+1)2. 1906. Учтите, что -у + ;с+у>1- 1912. Преобразуйте уравнение к виду 2sin*=— и используйте графический л метод решения. 344
1916. В точке jc = 1 графики функций пересекаются, имеют общую касательную и разный характер выпуклости. Значит, х=\ — единственный корень уравнения. 1917. Используйте ту же идею, что в № 1916. 1923. Из ~<jf<~ получаем-л/-~^-\/--—х >0, а тогда у-«—х + +*—1<0, т. е. корней нет. 1925. Преобразуйте уравнение к виду -yJx2 + x=^Jx+3—-j2 и учтите, что при *<— 3 левая и правая части представляют собой функции разного характера монотонности. 1932. Имеем cos2( -^- (sin *+л/2 cos2 хи . , откуда cos2( -j (sin x+-fi cos2 *)) = 1, Первое уравнение преобразуется к виду sin *+V2cos2 x=4nt что возможно лишь при л=0. 1939. Докажите, что sin^^^l, a ^Ooga \x\ ^ 1941. Преобразуйте уравнение к виду A sin x-\-B cos *=-—-, где Л = 1, В=— sin 15*, и учтите, что тогда ->JA2 + B2 <У2- 1947. Если / = arcctg х, то sin4 /-j-cos4 /= . 2 ; здесь левая часть не больше 1, а правая не меньше 1. 1949, 1950, 1951. Используйте графический метод решения неравенств. 1952, 1953, 1954, 1955. Докажите, что левая часть неравенства не превосходит I. 1957. Задача сводится к решению системы неравенств ( sin2 |^ cosf л^дг+—sinn*jj = — 1. 1958. Используйте ту же идею, что в № 1957. 1959 Преобразуйте второе уравнение системы к виду 9/ + qJt=-^—гтъ » /=92sinJf, /6 [1; 81]. Докажите далее, что наименьшее значение функции 9/-f 575 на [ 1; 81J совпадает 2*/+ 16г/ + 64 с наибольшим значением функции ——м "1 и сделайте отсюда вывод о возможных значениях х и t/. 1960. Преобразуйте систему к виду Г 4дс+^-!-Ь3.42^-1 I 4x+^l.42i/-|>-i и введите новые переменные. 1961, 1962, 1963. Воспользуйтесь тем, что наименьшее значение одной части уравнения совпадает с наибольшим значением другой части уравнения. 1968. Представьте уравнение в виде а 1 i-=28, а далее ^*"~" ' 4- а о а 345
1971, 1972. Рассмотрите уравнение как квадратное относительно х. 1974. Преобразуйте уравнение к виду (tg * + (sin y + cos J/))2+(l — sin 2*/)=0 {tg * + sin i/ + cos t/=0, sin 2i/= I. 1975. Рассмотрите уравнение как квадратное относительно sinjc. 1976. Рассмотрите уравнение как квадратное относительно cos у. 1977. Рассмотрите уравнение как квадратное относительно 2smx. 1978. Рассмотрите уравнение как квадратное относительно Iog2 (* + #). 1979. Рассмотрите уравнение как квадратное относительно cos—=-=^. 1980. Рассмотрите уравнение как квадратное относительно у. 1981. Докажите, что левая часть уравнения не меньше 4, а правая не больше 4. 1985. Докажите, что левая часть уравнения не больше 2, а правая не меньше 2. 1987, 1988. Используйте идею JSfe 1985. 1989. Используйте идею № 1981. 1990. Используйте идею № 1985. 1991. Докажите, что левая часть уравнения не меньше —, а правая не больше -г-. 1992. Преобразуйте уравнение к виду 4/2 + (6а—8) /+(13—6а)=0, где а=У4х—x2, /=cos (*+*/), и рассмотрите его как квадратное относительно /. 1993. Рассмотрите уравнение по отдельности в каждом из четырех возможных случаев: лг=О, jc=1, х= — 1, 0<*< 1. 1994. Докажите, что левая часть уравнения не меньше 4. 1995. Преобразуйте первое уравнение системы к виду 1996. Преобразуйте первое уравнение системы к виду 4/2-т-4 sin (x—y)t + \ =0, квадратному относительно /=sin (х+у). 1997. Преобразуйте первое уравнение системы к виду 4/2—4У5cos/-/+3=0, квадратному относительно /=cos (x -\- у). 1999. Докажите, что задача сводится к решению системы уравнений l. 2000. Докажите, что левая часть первого уравнения системы не меньше 4, а правая часть не больше 4. 2001. Так как х^\р-\-\у то левая часть неравенства не меньше 1. 2002. Докажите, что левая часть неравенства не больше 1, а правая часть не меньше 1. Это значит, что неравенство сводится к системе уравнений 2005. Преобразуйте неравенство к виду /sinx=l, и докажите, что оно сводится к системе уравнений ji_ __j 2006. Преобразуйте неравенство к виду Э46
((sin (x f Ski (* + !/)=-1, и докажите, что оно сводится к системе уравнений |з*г=1. 2008. Воспользуйтесь тем, что "Д/ у2+х ■ , — 1 > \у\. V |COS JC| 2009. Докажите, что у^\% и воспользуйтесь тем, что 2 arcsin (^2+|/)^л, 2010. Докажите, что |/<0, lg(l+t/)<0 и arcsin(2ul+^)<y. 2011. Докажите, что log | (1 + *)> — 1, и воспользуйтесь тем,
ПОСЛЕСЛОВИЕ Настоящее пособие предназначено для студентов физико-математических факультетов педагогических институтов и имеет своей целью дать студентам и преподавателям педвузов достаточно разнообразный материал для практических занятий по многосеместровому курсу элементарной математики. При написании этого пособия мы опирались на нашу книгу «Практикум по решению математических задач. Алгебра. Тригонометрия», опубликованную издательством «Просвещение» в 1984 г., существенно ее переработав и дополнив. Остановимся на тех изменениях и дополнениях, которые сделаны при переработке указанной книги. Прежде всего отметим включение в пособие нового раздела «Дополнительные задачи». В нем рассмотрены уравнения, системы уравнений и неравенства, которые не относятся к стандартным: комбинированные уравнения и неравенства; уравнения и неравенства, содержащие избыточное число переменных; уравнения и неравенства, решаемые с использованием свойств функций (монотонность, выпуклость, достижение наибольших и наименьших значений). В трех параграфах этого раздела имеется свыше 30 разнообразных лримеров, подробно разобранных в тексте, и более 200 упражнений, предлагаемых для самостоятельного решения. Включение этого раздела в книгу делает имеющийся в ней алгебраический и тригонометрический материал завершенным. Существенно пересмотрена вся система упражнений: более логичной стала их компоновка в каждом параграфе, увеличено число упражнений, значительно больше стало задач повышенной трудности. Теперь в книге свыше двух тысяч упражнений, подобранных таким образом, чтобы они в системе представляли собой надежный фундамент методико-математической подготовки учителя как для ведения уроков, так и для внеклассной работы в школе. Ко всем упражнениям в конце книги приведены ответы, а ко многим задачам из дополнительного раздела — и указания к решению. Редакционные или методические изменения по сравнению с нашей 348
предыдущей книгой коснулись практически всех параграфов, в ряде случаев переработка была радикальной — это относится, например, к § 11 главы I и к § 1 главы II. Предлагаемая вниманию читателя книга — не только и не столько задачник, сколько практикум. Это нашло свое отражение в структуре книги. Каждый параграф содержит необходимый теоретический материал (в справочной форме, в виде методических рекомендаций и т. д.) и довольно большое число (свыше 300) примеров с решениями. Поэтому книга будет полезна широкому кругу читателей: студентам и преподавателям педвузов, учителям математики общеобразовательных школ, лицам, интересующимся математикой или готовящимся к поступлению в высшие учебные заведения. Авторы.
ОГЛАВЛЕНИЕ Часть I. Алгебра. Глава I. Тождественные преобразования. $ 1 Разложение многочленов на множители 3 Упражнения (1—50) 5 § 2 Тождественные преобразования рациональных выражений .... 7 Упражнения (51 — 118) 12 $ 3 Тождественные преобразования иррациональных выражений ... 15 Упражнения (119—181) 20 $ 4 Тождественные преобразования показательных и логарифмических выражений . 23 Упражнения (182—215) 25 § 5 Доказательство неравенств 27 п. 1. Доказательство неравенств с помощью определения 27 п. 2. Синтетический метод доказательства неравенств 28 п. 3. Доказательство неравенств методом от противного ..... 31 п. 4. Доказательство неравенств методом математической индукции 32 Упражнения (216—268) 34 § 6 Сравнения значений числовых выражений 36 Упражнения (269—284) 38 Глава II. Решение уравнений, систем уравнений и неравенств. § 7 Равносильность уравнений 39 Упражнения (285—330) 45 § 8 Рациональные уравнения 47 Упражнения (331—382) ..." 52 § 9 Уравнения, содержащие переменную под знаком модуля 53 Упражнения (384—410) 57 $ 10 Системы рациональных уравнений 57 п. 1. Основные понятия 57 п. 2. Основные методы решения систем уравнений 63 п. 3. Однородные системы 67 п. 4. Симметрические системы 70 Упражнения (411—479) 72 § 11 Задачи на составление уравнений и систем уравнений 74 п. 1. Задачи на числовые зависимости 74 п. 2. Задачи на прогрессии 75 п. 3. Задачи на совместную работу 77 п. 4. Задачи на сплавы и смеси 80 п. 5. Задачи на движение 83 Упражнения (480—627) 88 § 12 Иррациональные уравнения и системы уравнений 101 п. 1. Решение иррациональных уравнений методом возведения обеих частей уравнения в одну и ту же степень 102 п. 2. Метод введения новых переменных 105 п. 3. Искусственные приемы решения иррациональных уравнений . 110 п. 4. Системы иррациональных уравнений 111 Упражнения (628—722) 112 § 13 Показательные уравнения 115
Упражнения (723—770) 121 § 14 Логарифмические уравнения 122 п. 1. Решение уравнений вида \oga f (x) = \ogag (х) и уравнений, сводящихся к этому виду 123 п. 2. Решение уравнений вида \oga(x)f M=logfl(A)g (*) и уравнений, сводящихся к этому виду 127 п 3. Разные логарифмические уравнения 128 Упражнения (771—851) 130 § 15 Системы показательных и логарифмических уравнений 132 Упражнения (852—881) 135 § 16 Рациональные неравенства 136 п. 1. Основные понятия 136 п. 2. Рациональные неравенства 137 п. 3. Системы и совокупности неравенств с одной переменной .... 142 п. 4. Неравенства, содержащие переменную под знаком модуля ... 147 п. 5. Задачи на составление неравенств 150 Упражнения (882—1004) 152 § 17 Иррациональные неравенства '. 157 Упражнения (1005—1057) 163 § 18 Показательные неравенства 164 Упражнения (1058—1101) 168 § 19 Логарифмические неравенства 169 Упражнения (1102—1188) 174 § 20 Уравнения, системы уравнений и неравенства с параметрами .... 176 Упражнения (1189—1272) 194 Часть II. Тригонометрия. Глава III. Тождественные преобразования. § 1 Тождественные преобразования тригонометрических выражений ... 198 Упражнения (1273—1386) 207 § 2 Тождественные преобразования выражений, содержащих обратные тригонометрические функции 212 Упражнения (1387—1433) 216 § 3 Доказательство неравенств 217 Упражнения (1434—1495) 225 Глава IV. Решение уравнений, систем уравнений и неравенств. § 4 Уравнения 227 Упражнения (1496—1644) 248 § 5 Системы уравнений 253 Упражнения (1645—1688) 262 § 6 Неравенства 264 Упражнения (1689—1743) • 272 § 7 Уравнения, системы уравнений и неравенства с параметрами .... 274 Упражнения (1744—1786) 282 Часть III. Дополнительные задачи. § 1 Комбинированные уравнения, системы уравнений, неравенства . . . 283 Упражнения (1787—1892) 295 § 2 Использование свойств функций при решении уравнений и неравенств . . 299 Упражнения (1893—1963) 307 § 3 Нестандартные уравнения и неравенства *.. 310 Упражнения (1964—2011) 317 Ответы 319 Указания к решению задач части III 344 Послесловие . ... 348